0% encontró este documento útil (0 votos)
91 vistas486 páginas

Aritmetica

Cargado por

Melvin Sermeño
Derechos de autor
© © All Rights Reserved
Nos tomamos en serio los derechos de los contenidos. Si sospechas que se trata de tu contenido, reclámalo aquí.
Formatos disponibles
Descarga como PDF, TXT o lee en línea desde Scribd
0% encontró este documento útil (0 votos)
91 vistas486 páginas

Aritmetica

Cargado por

Melvin Sermeño
Derechos de autor
© © All Rights Reserved
Nos tomamos en serio los derechos de los contenidos. Si sospechas que se trata de tu contenido, reclámalo aquí.
Formatos disponibles
Descarga como PDF, TXT o lee en línea desde Scribd

TEORÍA DE NÚMEROS

Metodología Problem Solving


Con aplicaciones en criptografía y programación Phyton

Gerard Romo Garrido

Toomates Coolección vol. 6


Toomates Coolección
Los libros de Toomates son materiales digitales y gratuitos. Son digitales porque están pensados para ser consultados mediante un ordenador,
tablet o móvil. Son gratuitos porque se ofrecen a la comunidad educativa sin coste alguno. Los libros de texto pueden ser digitales o en papel,
gratuitos o en venta, y ninguna de estas opciones es necesariamente mejor o peor que las otras. Es más: Suele suceder que los mejores docentes
son los que piden a sus alumnos la compra de un libro de texto en papel, esto es un hecho. Lo que no es aceptable, por inmoral y mezquino, es
el modelo de las llamadas "licencias digitales" con las que las editoriales pretenden cobrar a los estudiantes, una y otra vez, por acceder a los
mismos contenidos (unos contenidos que, además, son de una bajísima calidad). Este modelo de negocio es miserable, pues impide el
compartir un mismo libro, incluso entre dos hermanos, pretende convertir a los estudiantes en un mercado cautivo, exige a los estudiantes y a
las escuelas costosísimas líneas de Internet, pretende pervertir el conocimiento, que es algo social, público, convirtiéndolo en un producto de
propiedad privada, accesible solo a aquellos que se lo puedan permitir, y solo de una manera encapsulada, fragmentada, impidiendo el derecho
del alumno de poseer todo el libro, de acceder a todo el libro, de moverse libremente por todo el libro.
Nadie puede pretender ser neutral ante esto: Mirar para otro lado y aceptar el modelo de licencias digitales es admitir un mundo más injusto, es
participar en la denegación del acceso al conocimiento a aquellos que no disponen de medios económicos, y esto en un mundo en el que las
modernas tecnologías actuales permiten, por primera vez en la historia de la Humanidad, poder compartir el conocimiento sin coste alguno, con
algo tan simple como es un archivo "pdf". El conocimiento no es una mercancía.
El proyecto Toomates tiene como objetivo la promoción y difusión entre el profesorado y el colectivo de estudiantes de unos materiales
didácticos libres, gratuitos y de calidad, que fuerce a las editoriales a competir ofreciendo alternativas de pago atractivas aumentando la calidad
de unos libros de texto que actualmente son muy mediocres, y no mediante retorcidas técnicas comerciales.
Estos libros se comparten bajo una licencia “Creative Commons 4.0 (Atribution Non Commercial)”: Se permite, se promueve y se fomenta
cualquier uso, reproducción y edición de todos estos materiales siempre que sea sin ánimo de lucro y se cite su procedencia. Todos los libros se
ofrecen en dos versiones: En formato “pdf” para una cómoda lectura y en el formato “doc” de MSWord para permitir y facilitar su edición y
generar versiones parcial o totalmente modificadas.

¡Libérate de la tiranía y mediocridad de las editoriales! Crea, utiliza y comparte tus propios materiales didácticos

Toomates Coolección Problem Solving (en español):


Geometría Axiomática , Problemas de Geometría 1 , Problemas de Geometría 2
Introducción a la Geometría , Álgebra , Teoría de números , Combinatoria , Probabilidad
Trigonometría , Desigualdades , Números complejos , Funciones
Toomates Coolección Llibres de Text (en catalán):
Nombres (Preàlgebra) , Àlgebra , Proporcionalitat , Mesures geomètriques , Geometria analítica
Combinatòria i Probabilitat , Estadística , Trigonometria , Funcions , Nombres Complexos ,
Àlgebra Lineal , Geometria Lineal , Càlcul Infinitesimal , Programació Lineal , Mates amb Excel
Toomates Coolección Compendiums:
PAU España: Cataluña TEC Cataluña CCSS Valencia Galicia País Vasco Baleares
PAU Internacional: Portugal A Portugal B Italia UK (A Level) IB Francia (BAC)
Canguro: ESP CAT FR USA UK AUS
USA: Mathcounts AMC 8 10 12 AIME USAJMO USAMO TSTST TST ELMO Putnam
España: OME OMEFL OMEC OMEA OMEM CDP
Internacional: IMO OMI IGO SMT INMO CMO REOIM Arquimede HMMT BMO
Pruebas acceso: ACM4 , CFGS , PAP
Pizzazz!: Book A Book B Book C Book D Book E Pre-Algebra Algebra
AHSME: Book 1 Book 2 Book 3 Book 4 Book 5 Book 6 Book 7 Book 8 Book 9

¡Genera tus propias versiones de este documento! Siempre que es posible se ofrecen las versiones editables “MS
Word” de todos los materiales, para facilitar su edición. Descarga en los siguientes enlaces la versión ".doc" de
este documento:
[Link]/biblioteca/[Link] → [Link]

¡Ayuda a mejorar! Envía cualquier duda, observación, comentario o sugerencia a toomates@[Link]


¡No utilices una versión anticuada! Todos estos libros se revisan y amplían constantemente. Descarga totalmente
gratis la última versión de estos documentos en los correspondientes enlaces superiores, en los que siempre
encontrarás la versión más actualizada.

Consulta el Catálogo de libros completo en [Link]


Encontrarás muchos más materiales para el aprendizaje de las matemáticas en [Link]
Descarga toda la biblioteca Toomates Coolección en un solo archivo Aquí

Visita mi Canal de Youtube: [Link]


Versión de este documento: 26/06/2024
Índice.
Primera parte: Primeros pasos con números enteros.

1 Primeros pasos con los números enteros. → Archivo doc


1.1 Bases de numeración.
1.2 Cuadrados perfectos. Potencias perfectas.
1.3 Orden en los números enteros. Principio de inducción.
2 Primeros pasos con Python. → Archivo doc
3.1 Instalación del IDE spyder.
3.2 Operaciones aritméticas con Python.
3 Problemas de la primera parte. → Archivo doc

Segunda parte: Divisibilidad.

4 Divisibilidad. Mcd y mcm. → Archivo doc


4.1 Concepto de divisibilidad.
4.2 Divisibilidad y orden.
4.3 Máximo común divisor y mínimo común múltiplo.
4.4 El Teorema de Bezout (TDB).
4.5 Divisibilidad con números coprimos.
4.6 El algoritmo de Euclides (ADE).
4.7 Máximo común divisor con números coprimos.
4.8 Actividades con Python.
4.9 Problemas de introducción a la divisibilidad.
5 Números primos. → Archivo doc
5.1 Concepto de número primo.
5.2 El Teorema fundamental de la aritmética (TFA)
5.3 Resolución de problemas mediante identidades algebraicas.
6 Problemas de la segunda parte. → Archivo doc

Tercera parte: Aritmética modular.

7 Introducción a la aritmética modular. → Archivo doc


7.1 Primer ejemplo: Las horas del día.
7.2 Segundo ejemplo: El conjunto Z7.
7.3 Los conjuntos Zn.
7.4 Aplicación a la criptografía: El cifrado César.
7.5 Aplicación a la criptografía: El cifrado Hill.
7.6 Problemas de aritmética modular básica.
8 Inversos multiplicativos modulares. → Archivo doc
8.1 Concepto de inverso multiplicativo.
8.2 Existencia y unicidad de inversos multiplicativos.
8.3 Inversión mediante el ADE.
8.4 Inversión mediante exponenciación modular.
8.5 Cancelación modular.
8.6 División modular.
8.7 Divisores de cero.
8.8 Actividades con Python.
9 Congruencias lineales y sistemas de congruencias lineales. → Archivo doc
9.1 Congruencias lineales.
9.2 Sistemas de congruencias lineales (resolución directa).
9.3 Sistemas de congruencias lineales con módulos coprimos.
9.4 Sistemas de congruencias lineales con módulos no coprimos.
9.5 Congruencias lineales mediante sistemas de congruencias lineales.
9.6 Congruencias y sistemas de congruencias lineales con varias variables.
10 Congruencias cuadráticas. Residuos cuadráticos. → Archivo doc
10.1 Congruencias cuadráticas con módulos primos.
10.2 Congruencias cuadráticas con módulos compuestos.
10.3 Congruencias con potencias y polinomios.
10.4 El Teorema de Wilson.
10.5 Residuos cuadráticos. Ley de reciprocidad.
11 Problemas de la tercera parte. → Archivo doc

Cuarta parte: Exponenciación modular y sus aplicaciones.

12 Exponenciación modular. El problema del logaritmo discreto. → Archivo doc


12.1 Exponenciación modular.
12.2 Exponenciación modular optimizada (EMO).
12.3 El problema del logaritmo discreto (PLD).
12.4 Aplicación a la Criptografía: El sistema Diffie-Hellman (DH).
12.5 Aplicación a la Criptografía: El Criptosistema de ElGamal.
13 El pequeño Teorema de Fermat. El Teorema de Euler. → Archivo doc
13.1 El Pequeño Teorema de Fermat (PTF).
13.2 La función Phi de Euler. El Teorema de Euler.
13.3 Orden de un entero.
14 El problema de la primalidad. Encriptación RSA. → Archivo doc
14.1 El test de primalidad de Fermat.
14.2 Aplicación a la criptografía: El método RSA.
15 Raíces primitivas. Índices modulares. → Archivo doc

16 Problemas de la cuarta parte. → Archivo doc

Quinta parte. Aplicaciones.

17 Números factoriales. → Archivo doc

18 Números combinatorios. → Archivo doc

19 Números primos de Fermat y de Mersenne. → Archivo doc


19.1 Números primos de Fermat.
19.2 Números primos de Mersenne.
20 Número y suma de divisores de un entero. → Archivo doc
20.1 Número de divisores de un entero.
20.2 Suma de los divisores de un entero.
20.3 Números perfectos.
21 Encriptación mediante Curvas Elípticas. Encriptación bitcoin. → Archivo doc
21.1 Curvas elípticas sobre cuerpos en general.
21.2 Curvas elípticas sobre cuerpos finitos.
21.3 Protocolo de intercambio de claves Diffie-Hellmann en Curvas Elípticas (ECDH).
22 Ecuaciones diofánticas. → Archivo doc
22.1 Ecuaciones diofánticas lineales.
22.2 Ternas pitagóricas.
22.3 La ecuación diofántica x2-y2=k.
22.4 La técnica del descenso infinito de Fermat.
22.5 El método de la contradicción modular.
22.6 Resolución de ecuaciones diofánticas mediante factorización.
22.7 Resolución de ecuaciones diofánticas aplicando desigualdades.
22.8 Ecuaciones diofánticas en competiciones olímpicas.
22.9 Cuadrados perfectos.
22.10 Ecuaciones de Frobenius. Problema de las monedas.
Soluciones. → Archivo doc (1) , Archivo doc (2) , Archivo doc (3)

El capítulo 14 del Libro de Desigualdades està dedicado a la aplicación de las desigualdades en


la resolución de ecuaciones.
1 Primeros pasos con los números enteros.
La "Teoría de números" o "aritmética" estudia las propiedades de los números enteros.
Los conceptos teóricos de esta rama de las matemáticas pueden ser complicados, muy
complicados y terriblemente complicados. Sin embargo, muchos problemas se
resuelven con solo utilizar el sentido común, toda una serie de estrategias y conceptos
que, de tan obvios que son, los libros de teoría no dedican tiempo a explicarlos.

En este primer apartado se incluyen problemas cuya resolución no necesita ningún


concepto teórico previo, sólo el sentido común, la pura lógica, algunas formulitas de la
matemática elemental y los conceptos de divibilidad aprendidos en primero de ESO. Sin
embargo, no hay que despreciarlos. Es fundamental que el estudiante dedique a cada
problema tanto tiempo como sea necesario, y si no llega a resolverlo, estudie
detenidamente la solución que se presenta al final del libro.

1.1 Bases de numeración.


Definición. Base de numeración.
Diremos que n se escribe como an an1 an2 ... a1 a0 en base b  2 si
n  anb n  an 1b n 1  an  2b n  2  ...  a1b  a0

con 0  ai  b , ai  IN , an  0 .

Por ejemplo, 3562 en base 7 es el número 3  73  5  7 2  6  7  2  1318 , y se escribe


3562 7 .

1.1.1 MF
¿Cuál de los siguientes enteros se puede expresar como la suma de 100 enteros positivos
consecutivos?

(A) 1,627,384,950 (B) 2,345,678,910 (C) 3,579,111,300 (D) 4,692,581,470 (E) 5,815,937,260

ASHME 1997 #20

1.1.2 M
Consideremos el entero N  9  99  999  9999  ..  99
 ...
99
321 cifras

Calcula la suma de todas las cifras de N.

AIME I 2019 #1

1.1.3 M
Para cada entero positivo n , sea d n la cifra de las unidades de 1  2  3  ..  n .
2017
Determina el residuo cuando d
n 1
n se divide entre 1000.

AIME I 2017 #3
1.1.4 MF
Sea a0  2 , a1  5 , a2  8 , y para cada n  2 , define an recursivamente como el
residuo cuando 4an 1  an  2  an 3  se divide entre 11. Determina a2018  a2020  a2022 .

AIME II 2018 #2

1.1.5 F
Multiplicamos todos los números pares del 2 al 98 inclusive, excepto aquellos acabados
en 0. ¿Cuál será la cifra de las unidades del resultado?

(A) 0 (B) 2 (C) 4 (D) 6 (E) 8

AMC10 1999 Sample #14

1.1.6 M
Demostrar que si entre los infinitos términos de una progresión aritmética de números
enteros hay un cuadrado perfecto, entonces infinitos términos de la progresión son
cuadrados perfectos.

OME 1993-94 (Primera sesión) #1

1.1.7 MF
Un entero positivo N se representa como a b c en base 11, y se representa como
1 b c a , donde a, b, c representan dígitos, no necesariamente distintos. Determina el
valor mínimo de N expresado en base 10.

AIME I 2020 #3

1.1.8 MF
Sean x y , y x dos números enteros de dos dígitos. Demuestra que su suma es un
número compuesto.

1.1.9 MF Problema solucionado paso a paso en vídeo.

¿Cuántos números naturales de tres dígitos tienen la propiedad de que cuando sus
dígitos se escriben en orden inverso, el resultado es un número de tres dígitos que es 99
más que el número original?

(A) 80 (B) 72 (C) 64 (D) 81 (E) 8

Cangur B2 2021 #13, Kangaroo Student 2021 #13

Solución: [Link]

1.1.10 MF
Determina la cifra de las unidades del producto (55+1)·(510+1)·(515+1).

(A) 6 (B) 5 (C) 3 (D) 1 (E) 0

Cangur B2 2023 #5, Canguro N6 2023 #10


1.1.11 F Problema solucionado paso a paso en vídeo.

Determina la cantidad de números de dos dígitos con la siguiente propiedad: La suma de


dicho número y el número obtenido invirtiendo el orden de sus dígitos es 132.

(A) 5 (B) 7 (C) 9 (D) 11 (E) 12

AMC 8 2016 #11

Solución: [Link]

1.1.12 MF
La suma a + a + a + b + b + b + b de las siete cifras del número aaabbbb es igual al
número de dos cifras ab . ¿Cuánto vale a + b ?

(A) 10 (B) 11 (C) 8 (D) 9 (E) 12

Cangur B1 2019 #18, Canguro N5 2019 #18

1.1.13 MF
Un palíndromo es un número que se lee de la misma forma hacia delante y hacia atrás.
Determina el mayor entero menor de 1000 que es palíndromo tanto si es escrito en base
10 como si es escrito en base 8, por ejemplo, 292=444ocho.

AIME II 2023 #2

1.1.14 F
La suma de los dígitos del número N es tres veces la suma de los dígitos del número
N+1. ¿Cuál es la suma más pequeña posible de los dígitos de N?

(A) 3 (B) 9 (C) 12 (D) 15 (E) 27

Canguro N5 2024 #23, Cangur B1 2024 #23


1.2 Cuadrados perfectos. Potencias perfectas.
Diremos que un entero n es un cuadrado perfecto cuando podamos encontrar otro
entero m tal que n  m 2 .
De la misma forma, diremos que n es un cubo perfecto cuando podamos encontrar otro
entero m tal que n  m 3 .
En general, si podemos escribir n  m k diremos que n es una potencia perfecta de
grado k.

Los cuadrados perfectos son un tema recurrente en las competiciones matemáticas, y


serán tratados a lo largo de este libro aplicando diversas técnicas.

Por ejemplo, en 7.3.9 se ofrece un cuadro de las caracterizaciones de ciertas potencias


perfectas cuando se trabaja con aritmética modular.

Veamos aquí un problema motivador para que el estudiante analice la relación entre
cuadrados perfectos y la cantidad de divisores de un número:

1.2.1
Veinte estudiantes aburridos se dedican a abrir y cerrar las taquillas de un vestuario,
numeradas del 1 al 20. El primer estudiante abre todas las taquillas; el segundo
estudiante cierra todas las taquillas numeradas 2, 4, 6, 8, 10, 12, 14, 16, 18; El tercer
estudiante se dedica a las taquillas numeradas 3, 6, 9, 12, 15 y 18, si las encuentra
abiertas, las cierra, y si las encuentra cerradas, las abre. En general, el estudiante
número k se dedica a las taquillas múltiples de k: Si las encuentra abiertas, las cierra, y
si las encuentra abiertas, las abre. Determina las taquillas que quedarán abiertas después
de que hayan pasado los veiente estudiantes.
1.3 Orden en los números enteros. Principio de inducción.

Principio de la buena ordenación.


Todo conjunto S de números enteros no negativos contiene un elemento mínimo, es
decir, existe un elemento a  S tal que a  b para todo b  S .

1.3.1 D
a 2  b2
Demostrar que, si a, b son enteros positivos tales que es un entero, entonces
1  ab
a 2  b2
es un cuadrado perfecto.
1  ab
IMO 1988 #6

Teorema. Propiedad arquimediana de los números naturales.


Si a y b son números enteros positivos, entonces existe un entero positivo n tal que
na  b.

Demostración. Supongamos que no es cierto, es decir, que existen dos números a, b  0


tales que n a  b para todo n  0 . Consideremos el conjunto S  b  na , n  0 .
Está claro que es un subconjunto de números enteros positivos, pues
n a  b  0  b  na , y por tanto le podemos aplicar el Principio de la buena
ordenación, es decir, contendrá un elemento mínimo b  m a , para cierto m  0 .
Pero, por hipótesis, b  (m  1) a también pertenecerá a S, luego:
b  (m  1) a  b  ma  a  b  ma , pues a  1 , luego b  m a no puede ser el mínimo,
llegando a contradicción.
Así pues, la propiedad arquimediana de los números naturales debe ser cierta, pues su
negación nos lleva a contradicción.

Principio de Inducción.
Sea S un conjunto de números enteros positivos cumpliendo las dos condiciones
siguientes:
a) 1 pertenece a S.
b) Si n  S , entonces n 1 S
Entonces S es el conjunto de todos los enteros positivos: S  1, 2 , 3,... 

Demostración. Sea T  1, 2 , 3, ...  S , y supongamos que T   , es decir, que no está


vacío, o lo que es lo mismo, que no se cumple el Principio de Inducción.
Aplicando el Principio de la buena ordenación, T contendrá elemento mínimo,
llamémosle a .
Puesto que 1T , pues por hipótesis, 1 S , está claro que a  1 , luego 0  a  1  a .
El número a  1 tampoco pertenecerá a S, pues si a  1 S  a  1  1  a  S ,
contradiciendo la hipótesis. Pero a 1  a , llegando a contradicción, pues habíamos
supuesto que a era mínimo.

El principio de inducción es una herramienta muy poderosa para demostrar fórmulas


que nos serán muy útiles para solucionar una enorme variedad de problemas.
Ejemplo.
n(2n  1)(n  1)
12  22  32  ...  n2  para todo n  1, 2 , 3 , ...
6

Demostración. Sea S el conjunto de números enteros positivos para los que la fórmula
anterior es cierta.
Está claro que 1 pertenece a S, pues
1(2  1  1)(1  1)
12  1 
6
Supongamos que la fórmula anterior se cumple para un cierto valor n , y veamos que,
entonces, se cumplirá también para n  1 :
n(2n  1)(n  1)  n(2n  1) 
12  22  32  ...  n 2  n  1   (n  1) 2  (n  1)  (n  1)  
2

6  6 
 n(2n  1)  6(n  1) 
 (n  1)   (*)
 6 
n(2n  1)  6(n  1)  2n  n  6n  6  2n2  7n  6  (n  2)(2n  3)
2

 (n  2)(2n  3)  (n  1)(n  2)(2n  3) (n  1)(2(n  1)  1)(n  1  1)


(*)  (n  1)   
 6  6 6
k (2k  1)(k  1)
 , tomando k  n  1 , luego la fórmula también es válida para n  1 .
6
Así pues, S  1, 2 , 3,...  , es decir, la fórmula es válida para todos los enteros positivos.
2 Primeros pasos con Python.
2.1 Instalación del IDE spyder.
Instalación del entorno de programación (IDE) de Phyton “spyder”.

Primero descargamos e instalamos en nuestro ordenador la última versión del entorno


spyder (178 Mb) de la web [Link]

Una vez instalado el programa, ejecutamos el entorno Spyder:

¿Por qué spyder y no cualquier otro IDE de Python?


No hay ningún motivo. Es el primer IDE de Python que he encontrado con Google y he
podido instalar correctamente en mi portatil HP Presario CQ57 Notebook con AMD
1.30Ghz y 2 Gb de RAM, con Windows 7 Home Premium 64 bits.

Todos los problemas de este libro se pueden ejecutar en cualquier entorno de


programación Python.
El entorno de programación spyder.

Al ejecutar spyder nos tiene que aparecer una pantalla como esta:

Siempre trabajaremos de la misma manera:

Primer paso:
Escribiremos el programa en la columna de la izquierda.


Segundo paso:
Ejecutaremos el programa pulsando el botón “Play” de la barra de comandos:

o pulsando la tecla “F5”


Tercer paso:
Vemos el resultado en la columna de la derecha.
Programa 1. Visualizar resultados.

La función más importante de todo lenguaje de programación es print: Muestra algo por
pantalla (un texto, un número, el resultado de una operación…)

#
# Este es mi primer programa en Phyton
#
print("Hola a todos")

Recuerda los tres pasos:

Las tres primeras empiezan con “almohadilla” # y son comentarios: No hacen nada, solo
sirven de título. Debemos acostumbrarnos a poner en nuestros programas comentarios,
pues hacen que el programa sea más legible y agradable.
Programa 2. Los inevitables errores de sintaxis.

La única manera de aprender a programar es estudiando ejemplos, copiando ejemplos...


y después jugar con ellos: hacer pequeñas modificaciones...
Preguntarnos ¿Qué pasaría si cambio esto? Y ver qué pasa...

Es inevitable que aparezcan errores, que nos equivoquemos en algo. ¡No pasa nada!
Buscamos donde está el error y lo rectificamos, y de esta manera aprendemos.

Por ejemplo: Modificamos el programa anterior:

#
# Este es mi primer programa en Phyton
#
print("Hola a todo el mundo)

Lo ejecutamos y ¡oh! vemos que da error:

¿Qué habrá pasado? Estudiamos el código: Nos hemos equivocado al no cerrar con
comillas la frase que queríamos visualizar. La rectificamos y volvemos a ejecutar el
programa. Vemos que ahora sí está bien:

Practicar es obligatorio. Equivocarse es inevitable.


2.2 Operaciones aritméticas con Python.
Utilizaremos el operador "+" para la suma de dos números:

print(7+2) Salida: 9

Utilizaremos el operador "+" para la resta de dos números:

print(7-2) Salida: 95

Utilizaremos el operador "*" para la multiplicación de dos números:

print(7*2) Salida: 14

Utilizaremos el operador "/" para la división con decimales de dos números:

print(7/2) Salida: 3.5

Utilizaremos el operador "//" para determinar el cociente de la división exacta de dos números:

print(7//2) Salida: 3

Utilizaremos el operador "%" para determinar el residuo de la división exacta de dos números:

print(7%2) Salida: 1

Utilizaremos el operador "**" para determinar potencias:

print(7**2) Salida: 49

A partir de la versión 3.8 de Python incorpora, a partir de la versión 3.8, una función específica
de exponenciación modular:

print(pow(2,6,11)) Salida: 9
Programa 3. La operación "residuo de la división".

Sabemos que una operación muy importante en criptografía es hacer el residuo de una
división entera. En Python esta operación se codifica mendiante el operador %.

Observa y ejecuta el siguiente programa:

#
# Este es mi segundo programa de Python
#
print(19%5)

Una vez ejecutado, te tiene que aparecer en pantalla el resultado: 4

Desde un punto de vista matemático acabamos de hacer

19  4 mod 5
Programa 4. Las variables.

El objeto más importante de todo lenguaje de programación son las variables. Una
variable contiene una partícula de información que el ordenador almacena y puede
modificar. Las variables se definen mediante letras o palabras.

#
# Este es mi tercer programa de Python
#
n=42
print(n%5)

Te tiene que aparecer el resultado correcto: 2.

Desde un punto de vista matemático acabamos de hacer

42  2 mod 5
Programa 5. Bucles.

Mediante un bucle le pedimos al ordenador que repita una determinada acción, dando a una
determinada variable un rango: Desde un valor inicial hasta un determinado valor final.

Por ejemplo, queremos calcular los equivalentes modulares módulo 5 de todos los números del
0 al 12:

#
# Los equivalentes modulares módulo 5 del 0 al 12
#
for i in range (0,12):
print(i%5)
print("Final")

Programa 6. Mejorando la presentación.

La presentación quedará más elegante si añadimos algún texto:

#
# Los equivalentes modulares módulo 5 del 0 al 12
#
for i in range (0,12):
print(i," = ", i%5, " (mod 5)")
print("Final")
Programa 7. Tabla de la suma modular.

Podemos hacer un programa que calcule la tabla de la suma de un determinado número.

#
# Tabla de la suma módulo 7 de un número.
#
a=3
print("La tabla de la suma del ",a," módulo 7")
for i in range (0,6):
print(a,"+", i, "=" ,(a+i)%7 , " (mod 7)")
print("Final")

Programa 8. Tabla de la multiplicación modular.

Para la operación multiplicar se utiliza el símbolo asterisco: *


El siguiente programa muestra la tabla de multiplicar módulo 7 de un determinado
número.

#
# Tabla de la multiplicación módulo 7 de un número.
#
a=3
print("La tabla de la suma del ",a," módulo 7")
for i in range (0,6):
print(a,"*", i, "=" ,(a*i)%7 , " (mod 7)")
print("Final")

2.2.1 Ejercicio.
Realiza un programa Python que genere la tabla de la suma módulo 13 del número 4.

2.2.2 Ejercicio.
Modifica el programa anterior para que genere la tabla de la multiplicación módulo 13
del número 4.
3 Problemas de la primera parte.
3.1 F
Tenemos tres cartulinas y en cada una se ha escrito un número de cinco cifras. Como se
ve en la figura tres de las cifras están tapadas. La suma de los tres números es 57263.
¿Cuáles son las cifras ocultas?

A) 0, 2 y 2 B) 1, 2 y 9 C) 2, 4 y 9 D) 2, 7 y 8 E) 5, 7 y 8

Canguro N5 2019 #10, Cangur B1 2019 #10

3.2 F
Los enteros positivos a, b y c tienen cada uno tres cifras, y en cada entero la primera
cifra es la misma que la última. También cumplen que b  2a  1 y c  2b  1 . ¿Cuántos
valores distintos hay para el entero a?

A) 0 B) 1 C) 2 D) 3 E) más de 3

Canguro N5 2019 #26, Cangur B1 2019 #26

3.3 F
Tenemos tres cartulinas y en cada una se ha escrito un número de cuatro cifras. Como se
ve en la figura tres de las cifras están tapadas.

La suma de los tres enteros de cuatro cifras es 11126. ¿Cuáles son las cifras ocultas?

A) 1, 4 y 7 B) 1, 5 y 7 C) 3, 3 y 3 D) 4, 5 y 6 E) 4, 5 y 7

Canguro N6 2019 #6, Cangur B2 2019 #6

3.4 F
¿Cuál es la primera cifra (la situada más a la izquierda) del número entero positivo más
pequeño cuyas cifras suman 2019?

A) 2 B) 3 C) 4 D) 5 E) 6

Canguro N6 2019 #7, Cangur B2 2019 #7


3.5 F
Un gráfico consta de 16 vértices y algunos segmentos que los conectan, como en la
imagen.

Ahora hay una hormiga en el vértice A. En cada movimiento, puede caminar desde un
vértice a cualquier vértice vecino a lo largo de un segmento de conexión.
¿En cuál de los vértices P, Q, R, S, T puede estar la hormiga después de 2019
movimientos?

A) sólo P, R o S, no Q y T B) sólo P, R, S o T, no Q C) sólo Q D) sólo T


E) en cualquiera

Canguro N5 2019 #25, Cangur B1 2019 #25

3.6 M
Dado un entero positivo n , sea f (n) la suma de los dígitos de la representación de n
en base cuatro, y sea g (n) la suma de los dígitos de la representación de f (n) en base
ocho.
Por ejemplo: f (2020 )  f 133210 4   10  128 , y g (2020 )  1  2  3 .
Determina el valor mínimo de n de forma que la representación en base 16 de g (n) no
pueda ser representada usando solo los dígitos 0 a 9.

AIME II 2020 #5

3.7 MF
Los 25 enteros entre -10 y 14, inclusive, se pueden organizar para formar un cuadrado
de 5 por 5 en el que la suma de los números de cada fila, de cada columna y de las dos
diagonales sumen lo mismo. ¿Cuál es el valor de esta suma común?

(A) 2 (B) 5 (C) 10 (D) 25 (E) 50

AMC 12A 2020 #5

3.8 M
Determina el número de pares ordenados ( x, y ) de enteros que satisfacen la ecuación
x 2020  y 2  2 y

(A) 1 (B) 2 (C) 3 (D) 4 (E) hay infinitos pares


AMC 12B 2020 #8
3.9 M
Sean a y b números reales positivos cumpliendo la condición

log a  log b  log a  log b  100

y en donde los cuatro términos de la izquierda son enteros positivos, denotando por log
el logaritmo en base 10. Determina ab .

(A) 10 52 (B) 10 100 (C) 10 144 (D) 10 164 (E) 10 200

AMC 12A 2019 #15

3.10 F
Determina la cantidad de números enteros no negativos que se pueden escribir de la
forma
a7  37  a6  36  a5  35  a4  34  a3  33  a2  32  a1  31  a0  30

donde ai    1 , 0 , 1  para todo 0  i  7 .

(A) 512 (B) 729 (C) 1094 (D) 3281 (E) 59048

AMC 12A 2018 #13

3.11 M
Sea el triángulo ABC de lados AB  9 , BC  5 3 y AC  12 . Marcamos los puntos
A  P0 , P1 , P2 ,..., P2450  B en el segmento AB de forma que Pk se encuentra entre Pk 1
y Pk 1 para todo k  1,2,..., 2449 , y marcamos los puntos A  Q0 , Q1 , Q2 ,..., Q2450  C en
el segmento AC de forma que Qk se encuentra entre Qk 1 y Qk 1 para todo
k  1,2,..., 2449 . Además, todo segmento Pk Qk , con k  1,2,..., 2449 , es paralelo a BC .
Estos segmentos cortan el triángulo en 2450 regiones, consistiendo en 2449 trapecios y
un triángulo. Todas estas 2450 regiones tienen el mismo área. Determina el número de
segmentos Pk Qk , con k  1,2,..., 2450 cuya longitud es racional.

AIME II 2018 #7

3.12 F
Se toma aleatoriamente un número m del conjunto 11,13,15,17 ,19 , y se toma
aleatoriamente un número n del conjunto 1999, 2000, 2001, ..., 2018. Determina la
probabilidad de la cifra de las unidades de m n sea 1.

1 1 3 7 2
(A) (B) (C) (D) (E)
5 4 10 20 5
AMC 10A 2018 #19

3.13 D
Utilizando la igualdad 73 2  34 2  5  1297 , escribe 1297 como suma de dos cuadrados.
3.14 M
Determina la expresión equivalente a

2  322  32 24  34 28  38 216  316 232  332 264  364 


(A) 3127  2127 (B) 3127  2127  2  363  3  263 (C) 3128  2128 (D) 3128  3128 (E) 5127

AMC 10A 2021 #10

3.15 MF
La suma de los cinco números de tres cifras ABC, BCD, CDE, DEA y EAB es 2664.
¿Cuál es el valor de la suma de las cifras A, B, C, D y E?

A) 4 B) 14 C) 24 D) 34 E) 44

Canguro N6 2020 #5, Cangur B2 2020 #5

3.16 MF
Sean a, b y c enteros que satisfacen 1  a  b  c y a  b  c  1000000 . ¿Cuál es el mayor
valor posible de b?

A) 100 B) 250 C) 500 D) 1000 E) 2000

Canguro N6 2020 #7, Cangur B2 2020 #7

3.17 F
Sean a, b y c tres números enteros. ¿Cuál de los siguientes valores nunca puede ser igual
a a  b   (b  c) 2  (c  a) 2 ?
2

A) 0 B) 1 C) 2 D) 6 E) 8

Canguro N6 2020 #14, Cangur B2 2020 #14

3.18 F
El número entero 29...... tiene 100 cifras. ¿Cuántas cifras tiene su cuadrado?

A) 101 B) 199 C) 200 D) 201 E) no se puede saber

Canguro N6 2020 #15, Cangur B2 2020 #15

3.19 F
La sucesión f n viene dada por f1  1, f 2  3 y f n  2  f n  f n 1 para n  1 . ¿Cuántos de
los primeros 2020 términos de la sucesión son pares?

A) 673 B) 674 C) 1010 D) 1011 E) 1347

Canguro N6 2020 #18, Cangur B2 2020 #18


3.20 MF
En los cálculos mostrados, cada letra representa una cifra y se usan para formar algunos
números de dos cifras.

Los dos números de la izquierda suman 79. ¿Cuál es la suma de los cuatro números de
la derecha?

A) 79 B) 158 C) 869 D) 1418 E) 7979

Canguro N5 2020 #5, Cangur B1 2020 #5

3.21 MF
La suma de cuatro enteros consecutivos es 2. ¿Cuál es el menor de estos enteros?

A) -3 B) -2 C) -1 D) 0 E) 1

Canguro N5 2020 #6, Cangur N5 2020 #6

3.22 MF
Los años 2020 y 1717 se forman con un número de dos cifras repetido dos veces.
¿Cuántos años transcurrirán a partir de 2020 para encontrarnos en el siguiente año que
tenga esta misma propiedad?

A) 20 B) 101 C) 120 D) 121 E) 202

Canguro N5 2020 #7, Cangur B1 2020 #1, Kangaroo Junior 2020 #7

3.23 MF
El camino más corto para ir de A a C pasa por B. Paseando por este camino de A a C,
primero encontramos en el lado izquierdo del camino el poste de señales que se muestra
a la izquierda de la figura, y después, en el lado derecho del camino el otro poste.

¿Qué distancia estaba escrita en el cartel roto?

A) 1 km B) 2 km C) 3 km D) 4 km E) 5 km

Canguro N5 2020 #13, Cangur N5 2020 #13


3.24 F
En cada una de las nueve celdas de la figura se escribe un número de modo que la suma
de los tres números en cada diámetro es 13 y la suma de los ocho números en la
circunferencia es 40.

¿Qué número debe escribirse en la celda central?

A) 3 B) 5 C) 8 D) 10 E) 12

Canguro N5 2020 #15, Cangur B1 2020 #15

3.25 MF
Lucas comienza un viaje de 520 km en coche con 14 litros de combustible en el
depósito. Su automóvil consume 1 litro de combustible por cada 10 km. Después de
conducir 55 km, lee una señal de tráfico que muestra las distancias desde ese punto
hasta cinco estaciones de servicio en la carretera. Estas distancias son 35 km, 45 km, 55
km, 75 km y 95 km. La capacidad del depósito de combustible del coche es de 40 litros
y Lucas quiere detenerse solo una vez para llenarlo. ¿A qué distancia está la estación de
servicio donde debe repostar?

A) 35 km B) 45 km C) 55 km D) 75 km E) 95 km

Canguro N5 2020 #18, Cangur B1 2020 #18

3.26 F
Carmen etiquetó los vértices de una pirámide de base cuadrada con los números 1, 2, 3,
4 y 5, uno para cada vértice. Para cada cara calculó la suma de los números en sus
vértices. Cuatro de estas sumas son 7, 8, 9 y 10. ¿Cuál es la quinta suma?

A) 11 B) 12 C) 13 D) 14 E) 15
Canguro N5 2020 #25, Cangur B1 2020 #25
3.27 F
En cada una de las celdas de una tabla se escribe un número, de manera que las sumas
de los 4 números en cada fila y en cada columna sean iguales. ¿Qué número tiene la
celda sombreada?

A) 5 B) 6 C) 7 D) 8 E) 9

Canguro N5 2020 #27, Cangur B1 2020 #27

3.28 F
1 1 1 3
Determina las soluciones enteras de la ecuación    con 1  a  b  c
a b c 4
Olimpiada Matemática de Chile 2011

3.29 MD
Sea n  100 un entero. Iván escribe cada uno de los números n , n+1 , . . . , 2n en un
naipe diferente. Después de barajar estos n+1 naipes, los divide en dos pilas distintas.
Probar que al menos una de esas pilas contiene dos naipes tales que la suma de sus
números es un cuadrado perfecto.

IMO 2021 #1

3.30 F
Determina la cifra de las unidades de 13 2003 .

(A) 1 (B) 3 (C) 7 (D) 8 (E) 9

AMC 10A 2003 #16

Nota: En el apartado 7.6 aparecen más problemas del tipo “Determina la cifra de las unidades de...”

3.31 MF
¿Cuál es la última cifra del número 1234554321 + 1

(A) 1 (B) 5 (C) 6 (D) 2 (E) 0

Canguro N4 2002 #1

3.32 MF
Sean p y q dos números primos tales que p 2  q 2  365 . ¿Cuánto vale p  q ?

(A) 20 (B) 21 (C) 22 (D) 23 (E) 24


Canguro N4 2013 #4
3.33 M
Determina la raíz cuadrada del número de 4044 dígitos siguiente:
444
...4 888
...
8 9
2022 dígitos 2021 dígitos

3.34 D
Sean a, b, c números naturales tales que
      
ab c  ab2  c 2 b 2c  a 3  b 2 c a 2 c  b  a a 2b  c 3 
Demostrar que al menos uno de los números a, b, c es un cuadrado perfecto.

OMEFL Castilla y León 2021 #1

3.35 MF
¿De cuantas formas diferentes se pueden combinar billetes de 5$ y de 2$ para obtener
17$, sin importar el orden?

(A) 2 (B) 3 (C) 4 (D) 5 (E) 6

AMC 8 2002 #2

3.36 F
Julia divide el número 111 1 entre 3. El número de ceros que obtendrá será de
...
2004cifras

(A) 670 (B) 669 (C) 668 (D) 667 (E) 665

Cangur N1 2004 #27

3.37 M
Determina el entero positivo de tres dígitos a b c cuya representación en base nueve es
b c a (9 , donde a, b y c son dígitos no necesariamente distintos.

AIME I 2022 #1

3.38 M
Sean a , b , c , d , e , f , g , h , i enteros diferentes entre 1 y 9. El menor valor positivo
posible de
a b c  d e f
g  h i

m
se puede escribir como , donde m y n son enteros positivos y coprimos. Determina
n
m  n.

AIME I 2022 #7
3.39 F
Los números del 1 al 10 se escriben en los círculos de la figura, uno en cada círculo. La
suma de los números de la fila superior es 24, la suma de los números de la fila inferior
también es 24, y la suma de los números de la columna de la izquierda es 25. ¿Cuál es el
número que puede figurar en el círculo con la interrogación?

(A) 6 (B) 5 (C) 4 (D) 2 (E) Ninguno de los valores anteriores

Cangur B2 2022 #23

3.40 F Problema solucionado paso a paso en vídeo.

Determina la cantidad de parejas ordenadas b, c de enteros positivos para los que ni
x 2  bx  c  0 ni x 2  cx  b  0 tienen dos soluciones reales distintas.

(A) 4 (B) 6 (C) 8 (D) 12 (E) 16

AMC 12A Fall 2021 #17, AMC 10A 2021 #20

Solución: [Link]

3.41 F
Sea N el entero positivo más pequeño cuya suma de sus dígitos sea 2021. ¿Cuál es la
suma de los dígitos de N + 2021?

(A) 2021 (B) 2026 (C) 10 (D) 4042 (E) 12

Cangur B1 2021 #23, Kangaroo Junior 2021 #23

3.42 MF
¿Cuántos números de 3 dígitos se pueden formar utilizando los dígitos 1,3 y 5 y que
sean divisibles por 3? Puedes utilizar un dígito más de una vez.

(A) 6 (B) 27 (C) 18 (D) 7 (E) 9

Cangur B2 2021 #8, Kangaroo Student 2021 #8


3.43 MF
David escribe, en orden creciente, todos los números enteros del 2 al 2022 que usan solo
0 y 2. ¿Cuál es el número que se encuentra en el medio de su lista?

(A) 200 (B) 220 (C) 222 (D) 2000 (E) 2002

Cangur B2 2022 #6, Kangaroo Student 2022 #8

3.44 M
Un gimnasio tiene doce pesas diferentes de 1 kg a 12 kg, todas de números enteros. Las
divide en tres grupos de cuatro pesas cada uno. El peso total del primer grupo es de 41
kg y del segundo de 26 kg. ¿Cuál de las siguientes pesas está en el mismo grupo que la
pesa de 9 kg?

(A) 3 kg (B) 7 kg (C) 5 kg (D) 10 kg (E) 8 kg

Cangur B1 2022 #26, Kangaroo Junior 2022 #25

3.45 F
Para un entero positivo N, denotamos por p(N) el producto de los dígitos de N cuando
se escriben en forma decimal. Por ejemplo, p(23) = 2 × 3 = 6. ¿Cuál es el valor de la
suma p(10) + p(11) + p(12) + ... + p(99) + p(100) ?

(A) 5050 (B) 4050 (C) 5005 (D) 2025 (E) 4500

Cangur B2 2021 #16

3.46 F
Los números 1, 2, 7, 9, 10, 15 y 19 están escritos en una pizarra. Dos jugadores se
alternan para eliminan un número hasta que solo quede un número en la pizarra. La
suma de los números eliminados por uno de los jugadores es el doble de la suma de los
números eliminados por el otro jugador. ¿Cuál es el número que queda?

(A) 7 (B) 10 (C) 9 (D) 19 (E) 15

Cangur B2 2021 #22, Kangaroo Student 2021 #22


3.47 F
Los números del 1 al 6 se colocan en los círculos en las intersecciones de tres anillos
circulares. Se muestra la posición del número 6. Las sumas de los números de cada
anillo son las mismas.

¿Qué número se coloca en el círculo con el signo de interrogación?

(A) 5 (B) 1 (C) 3 (D) 2 (E) 4

Cangur B1 2021 #13, Kangaroo Junior 2021 #13

3.48 F
En una cuadrícula 5×5 como se muestra, la suma de los números en cada fila y en cada
columna es la misma. Hay un número en cada celda, pero algunos de los números no se
muestran. ¿Cuál es el número en la celda marcada con un signo de interrogación?

(A) 23 (B) 8 (C) 10 (D) 18 (E) 12

Cangur B2 2021 #17, Kangaroo Junior 2021 #17

3.49 F
¿Cuántos enteros positivos de tres cifras abc hay, tales que (a+b)c es un entero de tres
cifras que además es una potencia de 2?

(A) 15 (B) 16 (C) 18 (D) 20 (E) 21


Canguro N6 2017 #29, Cangur B2 2017 #29, Kangaroo Student 2017 #29
3.50 F
Ocho equipos participan en un torneo de fútbol. Cada equipo juega contra otro equipo
exactamente una vez. En cada partido, el ganador obtiene 3 puntos y el perdedor no
obtiene ningún punto. Si se empata un partido, cada equipo obtiene 1 punto. Al final del
torneo el número total de puntos obtenidos por todos los equipos es de 61. ¿Cuál es el
mayor número de puntos que podría haber obtenido el equipo campeón?

(A) 21 (B) 19 (C) 18 (D) 17 (E) 16

Cangur B1 2022 #22, Kangaroo Junior 2022 #29

3.51 MF
La suma de las dos últimas cifras del resultado del producto 1 · 2 · 3 · 4 · 5 · 4 · 3 · 2 · 1
es:

(A) 8 (B) 2 (C) 16 (D) 4 (E) 6

Cangur B2 2020 #1, Canguro N6 2020 #1

3.52 F
Determina todos los enteros positivos n para los cuales n 2  5n  6 es un cuadrado
perfecto.

OMEC 2023 #1

3.53 MF
Julia tira 5 dados y obtiene un total de 19 puntos. ¿Cuál es el número máximo de seises
que puede haber obtenido?

(A) 0 (B) 1 (C) 2 (D) 3 (E) 4

Cangur B2 2023 #2, Canguro N6 2023 #2

3.54 MF
Determina el número de enteros positivos x, y que satisfacen la ecuación x  2 y  210 .

(A) 29−1 (B) 29 (C) 29+1 (D) 29+2 (E) 0


Cangur B2 2023 #7, Canguro N6 2023 #6
3.55 MF
Encima de un reloj se pone un círculo gris con dos agujeros, tal y como se puede ver en
la figura. Hacemos girar este círculo alrededor de su centro de forma que el número 10
aparece en uno de sus agujeros. ¿Qué otros dos números pueden aparecer en el otro
agujero?

(A) El 2 o el 6 (B) El 3 o el 7 (C) El 3 o el 6 (D) El 1 o el 9 (E) El 2 o el 7

Cangur B1 2023 #1, Canguro N5 2023 #1

3.56 MF
Si m y n son dos números enteros positivos impares, ¿Cual de los siguientes números es
también impar?

(A) m·(n+1) (B) (m+1)·(n+1) (C) m+n+2 (D) m·n+2 (E) m+n

Cangur B1 2023 #3, Canguro N5 2023 #3

3.57 MF
Las edades de una familia formada por cinco miembros suman 80 años. Las dos
personas más jóvenes tienen 6 y 8 años. ¿Cuál era la suma de las edades de los
miembros de esta familia hace 7 años?

(A) 35 (B) 36 (C) 45 (D) 46 (E) 66


Cangur B1 2023 #7, Canguro N5 2023 #7

3.58 F
Después de haber jugado 200 partidas de ajedrez, he ganado exactamente un 49%.
¿Cuál es el mínimo número de partidas adicionales que tendré que jugar para que mi
porcentaje de partidas ganadas pueda aumentar hasta un 50%?

(A) 1 (B) 2 (C) 3 (D) 4 (E) 6

Cangur B1 2023 #10, Canguro N5 2023 #10


3.59 MF
Una cerca de madera está construida con tablones verticales y horizontales. Cada dos
tablones verticales consecutivos están unidos por 4 tablones horizontales.

¿Cuál de las siguientes cantidades puede corresponder al número de tablones de la


cerca?

(A) 95 (B) 96 (C) 97 (D) 98 (E) 99

Cangur B1 2023 #8, Canguro N5 2023 #8

3.60 M
Determina todos los números primos que se pueden expresar como suma y diferencia de
dos números primos.

New Zeland M.O. Camp 2013

3.61 F
Un conjunto de enteros positivos tiene las siguientes propiedades:
- La suma de sus elementos es 30.
- La única moda de la lista es 9.
- La mediana de la lista es un entero positivo que no aparece en dicho conjunto.

Determina la suma de los cuadrados de todos los elementos de este conjunto.

AIME II 2024 #2

3.62 F Problema solucionado paso a paso en vídeo

Un número de cuatro dígitos a b c d satisface la ecuación:


a b c d  a a  bb  c c  d d

¿Cuál es el valor de a?

(A) 2 (B) 3 (C) 4 (D) 5 (E) 6

Canguro 2024 N6 #30, Cangur 2024 B2 #30

Solución: [Link]
3.63 F
Tiré seis dados a la vez. No saqué un número diferente en cada dado. Pero tampoco
saqué seis del mismo número. Cuando multipliqué los números que salieron, obtuve un
número mayor que 15.000, que no era divisible por cuatro. ¿Cuál fue la suma de los
números que salieron al tirar los dados?

(A) 21 (B) 30 (C) 31 (D) 33 (E) 35

Canguro N5 2024 #29

3.64 F
Tengo un número de cuatro cifras distinto de cero N = pqrs. Cuando coloco una coma
decimal entre la q y la r, encuentro que el número resultante pq,rs es la media de los
números de dos cifras pq y rs. ¿Cuál es la suma de los dígitos de N?

(A) 14 (B) 18 (C) 21 (D) 25 (E) 27

Canguro N6 2024 #19, Cangur B2 2024 #19

3.65 F
Se escribe un número en cada uno de los doce círculos que se muestran. El número
dentro de cada cuadrado indica el producto de los números de sus cuatro vértices. ¿Cuál
es el producto de los números de los ocho círculos grises?

(A) 20 (B) 40 (C) 80 (D) 120 (E) 480

Canguro N5 2024 #16, Cangur B1 2024 #16, Cangur E4 2024 #24

3.66 MF
Cristina tiene un juego de cartas numeradas del 1 al 12. Coloca ocho de ellas en los
vértices de un octógono de tal manera que la suma de cada par de números que
comparten un lado sea múltiplo de 3. ¿Qué números no colocó Cristina?

(A) 1, 5, 9, 12 (B) 3, 5, 7, 9 (C) 1, 2, 11, 12 (D) 5, 6, 7, 8 (E) 3, 6, 9, 12

Canguro N5 2024 #19, Cangur B1 2024 #19

3.67 MF
Teodora escribió tres números enteros consecutivos en orden, pero en lugar de dígitos
usó símbolos, así que escribió □♢♢, ♡△△, ♡△□. ¿Qué escribiría después?

(A) ♡ ♡ ♢ (B) □ ♡ □ (C) ♡ △ ♢ (D) ♡ ♢ □ (E) ♡ △ ♡

Canguro N3 2023 #11


3.68 F
Bart escribió el número 1015 como una suma de números usando solo el dígito 7. Usó
el 7 un total de 10 veces, como se muestra en la figura. Ahora quiere escribir el número
2023 como una suma de números usando solo el dígito 7, usando el 7 un total de 19
veces. ¿Cuántas veces usará el número 77?

(A) 2 (B) 3 (C) 4 (D) 5 (E) 6

Canguro N3 2023 #17

3.69 MF
Cuatro animales, del más pesado al más ligero, son un pingüino, un perro, un gato y una
rana. Cada uno pesa un número entero de kilos y sus pesos son diferentes. El peso de los
cuatro juntos es de 18 kilos. Si el perro pesa 7 kilos. ¿Cuánto pesa el gato?

(A) 2 (B) 3 (C) 4 (D) 5 (E) 6

Canguro N4 2023 #5

3.70 MF
Hay un dado no convencional que en sus caras tiene los números 1, 2, 3, 4, 5 y 6, pero
las sumas de los lados opuestos son tres números primos diferentes. ¿Cuál de las
siguientes opciones es falsa?

(A) Uno de estos números primos puede ser 5


(B) El menor de estos números primos debe ser 3
(C) El mayor de estos números primos debe ser 11
(D) La suma de dos de estos números primos debe ser 10
(E) La suma de dos de estos números primos debe ser 18

Canguro N4 2023 #4
3.71 F
El circo tiene diez números de teléfono. Son números consecutivos a partir del que se
muestra. Desafortunadamente, en este momento un elefante está ocultando algunos de
los dígitos. ¿Cuántos de estos diez números de teléfono son múltiplos de 8?

(A) Ninguno (B) Uno (C) Dos (D) Tres (E) No es posible determinarlos

Canguro N4 2023 #10

3.72 F
Martín está parado en una fila. El número de personas en la fila es un múltiplo de 3. Se
da cuenta de que tiene tantas personas delante de él como detrás de él. Ve a dos amigos,
ambos de pie detrás de él en la fila, uno en el puesto 19 y el otro en el puesto 28. ¿En
qué posición de la fila está Martín?

(A) 14 (B) 15 (C) 16 (D) 17 (E) 18

Canguro N3 2023 #25

3.73 F
Determina el número de dígitos que tiene la representación en base 10 del número
85  510  15 5 .

(A) 14 (B) 15 (C) 16 (D) 17 (E) 18

AMC 12A 2023 #4, AMC 10A #5


4 Divisibilidad, máximo común divisor y mínimo común múltiplo.
4.1 Concepto de divisibilidad.
Definición. Divisor de un número.
Dados dos números enteros a, b , diremos que a divide a b, o que b es divisible entre a,
y escribiremos a | b , cuando exista un tercer número entero c tal que a c  b .
Puesto que a  0  0 , todo número entero es divisor de cero, incluso 0 | 0 .

Proposición. Propiedades básicas de la divisibilidad.


a) a | a para todo entero a (propiedad reflexiva)
b) a | b y b | c  a | c (propiedad transitiva)
c) a | b y a | c  a | x b  y c para cualquier par de enteros x, y
d) a | b y c | d  ac | bd . En particular, a | b  a | b c .
e) a | b y a | b  c  a | c

Observación.
a | a n , y por tanto a n | c  a | c , por la propiedad transitiva: a | a n | c  a | c

Criterios básicos de divisibilidad.


Entre 2: Cuando acaba en cero o cifra par.
Entre 3: Cuando la suma de sus cifras es múltiplo de 3.
Entre 4: Cuando el número formado con sus dos últimas cifras es un múltiplo de 4.
Entre 5: Cuando acaba en 0 o en 5.
Entre 9: Cuando la suma de sus cifras es múltiplo de 9.
Entre 11: Cuando la diferencia entre la suma de sus cifras pares y la suma de sus cifras
impares sea 0 o múltiplo de 11.

Teorema. Algoritmo de la división.


Para todo a  Z y b  N , existen q  Z y 0  r  b únicos, llamados respectivamente
cociente y residuo de la división, tales que a  q  b  r

Problema resuelto.
Aplicando el Algoritmo de la división, demuestra que todo cuadrado perfecto es
siempre de la forma 4k o 4k  1.

Solución. Aplicando el Algoritmo de la división, todo número n será de la forma


n  4b , n  4b  1, n  4b  2 , n  4b  3  4b  4  1  4(b  1)  1 .
Veamos su cuadrado, caso por caso:
n  4b  n 2  4b   44b 2 
2

n  4b  1  n 2  4b  1  16 b 2  8b  1  4(4b 2  2b)  1


2

n  4b  2  n 2  4b  2   16 b 2  16 b  4  4(4b 2  4b  1)
2

Sea cual sea el caso, siempre es de la forma 4k o 4k  1.

Nota: Este resultado es muy útil para resolver muchos problemas de ecuaciones diofánticas.
Nota: Ver 7.3.9 para más resultados como este, utilizando el lenguaje de la aritmética modular.
4.1.1 MF
El número de dígitos de 416525 (cuando está escrito en la base 10 usual) es

(A) 31 (B) 30 (C) 29 (D) 28 (E) 27

AHSME 1984 #9

4.1.2 F
Demuestra que el cuadrado de un número impar es siempre de la forma 8k  1

OPOS BALEARES 2018

4.1.3 F
El perímetro de un triángulo equilátero excede el perímetro de un cuadrado en 1989 cm.
La longitud de cada lado del triángulo excede la longitud de cada lado del cuadrado en d
cm. El cuadrado tiene perímetro mayor que 0. ¿Cuántos posibles enteros positivos no
son válidos para d?

(A) 0 (B) 9 (C) 221 (D) 663 (E) infinitos

ASHME 1989 #17

4.1.4 F
¿Cuántos números en base 10, N  a b c d satisfacen todas las tres condiciones
siguientes?
(i) 4000  N  6000
(ii) N es múltiplo de 5
(iii) 3  b  c  6

(A) 10 (B) 18 (C) 24 (D) 36 (E) 48

AHSME 1995 #12

4.1.5 F
La profesora Walter corrige un examen de matemáticas de sus cinco alumnos. Entra en
orden aleatorio las puntuaciones en una hoja de cálculo, que va recalculando la media
de la clase después de cada puntuación (sobre el número de alumnos ya introducidos, no
sobre el total de 5). La profesora se da cuenta de que, después de cada puntuación, la
media es siempre un entero. Las puntuaciones (presentadas en orden ascendente) son
71, 76, 80, 82 y 91. ¿Cuál fue la última puntuación que introdujo la profesora Walter?

(A) 71 (B) 76 (C) 80 (D) 82 (E) 91

AMC12 2000 #9
F
4.1.6
Determina los valores enteros de x para los cuales la expresión x 2  6 x es un cuadrado
perfecto, es decir, el cuadrado de un entero.
4.1.7 M
Determina la suma de todos los números enteros positivos b  1000 tal que el número
36 b (escrito en base b) es un cuadrado perfecto y el número 27 b (también escrito en
base b) es un cubo perfecto.

AIME II 2018 #3

4.1.8 F
Determina la suma de todos los enteros positivos n tales que n 2  85 n  2017 es un
entero.

AIME II 2017 #6

4.1.9 F
Demuestra que ningún número de la forma 11 ,111 ,1111 ,11111 , ... es un cuadrado
perfecto.

Indicación: Todo número de la forma 111 ...111 se puede escribir como


111 ...111  111 ...108  3  4k  3 .

4.1.10 MF
¿Para cuantos enteros n entre 1 y 100 el polinomio x 2  x  n factoriza en el producto
de dos factores lineales con coeficientes enteros?

(A) 0 (B) 1 (C) 2 (D) 9 (E) 10

ASHME 1989 #8

4.1.11 F
Sea S el número de pares ordenados de enteros ( a, b) con 1  a  100 y b  0 tales que
el polinomio x 2  ax  b pueda ser factorizado como producto de dos (no
necesariamente distintos) factores lineales con coeficientes enteros. Determina el
residuo cuando S se divide entre 1000.

AIME I 2018 #1

4.1.12 F
Una sucesión pucelana es una sucesión crecientes de dieciséis números impares
positivos consecutivos, cuya suma es un cubo perfecto. ¿Cuántas sucesiones pucelanas
tienen solamente números de tres cifras?

OME 2010 #1

4.1.13 M
a) Demuestra que el producto de dos números consecutivos es par.
b) Demuestra que el producto de tres números consecutivos es divisible entre 6.
c) Demuestra que n 5  n es divisible entre 30.
4.1.14 F
Determina la suma de todos los números primos entre 1 y 100 tal que sean
simultáneamente 1 mayor que un múltiplo de 4 y 1 menor que un múltiplo de 5

ASHME 1999 #4

4.1.15 MF
¿Cuántos enteros positivos b existen con la propiedad de que log b 729 sea un entero
positivo?

(A) 0 (B) 1 (C) 2 (D) 3 (E) 4

AMC12 2000 #7

4.1.16 F
Aplicando el Algoritmo de la división, demuestra que:
a) Un cuadrado perfecto es siempre de la forma 3k o 3k  1 .
b) Un número de la forma 3a 2  1 nunca puede ser un cuadrado perfecto.

4.1.17 F
N2  7
¿Para cuántos enteros N entre 1 y 1990 la fracción impropia no es irreducible?
N 4
ASHME 1990 #19

4.1.18 MF
Sea r el residuo cuando 1059, 1417 y 2312 se dividen entre d  1 . Determina el valor de
d r .
AHSME 1976 #15

4.1.19 F
Demuestra que existen infinitos números enteros n tales que n 2  23 es divisible por 24.

4.1.20 M
Determina todos los enteros positivos d tales que d divide n 2  1 y (n  1) 2  1 para
algún entero n.

4.1.21 F
Determina todos los enteros positivos n tales que el número que se obtiene eliminando
el último dígito es un divisor de n .

4.1.22 F
Calcula la suma de todos los enteros positivos n para los cuales la expresión
n7
n 1
es un entero.
HMMT 2021 #1
4.2 Divisibilidad y orden.

Proposición. Divisibilidad implica desigualdad.


a) Si a | b y b  0  a  b .

Y por tanto, si a y b son positivos:

b) a | 1  a  1
c) a | b  a  b
d) a | b y b | a  a  b
e) a | b  a  b

4.2.1 F
Sean 1  d1  d2  d3  ...  dk  n los divisores del entero positivo n.
Encuentra todos los números n tales que n  d 22  d 33 .
México 2008

4.2.2 M
Sea n  231 319 . Determina el número de enteros positivos de n 2 que sean menores que
n y que no sean divisores de n .

AIME 1995 #6

Indicación: Distribución de los divisores de n 2 .


Un dato que puede ser útil es que n es el divisor central de n 2 , es decir, el número de
divisores de n 2 menores que n es igual al número de divisores de n 2 mayores que n.

4.2.3 D
Encontrar las ternas a, b, n de números enteros positivos tales que
1 1 n
  .
a b ab

4.2.4 M
Dado n  IN , calcula la longitud del lado BC en el triángulo que se muestra en la
siguiente figura. ¿Existe algún valor de n para el que la longitud BC sea entero?
4.2.5 F
Sea n un número de cinco dígitos, y sean q y r el cociente y el residuo, respectivamente,
cuando n se divide entre 100. ¿Para cuántos valores de n se cumple que q  r es
divisible entre 11?

(A) 8180 (B) 8181 (C) 8182 (D) 9000 (E) 9090

AMC12A 2003 #18

4.2.6 D
Determina todos los enteros n  0 tales que existen enteros a, b con la propiedad
n 2  a  b y n3  a 2  b 2

Romanian Mathematical Olympiad 2004

4.2.7 MD
Dado un entero n  3 , demuestra que siempre es posible, eliminando a lo sumo dos
elementos del conjunto 1, 2 ,..., n  de forma que la suma de los restantes números sea
un cuadrado perfecto.

Romanian Mathematical Olympiad 2003

4.2.8 D
Determina todos los enteros positivos a, b, c tales que
ab  bc  ac  abc

4.2.9 F
Sea n un entero positivo. Cada uno de los números 1,2,3,...,2023 se pinta de algún
bonito color pero verificando la siguiente propiedad: si (a,b) es un par de enteros
diferentes tales que a es divisor de b, sus colores son diferentes. Encontrar el mínimo
número de colores que se precisa para que esa propiedad se cumpla.

OME Fase Local Extremadura 2023 #1


4.3 Máximo común divisor y mínimo común múltiplo.

Definición. Máximo común divisor. Números coprimos. Mínimo común múltiplo.


Dados dos números positivos a, b , el conjunto de divisores comunes de ambos no está
vacío, pues 1 es divisor de los dos, y está acotado superiormente, pues todo divisor
común es menor o igual que min( a, b) . Luego tiene sentido definir el máximo común
divisor de a y b como el mayor divisor positivo común a ambos, que denotaremos por
a, b .
Diremos que dos números son coprimos cuando sus únicos divisores comunes sean 1 y
1 , es decir, cuando a, b  1 .
Dados dos números positivos a, b , el conjunto de múltiplos comunes a ambos no está
vacío, pues a b es un múltiplo común, y está acotado inferiormente por max( a, b) .
Luego tiene sentido definir el mínimo común múltiplo de a y b como el menor de los
múltiplos comunes a ambos, y se denotará por a, b .

Proposición. Algunas propiedades básicas del mcd y del mcm.


a) a, b | a, b | a, b .
b) a, b  min( a, b)  a, b  max( a, b)  a, b.

Interpretación geométrica del máximo común divisor.


El máximo común divisor de a y b indica el número de puntos ( x, y ) con coordenadas
enteras en el segmento que une los puntos ( 0 , 0 ) y ( a , b ) , sin contar el inicial ( 0 , 0 ) .
Por ejemplo, (15 , 6 )  3 , y por tanto el segmento que une los puntos ( 0 , 0 ) y (15 , 6 )
pasa por tres puntos con coordenadas enteras, aparte del propio ( 0 , 0 ) :

15 
 5 ( 5 , 2 )
3  
  ( 5  5 , 2  2 )  (10 ,4 )
6
 2  (10  5 , 4  2 )  (15 ,6 )
3 
4.3.1 F
Diremos que un punto ( x, y ) del plano es “punto entero” cuando sus coordenadas sean
enteras. ¿Cuántos de estos puntos de este tipo hay (incluyendo ambos extremos) en el
segmento que cuyos extremos son ( 3 ,17 ) y ( 48 , 281 ) ?

ASHME 1989 #16

4.3.2 MF
Demostrar que (n, n  k ) | k , independientemente del valor de n . En particular,
(n, n  1)  1

4.3.3 M
Sean a, b, c enteros positivos tales que
a  b  c  23
y
Mcd (a, b)  Mcd (b, c)  Mcd (c, a)  9

Determina la suma de todos los valores distintos de a 2  b 2  c 2 .

(A) 259 (B) 438 (C) 516 (D) 625 (E) 687

AMC 12B Fall 2021 #16

4.3.4 F
Determina el máximo común divisor de todos los números de la forma
n3 (n  1)3 (n  2)3 (n  3)3 (n  4)3

donde n es un entero positivo.

(A) 2 933 (B) 2 6 3353 (C) 2 9 36 53 (D) 2832 53 (E) 2 9 3353

Cangur B2 2023 #27, Canguro N6 2023 #27

4.3.5 F
Un grupo de 50 estudiantes se sientan en círculo. Lanzan una pelota alrededor del
círculo. Cada estudiante que recibe la pelota se la lanza al sexto estudiante sentado en el
sentido contrario a las agujas del reloj desde donde está sentado, quien la atrapa.
Fernanda atrapa la pelota 100 veces. En ese tiempo, ¿cuántos estudiantes nunca logran
atrapar la pelota?

(A) 0 (B) 8 (C) 10 (D) 25 (E) 40

Canguro N4 2024 #24, Cangur E3 2024 #26


4.4 El Teorema de Bezout (TDB).

Lema.
( a , b ) divide a cualquier combinación lineal de a y b .

Demostración. Sea ax  by una combinación lineal de a y b .


(a, b) | a  (a, b) | ax
  (a, b) | ax  by
(a, b) | b  (a, b) | by 

Teorema. Teorema de Bezout (TDB).


Dados dos números enteros a, b no ambos cero, el máximo común divisor de a, b se
caracteriza por ser el elemento mínimo del conjunto no vacío

A   ax  by , x, y  Z , ax  by  0 

Luego existe, es único y siempre se puede escribir como combinación lineal de a y b :


Existen enteros x, y tales que (a, b)  ax  by

Demostración. Consideremos el conjunto anterior A   ax  by , x, y  Z , ax  by  0 .


Es un conjunto no vacío pues al menos aa  bb  a 2  b 2  0 pertenece a A (estamos en
todo momento suponiendo que a, b no son ambos cero).

Por el Principio de buena ordenación, A tendrá un mínimo, al que llamaremos d .


Vamos a demostrar que d  (a, b) .

Supongamos que d  a x1  b y1  0 para ciertos enteros x1, y1 .


Por el Algoritmo de la división, existirán enteros q, r tales que a  d q  r con
0r d.

Si r  0 , entonces
0  r  a  dq  a  (a x1  b y1 )q  a  a x1q  b y1q  a(1  x1q)  b y1q

y por tanto, r pertenece al conjunto A, tomando x2  1  x1q , y2   y1q . Pero r  d , lo


cual contradice la hipótesis de d como elemento mínimo. Luego r  0 y por tanto
a  d q , es decir, d divide al número a.

Con el mismo razonamiento se demuestra que d divide al número b, y por tanto d es


común divisor de a y b. Veamos que es el máximo común divisor.
Sea m otro común divisor de a y b, entonces, aplicando el lema anterior,
m | a x1  b y1  d  m | d  m  d .
Corolario.
Si x1 ,..., xn son números enteros y a es cualquier número entero positivo,
a x1,..., a xn   a x1,..., xn 
Demostración.
Sean d  a x1 ,..., a xn  y e  x1 ,..., xn  . Luego e | xi  a e | a xi  a e | d  a e  d .
Por el TDB, e  k1x1  ...  kn xn  a e  k1 ax1   ...  kn axn 
Es decir, ae es combinación lineal de a x1,..., a xn , y por tanto es un múltiplo de d ,
luego ae  d . Y, puesto que anteriormente hemos demostrado que a e  d , llegamos a
ae  d .

Corolario.
( a, b)  1  Existe una combinación lineal ax  by  1

Demostración.  Es el TDB.
 (a, b) | ax  by  1  (a, b) | 1  (a, b)  1

4.4.1 M
21 n  4
Demuestra que la fracción es irreducible para todo número natural n.
14 n  3
IMO 1959 #1

4.4.2 M
Los números de la sucesión 101 ,104 ,109 ,116 , ... son de la forma an  100  n 2 ,
n  1, 2, ...
Para cada n , sea d n   an , an1  . Determina max n 1 d n .

AIME 1985

Teorema.
Si d   a1 , a2 , ..., an  entonces  a1 , a2 , ..., an

 1
d d d 

Demostración.
a a a 
d  a1 , a2 , ..., an   d | ai  ai  ki d . Sea d '   1 , 2 , ..., n  y supongamos que
d d d 
d ' 1 .
a a a  a a
d '   1 , 2 , ..., n   d '| i  i  k 'i d '  ai  k 'i d ' d  d ' d | ai  d ' d | d
d d d  d d
Lo cual es imposible suponiendo d ' 1 . Luego d ' 1 .
4.5 Divisibilidad con números coprimos.

Teorema.
Si (a, b)  1 , entonces:
a | c
a)   ab | c b) a | bc  a | c
b | c
d | ac d | ac
c) d |c d)   d | ce
d | bc  d | be 

Demostración.
a)
a | c  c  aa' 

b | c  c  bb' 
(a, b)  1  1  ax  by 
 c  c  1  c(ax  by)  cax  cby  bb' ax  aa' by  ab(b' x  a ' y )  ab | c
b)
a | bc  bc  ak1
d | bc  bc  dk2
(a, b)  1  1  k3a  k4b  c  k3ac  k4bc  k3ac  k4 ak1  ak3c  k4 k1   a | c

Por el TDB, (a, b)  1  1  ax  by para ciertos enteros x, y . Luego c  acx  bcy , y


puesto que trivialmente a | acx y por hipótesis a | bcy , se deduce a | acx  bcy  c .
c)
d | ac  ac  dk1
d | bc  bc  dk2
(a, b)  1  1  k3a  k4b  c  k3ac  k4bc  k3dk1  k4 dk2  d (k3k1  k4 k2 )  d | c

d)
d | ac  d | a(ce)
  d | ce aplicando el apartado anterior.
d | be  d | b(ec) 

Observación. El apartado a) del teorema anterior se puede generalizar:


Si a1 , a2 ,..., an son enteros tales que (ai , a j )  1 si i  j , entonces
a1 | b , a2 | b , ..., an | b  a1a2 ...an | b

4.5.1 D
Sean m, n dos enteros positivos diferentes. Demostrar que

(m, n)  (m  1, n  1)  (m  2, n  2)  m  n  1

Indian National Mathematical Olympiad 2019 #3 (parcial)


4.6 El algoritmo de Euclides (ADE).

Lema.
(a, ab  c)  (a, c)

Demostración.
Sean n  (a, ab  c) y m  ( a, c ) .
n  (a, ab  c)  n  xa  y (ab  c)  xa  aby  yc  a( x  by)  yc , luego (a, c) | n .
Y por otro lado,
m | a  m | ab
m  ( a, c )     m | ab  c  m | (a, ab  c)  n .
m | c 
Así pues, n | m y m | n , y por tanto n  m .

Teorema. Algoritmo de Euclides (ADE).


El algoritmo de Euclides es un método efectivo para calcular el máximo común divisor
de dos números a y b que se basa en el algoritmo de la división y en la proposición
anterior:
Si b  a k  c , entonces (a, b)  (a, a k  c)  (a, c)

Suponiendo a  b , podemos dividir a entre b para expresar a  kb  r , con r  b y


así
(a, b)  (bk  r , b)  (r , b)

Este mismo proceso repetiremos una y otra vez, con números más y más pequeños,
hasta que el máximo común divisor se haga evidente.

Ejemplo 1.
Calcula  29 , 23  mediante el Algoritmo de Euclides.

Solución.
29  1  23  6  (29,23)  (1  23  6 , 23 )  ( 6 , 23 )
23  3  6  5  (23,6)  ( 3  6  5 , 6 )  ( 5 , 6 )
6  1  5  1  (6,5)  (1  5  1, 5 )  (1, 5 )  1
Así pues,  29 , 23   1

Ejemplo 2.
Calcula  3456, 246  mediante el Algoritmo de Euclides.

Solución.
( 3456 , 246 )  (13  246  158 , 246 )  (158 , 246 )
( 158 , 246 )  ( 158 ,1  158  88 )  ( 158 , 88 )
Y de la misma manera:
( 158 , 88 )  ( 70 , 88 )  ( 18 , 70 )  ( 16 ,18 )  ( 2 ,16 )  2
Luego  3456, 246   2
Proposición.
c | a a | c
a)   c | a, b  b)   a, b | c
c | b b | c

Demostración.
a)  Por el TDB, a, b  ax  by  ca' x  cb' y  c(a' x  b' y)  c | a, b
 Por transitividad: c | (a, b), (a, b) | a  c | a
b)  Sea k  a, b. Supongamos que k  0 , (es decir, que a, b  0 )
Por reducción al absurdo, supongamos que k | c .
Entonces existirán q y 0  r  k tales que c  kq  r .
Luego r  c  kq múltiple común de a y b , pues lo son c y k , pero r  k ,
contradiciendo la hipótesis de k como menor múltiple común de a y b .
 Por transitividad: a, b | c, a | a, b  a | c .

Corolario.
a |c
  (a, b) | c, d 
b | d

Demostración.
( a, b) | a | c 
  ( a , b ) | ( c, d )
( a, b) | b | d 
4.7 Máximo común divisor con números coprimos.

Proposición.
(a, c)  1  (ab, c)  (b, c)

Demostración.
Por el TDB, (a, c)  1 implica que existen dos enteros x, y tales que 1  ax  cy .
Sea g  (b, c) y h  (ab, c) . Los valores g, h no son negativos.
g  (b, c) | b | ab
  g | (ab, c)  h
g  (b, c) | c 
h  (ab, c) | ab | abx
  h | abx  cby  b(ax  cy)  b  1  b
h  (ab, c) | c | cby 
h  (ab, c) | c 
  h | (b, c)  g
h|b 
Llegamos a h | g y g | h , y puesto que no son negativos, se deduce que g  h .

Corolario.
(a, c)  1
  (ab, c)  1
(b, c)  1

Demostración.
 Por la proposición anterior, (a, c)  1  (ab, c)  (b, c)  1 .

d | a  d | ab
  d | (ab, c)  1  d  1 , luego (a, c)  1 , y con un razonamiento similar
d |c 
se demuestra que (b, c )  1

Observación. Esta propiedad se puede generalizar fácilmente por inducción:

(a1, c)  (a2 , c)  ...  (an , c)  1  (a1a2 ...an , c)  1

Teorema. El Lema de Euclides.


a | bc 
a|c
(a, b)  1

Demostración.
a | bc
  a | (a, bc)
a|a 
Por otro lado, por 4.8, (a, b)  1  (a, bc)  (a, c) .
Luego a | (a, bc)  (a, c) | c , es decir, a | c .
Corolario.
Si c  0 , (ca, cb)  c(a, b)

Demostración.
c y ( a , b ) son no negativos, luego c(a, b) es no negativo. Luego, aplicando 4.6a
(a, b) | a  c(a, b) | ca
  c(a, b) | (ca, cb)
(a, b) | b  c(a, b) | cb 
Por otro lado, aplicando el TDB, existirán enteros x, y tales que (a, b)  ax  by .
(ca, cb) | ca | cax
  (ca, cb) | cax  cby  c(ax  by)  c(a, b)  (ca, cb) | c(a, b)
(ca, cb) | cb | cby

Así pues, c(a, b) | (ca, cb) y (ca, cb) | c(a, b) , y puesto que ambos son no negativos,
llegamos a (ca, cb)  c(a, b) .

Corolario.
Si a y b son enteros positivos, (a, b)  1  (c, a )(c, b)  (c, ab)

Demostración.
Floor and arithmetic functions (Darij Grinberg, 2016) pág. 12
4.8 Actividades con Python.
El Algoritmo de Euclides Extendido (ADE).

def egcd(a, b):


if a == 0:
return (b, 0, 1)
else:
g, y, x = egcd(b % a, a)
return (g, x - (b // a) * y, y)

print(egcd(117,244))

Salida: (1, 73, -35)


4.9 Problemas de introducción a la divisibilidad.

4.9.1
Eduardo colecciona 2004 piezas cónicas. Las coloca en montones de 5 cada uno.
¿Cuántos montones de 5 piezas tiene?

(A) 5 (B) 400 (C) 401 (D) 402 (E) 404

Cangur N1 2004 #5, Canguro N1 2004 #5

4.9.2
Tenemos 16 lápices, 12 gomas de borrar y 5 tijeras. Con este material queremos hacer
paquetes como el que indica la figura. ¿Qué objetos faltan para hacer paquetes
completos?

(A) 1 lápiz y 1 goma (B) 2 lápices y 1 goma (C) 2 lápices y 1 tijera (D) 2 lápices y 2
gomas (E) 2 gomas y 2 tijeras.

Cangur P5 2016 #7

4.9.3
La cabina de pasajeros de un avión tiene 108 asientos. Hay un asiento vacío por cada
dos asientos ocupados. ¿Cuántos pasajeros hay en el avión?

(A) 36 (B) 42 (C) 56 (D) 64 (E) 72

Cangur N1 2001 #5, Canguro N1 2001 #5

4.9.4
En una tienda se venden los globos en paquetes de 5 globos, o de 10 globos o de 25
globos. Mateo quiere comprar 70 globos. ¿Cuál es el menor número de paquetes que
comprará?

(A) 3 (B) 4 (C) 5 (D) 6 (E) 7

Cangur P6 2017 #7

4.9.5
Claudia tiene 10 hojas de papel y corta alguna en cinco partes cada una. Después de
hacerlo le quedan 22 piezas en total. ¿Cuántas hojas ha cortado?

(A) 3 (B) 2 (C) 6 (D) 7 (E) 8

Cangur P6 2020 #9
4.9.6
Miguel quiere preparar 24 madalenas para su fiesta de aniversario. Para cocinar seis
madalenas necesita dos huevos. Los huevos se venden en cajas de seis. ¿Cuántas cajas
tiene que comprar Miguel, como mínimo?

(A) 2 (B) 8 (C) 4 (D) 1 (E) 3

Cangur E1 2020 #4

4.9.7
María eligió un número entero y lo multiplicó por 3. ¿Cuál de los siguientes números no
pudo ser el resultado obtenido?

(A) 103 (B) 105 (C) 204 (D) 444 (E) 987

Canguro N1 2005 #9, Cangur N1 2005 #10

4.9.8
Éste es un trozo de una tabla de multiplicar

y éste es otro, donde, desafortunadamente, han desaparecido algunos números.

¿Cuál es el número en la casilla con la interrogación?

(A) 54 (B) 56 (C) 65 (D) 36 (E) 42

Cangur N1 2008 #9, Canguro N1 2008 #7

4.9.9
¿Cuál es el menor entero positivo divisible por 2, 3, y 4?

(A) 1 (B) 6 (C) 12 (D) 24 (E) 36

Cangur N1 2003 #4, Canguro N1 2003 #4

4.9.10
¿Cuántos números de 2 cifras son divisibles por 2 y por 7?

(A) 8 (B) 7 (C) 6 (D) 5 (E) 4


Canguro N1 2000 #12, Cangur N1 2000 #11
4.9.11
Determina la probabilidad de que, tomando aleatoriamente un múltiple de 864, sea
también divisible entre 1944.

HMMT 2002

4.9.12
Se forma un número de nueve cifras colocando aleatoriamente los dígitos del 1 al 9.
¿Cuál es la probabilidad de que el número resultante sea divisible por 18?

(A) 1/2 (B) 4/9 (C) 5/9 (D) 1/3 (E) 3/4

Canguro N5 2020 #21, Kangourou J 2020 #19

4.9.13 MF
¿Cuál es el máximo común divisor de 2 2021  2 2022 y 32021  32022 ?

(A) 2 2021 (B) 1 (C) 2 (D) 6 (E) 12


Cangur B2 2022 #13, Kangaroo Student 2022 #13

4.9.14 MF
¿Cuántos números enteros positivos de tres dígitos son divisibles por 13?

(A) 68 (B) 69 (C) 70 (D) 76 (E) 77


Cangur B2 2022 #2, Kangaroo Student 2022 #2

4.9.15 MF
Una vez conocí a seis hermanos cuyas edades eran seis números enteros consecutivos.
Le hice a cada uno de ellos la pregunta: “¿Cuántos años tiene tu hermano mayor?”.
¿Cuál de las siguientes podría no ser la suma de sus seis respuestas?

(A) 233 (B) 205 (C) 167 (D) 125 (E) 95

Cangur B1 2022 #13

4.9.16 MF
¿Cuántos polígonos regulares hay cuyos ángulos (en grados) son números enteros?

(A) 17 (B) 18 (C) 22 (D) 25 (E) 60

Cangur N4 2015 #25, Canguro N6 2015 #24, Kangaroo Student 2015 #24

4.9.17 MF
¿De cuántas formas diferentes podemos dar valores enteros positivos a la pareja de
a 7
números a y b de forma que la igualdad  sea cierta?
5 b

(A) 0 (B) 1 (C) 2 (D) 3 (E) 4


Cangur B1 2023 #9, Canguro N5 2023 #9
4.9.18 MF
Cada tercer escalón de una escalera de 2023 peldaños está pintado de negro. Los siete
primeros peldaños se muestan en la siguiente figura:

Anna sube la escalera de uno en uno comenzando por el pie derecho o por el pie
izquierdo, y alternándolos en cada paso. Determina el número de peldaños negros que
habrá pisado con el pie derecho.

(A) 333 (B) 336 (C) 337 (D) 674 (E) Depende del pie con el que comience

Cangur B1 2023 #17, Canguro N5 2023 #17

4.9.19 MF
Un número de dos cifras se llama sp (sin potencias) si ninguna de sus cifras se puede
escribir como una potencia de exponente mayor que 1 de un número entero. Por
ejemplo, 53 es sp y 54 no lo es porque 4 = 2² . ¿Cuál de los siguientes números es
divisor común del más grande y del más pequeño de los números sp de dos cifras?

(A) 3 (B) 5 (C) 7 (D) 11 (E) 13

Cangur B1 2023 #18, Canguro N5 2023 #18

4.9.20 MF
Un número palíndromo de tres cifras tiene la forma 'aba', donde las cifras a y b pueden
ser iguales o diferentes. ¿Cuál es la suma de los dígitos del mayor palíndromo de tres
cifras que también es múltiplo de 6?

(A) 16 (B) 18 (C) 20 (D) 21 (E) 24

Canguro N5 2024 #8, Cangur B1 2024 #8

4.9.21 F
El granjero Felipe vende huevos de gallina y de pato. Tiene cestas con 4, 6, 12, 13, 22 y
29 huevos. Cada cesta contiene una combinación de ambos tipos de huevos. Su primer
cliente compra todos los huevos de una de las cestas. Felipe se da cuenta de que la
cantidad de huevos de gallina que le quedan es el doble de la cantidad de huevos de
pato. ¿Cuántos huevos compró el cliente?

(A) 4 (B) 12 (C) 13 (D) 22 (E) 29

Canguro N4 2024 #26, Cangur E3 2024 #27


5 Números primos.
5.1 Concepto de número primo.
Definición. Número primo.
Todo número es divisible por sí mismo y por la unidad. Diremos que un número natural
p  1 es primo cuando sus únicos divisores positivos sean él mismo y la unidad. Por
ejemplo, son primos los números 5, 13, 59 o 397. Llamamos compuestos a los números
que no son primos. El número 1 no se considera ni primo ni compuesto.

Proposición. Algunas propiedades de los números primos.


a) Si p es primo y a | p , entonces a  1 ,  p .
b) Si p y q son primos, entonces p | q  p  q .
c) Todos los primos son impares excepto el 2.
d) El 2 y el 3 son los únicos primos cuya diferencia es 1.
e) Si p es primo y a, b  N , entonces p | a b  p | a .

5.1.1 Problema resuelto.


Demuestra que si p  3 es primo, entonces 24 | p 2  1.

Solución.
Por el algoritmo de la división, todo número se puede representar como 6n , 6n  1 o
6n  2 . Si es primo, la única opción aceptable es 6n  1 , pues las otras son divisibles
entre 2 o 3. Luego:
p  6n  1  p 2  6n  1  36 n 2  12 n  1  p 2  1  36 n 2  12 n  12 n(3n  1)
2

Está claro que o bien n es par o bien 3n  1 es par, luego 24 | p 2  1.

5.1.2 Problema resuelto.


Determina la suma de todos los números primos entre 1 y 100 que son simultáneamente
1 más que un múltiplo de 4 y 1 menos que un múltiplo de 5.

AHSME 1999 #3
Solución.
p  4a  1
  4a  1  5b  1  5b  4a  2  2a  1
p  5b  1 
 2 | 5b  2 | b  b  2c  p  5(2c)  1  10 c  1
Con p  10 c  1 ya tenemos un conjunto de candidatos suficientemente pequeño como
para proceder a testearlos, uno por uno:
c  1  p  10  1  1  9 (no es primo) c  2  p  10  2  1  19  4  4  3
c  3  p  10  3  1  29  4  7  1 c  4  p  10  4  1  39  4  9  3
c  5  p  10  5  1  49 c  6  p  10  6  1  59  4  14  3
c  7  p  10  7  1  69 c  8  p  10  8  1  79  4  19  3
c  9  p  10  9  1  89  4  22  1 c  10  p  10  10  1  99
Luego la suma es 29+89=118.
Corolario. Corolario al Lema de Euclides.
Si p es primo y p | ab entonces p | a o p | b .

Demostración.
Basta aplicar el Lema de Euclides teniendo en cuenta que ( p, a )  ( p, b)  1 .

5.1.3 M
Sean x, y enteros. Demostrar que 2 x  3 y es divisible entre 17 si y solo si 9 x  5 y es
divisible entre 17.

Corolario.
a) a es par si y solo si a n es par
b) a es impar si y solo si a n es impar.

Demostración.
 
a)  a  2k  a n  (2k )n  2 2n 1 k n
 Basta aplicar el Lema de Euclides con p  2 .
n
 n
b)  a  2k  1  a n  (2k  1) n  1    2k  impar.
k

k 1  k 

 Por el apartado a, si a es par entonces a n es par, luego si a n es impar,


necesariamente a debe ser par.

Corolario.
Hay infinitos números primos.

Demostración. Entre otras muchas demostraciones de este resultado, la de Euclides es


un ejemplo de elegancia:
Supongamos, por el contrario, que existe una cantidad finita de números primos. Sean
estos
1  p1  p2  ...  pn
Consideremos el número n  p1  p2  pn  1.
No puede ser primo, pues n  pn , luego existirá al menos un k tal que pk | n , pero
también se cumple trivialmente pk | n  1  p1  p2  pn , y por tanto
pk | n  (n  1)  1  pk | 1  pk  1
lo cual es imposible. Así pues, no es posible que exista un número finito de primos.

Observación.
La densidad de los números primos (es decir, la proporción de números primos respecto
al total de naturales) es un resultado que se conoce desde hace 100 años, y es un
teorema de la llamada teoría de números analítica:
 n 
lim 1
n  n / log n

Donde  n denota el número de primos menores o iguales a n. Este resultado fue
demostrado por Hadamard y de la Vallé Poussin en 1896.
5.1.4 F
Determina cuántos números primos hay en los primeros diez números de la secuencia
121, 11211, 1112111...

(A) 0 (B) 1 (C) 2 (D) 3 (E) 4

AMC 12B 2022 #3

5.1.5 MF
¿Cuál es el número entero más pequeño que podemos obtener como resultado de
calcular una media aritmética de cinco números primos diferentes?

(A) 6 (B) 5 (C) 12 (D) 2 (E) 30

Cangur B1 2023 #20, Canguro N5 2023 #20

5.1.6 MF
Andrés delimita un campo rectangular con 40 m de valla. Las longitudes de los lados
del campo son números primos. ¿Cuál es la superficie máxima posible del campo?

(A) 99 m² (B) 96 m² (C) 91 m² (D) 84 m² (E) 51 m²

Canguro N5 2024 #10, Cangur B1 2024 #10


5.2 El Teorema fundamental de la aritmética (TFA).

Teorema. Teorema fundamental de la aritmética (TFA).


a) Todo entero n  1 se puede representar como producto de números primos.
b) Ordenando los pi , obtenemos lo que denomina descomposición canónica del
número n :
n  p1a1 p2a 2 ... pka k con p1  p2  ...  pk y 0  ai

Por ejemplo: 18  2  32 , 84  22  3  7 , 4576  2  11  13 , 32716  2  8179

La descomposición canónica de un número es única.

Demostración. a) Sea n  1 . Si n es primo, ya hemos acabado. Supongamos que n es


compuesto y sea p1 su menor divisor. Está claro que p1 tiene que ser primo.
Luego n  p1n1 , para cierto 1  n1  n . De nuevo, si n1 es primo ya hemos acabado.
Supongamos que es compuesto y sea p2 su menor divisor. Está claro que p2 tiene que
ser primo, y n  p1 p2n2 para cierto 1  n2  n1  n .
Este proceso no puede continuar indefinidamente, pues antes de n pasos debemos
encontrar un px primo, y n  p1 p2 ... ps .

b) Supongamos que n  p1a1 p2a 2 ... psa s  q1b1 q2b2 ...qtat .


Por el Corolario al Lema de Euclides, todo pi será un q j y todo q j será un pi .
Esto implica que s  t . Puesto que, además, p1  p2  ...  ps y q1  q2  ...  qs ,
tenemos que pi  qi para todo 1  i  s :
n  p1a1 p2a 2 ... psa s  p1b1 p2b2 ... psa s
Veamos que también los exponentes coinciden.
Si ai  bi para cierto 1  i  s , entonces, dividiendo entre pibi ambos lados de la
igualdad, tenemos
p1a1 p2a 2 ... piai  bi psa s  p1b1 p2b2 ... pibi11 pibi11 ... psa s

lo cual es absurdo, pues pi divide la parte izquierda pero no la parte derecha de la


igualdad.

Con razonamiento similar demostramos que tampoco se puede dar ai  bi , y por lo tanto
llegamos a la conclusión de que ai  bi para todo 1  i  s , es decir, las dos
descomposiciones canónicas coinciden.
Proposición.
Dado un número en descomposición canónica n  p1a1 p2a2 ... pkak ,
a) Todo divisor de n es de la forma d  p1e1 p2e2 ... pkek , con 0  ei  ai .
b) El número de divisores de n es a1  1a2  1...ak  1 .

Demostración.
a) Todo número de la forma d  p1e1 p2e2 ... pkek , con 0  ei  ai es un divisor de n, y son
todos diferentes por el TFA.
b) Basta aplicar el principio fundamental del recuento.

Teorema. Descomposición canónica del mcd y del mcm.


Se puede demostrar fácilmente que si a  p1a1 p2a 2 ... pna n y b  p1b1 p2b2 ... pnbn
(en donde algunos de los ai y bi pueden ser 0)

Entonces:
a) (a, b)  p1min( a1 ,b1 ) p2min( a2 ,b2 ) ... pnmin( an ,bn )
b) a, b  p1max( a1 ,b1 ) p2max( a2 ,b2 ) ... pnmax( an ,bn )
c) ab  (a, b)  a, b

Demostración.
c) Basta aplicar el anterior teorema teniendo en cuenta que
x  y  min( x, y )  max( x, y ) .

Proposición.
a) Si p | mcm (a, b) y p | a  b entonces p | mcd (a, b) .
b) Si p | mcda, b entonces p | mcm (a, b) y p | a  b .
c) mcda, b  mcda  b, mcm(a, b)

Demostración.
a) Puesto que mcm (a, b)  p1max( a1 ,b1 ) p2max( a2 ,b2 ) ... pnmax( an ,bn ) , si p | mcm (a, b)
entonces p | a o p | b .
Si p | a , puesto que p | a  b , se deduce que p | b , y por tanto p | mcd (a, b) .
Si p | b , con un razonamiento similar llegamos igualmente a p | mcd (a, b) .
b) Ya fue demostrado anteriormente que p | mcd (a, b)  p | a  b .
Puesto que max( x, y )  min( x, y ) , está claro que p | mcd (a, b)  p | mcm (a, b)

Proposición.
Sean a, b números positivos coprimos. Entonces:
a) Los divisores positivos de ab son de la forma dd ' , con d | a , d ' | b , d , d '  0 .
b) Si ab  c n , con c  0 , entonces a  u n y b  v n para ciertos enteros u, v .

Demostración.
a) Puesto que a y b son coprimos, sus respectivas descomposiciones factoriales
canónicas serán de la forma a  p1a1 p2a2 ... prar , b  q1b1 q2b2 ...qsbs , con pi  q j .
Luego la descomposición factorial del producto será
ab  p1a1 p2a2 ... prar q1b1 q2b2 ...qsbs
Por lo tanto, los divisores de ab serán de la forma
  
p1a '1 p2a '2 ... pra 'r q1b '1 q2b '2 ...qsb ' s  p1a '1 p2a '2 ... pra 'r q1b '1 q2b '2 ...qsb ' s con a'i  ai y b' j  b j .
es decir, son el producto de un divisor positivo de a y otro de b.

b) De nuevo, a  p1a1 p2a2 ... prar , b  q1b1 q2b2 ...qsbs , con pi  q j . Por lo tanto
c n  ab  p1a1 p2a2 ... prar q1b1 q2b2 ...qsbs
Supongamos que un primo p tal que p | c . Entonces
p | c  p | c n  ab  p1a1 p2a 2 ... pra r q1b1 q2b2 ...qsbs , luego p es un pi o un q j , y por tanto c se
escribe como c  p1a '1 p2a '2 ... pra 'r q1b '1 q2b '2 ...qsb ' s .
Luego c n  p1na'1 p2na'2 ... prna'r q1nb'1 q2nb'2 ...qsnb' s , y por la unicidad de la descomposición
canónica, se sigue que na'i  ai y nb' j  b j , y por tanto

a  p1na'1 p2na'2 ... prna' r  p1a '1 p2a '2 ... pra 'r 
n

, y b  q1nb'1 q2nb' 2 ...qsnb' s  q1b '1 q2b ' 2 ...qsb ' s .
n

Proposición.
Sean a, b, c enteros positivos tales que a n  b n c para algún entero positivo n. Entonces
c  d n para algún entero d . En particular, si c no es la potencia n-ésima de un número
entero, entonces n c no es un número racional.

Demostración.
Si a  p1a1 p2a2 ... prar es la descomposición de a en factores primos, entonces la
descomposición de a n en factores primos será a n  p1na1 p2na2 ... prnar
Puesto que b | a n , la descomposición de b en factores primos será de la forma
b  p1b1 p2b2 ... prbr , con 0  bi  n ai
Y por el mismo motivo tendremos c  p1c1 p2c2 ... prcr , con 0  ci  n ai
Luego a n  bnc  p1nb1 p2nb2 ... prnbr p1c1 p2c2 ... prcr  p1nb1  c1 p2nb2  c2 ... prnbr  cr
Y por unicidad de la descomposición en factores primos tendremos
nbi  ci  nai  ci  nai  nbi  nai  bi   0  ai  bi
Luego d  p1a1 b1 p2a2 b2 ... prar br es un número entero que verifica d n  c .

Para la última parte, si n c fuera racional, se podría escribir como n c  a / b , con a, b


enteros positivos. Por lo tanto, a n  b n c , y por el resultado anterior se deduce que c debe
ser una potencia n-ésima de un entero, llegando a contradicción.

Fuente de estos últimos resultados: Apuntes de Teoría Elemental de Números, por Enrique Arrondo.
5.2.1 F
Determina todas las parejas de enteros positivos ( m, n ) tales que m 2 n  20 20 .

AIME II 2020 #1

5.2.2 F
Existen enteros positivos A, B y C, sin factores comunes mayores que 1, tales que
A log 200 5  B log 200 2  C
Determina A  B  C .
ASHME 1995 #24

5.2.3 M
Demostrar que existe un único número natural n tal que 28  211  2n es un cuadrado
perfecto.

5.2.4 F
Si 1998 se escribe como producto de dos enteros positivos cuya diferencia es lo más
pequeña posible, entonces la diferencia es:

(A) 8 (B) 15 (C) 17 (D) 47 (E) 93


AHSME 1998 #6

5.2.5 M
Determina los tres números naturales consecutivos más pequeños cuya suma es un
cuadrado perfecto y un cubo perfecto de números naturales.

5.2.6 M
Halla todas las sucesiones finitas de n números naturales consecutivos a1 , a2 ,..., an , con
n  3 , tales que a1  a2 ,...  an  2009
OME 2009 #1

5.2.7 MF
En el año 2001, los Estados Unidos acogieron las Olimpiadas Matemáticas. Sean
I , M , O enteros positivos tales que I  M  O  2001 . ¿Cuál será el valor más grande
posible de la suma I  M  O ?

(A) 23 (B) 55 (C) 99 (D) 111 (E) 671


AMC12 2000 #1
5.2.8 F
Existen enteros positivos A, B, C , sin factores comunes mayores que 1, tales que
A log 200 5  B log 200 2  C .

¿Cuál es el valor de A  B  C ?

(A) 6 (B) 7 (C) 8 (D) 9 (E) 10

AHSME 1995 #24


5.2.9 M
¿Cuántos conjuntos de tres elementos a, b, c  de enteros positivos verifican
a  b  c  2310 ?
(A) 32 (B) 36 (C) 40 (D) 43 (E) 45

AHSME 1995 #29

5.2.10 F
Existe un número primo p tal que 16 p  1 es el cubo de un entero positivo. Determina p.

AIME I 2015 #3

5.2.11 F
Cuando los números 702, 787 y 855 son divididos entre el entero positivo m, el resto es
siempre el mismo entero positivo r. Cuando los números 412, 722 y 815 son divididos
entre el entero positivo n, el resto es siempre el mismo entero positivo s  r . Determina
mnr  s.

AIME I 2017 #2

5.2.12 M
Determina el número de polinomios de segundo grado f (x) con coeficientes enteros y
ceros enteros tales que f (0)  2010 .

AIME II 2010 #10

5.2.13 M
Determina todos los primos p para los cuales la ecuación x 2  px  444 p  0 tiene
soluciones enteras.
ASHME 1987 #23

5.2.14 M
Demostrar que en cualquier conjunto de n  1 números entre 1 y 2n siempre podemos
encontrar dos elementos tales que el menor divide al mayor.

5.2.15 F
Determina el número de valores de k sabiendo que 1212 es el mínimo común múltiplo de
66, 88 y k.

AIME 1998

5.2.16 F
Denotamos por [r,s] el mínimo común múltiplo de los enteros positivos r y s. Determina
el número de triplas ordenadas a, b, c tales que [a,b]=1000, [b,c]=2000 y [c,a]=2000.

5.2.17 F
Determina 3x 2 y 2 si x, y son enteros tales que y 2  3x 2 y 2  30 x 2  517

AIME 1987 #5
5.2.18 M
Determina p sabiendo que es un número primo tal que 16 p  1 es el cubo de un
número entero.

AIME I 2015 #3

5.2.19 M
Determina todos los enteros a, b  0 tales que
(a, b)  a, b  a  b  6

5.2.20 M
Halla dos enteros positivos a y b conociendo su suma y su mínimo común múltiplo.
Aplícalo en el caso de que la suma sea 3972 y el mínimo común múltiplo 985928.

OME 2008 #1

5.2.21 F
Determina todos los enteros n tales que n  50 y n  50 son ambos cuadrados
perfectos.

5.2.22 F
Determina el número de pares ordenados de enteros positivos a, b cuyo mínimo común
múltiplo sea 23571113 .

5.2.23 F
13!
La suma de todos los enteros positivos m tales que es un cuadrado perfecto se
m
puede escribir como 2a 3b 5c 7 d 11e13 f , donde a, b, c, d, e y f son enteros positivos.
Determina a  b  c  d  e  f .

AIME 2023 I #4

5.2.24 M
Aníbal ha separado los números enteros del 1 al 25 en dos grupos. Después ha quitado
algunos enteros de los grupos para que los productos de los números de cada grupo sean
iguales. ¿Cuál es el mínimo de números que Aníbal puede eliminar.

(A) 4 (B) 5 (C) 6 (D) 7 (E) 8

Canguro N5 2024 #27, Cangur B1 2024 #27

5.2.25 F
Supongamos que a, b, c, y d son enteros positivos satisfaciendo las siguientes
relaciones: abcd  26  39  57 , mcm(a, b)  23  32  53 , mcm(a, c)  23  33  53 ,
mcm (a, d )  23  33  53 , mcm(b, c)  21  33  52 , mcm(b, d )  22  33  52 ,
mcm (c, d )  22  33  52 . Determina Mcd (a, b, c, d )

(A) 30 (B) 45 (C) 3 (D) 15 (E) 6


AMC 12B 2023 #24
5.3 Resolución de problemas mediante identidades algebraicas.

Encuentra una tabla de factorizaciones y identidades notables al final del Dossier


de Problemas de Álgebra

Proposición. La primera identidad algebraica fundamental.


La identidad algebraica


x n  y n  x  y  x n 1  x n  2 y  x n 3 y 2  ...  x y n  2  y n 1 
es fundamental en la resolución de todo tipo de problemas de Aritmética.
Su demostración es fácil: Basta con desarrollar el producto de la derecha:

x  y  x n 1  x n  2 y  x n 3 y 2  ...  x y n  2  y n 1  
 x n  x n 1 y  x n  2 y 2  ...  x 2 y n  2  xy n 1  x n 1 y  x n  2 y 2  x n  3 y 3  ...  x y n 1  y n
 xn  yn
En particular, tomando y  1 obtenemos una identidad muy útil:
n1 n 2 x n1  1
x x
n
x  ...  x  1 
x 1

De la primera identidad se desprende directamente el siguiente resultado:

a) a  b | a n  bn para cualquier n  1.
Por ejemplo, sin necesidad de ningún cálculo, 8767 2345  8101 2345 es divisible entre 666.

Que se puede generalizar para obtener el siguiente resultado:

b) Si d | n con d y n positivos, entonces a d  b d | a n  b n para todo entero a, b .


En efecto, supongamos que n  k d para cierto entero k . Entonces
a n  bn  a k d  bk d  a d    b   a
k d k d
 
 bd a d 1  a d  2b  a d 3b2  ...

Proposición. La segunda identidad algebraica fundamental.


Otra identidad algebraica muy útil es


Si n es impar, x n  y n  x  y  x n 1  x n  2 y  x n 3 y 2  ...  x y n  2  y n 1 
En efecto, Si n es impar, x n  y n  x n   y  , y basta aplicar la identidad anterior.
n

De esta segunda identidad se desprende de forma directa el siguiente resultado:

c) Si n es impar, a  b | a n  bn
Que se puede generalizar para obtener el siguiente resultado:

e) Si d | n con d y n positivos, y n / d es impar, entonces a d  b d | a n  b n para todo


entero a, b .
Proposición.
Si 2n  1 es primo, entonces n es una potencia de 2.

Demostración. Supongamos, por el contrario, que n no es una potencia de 2, es decir,


que podemos escribir n  k  h , con k  1 impar.

Entonces
 k
 
k
2n  1  2k h  1  2h  1  2h  1k  2h  1 2h    k 1
 2h k 2
 
 2h
k 3

 ...  2h  1

Y por tanto nuestro número es divisible entre 2h  1 .

Observación 1: Este resultado será fundamental en el Tema 15, como base para definir
los “primos de Fermat”.

Observación 2: El recíproco no es cierto. Por ejemplo: 641 | 232  1 .

En efecto, utilizando que 641  2 4  54  5  2 7  1 ,


232  1  2 28 2 4  1  2 28 2 4  54  54   1  2 28 2 4  54   2 28 54   1 
   
 2 28 2 4  54  2 7  5  1  2 28  641  641  1  1  2 28  641  641  639 640 2  1 
4 4
 
 6412 28
 639 640  1
2

En donde hemos utilizado que:


640 4  1  640 4  14  640  1640 3  640 2  640  1  639 640 3  640 2  641  
   
 639 640 2 640  1  641  639 640 2  641  641  639  641 640 2  1  
(En el Tema 6 volveremos a demostrar este resultado aplicando la aritmética modular)

5.3.1 F
Determina todos los números primos de la forma n3  1 , para enteros n  1 .

5.3.2 M
Demostrar que 2903 n  803 n  464 n  261 n es divisible entre 1897 para todo natural n .

EOTVOS 1899

5.3.3 M
Demuestra que n 4  4 con n  IN es primo si y solo si n  1 .

5.3.4 M
Determina todos los enteros n  1 para los cuales n 4  4n es un número primo.

5.3.5 F
Demuestra que, para todo n  IN , n 2 divide a n  1  1 .
n

5.3.6 F
Demostrar que 1001 divide 11993  21993  31993  ...  1000 1993
5.3.7 M
Demuestra que 7 divide al número 147  247  347  447  547  647

5.3.8 F
Si al cuadrado de un número de dos dígitos se le resta el cuadrado del número formado
invirtiendo el orden de sus dígitos, entonces el resultado no siempre será divisible por:

(A) 9 (B) El producto de los dígitos. (C) La suma de los dígitos.


(D) La diferencia de los dígitos. (E) 11

ASHME 1957 #24

5.3.9 MD
Demuestra que, para todo entero n  1 , el número n 5  n 4  1 no es primo.

5.3.10 M
Determina todos los enteros positivos a, b para los cuales a 4  4b 4 es primo.

5.3.11 MF
Sean aº y bº los ángulos agudos de un triángulo rectángulo, con a  b y ambos números
primos. ¿Cuál es el menor valor posible de b?

(A) 2 (B) 3 (C) 5 (D) 7 (E) 11

AMC 12B 2020 #4

5.3.12 F
Determina los números p , q primos para los que sabemos que la ecuación
x2  p x  q  0

tiene dos soluciones enteras positivas.

AHSME 1976

5.3.13 D
Factoriza el número 27000001.

5.3.14 MF
Determina todos los pares de enteros positivos ( x, y ) tales que x 2  y 2  23

5.3.15 MF
Tomamos dos números primos diferentes p, q entre 4 y 18. ¿Qué resultado podemos
obtener al restar su suma a su producto?

(A) 23 (B) 60 (C) 119 (D) 180 (E) 231

AMC 12 2000 #6
5.3.16 MF
Sean m, n enteros tales que m 2  3m 2 n 2  30 n 2  517 . Determina 3m 2 n 2 .

AIME 1987 #5

5.3.17 M
Existen dos únicos enteros positivos x , y safisfaciendo la ecuación

x 2  84 x  2008  y 2
Determina x  y .

AIME I 2008 #4

5.3.18 F
Expresa el número 181 2 como la diferencia de los cubos de dos enteros positivos
consecutivos.

5.3.19 F
Determina los pares ordenados ( x, y ) de números enteros positivos que satisfacen la
ecuación
1 1 1 2
  
x y xy 5

5.3.20 MF
Hallar la suma de los factores primos de 36 - 26.

(A) 19 (B) 31 (C) 54 (D) 102 (E) 138

Canguro N4 2016 #5

5.3.21 MF Problema solucionado paso a paso en vídeo.

Tenemos dos números enteros positivos x , y que cumplen

x  1 y  1  77
Si x > y, determina el valor de x.

(A) 6 (B) 7 (C) 8 (D) 10 (E) 11

Cangur B1 2022 #5

Solución: [Link]
5.3.22 MF Problema solucionado paso a paso en vídeo.

El mayor factor primo de 16384 es 2 porque 16384  214 . Determina la suma de los
dígitos del mayor divisor primo de 16383.

(A) 3 (B) 7 (C) 10 (D) 16 (E) 22

AMC 12B Fall 2021 #6

Solución: [Link]

5.3.23 F Problema solucionado paso a paso en vídeo.

Los números enteros a y b son cuadrados perfectos. Su diferencia, a − b, es un número


primo. ¿Cuál de estos números puede ser b?

(A) 10000 (B) 256 (C) 144 (D) 100 (E) 900

Cangur B1 2021 #26, Kangaroo Junior 2021 #26

Solución: [Link]

5.3.24 MF
El producto de las diez cifras de un número es 15. Determina la suma de estas diez
cifras.

(A) 8 (B) 12 (C) 15 (D) 16 (E) 20

Cangur B2 2022 #4, Kangaroo Student 2022 #6

5.3.25 F
Determina la cantidad de enteros positivos de dos dígitos que son factores de 224  1 .

(A) 4 (B) 8 (C) 10 (D) 12 (E) 14

AMC 12B 2011 #15

5.3.26 F
El número 332  1 tiene exactamente dos divisores que son mayores de 75 y menores de
85. Determina su producto.

(A) 5852 (B) 6560 (C) 6804 (D) 6888 (E) 6972

Cangur N4 2008 #29, Kangaroo Student 2008 #29


6 Problemas de la segunda parte.
6.1 MF
Demuestra que, para todo entero n , 120 divide a n 5  5n 3  4n .

6.2 MD
Demuestra que n 2  3n  5 no es divisible entre 121.

6.3 MD
Demuestra que 34  45 es el producto de dos enteros, cada uno de ellos mayor que
5 6

10 2002 .

6.4 F
Si a y b son enteros positivos, pruebe que 19 divide a 11a  2b si y solo si 19 divide a
18a  5b .
Olimpiada Matemática Mexicana 1988

6.5 MF
n2  n  1
Demuestra que si n es un entero positivo, entonces es una fracción
n 2  2n
irreducible.

6.6 MF
x3  3x  2
Determina los enteros x para los cuales sea entero.
2x  1

6.7 F
Existe un número primo p tal que 16 p  1 es el cubo de un entero positivo. Determina
p.

AIME I 2015 #3

6.8 MD
Dado un número primo p , determina todos los enteros k  0 para los cuales k 2  pk
es un entero positivo.

OME 1997 #4

6.9 MF
¿Cuántos cuadrados perfectos positivos menores de 10 6 son múltiplos de 24?

AIME I 2007 #1

6.10 MF
x2
Determina qué valores enteros de x hacen entera la función f ( x) 
x6
6.11 F
La suma de las siete cifras del número aaabbbb es igual al número de dos cifras ab.
¿Cuánto vale la suma a + b?

A) 8 B) 9 C) 10 D) 11 E) 12

Canguro N5 2019 #18, Cangur B1 2019 #18

6.12 F
Se empaquetan 60 manzanas y 60 peras en cajas con igual número de manzanas y
distinto número de peras en cada caja. ¿Cuál es el mayor número posible de cajas que se
pueden empaquetar de esta manera?

A) 20 B) 15 C) 12 D) 10 E) 6

Canguro N5 2019 #19, Cangur B1 2019 #19

6.13 M
¿Cuál es el menor número de elementos que tenemos que quitar del conjunto:

{10, 20, 30, 40, 50, 60, 70, 80, 90}

de modo que el producto de los elementos que quedan en el conjunto es un cuadrado


perfecto?

A) 1 B) 2 C) 3 D) 4 E) 5
Canguro N5 2019 #28, Cangur B1 2019 #28

6.14 M
¿Cuántos de los números del 210 al 213 , ambos inclusive, son divisibles por 210 ?

A) 2 B) 4 C) 6 D) 8 E) 16

Canguro N6 2019 #10, Cangur B2 2019 #10

6.15 M
¿Cuál es la mayor potencia de 3 que es divisor del número 7! + 8! + 9! ?

A) 32 B) 34 C) 35 D) 36 E) una potencia de 3 mayor que 36

Canguro N6 2019 #11, Cangur B2 2019 #11

6.16 M
Un entero positivo n se llama “bueno” si su divisor más grande (excluyendo n) es igual
a n  6 . ¿Cuántos enteros positivos “buenos” hay?

A) 1 B) 2 C) 3 D) 6 E) infinitos

Canguro N6 2019 #16, Cangur B2 2019 #16


6.17 M
Sea a la suma de todos los divisores positivos de 1024 y b el producto de todos los
divisores positivos de 1024. Entonces

A) (a  1)5  b B) (a  1) 5  b C) a 5  b D) a 5 1  b E) a 5 1  b

Canguro N6 2019 #21, Cangur B2 2019 #21

6.18 M
Los vértices de la red mostrada se numeran del 1 al 10. La suma S de los números de los
cuatro vértices de cada cuadrado es la misma. ¿Cuál es el menor valor posible de S?

A) 18 B) 19 C) 20 D) 21 E) 22

Canguro N6 2019 #23, Cangur B2 2019 #23

6.19 F
¿Para cuántos enteros n es n 2  2n  3 un número primo?
A) 1 B) 2 C) 3 D) 4 E) infinitos

Canguro N6 2019 #26 , Cangur B2 2019 #26

6.20 F
Este año, el número de chicos en mi clase ha aumentado respecto del curso pasado un
20% y el número de chicas ha disminuido un 20%. Ahora tenemos un estudiante más
que antes. ¿Cuál de los siguientes podría ser ahora el número de estudiantes en mi
clase?

A) 22 B) 26 C) 29 D) 31 E) 34
Canguro N6 2019 #12, Cangur B2 2019 #12

6.21 D
ab
Para calcular , Sara teclea en la calculadora a  b  c  , y el resultado es 11 (a, b,
c
y c son enteros positivos). Luego teclea b  a  c  , y se sorprende al ver que el
resultado es 14. Se da cuenta de que la calculadora está diseñada para calcular las
ab
divisiones antes que las sumas. ¿Cuál es el resultado correcto de ?
c

A) 1 B) 2 C) 3 D) 4 E) 5

Canguro N6 2019 #20, Cangur B2 2019 #20


6.22
Sea S el conjunto de todos los enteros positivos N con la siguiente propiedad: Los
últimos cuatro dígitos de N son 2020, y cuando se eliminan estos cuatro dígitos, el
resultado es un divisor de N. Por ejemplo, 42020 pertenece a S porque 4 es divisor de
42020. Determina la suma de todos los dígitos de todos los números de S. Por ejemplo,
el número 42020 contribuye con 4+2+0+2+0=8 a este total.

AIME I 2020 #4

6.23 D
Sean m y n enteros positivos satisfaciendo las siguientes condiciones:
- El máximo común divisor ( m  n , 210 )  1 .
- m m es un múltiplo de n n .
- m no es un múltiplo de n .

Determina el menor valor posible de m  n .

AIME I 2020 #10

6.24 MF
Determina la cantidad de números enteros positivos de cuatro cifras (es decir, enteros
entre 1000 y 9999, inclusive) que tienen solo cifras pares y son divisibles entre 5.

(A) 80 (B) 100 (C) 125 (D) 200 (E) 500

AMC 12A 2020 #4

6.25 F
Para todos los enteros n  9 , el valor de
(n  2)!(n  1)!
n!

(A) es un múltiple de 4 (B) es un múltiple de 10 (C) es un número primo


(D) es un cuadrado perfecto (E) es un cubo perfecto.

AMC 12B 2020 #6

6.26 D
Determina el número de enteros positivos n múltiplos de 5 tales que el mínimo común
múltiplo de 5! y n sea igual a 5 veces el máximo común divisor de 10! y n.

(A) 12 (B) 24 (C) 36 (D) 48 (E) 72

AMC 12A 2020 #21


6.27 D
Sean x, y enteros positivos satisfaciendo el siguiente sistema de ecuaciones:
log 10 x  2 log 10 Mcd ( x, y )   60

log 10 y  2 log 10 mcm ( x, y )   570

Sea m el número de factores primos en la factorización de x (no necesariamente


diferentes), y sea n el número de factores primos en la factorización de y (no
necesariamente diferentes). Determina 3m  2n .

AIME I 2019 #7

6.28 D
Diremos que un entero positivo n es k-guapo si n tiene exactamente k divisores
positivos y es divisible entre k. Por ejemplo, 18 es 6-guapo. Sea S la suma de todos los
enteros positivos menores de 2019 que sean 20-guapos. Determina S/20.

AIME II 2019 #9

6.29 MF
Consideremos la afirmación "Si n no es primo, entonces n  2 es primo". ¿Cuáles de los
siguientes valores de n es un contraejemplo de esta afirmación?

(A) 11 (B) 15 (C) 19 (D) 21 (E) 27

AMC 12B 2019 #2

6.30 MF
Los caramelos de una tienda cuestan valores enteros en céntimos. Casper tiene
exactamente suficiente dinero para comprar 12 caramelos rojos, 14 caramelos verdes,
15 caramelos azules, o n caramelos púrpura (una de las cuatro opciones). Un caramelo
púrpura cuesta 20 céntimos. Determina el menor valor posible de n .

(A) 18 (B) 21 (C) 24 (D) 25 (E) 28

AMC 12B 2019 #5

6.31 M
Sea S el conjunto de todos los divisores enteros positivos de 100 000. ¿Cuántos
números son el producto de dos elementos diferentes de S?

(A) 98 (B) 100 (C) 117 (D) 119 (E) 121

AMC 10B 2019 #19, AMC 12B 2019 #14


6.32 MF
Determina el número de enteros n (no necesariamente positivos) para los cuales
n
2
4000   
5
es un entero.

(A) 3 (B) 4 (C) 6 (D) 8 (E) 9

AMC 12A 2018 #7

6.33 MF
Sea S un subconjunto de 6 elementos de 1,2,...,12  con la propiedad de que si a y b
son elementos de S y a  b , entonces b no es un múltiple de a . Determina el menor
valor posible de los elementos de S.

(A) 2 (B) 3 (C) 4 (D) 5 (E) 7

AMC 12A 2018 #12

6.34 M
Fijado un entero positivo n , y tres dígitos a, b, c diferentes de cero, sea An el entero de
n dígitos todos ellos iguales a a , sea Bn el entero de n dígitos todos ellos iguales a b ,
y sea Cn el entero de 2n dígitos (no de n dígitos) todos ellos iguales a c . Determina el
mayor valor posible de a  b  c para el que existen al menos dos valores de n tales que
Cn  Bn  An2 .

(A) 12 (B) 14 (C) 16 (D) 18 (E) 20

AMC 12A 2018 #25

6.35 F
Mary toma un número entero par de 4 dígitos n y escribe todos sus divisores en orden
n
creciente, de izquierda a derecha: 1, 2, ..., , n . En un cierto momento, Mary escribe el
2
número 323 como divisor de un cierto n. ¿Cuál es el menor entero divisor de n que
Mary escribirá a la derecha de 323?

(A) 324 (B) 330 (C) 340 (D) 361 (E) 646

AMC 12B 2018 #19


6.36 D
5 p 4
Sean p y q enteros positivos tales que   y siendo q tan pequeño como sea
9 q 7
posible. Determina q  p .

(A) 7 (B) 11 (C) 13 (D) 17 (E) 19


AMC 12B 2018 #17
6.37 MD
Supongamos que la ecuación cuadrática x 2  a x  b  0 tiene dos raíces reales, ambas
múltiples de 3, y que la ecuación x 2  bx  9a  0 tiene una raíz doble. Determina a y
b.

6.38 M
Considera el siguiente par de números naturales de 4 cifras:

(m, n) = (2601, 1600) (es decir, m = 2601, n = 1600).

Fíjate que m y n verifican las siguientes propiedades:


1. Son números de 4 cifras (esto es, entre 1000 y 9999).
2. Son cuadrados perfectos.
[Link] las mismas cifras en exactamente dos de las cuatro posiciones. (En nuestro
ejemplo, en la segunda y tercera posiciones.)
4. En las dos posiciones en las que las cifras son distintas, la cifra que aparece en m es
igual a la que aparece en n más 1.
Encuentra todos los pares de números naturales que verifican estas cuatro propiedades.
OMEFL Aragón 2021 #2

6.39 F
8n  3
Determinar todos los enteros positivos n para los que la fracción
17 n  9
es irreductible.

OMEC 2021 #4

6.40 MF
¿Cuál de los siguientes números no es divisible por 3, cualquiera que sea el número
entero n?

A) 5n  1 B) n 2 C) n(n  1) D) 6n  1 E) n 3  2

Canguro N6 2020 #10, Cangur B2 2020 #10


6.41 F
Un número N es divisible por todos los enteros del 2 al 11, excepto por dos de ellos.
¿Cuál de las siguientes parejas de números puede ser la excepción?

A) 2 y 3 B) 4 y 5 C) 6 y 7 D) 7 y 8 E) 10 y 11

Canguro N6 2020 #24, Cangur B2 2020 #24

6.42 M
Hay 71 canicas en una caja. Un juego consiste en sacar 30 canicas de la caja, si las hay,
o meter 18. Se puede jugar indefinidamente. ¿Cuál es la menor cantidad de canicas que
puede quedar en la caja en algún momento?

A) 1 B) 3 C) 5 D) 7 E) 11
Canguro N6 2020 #26, Cangur B2 2020 #26
6.43 M
Hay n números primos p1, p2 , ..., pn diferentes en la fila inferior de la tabla piramidal,
como se ve en la figura.

El producto de dos números contiguos de una fila se coloca en la fila superior, en la


casilla situada exactamente sobre ellos. El número K  p11  p2 2  ...  pn n está en la
casilla de la última fila. Si  2  8 , ¿cuántos números hay en la tabla que sean divisibles
por p4 ?

A) 4 B) 16 C) 24 D) 28 E) 36

Canguro N6 2020 #29, Cangur B2 2020 #29

6.44 F
Marta puso un signo de multiplicación entre las cifras segunda y tercera del número
2020 y observó que el producto resultante 20  20 es un cuadrado perfecto. ¿Cuántos
números entre 2010 y 2099 (incluido 2020) tienen la misma propiedad?

A) 1 B) 2 C) 3 D) 4 E) 5

Canguro N5 2020 #16, Cangur B1 2020 #16

6.45 F
Se forma un número de nueve cifras colocando aleatoriamente los dígitos del 1 al 9.
¿Cuál es la probabilidad de que el número resultante sea divisible por 18?

1 4 5 1 3
A) B) C) D) E)
2 9 9 3 4
Nota: En este problema se supone que no se repiten las cifras.

Canguro N5 2020 #21, Cangur B1 2020 #21, Kangourou J 2020 #19

6.46 M
Ocho enteros positivos consecutivos de tres cifras tienen la siguiente propiedad: cada
uno de ellos es divisible por la cifra de las unidades. ¿Cuál es la suma de las cifras del
menor de los ocho enteros?

A) 10 B) 11 C) 12 D) 13 E) 14

Canguro N5 2020 #30, Cangur B1 2020 #30


6.47 MF
A lo largo de cuatro días, Linda viaja durante una hora, a una velocidad equivalente a
avanzar una milla en un número entero de minutos. Después del primero, su velocidad
disminuye de forma que el número de minutos que necesita para avanzar una milla se
incrementa en 5 minutos respecto del día anterior. En cada uno de los cuatro días la
distancia recorrida es también un número entero de millas. Determina el número total de
millas recorridas después de los cuatro días.

(A) 10 (B) 15 (C) 25 (D) 50 (E) 82

AMC 8 2017 #23

6.48 MF
n 1 n 5
Si   
n 1 n n  1 k

Para ciertos enteros positivos n y k, ¿Cuál es el valor de k?

(A) 1 (B) 5 (C) 24 (D) 25

SAT

6.49 MD
Determina todos los enteros a, b, c con 1  a  b  c para los cuales (a  1)(b  1)(c  1)
divide a abc  1

IMO 1992 #1

6.50 M
El número de pares de enteros ( x, y ) con 0  x  y tales que 1984  x  y es:

(A) 0 (B) 1 (C) 3 (D) 4 (E) 7

ASHME 1984 #28

6.51 M
El número n se escribe en base 14 como a b c , se escribe en base 15 como a c b y se
escribe en base 6 como a c a c , con a  0 . Determina el número n en base 10.

AIME I 2018 #2

6.52 F
El número de triples a, b, c de números enteros positivos que satisfacen el sistema
ab  bc  44

ac  bc  23

es
(A) 0 (B) 1 (C) 2 (D) 3 (E) 4
ASHME 1984 #20
6.53 F
Determina el número de 7-tuplas de números positivos (a, b, c, d , e, f , g ) que satisfacen
el siguiente sistema de ecuaciones:
abc  70

cde  71
efg  72

AIME II 2019 #3

6.54 M
4 2
Resuelve la siguiente ecuación diofántica  1
m n
ASHME 1993 #19

6.55 M
Resuelve el siguiente sistema de ecuaciones, con a, b, c enteros.

 a  b  c  19

 ab  c  97

ASHME 1997 #28

6.56 F

Resuelve la ecuación con solución entera x2  x  1  x2
1

ASHME 1985 #21

6.57 D
Determina todas las soluciones enteras de la ecuación 2 x  1  3 y

6.58 MF
Si x , y son enteros mayores que 1, y se cumple y  x  4 y   7 , determina el valor de x.

SAT Hard Problem

6.59 MF
Si x, y, z son enteros positivos tales que x y  24 , x z  48 , y z  72 , determina
x yz.

(A) 18 (B) 19 (C) 20 (D) 22 (E) 24

AMC 10 2001 #10


6.60 F
¿Qué proporción de todos los divisores de 7! son números impares?

(A) 1/5 (B) 1/2 (C) 1/3 (D) 1/4 (E) 1/6

Cangur B2 2021 #11

6.61 F
Sea N  34  34  63  270 . Determina la razón de la suma de los divisores impares de N
entre la suma de los divisores pares de N.

(A) 1:16 (B) 1:15 (C) 1:14 (D) 1:8 (E) 1:3

AMC 10B 2021 #12 , AMC 12B 2021 #7

6.62 MF
Determina el menor número primo que divida a la suma 311  513 .

(A) 2 (B) 3 (C) 5 (D) 311  513 (E) Ninguno de los anteriores

ASHME 1974 #8

6.63 F
Sean m y n dos enteros positivos tales que satifacen 5m  n | 5n  m

Demuestre que m es divisor de n.

Holanda Selectivo EGMO 2015

6.64 F
¿Para cuántos valores del número real b tiene la ecuación x 2  b x  80  0 dos
soluciones distintas que son números enteros positivos pares?

(A) 0 (B) 1 (C) 2 (D) 3 (E) infinitos

Canguro N6 2006 #23, Cangur N4 2006 #22

6.65 F
Hallar cuántos números reales positivos “ a ” hay de manera que la ecuación cuadrática
x 2  a x  2007  0 tenga dos raíces enteras.

(A) 3 (B) 4 (C) 6 (D) 8 (E) otra respuesta

Canguro N6 2007 #26, Cangur N4 2007 #26

6.66 MF
Los enteros m y n verifican 6  m6  n  12 . ¿Cuántos valores puede tomar m?

(A) 6 (B) 7 (C) 12 (D) 13 (E) Ninguno de los anteriores

Canguro N4 2013 #16

6.67 F
¿Cuántos divisores de cuatro cifras tiene el número 1022 ?

(A) 2 (B) 3 (C) 4 (D) 5 (E) 6


Canguro N6 2005 #17
6.68 F Problema solucionado paso a paso en vídeo.

¿Cuál es la mayor potencia de 2 que divide a 2552 – 1?

(A) 21 (B) 28 (C) 29 (D) 2127 (E) 2254

Canguro N4 2010 #10

Solución: [Link]

6.69 F
Si las dos soluciones de la ecuación x 2  63 x  k  0 son números primos, el número de
posibles valores de k es

(A) 0 (B) 1 (C) 2 (D) 4 (E) más de 4

AMC 10A 2002 #14, AMC 12A 2002 #12

6.70 F
Determina el número de divisores de 2020 que tienen, a su vez, más de tres divisores.
(Por ejemplo, 12 tiene 6 divisores: 1, 2, 3, 4, 6 y 12).

(A) 6 (B) 7 (C) 8 (D) 9 (E) 10

AMC 8 2020 #17

6.71 M
Calcula el siguiente máximo común divisor:  2002  2 , 2002 2
 2 , 2002 3  2 , ... 
HMMT 2002

6.72 F

Determina los valores de n para los cuales la expresión



16 n 2  n  1 2

es un entero.
2n  1
Maclaurin Olympiad 2018

6.73 F
Determina los números p, q primos que satisfacen la ecuación p 2  2q 2  1 .

6.74 M
5
Los adultos representan un del total de asistentes a un concierto. Después de la
12
11
llegada de 50 asistentes más, los adultos representan un del total de asistentes a un
25
concierto. Determina el mínimo número de adultos que hay en el concierto después de
la llegada del autobús.
AIME II 2022 #1
6.75 F
Determina el número de rectas en el plano cuyo punto de corte con el eje X es un entero
positivo primo, su punto de corte con el eje Y es un entero positivo y además pasan por
el punto  4 , 3  .

(A) 0 (B) 1 (C) 2 (D) 3 (E) 4

ASHME 1994 #28

6.76 MF
Determina el número de enteros n para los cuales n 2  2n  3 es un número primo.

(A) 1 (B) 2 (C) 4 (D) 3 (E) Infinitos

Cangur B2 2019 #26

6.77 MF
¿Cuántos números primos p hay de forma que p² + 2 también sea un número primo?

(A) 0 (B) 1 (C) 3 (D) 5 (E) Infinitos

Cangur B2 2016 #21

6.78 MF
¿Cuántos números primos p cumplen que p 4  1 también es un número primo?
(Observación: 1 no es un número primo).

(A) Ninguno (B) Uno (C) Dos (D) Tres (E) Infinitos

Cangur N4 2008 #4

6.79 M
Resuelve en enteros la ecuación
m 3  24 m 2  2022  3n

Admission Test Summer Program math Olympiad

6.80 MD
Determina todas las ternas  p, x, y  de números naturales con p primo que cumplen
p x  y4  4

India National Mathematical Olympiad 2008 #2

6.81 F
Determina la suma de todos los enteros positivos n para los cuales n 2  19 n  99 es un
cuadrado perfecto.

AIME 1999 #3
6.82 M
Resuelve en enteros la ecuación
m 3  24 m 2  2022  3n

Awesomemath 2022 part1 Admission Test Summer Program #6

6.83 F
Determina el número de pares ordenados x, y  de enteros para los que se cumple
x 2  y 2  2000 2 .

AIME II 2000 #2

6.84 F
Determina todos los valores de n enteros para los cuales n 2  97 n  85 es un cuadrado
perfecto.

6.85 F
Determina todas las parejas de enteros ( m, n ) que verifican 2 m  2 n  2016

Pakistan Math Olympiad

6.86 F
¿Cuántos números enteros y positivos de cinco cifras cumplen que el producto de sus
cifras es igual a 1000?

(A) 30 (B) 60 (C) 10 (D) 40 (E) 20

Cangur B1 2021 #28, Kangaroo Junior 2021 #28

6.87 F
El entero positivo N es tal que el producto de sus dígitos es 20. ¿Cuál de los siguientes
no podría ser el producto de los dígitos de N+1?

(A) 40 (B) 30 (C) 25 (D) 35 (E) 24

Cangur B1 2022 #28, Kangaroo Junior 2022 #27

6.88 F
Determina la cantidad de combinaciones posibles a, b y c de tres enteros positivos que,
al ser multiplicados entre ellos, su producto es 100 y suponiendo que a  b  c .

(A) 0 (B) 1 (C) 2 (D) 3 (E) 4

AMC 8 2022 #3
6.89 F
Dado n entero positivo, definimos por f (n) el cociente obtenido al dividir la suma de
todos los divisores de n entre el propio n. Por ejemplo,

f (14)  1  2  7  14   14 
12
7

Determina f (768 )  f (384 ) .

(A) 1/768 (B) 1/192 (C) 1 (D) 4/3 (E) 8/3

AMC 12B Fall 2021 #12

6.90 F
¿Para cuántos enteros a entre 1 y 25 el producto a (a  1)( a  2) es múltiplo de 84?

(A) para ninguno (B) 1 (C) 2 (D) 4 (E) 6

Canguro N5 2009 #26

6.91 M
Determina el número de pares ( x, y ) de enteros positivos con x  y satisfaciendo
x, y   5! y x, y  50!
CMO 1997 #1

6.92 M Problema solucionado paso a paso en vídeo.

Hallar la suma de todos los enteros positivos x menores que 100 para los que x² – 81 es
múltiplo de 100.

(A) 200 (B) 100 (C) 90 (D) 81 (E) 50

Cangur N4 2011 #19, Canguro N6 2011 #19, Kangaroo Student 2011 #26

Solución: [Link]

6.93 F
a, b i c son números enteros y positivos de manera que a2=2b3=3c5. Determina el menor
número de divisores de a·b·c (incluyendo 1 y a·b·c)?

(A) 30 (B) 49 (C) 60 (D) 77 (E) 1596

Cangur N4 2011 #28, Canguro N6 2011 #28, Kangaroo Student 2011 #28

6.94 MF
Resuelve la ecuación 5a  5b  3000 con a, b enteros positivos.
6.95 MF
Determina el número de valores de k para los que el polinomio x 2  k x  36 tiene dos
raíces enteras distintas.

(A) 6 (B) 8 (C) 9 (D) 14 (E) 16

AMC 12B 2022 #4

6.96 F
El mínimo común múltiplo de un entero positivo n y 18 es 180, y el máximo común
divisor de n y 45 es 15. Determina la suma de los dígitos de n.

(A) 3 (B) 6 (C) 8 (D) 9 (E) 12

AMC 12A 2022 #4, AMC 10A 2022 #7

6.97 F
a) Sea n un entero. Demustra que si n es racional, entonces n es un cuadrado perfecto.
b) Sean n y m enteros. Demuestra que si n  m es un entero, entonces n y m son
cuadrados perfectos.

6.98 M
Sea S el conjunto de todos los números racionales positivos r tales que cuando los dos
números r y 55r se escriben en forma de fracción irreducible, la suma del numerador y
del denominador de la primera es igual a la suma del numerador y del denominador del
segunda. La suma de los elementos de S se puede escribir de la forma p/q, donde p y
que son enteros positivos coprimos. Determina p+q.

AIME II 2023 #5

6.99 F
13 !
La suma de todos los enteros positivos m tales que es un cuadrado perfecto se
m
puede escribir como 2a 3b 5c 7 d 11e13 f , donde a, b, c, d, e y f son enteros positivos.
Determina a  b  c  d  e  f .

AIME 2023 I #4

6.100 MF
Determina el número de enteros positivos que son divisores de 220·323 pero no son
divisores de 210·320.

(A) 13 (B) 30 (C) 273 (D) 460 (E) Ninguna de las anteriores

Cangur B2 2023 #20, Canguro N6 2023 #20


6.101 M Problema solucionado paso a paso en vídeo .

El producto de seis números consecutivos es un número de 12 cifras de la forma

ab b cdd cdd ab b

donde las cifras a, b, c y d son cuatro números consecutivos no necesariamente


ordenados. Determina el valor de la cifra d.

(A) 1 (B) 2 (C) 3 (D) 4 (E) 5

Cangur B2 2023 #30, Canguro N6 2023 #30

Solución: [Link]

6.102 M
Determina el número de enteros positivos de tres cifras con la propiedad de que si
restamos del número la suma de sus cifras, el resultado es un número de tres cifras
iguales.

(A) 2 (B) 3 (C) 8 (D) 20 (E) 30

Cangur B1 2023 #27, Canguro N5 2023 #27

6.103 F
¿Cuántos números positivos de tres cifras hay que sean iguales al producto de sus tres
cifras multiplicado por 5?

(A) 4 (B) 3 (C) 2 (D) 1 (E) Ninguno

Cangur B2 2022 #22, Kangaroo Student B2 2022 #22

6.104 MF
Llamamos “primo2” a un entero positivo n, si tiene exactamente tres divisores
diferentes, el 1, el 2 y el propio n. ¿Cuántos números “primo2” diferentes hay?

(A) 0 (B) 1 (C) 2 (D) 3 (E) 4

Canguro N6 2023 #5

6.105 F
A, B y C son dígitos distintos, siendo A y C distintos de 0, tales que seis veces el
número de tres dígitos ABC es igual al número de tres dígitos CCC. Encuentra el valor
de A + B + C.

(A) 11 (B) 13 (C) 17 (D) 14 (E) 15

Canguro N4 2023 #22


6.106 D Problema solucionado paso a paso en vídeo .

Sea n un entero mayor que 1. Los enteros positivos divisores de n son d1 , d 2 , ..., d k con
1  d1  d 2  ...  d k  n

Se define D  d1d 2  d 2 d3  ...  d k 1d k .


a) Demostrar que D  n 2 .
b) Determinar todos los números n tales que D es un divisor de n 2 .

IMO 2002 Problema 4

Solución: [Link]

6.107 D
Determina todos los enteros compuestos n>1 que satisfacen la siguiente propiedad:
Si d1 , d 2 , ..., d k son todos los divisores positivos de n con 1  d1  d 2  ...  d k  n ,
entonces d i divide a di 1  di 2 para cada 1  i  k  2 .

IMO 2023 #1

6.108 F Problema solucionado paso a paso en vídeo .

Determina el número de enteros positivos n menores de 2017 para los cuales


n 2 n3 n 4 n5 n 6
1 n     
2! 3! 4! 5! 6!
Sea un entero.

AIME II 2017 #8

Solución: [Link]

6.109 F Problema solucionado paso a paso en vídeo .

Determina todos los enteros positivos compuestos n para los cuales es posible ordenar
todos sus divisores mayores o iguales que 1 de forma que dos elementos adyacentes no
sean coprimos.

USAMO 2005 #1

Solución: [Link]
6.110 M Problema solucionado paso a paso en vídeo .

Consideramos 2024 números primos distintos p1 , p2 , ..., p2024 tales que


p1  p2  ...  p1012  p1013  p1014  ...  p2024
Sea A  p1 p2 ...p1012 y B  p1013 p1014... p2024 . Demuestra que A  B  4 .
OME 2024 #1

Solución:

6.111 F
Supongamos que m y n son números enteros con 0 < m < n. Sean P = (m, n), Q = (n,
m), y O = (0, 0). ¿Para cuántos pares de valores m y n, el área del triángulo OPQ será
igual a 2024?

(A) 4 (B) 6 (C) 8 (D) 10 (E) 12

Canguro N5 2024 #30, Cangur B1 2024 #30

6.112 MF
¿Cuál de estos números enteros es dos unidades menos que un múltiplo de diez, dos
unidades más que un cuadrado y dos veces un número primo?

(A) 78 (B) 58 (C) 38 (D) 18 (E) 6

Canguro N6 2024 #2, Cangur B2 2024 #2

6.113 MF
Un estudiante comenzó con el número 1 y lo multiplicó por 6 o por 10. A continuación,
multiplicó el resultado por 6 o por 10, y continuó este procedimiento muchas veces.
¿Cuál de los siguientes valores no puede ser uno de los números obtenidos?

(A) 2100320580 (B) 290320580 (C) 290320570 (D) 2110380530 (E) 250550
Canguro N6 2024 #9, Cangur B2 2024 #9
6.114 MF
Solo una de estas afirmaciones sobre un cierto número entero positivo n es cierta. ¿Qué
afirmación es cierta?
(A) n es divisible por 3
(B) n es divisible por 6
(C) n es impar
(D) n = 2
(E) n es primo

Canguro N6 2024 #11, Cangur B2 2024 #11

6.115 F
La hija de María dio a luz hoy a una niña. Dentro de dos años, el producto de las edades
de María, su hija y su nieta será igual a 2024. Las edades de María y su hija son
números pares. ¿Qué edad tiene María ahora?

(A) 42 (B) 44 (C) 46 (D) 48 (E) 50

Canguro N5 2024 #14, Cangur B1 2024 #14

6.116 F
La factorización en números primos del número. n! = 1·2 · · · (n-1)·n, es de la forma
que se muestra en la imagen. Los números primos se escriben en orden creciente. La
tinta ha cubierto algunos de los números. ¿Cuál es el exponente que tiene el número 17?

(A) 1 (B) 2 (C) 3 (D) 4 (E) 5

Canguro N5 2024 #21, Cangur B1 2024 #21

6.117 F
La pingüina Paula va a pescar todos los días y siempre trae doce peces para sus dos
polluelos. Cada día, le da al primer polluelo que ve, siete peces y al segundo cinco
peces, que se comen. En los últimos días un polluelo ha comido 44 peces. ¿Cuántos se
ha comido el otro polluelo?

(A) 34 (B) 40 (C) 46 (D) 52 (E) 58

Canguro N4 2024 #14, Cangur E4 2024 #5


6.118 F
Damián quiere completar la figura de modo que cada cuadro contenga el producto de
los valores que se encuentran en los dos cuadros que están por debajo y cada cuadro
contenga un número entero positivo. Quiere que el valor en el cuadro superior sea 720.
¿Cuántos valores diferentes puede tomar el número entero n?

(A) 1 (B) 4 (C) 5 (D) 6 (E) 7

Canguro N4 2024 #25, Cangur E3 2024 #25

6.119 F
Determina el número de cuadrados perfectos positivos menores de 2023 divisibles entre
5.

(A) 8 (B) 9 (C) 10 (D) 11 (E) 12

AMC 12A 2023 #3, AMC 10A 2023 #3

6.120 M
Supongamos que a, b y c son enteros positivos tales que
a b c
 
14 15 210
¿Cuál de las siguientes afirmaciones es necesariamente cierta?

I. Si Mcd ( a,14 )  1 o Mcd (b,15)  1 o ambos, entonces Mcd (c,210 )  1


II. Si Mcd (c,210 )  1 , entonces Mcd ( a,14 )  1 o Mcd (b,15)  1 o ambos.
III. Mcd (c,210 )  1 si y solo si Mcd (a,14 )  Mcd (b,15)  1 .

(A) I,II y III (B) I solamente (C) I y II solamente (D) III solamente (E) II y III
solamente

AMC 12B 2023 #15

6.121 F
Sea f la única función definida para todos los enteros positivos tal que
n

d |n
d  f   1
d 
para cualquier entero n, donde esta suma recorre todos los divisores positivos de n.
Determina f 2023 .

(A) -1536 (B) 96 (C) 108 (D) 116 (E) 144


AMC 12A 2023 #22
7 Aritmética modular básica.

Para ver aplicaciones de la aritmética modular para simplificar problemas de


combinatoria, ver, por ejemplo los problemas #1.6.37 y #1.6.40 de
[Link]

El lenguaje de las congruencias nos permite abordar con éxito problemas aparentemente
muy difíciles. Las congruencias es un lenguaje, una técnica, y por tanto solo se aprende
practicando, jugando con ellas durante mucho tiempo.

Estamos acostumbrados a trabajar con conjuntos numéricos infinitos.

Por ejemplo, los números naturales:


N  0 ,1, 2 , 3, 4 , ... 

Los números enteros:


Z  ...,  4 ,  3,  2 ,  1, 0 ,1, 2 , 3, 4 , ... 

Los números racionales:


 1 3 7  11 
Q   ...,  4 ,  3 ,  2 ,  1, 0 ,1, 2 , 3 , 4 , ..., , , , , ... 
 2 2 3 5 

Todos estos conjuntos tienen una cantidad infinita de elementos.

Pero también es perfectamente válido disponer de conjuntos finitos de elementos y


hacer operaciones en ellos. En los siguientes apartados vamos a ver unos ejemplos
introductorios.
7.1 Primer ejemplo: Las horas del día.
El día tiene 24 horas:
D  0 ,1, 2 , 3, 4 , 5, 6 , 7 , 8,..., 22, 23 

La hora 24 sería medianoche, es decir, la hora 0. Así pues, podemos decir que la hora 24
y la hora 0 son la misma hora, es el mismo elemento de D. Escribiremos
24  0

Para indicar que la hora 24 y la hora 0 son equivalentes.

De la misma forma, podemos decir que la hora 25 es la hora 1, la hora 26 es la hora 2, la


hora 27 es la hora 3...

25  1 , 26  2 , 27  3

Utilizaremos el símbolo  , que se parece mucho al símbolo  , para expresar la idea de


que no son iguales, pero son equivalentes: En la práctica, es como si fueran iguales.

Como calcular el equivalente modular.


¿Qué hora sería la hora 67 ? Han pasado dos días completos y quedan 17 horas del
tercero:
67  24  2  17

Luego 67  17

Lo que estamos haciendo es realizar la división entera entre 24 y tomar el resíduo:

¿Qué hora será la hora 349? Realizamos la división entera y tomamos el resíduo:

Escribimos 349  14

7.1.1 Ejercicio.
Determina los correspondientes equivalentes modulares en el conjunto D de las horas
del día:
a) 7 b) 23 c) 29 d) 168 e) 773

7.1.2 Ejercicio.
Determina los correspondientes equivalentes modulares en el conjunto D de las horas
del día:
a) 15 b) 21 c) 30 d) 200 e) 1441
Representación circular.
Puede ser interesante representar las horas en forma circular:

El número de vueltas completas (el cociente de la división) no cuenta para nada, y nos
quedamos con el resíduo.

Como sumar con aritmética modular.


Si son las 17 horas (las cinco de la tarde) y quiero ver una película de 3 horas, ¿A qué
hora acabará la película? Muy fácil, basta hacer una simple suma:

17  3  20

Acabará a las 20 horas (las ocho de la tarde).

¿Pero si son las 22 horas, las diez de la noche?

22  3  25

Nadie dice "Acabará a las 25", todo el mundo dice "acabará a la una (de la
madrugada)", ¡y estamos tan acostumbrados que lo hacemos mentalmente!

Primero realizamos la suma y después calculamos el resíduo:

Escribiremos 22  3  1

7.1.3 Ejercicio.
Calcula (en el conjunto D de horas del día)
a) 5  4 b) 8  15 c) 15  9 d) 20  18

7.1.4 Ejercicio.
Calcula (en el conjunto D de horas del día)
a) 3  2 b) 18  15 c) 25  12 d) 22  17
Como hacer operaciones en aritmética modular.
Lo mismo que hemos hecho con las sumas lo podemos hacer con las restas,
multiplicaciones y potencias: Basta con realizar la operación usual en Z y quedarnos
con el residuo modular. Por ejemplo, en el conjunto
D  0,1, 2 , 3, 4 , 5, 6 , 7 , 8,..., 22, 23 

Podemos restar. Por ejemplo: 32  5  3


Primero hemos hecho la resta "normal": 32  5  27 y luego calculamos el equivalente
modular del resultado:

Podemos multiplicar. Por ejemplo: 9  4  36  12


Primero hemos hecho la multiplicación "normal": 9  4  36 y luego calculamos el
equivalente modular del resultado:

Podemos dividir. Por ejemplo: 144  2  0


Primero hemos hecho la división "normal": 144  2  72 y luego calculamos el
equivalente modular del resultado:

Podemos hacer potencias. Por ejemplo: 62  12


Primero hemos hecho la potencia "normal": 62  36 y luego calculamos el equivalente
modular del resultado:

7.1.5 Ejercicio.
Realiza las siguientes operaciones (en el conjunto D de horas del día)
a) 30  5 b) 5 7 c) 42  3 d) 5 2
e) 100  1 f) 6 4 g) 720  9 h) 4 3

7.1.6 Ejercicio.
Realiza las siguientes operaciones (en el conjunto D de horas del día)
a) 70  20 b) 6  5 c) 81  3 d) 10 2 e) 2 5

7.1.7
Hoy es jueves. ¿Qué día de la semana será de aquí a 2023 días?

(A) Martes (B) Miércoles (C) Jueves (D) Viernes (E) Sábado
Cangur B1 2023 #2, Canguro N5 2023 #2
7.2 Segundo ejemplo: El conjunto Z7.

Definición de Z7.
Definimos "Zeta siete" como el conjunto de los números del 0 al 6:

Z 7   0 ,1, 2 , 3 , 4 , 5 , 6 

Visualmente podemos considerar este conjunto como una noria de siete


compartimentos, en la que sumar quiere decir "girar hacia la derecha" y restar quiere
decir "girar hacia la izquierda":

Está claro que en Z 7 sumar o restar 7 es no hacer nada: Damos una vuelta completa y
nos quedamos donde estábamos, Escribiremos, por ejemplo:

3  7  3 (mod 7) 5  7  5 (mod 7)

En Z 7 los múltiples de 7 son todos equivalentes a cero:

7  0 (mod 7) , 14  0 (mod 7) , 21  0 (mod 7) ...

También los negativos:

 7  0 (mod 7) ,  14  0 (mod 7) ,  21  0 (mod 7) ...

Esto lo vemos claro con números pequeños. Para números grandes debemos utilizar el
método del residuo:

3421  5 (mod 7) porque al hacer la división entera 3421 entre 7 obtenemos un residuo
igual a 5:
Tablas de la suma y la multiplicación en Z7.

Ahora podemos hacer la tabla de la suma "módulo 7":

Observa detenidamente esta tabla. Por ejemplo: Fila "4" columna "5":

4  5  9  2 (mod 7)

Observamos las propiedades tradicionales de la suma: Es una operación conmutativa, el


cero no hace nada (por eso se llama "elemento neutro") ...

También podemos hacer la tabla de la multiplicación "módulo 7":

Observa detenidamente como se ha construido esta tabla. Por ejemplo: Fila "3",
columna "6":

3  6  18  4 (mod 7)

Y en esta tabla observamos las propiedades tradicionales de la multiplicación: Es una


operación conmutativa, el uno no hace nada (es el elemento neutro), cualquier número
multiplicado por cero da cero...

7.2.1 Ejercicio.
Completa la tabla de la suma de Z 5   0 ,1, 2 , 3 , 4 
7.2.2 Ejercicio.
Completa la tabla de la multiplicación de Z 5   0 ,1, 2 , 3 , 4 .
7.3 Los conjuntos Zn.

7.3.1 Definición. Congruencias.


Diremos que a  b (mod n) , y diremos que "a es congruente con b módulo n" cuando
sucede alguna de estas condiciones equivalentes:
a) n | (a  b)
b) a  k n  b para cierto entero k .
c) a y b dejan el mismo residuo cuando son divididos entre n.

Por ejemplo: 24  3 (mod 7) , pues 24  3  7  3 , 34  4 (mod 5) , pues 34  6  5  4


En particular: a  0 (mod n)  n | a

7.3.2 Ejemplo.
Determina x tal que 5 x  6 (mod 8) .

Solución.
Vamos probando, uno por uno:
5  1  5  0  8  5  5 (mod 8) 5  2  10  1  8  2  2 (mod 8)
5  3  15  1  8  7  7 (mod 8) 5  4  20  2  8  4  4 (mod 8)
5  5  25  3  8  1  1 (mod 8)
5  6  30  3  8  6  6 (mod 8)  La solución es x  6 (mod 8)
5  7  35  4  8  3  3 (mod 8)

7.3.3 Proposición. Propiedades básicas de las congruencias.


Sea n  0 fijo, y a, b, c, d enteros arbitrarios. Entonces se cumple:

a) a  a (mod n) (Propiedad reflexiva).


b) Si a  b (mod n)  b  a (mod n) (Propiedad simétrica).
c) Si a  b (mod n) y b  c (mod n)  a  c (mod n) (Propiedad transitiva).
d) Si a  b (mod n) y c  d (mod n)  a  c  b  d (mod n) y a c  b d (mod n) .
e) Si a  b (mod n)  k a  k b (mod n) para cualquier entero k

f) Si a  b (mod n)  a k  b k (mod n) para cualquier entero positivo k .

7.3.4 Ejemplo.
¿ Es 2414  1514 divisible entre 9?

Solución.
24  15 mod 9  2414  1514 mod 9  2414  1514  0 mod 9  9 | 2414  1514 , luego
sí es divisible.
7.3.5 Observación. Qué funciona y qué no funciona con congruencias.
Las propiedades anteriores nos permiten trabajar con congruencias prácticamente igual
a como trabajamos con números, pero no todo lo que hacíamos con números funciona
ahora con congruencias:

a) Cancelación de términos:
No funciona en general la cancelación de términos (el "tachar" de toda la vida).
Por ejemplo: 2  4  2  1 (mod 6) , pero 4  1 (mod 6)
217  21 (mod 7) , pero 216  1 (mod 7)

Aunque existe una Regla de cancelación:


Si (c, n)  1 , entonces ca  cb (mod n)  a  b (mod n)

b) Principio del producto nulo:


No existe tampoco el principio del producto nulo en general. Por ejemplo:
4  3  0 (mod 12 ) , pero 4  0 (mod 12 ) y 3  0 (mod 12 )

Pero sí se verifica cuando el módulo es un número primo:


Si p es primo, a  b  0 (mod p )  a  0 (mod p ) o b  0 (mod p )

7.3.6 Proposición. Modificaciones en el módulo.


 m
a) Si ( a, m)  d  1 , entonces ax  ay (mod m)  ax  ay  mod  .
 d
b) Si a  b (mod n) y d | n  a  b (mod d )
c) a  b (mod n) y a  b (mod m)  a  b  mod n, m  .
d) Si n1 , n2 ,... , nk   1 , a  b (mod ni ) i  1,..., k  a  b (mod n1n2 ...nk )

Demostración.
a) ax  ay (mod m)  m | ax  ay   m | ax  y   | x  y 
m a
d d
b) a  b (mod n m)  n m | (a - b)  n | (a - b)  a  b (mod n)
c)
a  b (mod n)  n | (a  b) 
  n, m | (a  b)  a  b  mod n, m  )
a  b (mod m)  m | (a  b)
(ver 4.6b)
d) Es un caso particular de c.
Trabajar con congruencias es una técnica muy potente para resolver problemas de
Teoría de Números, como se puede ver en los siguientes ejemplos:

7.3.7 Problema resuelto.


Demostrar que 41 divide 220  1

Solución. En primer lugar vemos que 25  32  9 (mod 41)


 
Luego 220  25   9 (mod 41)   9  9 (mod 41)  81  81 (mod 41)
4 4 2 2

Pero, por otro lado, 81 (mod 41)  1 (mod 41)


Luego 81  81 (mod 41)  (1)(1) (mod 41)  1 (mod 41)
Finalmente, 220  1  1  1 (mod 41)  0 (mod 41)  41 | 220  1 , tal y como queríamos
ver.

7.3.8 Problema resuelto.


Determina el residuo al dividir 1! 2! 3! 4! ...  99!100! entre 12.

Solución. Observamos que 4! 24  0 (mod 12 ) , luego, para todo k  4 ,


k! 4!5  6...  k  0  5  6...  k  0 (mod 12 )
luego
1!2!3!4!...  99!100! 1!2!3!4!0  ...  0  0  1!2!3!4! (mod 12 )

Y nuestro problema se reduce a encontrar el residuo al dividir 1!2!3!4! 33 entre 12,


que es 9.

7.3.9 Proposición. Potencias perfectas y congruencias.


Sea n un entero. Entonces:
a) n 2   0 , 1 mod 3 b) n 2   0 , 1 mod 4 c) n 2   0 ,  1 mod 5
d) n 2   0 , 1 , 4 mod 8 e) n 3   0 ,  1 mod 9 f) n 4   0 , 1 mod 16 

Demostración.
Basta ir estudiando casos, como se hizo en el problema propuesto del Apartado 4.1, que
correspondería a la demostración del apartado b.
7.4 Aplicación a la criptografía: El cifrado César.

Descripción del cifrado César.


El primer uso documentado de una cifra monoalfabética por sustitución con propósitos
militares aparece en el documento "La guerra de las Galias" de Julio César.

En dicho libro, Julio César describe cómo envía un mensaje cifrado a Cicerón, que se
encontraba sitiado y a punto de rendirse, aplicando una sustitución simple a las letras
del texto en claro de forma que el mensaje fuera ininteligible para el enemigo.

Aunque en aquella ocasión César sustituyó las letras romanas por letras griegas, es
común asociarle a su sistema de cifra un desplazamiento de 3 espacios a la derecha en el
alfabeto, tal y como lo describe Suetonio en su libro "Vidas de los Césares" en la
entrada LVI sobre Cayo Julio César.

Consistía en escribir el mensaje con un alfabeto que estaba formado por las letras del
alfabeto latino normal desplazadas tres posiciones a la derecha. Con nuestro alfabeto el
sistema quedaría así:

Alfabeto en claro: ABCDEFGHIJKLMNÑOPQRSTUVWXYZ


Alfabeto cifrado: DEFGHIJKLMNÑOPQRSTUVWXYZABC

Por ejemplo, si se quiere enviar el mensaje ATACARALAMANECER, lo que se


escribirá realmente es DWDFDUDÑDODPHFHU

El receptor del mensaje conocía la clave secreta de éste (es decir, que estaba escrito con
un alfabeto desplazado tres posiciones a la derecha), y podía descifrarlo fácilmente
haciendo el desplazamiento inverso con cada letra del mensaje. Pero para el resto de la
gente que pudiese accidentalmente llegar a ver el mensaje, el texto carecía de ningún
sentido.

Es un cifrado muy débil y poco seguro, pero en la época de Julio César no era de
conocimiento general la idea de ocultar el significado de un texto mediante cifrado. De
hecho, que un mensaje estuviese por escrito ya era un modo de asegurar la
confidencialidad frente a la mayoría de la población analfabeta de la época.

Como dato curioso, más de 1500 años después, un cifrado similar al de César fue
utilizado por la reina María Estuardo de Escocia, para conspirar junto con los españoles
contra su prima Isabel I (en realidad, fue incitada a conspirar por agentes al servicio de
Isabel I; una trampa bien urdida.) Los mensajes cifrados de María fueron fácilmente
descifrados mediante sencillos análisis estadísticos por los agentes de Isabel I, y así pues
quedó al descubierto la conspiración de la reina escocesa. Junto con la pérdida del
secreto de la comunicación, María perdió la cabeza en su ejecución el 8 de febrero de
1587. Después de esto el cifrado César quedó definitivamente descartado como método
de cifrado seguro para los gobernantes del mundo. Desde entonces a hoy, los cifrados
usados por los estados para preservar sus secretos han mejorado considerablemente.

El cifrado César desde un punto de vista matemático.

Lo que a nosotros nos interesa del cifrado César es que es un claro ejemplo de
utilización de la aritmética modular para garantizar la confidencialidad de la
información mediante el cifrado o encriptación. Matemáticamente, las veintisiete letras
del alfabeto castellano las podemos interpretar como el conjunto de números naturales
del 0 al 26:

A  0 , B  1, C  2 , D  3 , ..., Z  27

Y por tanto podemos suponer que trabajamos en el conjunto

Z 27   0 ,1, 2 , 3 , 4 , 5 , 6 , 7 , 8 , ..., 26 , 27 

El cifrado será la función


f : Z 27  Z 27
x  x3 mod 27 
Y el descifrado será su función inversa:

g : Z 27  Z 27
x  x3 mod 27 
Observamos que en vez de utilizar el 3 podemos utilizar cualquier otro número k , que
sería la clave de encriptación: Un valor que sólo conocerían el emisor y el receptor del
mensaje, y sin el cual (supuestamente) nadie podría descifrar el mensaje. Así pues,
podemos decir que el método César es un método de encriptación de clave privada.

Cifrado:
f : Z 27  Z 27
x xk mod 27 
Descifrado:
g : Z 27  Z 27
x xk mod 27 
Criptoanálisis de los Métodos de Cifrado Monoalfabéticos.

El cifrado monoalfabético constituye la familia de métodos criptográficos más simple


de criptoanalizar, puesto que las propiedades estadísticas del texto claro se conservan en
el criptograma. Supongamos que, por ejemplo, la letra que más aparece en Castellano es
la E. Parece lógico que la letra más frecuente en el texto codificado sea aquella que
corresponde con la E. Emparejando las frecuencias relativas de aparición de cada
símbolo en el mensaje cifrado con el histograma de frecuencias del idioma en el que se
supone está el texto claro, podremos averiguar fácilmente la clave.

Distribución de frecuencias de letras en español para un texto literario:

E - 16,78% R - 4,94% Y - 1,54% J - 0,30% A - 11,96%


U - 4,80% Q - 1,53% Ñ - 0,29% O - 8,69% I - 4,15%
B - 0,92% Z - 0,15% L - 8,37% T - 3,31% H - 0,89%
X - 0,06% S - 7,88% C - 2,92% G - 0,73% K - 0,00%
N - 7,01% P - 2,77% F - 0,52% W - 0,00% D - 6,87%
M - 2,12% V - 0,39%

Ejercicio 1.
Cifrad con el sistema de clave privada de César y clave secreta k = 5 el mensaje m =
VOYALCINE (si al buscar el cifrado de un carácter se llega al final del alfabeto, se
sigue la cuenta por el principio del alfabeto). ¿Qué nos han contestado si el mensaje
recibido es c = DTRTATD?

Ejercicio 2.
Se ha utilizado cifrado César con clave privada k = 6 para transmitir un nombre en
secreto. Se ha recibido BÑIZUX. ¿Cuál es el nombre secreto?

Ejemplo.
Un ejemplo literario y cinematográfico muy conocido es el del nombre HAL del
ordenador que aparece en la novela 2001 de Arthur C. Clarke y en la correspondiente
película dirigida por Stanley Kubrick, "2001 una odisea en el espacio". No es nada
complicado obtener el nombre de una famosa empresa si sabemos que el nombre del
maléfico ordenador (HAL) es el cifrado con clave k = 26 del nombre de la empresa.
Cifrado César con Python.

#
# Generador de puntos de la curva secp256k1 módulo p por Brute-Force
#
p=257
Total=0
for x in range (0,p):
for y in range (0,p):
v=y**2-x**3-7
if v%p==0:
print(x,",",y)
Total=Total+1
print("Total de puntos:",Total)

El siguiente programa es un ejemplo de un programa Python que encripta de textos


mediante el método César:

#
# Encriptador metodo Cesar
#
Clave=3
Frase1="ELATAQUESERAESTANOCHE"
Frase2=""
for c in Frase1:
n=ord(c)-65
m=(n+Clave)%26
Letra=chr(m+65)
Frase2=Frase2+Letra
print("Frase sin encriptar: ",Frase1)
print("Frase encriptada: ",Frase2)

Salida:

Frase sin encriptar: ELATAQUESERAESTANOCHE


Frase encriptada: HODWDTXHVHUDHVWDQRFKH

El programa es un bucle que toma uno a uno cada carácter de Frase1 y lo deposita en la
variable c.
for c in Frase1:

Para convertir el carácter c en su correspondiente valor numérico se utiliza la función


ord(c), que devuelve el código UNICODE del carácter que recibe como parámetro. El
problema está en que los códigos UNICODE de las letras mayúsculas empiezan en 65:

A → 65 , B → 66 , C → 67 …

Por lo que debemos restar 65 para así empezar desde cero:

A → 65-65=0 , B → 66-65=1 , C → 67-65=2 …

n=ord(c)-65
Ahora ya podemos realizar la encriptación, sumando a n el valor de la Clave y
determinando su equivalente módulo 26. Dejando el resultado en la variable m.
m=(n+Clave)%26

Por último, convertimos el valor numérico m en su correspondiente letra, mediante la


función chr(n), que devuelve el carácter UNICODE correspondiente al valor numérico
que recibe como parámetro. De nuevo, debemos tener en cuenta que los caracteres
UNICODE de las letras mayúsculas empiezan en 65, por lo que debemos sumar 65 al
valor de m. Dejamos la letra obtenida en la variable Letra.
Letra=chr(m+65)

Ya solo queda añadir Letra a la cadena de salida Frase2. Utilizamos para ello el
comando +, que en principio realiza la suma de dos números, pero que en el contexto de
dos cadenas de caracteres lo que hace es juntar las dos cadenas de caracteres.
Frase2=Frase2+Letra

Por último, se visualizan las dos frases: La frase de entrada sin encriptar y la frase de
salida encriptada.
print("Frase sin encriptar: ",Frase1)
print("Frase encriptada: ",Frase2)

El correspondiente programa desencriptador es prácticamente idéntico al anterior:

#
# Desencriptador Metodo Cesar
#
Clave=3
Frase1="HODWDTXHVHUDHVWDQRFKH"
Frase2=""
for c in Frase1:
n=ord(c)-65
m=(n-Clave)%26
Letra=chr(m+65)
Frase2=Frase2+Letra
print("Frase encriptada: ",Frase1)
print("Frase desencriptada: ",Frase2)

Salida:
Frase encriptada: HODWDTXHVHUDHVWDQRFKH
Frase desencriptada: ELATAQUESERAESTANOCHE

El único cambio significativo está en la línea 9, en donde en vez de sumar la Clave, la


restamos.
Mejora al programa anterior. El comando if

En el programa anterior hemos supuesto que los caracteres de la frase de entrada son
siempre letras mayúsculas.

Supongamos que queremos poder recibir cualquier carácter, y encriptar solo las letras
mayúsculas.

El comando if permite realizar una acción solo si se cumple una determinada condición,
y suele venir acompañado del comando else, que indica la acción a realizar en caso
contrario.

En nuestro caso, tomamos la letra c y la encriptamos si es una letra mayúscula (A, B,


C,…, Z) y la dejamos tal cual en caso contrario:

#
# Encriptador Metodo Cesar (ver. 2)
#
Clave=3
Frase1="ATACAR A LAS 4:00 DE ESTA NOCHE"
Frase2=""
for c in Frase1:
if [Link]():
n=ord(c)-65
m=(n+Clave)%26
Letra=chr(m+65)
else:
Letra=c
Frase2=Frase2+Letra
print("Frase sin encriptar: ",Frase1)
print("Frase encriptada: ",Frase2)

Salida:

Frase sin encriptar: ATACAR A LAS 4:00 DE ESTA NOCHE


Frase encriptada: DWDFDU D ODV 4:00 GH HVWD QRFKH
7.5 Aplicación a la criptografía: El cifrado Hill.

Introducción.

El método César es un método de cifrado por sustitución monoalfabética. Este tipo de


cifrados se basa en sustituir cada letra del texto original por otra del alfabeto.
Estos métodos representan un gran agujero en su seguridad al poderse determinar con
gran facilidad la frecuencia de repetición de cada una de las letras que posteriormente se
confronta con las estándar de aparición en el castellano, con lo que con un texto
suficientemente largo el mensaje y la clave quedan totalmente al descubierto.

Para evitar los ataques mendiante análisis de frecuencias cifraremos no cada letra
individualmente, sino en grupos de dos o más, son los llamados cifrados por sustitución
polialfabética, como por ejemplo el método Hill que describiremos en este apartado.
Este sistema fue inventado por Lester S. Hill en 1929, mediante transformaciones
matriciales.

El tamaño de los grupos elegidos para hacer las sustituciones depende del grado de
seguridad que deseemos. Aquí realizaremos la presentación del método con matrices
2 2 por su simplicidad, eligiendo una matriz 4 4 la respuesta es casi plana para la
frecuencia de cada letra. Es decir, en un texto cifrado por este método todas las letras
tienen prácticamente la misma probabilidad de aparecer, con lo cual un estudio de sus
frecuencias será totalmente inútil, puesto que todas darán casi el mismo resultado.

Nota importate: Se supone en el lector conocimientos básicos de álgebra lineal, en


especial las operaciones con matrices y el concepto de matriz inversa (ver dossier GL)

En IR:
A es invertible  det A  0 , en cuyo caso A1 
1
 Adj AT
det A
En Z n :
A es invertible  det A es invertible módulo n  ( det A , n )  1
en cuyo caso A1  det A  Adj A
1 T
Descripción del método.

Dividiremos el texto en grupos de dos letras, que pasaremos a sus equivalentes


numèricos Z 26 como hemos hecho en el apartado anterior. Para cifrar solo necesitamos
una matriz que utilizaremos como clave, y descifrar utilizaremos su matriz inversa.

 35  12    135 180 
Por ejemplo, tomamos la matriz A    y su inversa A 1   
 3 9    45 525 

Las transformamos a módulo 26:

 9 14   21 24 
A    y su inversa A 1   
 3 17  7 5

Ahora tomaremos un mensaje en claro: ELCOCHELLEVADOSANTENAS y lo


dividiremos en grupos de dos letras.

Primer bloque: "EL"

1. Determinamos sus correspondientes valores numéricos en Z 26 :

E4
L  11

Obteniendo así el siguiente vector columna:

4
u   
11 

2. Le aplicamos la transformación lineal de la matriz A, es decir, multiplicamos la


matriz clave por el vector u :

 35  12  4  190   8 
v  Au           mod 26 
 3  9 11  199  17 

3. Obtenemos las letras asociadas a los valores numéricos:

8I
17  R

Ya hemos obtenido las dos primeras letras de nuestra frase encriptada: "IR"

La operación se repite con todos los grupos de letras sucesivamente, obteniendo el


siguiente texto en clave: "IRGKMVIRZXHLPNGCTYKZSU"
El método de desenciptación es el mismo, solo que ahora utilizaremos la matriz inversa

 21 24 
A 1   
7 5

Primer bloque: "IR"


1. Determinamos sus correspondientes valores numéricos en Z 26 :

I 8
R  17

Obteniendo así el siguiente vector columna:

8
v   
17 

2. Le aplicamos la transformación lineal de la matriz A 1 :

 21 24  8   576   4 
u  A 1v           mod 26 
 7 5 17   141  11 

3. Obtenemos las letras asociadas a los valores numéricos:

4E
11  L

Ya hemos obtenido las dos primeras letras de nuestra frase desencriptada: "EL". Esta
operación se realiza con todos los grupos de dos letras recibidos en clave hasta que se
forme el mensaje en claro ELCOCHELLEVADOSANTENAS, que coincide con el
mensaje original, tal y como era de esperar.

Este método, como hemos dicho, se puede aplicar con qualquier tamaño de matriz
cuadarada, y cuanto mayor sea éste, mejor será la seguridad, aunque aumentará también
la complicación matemática.

Fuente:Revista Microhobby nº 165.


Codificación Python.

#
# Encriptador/Desencriptador metodo Hill
#
Frase1="ELCOCHELLEVADOSANTENAS"
Frase2=""
a11=9
a12=14
a21=3
a22=17
if len(Frase1)%2==1:
Frase1=Frase1+"X"
c=0
while(c<=len(Frase1)-2):
l1=Frase1[c:c+1]
l2=Frase1[c+1:c+2]
u1=ord(l1)-65
u2=ord(l2)-65
v1=(a11*u1+a12*u2)%26
v2=(a21*u1+a22*u2)%26
Letra1=chr(v1+65)
Letra2=chr(v2+65)
Frase2=Frase2+Letra1
Frase2=Frase2+Letra2
c=c+2
print("Frase entrada: ",Frase1)
print("Frase salida: ",Frase2)

SALIDA
Frase entrada: ELCOCHELLEVADOSANTENAS
Frase salida: IRGKMVIRZXHLPNGCTYKZSU

Comentarios:
Línia 10:Puesto que trabaja por parejas de letras, sería problemático si la cadena de
entrada tuviera una longitud impar. Resolvemos este problema añadiendo en dicho caso
una letra final.

El resto de programa no tiene mayor dificultad: Es un bucle que recorre la frase (línea
13) tomando parejas de letras, convirtiendo estas letras en sus respectivos equivalentes
numéricos, multiplicando por la matriz y generando las respectivas letras.
El programa desencriptador es el mismo programa, en el que únicamente hemos
cambiado las matriz por su inversa:

#
# Encriptador/Desencriptador metodo Hill
#
Frase1="IRGKMVIRZXHLPNGCTYKZSU"
Frase2=""
a11=21
a12=24
a21=7
a22=5
if len(Frase1)%2==1:
Frase1=Frase1+"X"
c=0
while(c<=len(Frase1)-2):
l1=Frase1[c:c+1]
l2=Frase1[c+1:c+2]
u1=ord(l1)-65
u2=ord(l2)-65
v1=(a11*u1+a12*u2)%26
v2=(a21*u1+a22*u2)%26
Letra1=chr(v1+65)
Letra2=chr(v2+65)
Frase2=Frase2+Letra1
Frase2=Frase2+Letra2
c=c+2
print("Frase entrada: ",Frase1)
print("Frase salida: ",Frase2)

SALIDA:
Frase entrada: IRGKMVIRZXHLPNGCTYKZSU
Frase salida: ELCOCHELLEVADOSANTENAS
7.6 Problemas de aritmética modular básica.
7.6.1 MF
Si n ! denota el producto de todos los números del 1 al n, ¿Cuál es el residuo de
1! 2! 3! ...  n!
al dividirlo entre 9?

7.6.2 MF
Encuentra un ejemplo que demuestre que a 2  b 2 (mod n) no implica a  b (mod n) .

7.6.3 F
Determina los residuos cuando 2 50 y 4165 son divididos entre 7.

7.6.4 F
Utilizando la teoría de congruencias, demuestra que 89 | 244  1 y 97 | 2 48  1 .

7.6.5 F
Determina el último dígito de 91003  7902  3801 .

7.6.6 MF
Demuestra que para todo n  IN , el número an  11n  2  12 2 n 1 es divisible entre 133.

7.6.7 MF
La cifra de las unidades de 2137 753 es:

(A) 1 (B) 3 (C) 5 (D) 7 (E) 9

AHSME 1961 #28

7.6.8 F
El dígito de las unidades de 31001 71002131003 es:

(A) 1 (B) 3 (C) 5 (D) 7 (E) 9

AHSME 1983 #14

7.6.9 F
Demuestra que 186  1 (mod 7k ) para k  1,2,3.

7.6.10 MF
Demuestra que si n es impar, entonces n 2  1 (mod 8)

7.6.11 F
Determina el número de enteros n , 1  n  25 , tales que n 2  3n  2 es divisible entre
6.
7.6.12 F
Demuestra que 2 n  6  9 n siempre es divisible entre 7, para todo entero positivo n.

7.6.13 M
Sea an  6  8 . Determina el residuo cuando a83 se divide entre 49.
n n

AIME 1983 #6

7.6.14 F
Determina el residuo al dividir 9  99  999  ...  99
 9 entre 1000.
...
999 nueves

AIME I 2010 #2

7.6.15 F
Consideremos el esquema triangular de números 0 ,1, 2 , 3... a lo largo de los lados y con
números interiores obtenidos sumando los dos números superiores de la fila anterior.
Las filas 1 a 6 se muestran en el siguiente esquema:

Sea f (n) la suma de los números de la fila n. ¿Cuál es el residuo cuando dividimos
f (100 ) entre 100?

AHSME 1995 #27

7.6.16 M
Sea k  2008 2  22008 . ¿Cuál es el dígito de las unidades de k 2  2 k ?

(A) 0 (B) 2 (C) 4 (D) 6 (E) 8

AMC12A 2008 #15, AMC 10A 2008 #24

7.6.17 F
Determina el menor entero positivo n tal que n y 107 n tienen las dos últimas cifras
iguales.

HMMT 2008 #2

7.6.18 M
Demuestra que si x, y, z son enteros cumpliendo x 2  y 2  3z 2 , entonces x  y  z  0 .

7.6.19 F
Determina los enteros n tales que n  2 divide n  18  .
2

PUMaC 2007/NT #B2


7.6.20 MF
Demuestra que para todo natural n , el número 2n  1 no es divisible entre 7.

IMO 1967 #1 (apartado b)

7.6.21 MF
   ...
7
Determina el último dígito de  7 7
7 7
en donde aparece 1001 veces el número 7.

7.6.22 F
Dados cuatro números enteros diferentes a, b, c, d , demuestra que
a  ba  ca  d b  cb  d c  d 
es divisible entre 12

7.6.23 F
Tomamos todos los enteros del 19 al 92 y los escribimos consecutivamente para formar
el número
N  192021 909192

Sea 3 k la mayor potencia de 3 que es factor de N. Determina k.

ASHME 1992 #17

7.6.24 F
El día 300 del año N cae en martes. El día 200 del año N  1 cae también en martes.
Determina en qué día de la semana cae el día 100 del año N  1 .

AMC 12 2000 #18

7.6.25 M
Determina todos los números naturales 1  n  25 tales que n 2  3n  2 es divisible
entre 6.

7.6.26 F
Sean 74 A52B1 y 326AB4C dos números de 7 dígitos, ambos múltiples de 3.
Determina la suma de todos los posibles valores de C.

AMC 8 2014 #21

7.6.27 D
Demuestra el criterio de divisibilidad del 9 y del 11 aplicando el lenguaje y técnicas de
la aritmética modular.
7.6.28 F
Sobre una mesa hay una torre hecha de bloques numerados del 1 al 90, ordenados en
orden creciente de abajo arriba. Roberto los va cogiendo de tres en tres, comenzando
por los tres de arriba y los va apilando, ordenadamente, y así construye otra torre, como
se puede ver en la siguiente figura:

Cuando los haya acabado de colocar todos, ¿Cuántos bloques habrá entre el 39 y el 40?

(A) 0 (B) 1 (C) 2 (D) 3 (E) 4

Cangur B1 2023 #16, Canguro N5 2023 #16

7.6.29 F
Presentamos las filas 1, 2, 3, 4 y 5 de una tabla de enteros:

En la que cada fila se forma poniendo 1 en los extremos y en las posiciones intermedias
sumandos los dos números inmediatamente superiores de la fila anterior y añadiendo 1.
Determina la cifra de las unidades de la suma de los 2023 números de la fila 2023.

(A) 1 (B) 3 (C) 5 (D) 7 (E) 9

AMC 12A 2023 #20


8 Inversos multiplicativos modulares.
8.1 Concepto de inverso multiplicativo.
Concepto de inverso multiplicativo. Primeros ejemplos.
Las operaciones de sumar, restar y multiplicar no son problemáticas en aritmética
modular: Basta realizar la operación correspondiente en los naturales y después
determinar el equivalente modular del resultado. Pero con la división las cosas se
complicarán, y mucho. Necesitamos definir y estudiar los llamados inversos
multiplicativos.

En el conjunto Q de números racionales, dado cualquier número diferente de cero, por


1 1
ejemplo 3, podemos determinar la fracción , que cumplirá la condición: 3   1
3 3
1 7
Así pues, diremos que es el inverso multiplicativo de 3. De la misma forma, es el
3 5
5
inverso multiplicativo de , por poner otro ejemplo.
7

En general, dado un número n  0 , definimos su inverso multiplicativo n 1 como el


número tal que
n  n 1  1

Diremos que un número es invertible cuando exista su inverso multiplicativo. Está


claro que el 0 nunca es invertible.

Notación: Para referirnos al inverso multiplicativo de k nunca utilizaremos la notación


1
como fracción , siempre utilizaremos la notación exponencial: k 1 .
k

Ejemplo.
Observemos la tabla de multiplicar de Z 5 :

El inverso multiplicativo de 2 es 3, porque 2  3  6  1 mod 5 , y escribiremos


2 1  3 mod 5

El inverso multiplicativo de 4 es el propio 4, porque 4  4  16  1 mod 5 , y


escribiremos
4 1  4 mod 5
8.1.1
Observa atentamente la tabla de multiplicar de Z 7 :

Determina los inversos multiplicativos de 1, 2, 3, 4, 5 y 6. Utiliza la notación


exponencial para denotarlos.

Sobre la (no) existencia de inversos multiplicativos.


Observa ahora la tabla de multiplicar de Z 8 :

Observamos que el 2 no tiene inverso multiplicativo: No existe ningún número que


multiplicado por 2 dé 1. Diremos que el 2 no es invertible en Z 8 .

Tampoco tienen inversos multiplicativos el 4 ni el 6. Así pues, no siempre existen


inversos multiplicativos modulares.

Más ejemplos.
El inverso de 3 módulo 4 es 3 porque 3  3  9  1 (mod 4) .
El inverso de 3 módulo 5 es 2 porque 3  2  6  1 (mod 5) .

8.1.2
Determina totos los elementos invertibles de Z 6 y sus correspondientes inversos.
8.2 Existencia y unicidad de inversos multiplicativos.

8.2.1 Teorema. Existencia y unicidad de inversos modulares.


Sean a, b enteros, se cumple (a, b)  1 si y solo si existe un entero x tal que
a x  1 (mod b) , y este valor es único módulo b.

Demostración. Aplicando el TDB, si (a, b)  1 existirán x, y tales que a x  b y  1 , y


por tanto ax  1 (mod b) .
Veamos la unicidad: Supongamos que existen dos valores x, x ' tales que
a x  1 (mod b) y a x'  1 (mod b) . Entonces
x'  1x'  a xx'  a x' x  1x  x (mod b)

Recíprocamente, supongamos que a x  1 (mod b) , es decir, a x  1  yb para cierto


entero y , y por tanto a x  yb  1  a x  ( y )b  1 .
Aplicando el TDB se deduce que (a, b)  1 .

La congruencia lineal a x  1 (mod n) tiene solución si y solo si ( a, n) | 1 , es decir,


cuando (a, n)  1 , así pues, existirá el inverso multiplicativo de a si y solo si (a, n)  1 , y
será único modulo n.

Definición. Cuerpo.
Diremos que Z n es un cuerpo cuando todos los elementos de Z n excepto el cero
tengan su correspondiente inverso multiplicativo.
Así pues, si n es primo, siempre se cumplirá (a, n)  1 , y por tanto Z n será un cuerpo.

Ejercicio resuelto.
Determina el inverso de 9 módulo 82.

Solución: Observamos que 9  9  81  1 (mod 82 ) , luego 9  (9)  81  1 (mod 82 ) .


El inverso de 9 módulo 82 es  9  82  9  73 (mod 82 ) .

Ejercicio resuelto.
Determina todas las parejas de inversos módulo 20.

Solución: Sabemos que serán todos los números coprimos con 20, es decir:
1, 3, 7 , 9,11,13,17,19 
Ahora solo nos queda agruparlos por parejas (o consigo mismo)
El 1 es inverso de sí mismo, pues 1  1  1  1 (mod 20)  11  1 (mod 20)
Vemos que 3  7  21  1 (mod 20)  31  7 (mod 20) , 71  3 (mod 20)
Vemos que 17  13  221  1 (mod 20)  17 1  13 (mod 20) , 131  17 (mod 20)
El 11 es inverso de sí mismo, pues 11  11  121  1 (mod 20)  111  11 (mod 20)
El 9 es inverso de sí mismo, pues 9  9  81  1 (mod 20)  91  9 (mod 20)
Finalmente, el 19 también es inverso de sí mismo, pues
19  19  381  1 (mod 20)  19 1  19 (mod 20)
Otra manera de verlo es observar que 19  1 (mod 20)  19 2  (1)2  1 (mod 20)
8.3 Inversión mediante el ADE.

Determinación de inversos multiplicativos mediante el ADE.


Dado un elemento a  Z n , a  0 , cumpliendo (a, n)  1 , sabemos que será invertible, y
el ADE nos ofrece la combinación lineal
xa  bn 1

Luego x a  b n  1  x a  1 mod n  a  x 1 mod n

Ejemplo resuelto.
Determina 34 1 mod 143 

Solución. Sabemos que, puesto que 34 , 143  1 , existirán enteros x, y tales que
34 x  143 y  1 , y por tanto 34x  1  143y  34x  1 mod 143 .
El Algoritmo de Euclides (4.17) nos permite determinar esta combinación lineal:
143  34  4  7  143,34  34  4  7,34  7,34
34  4  7  6  7,34  7,4  7  6  7,6  1 , luego:
1  7  6  7  34  4  7   7  34  4  7  34  5  7  34  5  143  34  4 
 34  5  143  34  4  5  5  143  21  34
Y por tanto 34 1  21  122 mod 143 

Ejemplo resuelto.
Determina el inverso de 11 en Z 20 .

Solución.
20  111  9 11  1 9  2  9  4  2  1  2  2 1

Por lo tanto mcd (11 , 20 )  1

Ahora sustituimos de abajo hacia arriba los restos:


1  9  4  2  9  411  9   9  4 11  4  9  5  9  4 11 
 5  20  11  4 11  5  20  9 11

Luego
1  5  20  9  11  9  11 (mod 20)  111  9  9  20  11 (mod 20)

8.3.1
Determinar el inverso de 117 módulo 244.
Problema resuelto.
Demostrar que las seis últimas cifras de 7 9999 son 857143.

Solución. Las claves de este problema son demostrar que


7  79999  710000  1 mod 10 6  
y observar que
6000001 6 10 6
857143 
7

7
 7  857143  1 mod 10 6  

 
Demostrar que 710000  1 mod 10 6 se puede hacer "por fuerza bruta", o mediante el
Binomio de Newton: 7 4  2401  24  10 2  1 , luego
 
710000  7 4
2500

 24  10 2  1 
2500

 2500   2500   2500 
 1    24  10 2   
 24  10 2
  
2

3
 24  10 2  ... 
 1   2   3 
 2500  3
2

 1  2500  24  10 2  1250  2499 24  10 2   
 24  10 6  ...
 3 
 1  3  2  5  2  3  5  7  17  10  ...
7 6 7 3 4 2 4

 1  3  2  10 6  2 5  33  5 2  7 2  17  10 6  ...
  
 1  3  2  2 5  33  5 2  7 2  17  ...  10 6  710000  1 mod 10 6 
6000001 6  10 6
Por otro lado, observamos que 857143  
7 7
Con lo cual:
710000  1 mod 10 6   710000  k  10 6  1  k  10 6  6  10 6  6  10 6  1 
 (k  6)  10 6  6  10 6  1

 
(k  6)  10 6  710000  6  10 6  1 ambos múltiples de 7, luego k  6 será también
múltiplo de 7, es decir: 710000  7 k '10 6  6  10 6  1

Y por tanto, finalmente:


710000 7k '10 6  6  10 6  1
7 9999 
  k '10 6  857143
7 7
tal y como queríamos ver.

Fuente: Jerónimo Vega Guillén en Facebook.

Observamos que

7  7 9999  1 mod 10 6
 
 7 9999  857143 mod 10 6  
6 

7  857143  1 mod 10  
Es consecuencia del teorema anterior, puesto que 7 ,10 6  1  
Corolario. Lema de Gauss.
Si a, b, c son enteros tales que a | bc y (a, b)  1 , entonces a | c .

Demostración. (a, b)  1 , luego existirá un x tal que bx  1 (mod a ) . Pero por otro lado,
a | bc  0  bc  xbc  cbx  c1  c (mod a)  a | c .

Corolario.
a) (a, b1 )  (a, b2 )  ...  (a, bn )  1   a , b1  b2  ...  bn   1
 
b) En particular, (a, b)  1  a , bk  1 para todo k  1 .

Demostración. a) (a, b1 )  (a, b2 )  ...  (a, bn )  1, luego existirán x1 ,..., xn tales que
xibi  1 (mod a) . Multiplicando tenemos que x1x2 ...xn b1b2 ...bn   1  1  ...  1  1 (mod a) ,
y por tanto  a , b1  b2  ...  bn   1 por el recíproco del Teorema del inverso modular.
b) Basta tomar en el apartado anterior b1  b2  ...  bk  b

Corolario.
a n | bn  a | b

Demostración. Si a  0 o b  0 está claro que a | b . Supongamos que a, b  0 .


Sea d  (a, b)  a  du , b  dv con (u , v)  1 .
Luego a n | b n  du  | dv  d nu n | d nv n  u n | v n  u | v n
n n

Por el corolario anterior, (u, v)  1  (u, v n )  1 , y por tanto u  1 , y en consecuencia


a  d , y por lo tanto divide a b  dv .

Corolario.
Si a | c y b | c con (a, b)  1 , entonces ab | c

Demostración. a | c  c  a d para cierto d .


b | ad 
  b | d por el Lema de Gauss, y por tanto a b | a d  c , como queríamos ver.
(a, b)  1

Teorema.
Sean a, b enteros positivos coprimos, tales que su producto es una potencia de grado n ,
es decir, ab  c n para cierto entero positivo c . Entonces a y b también son ambos
potencias de grado n .

Demostración. Sea d  (a, c) , y escribimos a  du y c  dv para ciertos u, v coprimos.


a  du 
n
 dub  (dv) n  d n v n  ub  d n 1v n (*)
ab  c 

De la igualdad anterior se desprende que u | d n 1v n , pero por hipótesis,


u
(u, v)  1  (u, v n )  1 , y por tanto u | d n 1 . Así pues, v n  n 1 b , es decir, b | v n .
d
Pero de la igualdad (*) también se deduce que v n | ub , y puesto que (u, v n )  1 ,
llegamos a v n | b , y finalmente:
b | v n 
b v
n
n
v | b 
Sustituyendo en (*) se deduce u  d n 1  a  d n , con lo que se concluye la
demostración.

Problema resuelto.
Demuestra que el producto de tres números consecutivos nunca puede ser una potencia
perfecta.

Solución. Supongamos que (n  1)n(n  1)  a k , para ciertos enteros a, k  1 .


Entonces podemos escribir n(n 2  1)  a k . Puesto que (n, n 2  1)  1 podemos aplicar el
teorema anterior: n  bk , n2  1  c k para ciertos enteros b, c  1 .
  2
  k
 
n2  1  ck  1  n2  ck  bk  ck  b2  ck  b2  c b2( k 1)  ...  ck 1 
Lo cual es imposible pues b 2  c  1 y b 2 ( k 1)  ...  c k 1  k  1 .
8.4 Inversión mediante exponenciación modular.

Determinación de inversos multiplicativos mediante exponenciación modular.


En 13.1 introduciremos El Pequeño Teorema de Fermat (PTF):

si p es primo y p | a , entonces a p 1  1 mod p 

Luego

a  a p 2  a p 1  1 mod p   a 1  a p2 mod p 

Y para determinar a p  2 podemos aprovechar la exponenciación modular "pow",


implementada en Python a partir de su versión 3.8:

pow(a,p-2,p)
8.5 Cancelación modular.
Trabajando en la aritmética convencional con enteros, sabemos que siempre podemos
tachar factores:

ab  ac  b  c

Pero no ocurre lo mismo en aritmética modular. Por ejemplo:

10  5  10 11 mod 12 pero 5  11 mod 12

Aquí no podemos tachar (cancelar) el factor 10 repetido.

Lema. Cancelación de términos en aritmética modular.


Si (a, n)  1 , entonces a r  a s mod n  r  s mod n .

Demostración.
Si (a, n)  1 entonces existirá un elemento b tal que ab  1 mod n , y por tanto:
a r  a s mod n  ba r   ba s   mod n

Ahora, aplicando las propiedades asociativa y conmutativa de la multiplicación


modular, por un lado, ba s   (ab)s  1s  s mod n , y por otro
ba r   (ba)r  (ab)r  1r  r mod n , con lo que llegamos a r  s mod n .
8.6 División modular.

División modular.
La división de dos números a  b , con b  0 no se considera propiamente una
operación, como la suma o el producto, sino que se interpreta como una forma rápida de
escribir el producto de a por el inverso de b :

a  b  a  b 1

Ejemplo.
Supongamos que queremos calcular 3  2 mod 7

Observamos la tabla de la multiplicación en Z 7 :

El inverso multiplicativo de 2 es 4, luego

3  2  3  2 1  3  4  12  5 mod 7
8.7 Divisores de cero.

Teorema. Definición y caracterización de los divisores de cero.


Dado un elemento x de Z n , diremos que es un divisor de cero cuando exista un
elemento y  0 de Z n tal que x  y  0 . Todo elemento x de Z n es un divisor de cero si
y solo si  x , n   1, es decir, cuando no sea invertible.

Demostración.
Supongamos que d   x , n   1. Luego x
 n  x   0 mod n
n x n
d d d
n n n
Y se cumple  0 pues d  1   n y por lo tanto no puede ser un múltiplo de
d d d
n.

Ejemplo.
En Z15 las clases invertibles son 1, 2, 4, 6, 7, 8, 11, 13 y 14 y las clases divisores de
cero son las restantes: 3, 5, 6, 9, 10 y 12.
Por ejemplo: 10  3  0 mod 15 .
8.8 Actividades con Python.
Inverso multiplicativo con Python.
Podemos utilizar la función egcd(a,b) definida anteriormente para construir la función
modinv(a,m):

def egcd(a, b):


if a == 0:
return (b, 0, 1)
else:
g, y, x = egcd(b % a, a)
return (g, x - (b // a) * y, y)

def modinv(a, m):


g, x, y = egcd(a, m)
if g != 1:
raise Exception('No existe inverso modular')
else:
return x % m

División modular en Python.

def egcd(a, b):


if a == 0:
return (b, 0, 1)
else:
g, y, x = egcd(b % a, a)
return (g, x - (b // a) * y, y)

def modinv(a, m):


g, x, y = egcd(a, m)
if g != 1:
raise Exception('No existe inverso modular')
else:
return x % m

def moddiv(a,b,m):
invb=modinv(b, m)
return (a*invb) % m

print(moddiv(3,2,7))
9 Congruencias lineales y sistemas de congruencias lineales.
Llamaremos congruencias a las ecuaciones que se plantean en términos modulares.
Como es habitual, diremos que una congruencia es lineal cuando sus incógnitas estén
elevadas únicamente a la primera potencia.

9.1 Congruencias lineales.


9.1.1
Resuelve las siguientes congruencias lineales:
a) x  4  0 mod 5 b) x  1 1 mod 5 c) x  3  1 mod 5 d) x  12  3 mod 5

Congruencia lineal.
Toda congruencia lineal se puede reducir a una congruencia de la forma a x  b (mod n)
Diremos que el entero x0 satisface la congruencia lineal a x  b (mod n) cuando
a x0  b (mod n)

O equivalentemente: a x0  b (mod n)  a x0  ny0  b  a x0  n y0  b

Es decir, buscamos soluciones x0 , y0  de la ecuación lineal diofántica


a x0  n y0  b

Ejercicio resuelto.
Suponiendo que 5x  6 mod 8 , determina x.

Solución: 5x  6 mod 8  x  5 16 mod 8

Determinamos 5 1 mod 8 por el método del EGCD.

8  5  3 , 5  3  2 , 3  2 1

Luego
1  3  2  3  (5  3)  3  5  3  2  3  5  28  5  5  2  8  2  5  5 
 2 8  35
1  2  8  3  5  1  3  5 mod 8  5 1  3 mod 8
x  5 16  (3)  6  18  2  6 mod 8

9.1.2
Resuelve las siguientes congruencias lineales:
a) 3x  1 mod 5 b) 3x  2 mod 5 c) 2 x  3 mod 5 d) 12x  4 mod 5
e) 2x  4  2 mod 5
9.1.3 Teorema. Existencia de soluciones de una congruencia lineal.
La congruencia lineal a x  b (mod n) tiene solución si y solo si d | b , donde
d  (a, n) , en cuyo caso existen d soluciones diferentes en Z n , todas ellas de la forma

n n n n
xo , x1  x0  , x2  x0   2 , x3  x0   3 , ... , xd 1  x0   (d  1)
d d d d

Donde x0 es una solución particular de la ecuación.

Ejemplo.
Resuelve la congruencia 18 x  30 (mod 42 )

Solución. d  (18,42 )  6 , y 6 | 30 , luego la ecuación anterior tiene seis soluciones.


Por tanteo, vemos que 18  4  72  42 1  30  4 es una solución de la ecuación.
Luego las soluciones serán:
x0  4
x1  4  (42 / 6) 1  4  7  11, efectivamente: 18  11  198  4  42  30
x2  4  (46 / 6)  2  4  14  18 , efectivamente: 18 18  324  7  42  30
x3  4  (46 / 6)  3  4  21  25 , efectivamente: 18  25  450  10  42  30
x4  4  (46 / 6)  4  4  28  32 , efectivamente: 18  32  576  13  42  30
x5  4  (46 / 6)  5  4  35  39 , efectivamente: 18  39  630  16  42  30

Las soluciones son: 4 ,11,18, 25, 32, 39 

9.1.4 F
Resuelve la congruencia 9 x  21 (mod 30 )

9.1.5 F
Resuelve la congruencia lineal 3x  7 (mod 10 )
Propiedad. Simplificación de congruencias lineales.

k a k b mod k n  a  b mod n

Problema resuelto.
Resuelve la congruencia
39x  52 mod 130

Solución. Esta congruencia tiene solución pues d   39,130   13 y 13 | 39 .


Simplificamos la congruencia dividiendo por 13:
39x  52 mod 130  3x  4 mod 10
El inverso multiplicativo módulo 10 de 3 es 7, luego
3x  4 mod 10  7  3x  7  4  28  8 mod 10  x  8 mod 10

Luego las soluciones de 39x  52 mod 130 son todos los conjuntos de la forma
8  130k  ...,  122, 8,138, 268, ... 
18  130k  ...,  112,18, 438, 278, ... 
...
128  130k  ...,  132,2 ,128, 268, ... 

Problema resuelto.
Resuelve la congruencia
140x  56 mod 252

Solución. Esta congruencia tiene solución pues d  140,252  28 y 28 | 56 .


Vemos que se puede simplificar pues todos los números involucrados son múltiples de
28:
140x  56 mod 252  5x  2 mod 9

Para resolver esta última congruencia vemos que el inverso de 5 módulo 9 es 2:


5  2  10  1 mod 9

Luego
5x  2 mod 9  x  51  2  2  2  4 mod 9

Las 28 soluciones módulo 252 son, por tanto:

4 , 4  9 , 4  18 , 4  27 , 4  36 , ...
9.2 Sistemas de congruencias lineales (resolución directa).
Queremos resolver un sistema de congruencias lineales:

a1 x  b1 mod m1 
a x  b mod m 
 2 2 2

...
ar x  br mod mr 

En donde vamos a suponer que los módulos mi son todos coprimos entre ellos.

9.2.1 Problema solucionado paso a paso en vídeo

El residuo de la división de 2021 entre 6, entre 7, entre 8 y entre 9 es 5 en los cuatro


casos. ¿Cuántos enteros positivos menores de 2021 hay que tengan esta propiedad?

(A) 1 (B) 3 (C) 2 (D) 4 (E) Cap

Cangur B1 2021 #15, Kangaroo Junior 2021 #14

Solución: [Link]

9.2.2 Problema solucionado paso a paso en vídeo

El número N tiene dos dígitos. Cuando N se divide entre 9, el residuo es 1. Cuando N se


divide entre 10, el residuo es 3. Determina el residuo cuando N se divide entre 11.

(A) 0 (B) 2 (C) 4 (D) 5 (E) 7

AMC 8 2016 #5

Solución: [Link]

9.2.3 Problema solucionado paso a paso en vídeo

Determina la cantidad de enteros positivos de tres dígitos con la siguiente propiedad:


Cuando se divide entre 6, da residuo 2, cuando se divide entre 9, da residuo 5, y cuando
se divide entre 11 da residuo 7.

(A) 1 (B) 2 (C) 3 (D) 4 (E) 5

AMC 8 2018 #21

Solución: [Link]
9.2.4 Ejemplo.
Resuelve el sistema
 x  0 mod 2

 x  0 mod 5

Solución. De la primera congruencia deducimos que x será un múltiple de 2, y de la


segunda que x será un múltiplo de 5, luego x será un múltiplo de 10:
x  0 mod 10

Visualmente:

9.2.5 Ejemplo.
Resuelve el sistema
 x  1 mod 3

 x  0 mod 7 

Solución. De la segunda congruencia deducimos que x debe ser un múltiplo de 7:


7, 14, 21, 28, 35, 42, 49, 56, 63, 70...
De la primera congruencia deducimos que x debe ser de la forma 3a  1 . Los valores de
la lista anterior que cumplen esta propiedad son:
7, 28, 49, 70...
Vemos que estos números son todos de la forma x  21b  7 , y que 21   3 , 7 . Así
pues, parece ser que las soluciones son:
x  7 mod 21 .

Demostraremos ahora que, efectivamente, x  7 mod 21 son soluciones del sistema.
x  21b  7  3  7 x  2  3  1  1 mod 3 , y por lo tanto satisface la primera congruencia
del sistema.
x  21b  7  3  7 x  1 7  0 mod 7 , y por lo tanto también satisface la segunda
congruencia del sistema.

Visualmente:

9.2.6 Ejemplo.
Resuelve el sistema
 x  3 mod 4

 x  2 mod 7 

Solución.
 x  3 mod 4  x  4a  3
  4a  3  7b  2  4a  1  7b  4a  1  0 mod 7 
 x  2 mod 7   x  7b  2
4a  1  0 mod 7  4a  1 mod 7  4a  6 mod 7
Esta última congruencia la podemos resolver con los métodos del apartado anterior: El
inverso multiplicativo de 4 módulo 7 es 2, luego:
a  2  6  12  5 mod 7  a  7c  5
Sustituyendo más arriba tenemos:
x  4a  3  47c  5  3  28c  20  3  28c  23  x  23 mod 28
Visualmente:

9.2.7 Ejemplo.
 x  3 (mod 4)

Resuelve el sistema  x  1 (mod 5)
 x  2 (mod 3)

Solución.
(Más adelante, mediante el Teorema chino del residuo, se verá que la solución existe y
es única mod 60, pues (4,5)  (5,3)  (4,3) )
x  3 (mod 4)  x  4a  3 
x  1 (mod 5)  4a  3  1 (mod 5)  4a  2 (mod 5)  2  5 (mod 5)  3 (mod 5)
4a  3 (mod 5)  4  4a  4  3 (mod 5)  16 a  12 (mod 5)  a  2 (mod 5) 
a  5b  2  x  4a  3  45b  2   3  20b  11
Luego:
x  2 (mod 3)  20b  11  2 (mod 3)  20b  9 (mod 3)  0 (mod 3)
 b  0 (mod 3)  b  3c
Finalmente:
x  20b  11  20  3c  11  11  60c (mod 60 ) . En efecto:
11  4  2  3 , 11  5  2  1 , 11  3  3  2

Visualmente:

9.2.8 Ejemplo resuelto.


Resuelve el sistema
x  3 mod 5

x  4 mod 11
Solución.
x  3 mod 5  x  5k  3 
  5k  3  11q  4  5k  11q  1
x  4 mod 11  x  11q  4
Mediante el EGCD o probando vemos que 5(2)  11(1)  1 , luego
x  5(2)  3  7 
  x  7  48 mod 55 
x  11(1)  4  7
9.3 Sistemas de congruencias lineales con módulos coprimos.

9.3.1 Teorema. Teorema chino del residuo. (Primera versión, con coeficientes = 1 )
Sean m1 , m2 ,..., mr enteros positivos tales que mi , m j   1 si i  j . Entonces el sistema
de congruencias lineales
 x  b1 mod m1 
 x  b mod m 
 2 2

...
 x  br mod mr 

Tiene una única solución x módulo el entero m1  m2  ...  mr .

y se obtiene siguiendo los siguientes pasos:


Paso 1: Sea N  m1  m2  ...  mr ,
Paso 2: Sean N1  N / m1 , N2  N / m2 , ... , Nr  N / mr .
Paso 3: Resolver las congruencias lineales:
N1 y1  1 mod m1  , N2 y2  1 mod m2  , ... , Nr yr  1 mod mr 

Paso 4: x  N1 y1 b1  N2 y2 b2  ...  Nr yr br (mod N ) es la única solución del


sistema.

Demostración.
Sea N  m1  m2  ...  mr , y sean N1  N / m1 , N2  N / m2 , ... , Nr  N / mr .
Es decir, N k  m1  m2  ...  mk 1  mk 1  ...  mr , y puesto que mi , m j   1 si i  j , está claro
que N k , mk   1, y por tanto, aplicando el Teorema 8.2.1, cada N k tendrá un inverso
multiplicativo módulo mk , al que llamaremos yk .
Sea el entero x  N1 y1 b1  N2 y2 b2  ...  Nr yr br , y vamos a demostrar que este entero
satisface las condiciones del enunciado.
Puesto que N k  m1  m2  ...  mk 1  mk 1  ...  mr , está claro que Ni  0 mod m j  si i  j , y
por tanto x  N k yk bk  bk mod mk  , para todo k, es decir, este número satisface todas
las congruencias del enunciado.
Veamos que esta solución es única módulo N.
Supongamos que existe otra solución y módulo N. Para cualquier valor de k, 1  k  r ,
x  y mod N   N | x  y  m1  m2  ...  mr | x  y 
  mk | x  y  x  y mod mk 
mk | N 
Y por tanto y  x  bk mod mk  , es decir, y también satisface el sistema de
congruencias.
Veamos ahora que todas las soluciones posibles son de esta forma. Supongamos que dos
enteros x e y son soluciones del sistema de congruencias, entonces, para todo k,
1 k  r ,
y  x  bk mod mk   x  y  0 mod mk   mk | x  y
Y, puesto que los mk son coprimos dos a dos, tendremos que
N  m1  m2  ...  mr | x  y  x  y mod N 
9.3.2 Ejemplo. El problema de Sun-Tsu.
El Teorema chino del residuo debe su nombre en honor al siguiente problema del siglo I
DC: Determina un número cuyos residuos son 2, 3 y 2 al dividirlo entre 3, 5 y 7,
respectivamente.

Nota: Este mismo problema aparece en las Introductio Arithmeticae del matemático
griego Nicómano de Gerasa, alrededor del 100 DC.

Solución. Se trata de resolver el sistema de congruencias lineales


 x  2 (mod 3)

 x  3 (mod 5)
 x  2 (mod 7)

Paso 1: N  3  5  7  105
105 105 105
Paso 2: N1   35 , N2   21 , N3   15
3 5 7
Paso 3: Las congruencias lineales 35 y1  1 (mod 3) , 21y2  1 (mod 5) y 15 y3  1 (mod 7)
tienen soluciones y1  2 , y2  1 y y3  1 .

Paso 4: x  35  2  2  21 1 3  15 1 2  233 será solución del sistema módulo 105, y
233  23 (mod 105 )

Y por tanto 23 es la única solución del sistema (módulo 105).

En efecto: 23  7  3  2 , 23  4  5  3 , 23  3  7  2

9.3.3 Ejemplo resuelto.


Resolver el sistema de congruencias
 x  2 (mod 4)

 x  7 (mod 9)

Solución.
Claramente ( 4,9)  1 y por tanto el sistema tiene solución.
Paso 1: N  4  9  36
36 36
Paso 2: N1   9 , N2   4.
4 9
Paso 3: Resolvemos las ecuaciones 9 y1  1 (mod 4) y 4 y2  1 (mod 9)
Puesto que 9 1  2  4  1 , y 4  7  28  3  9  1 , tenemos que y1  1 , y2  7 son
soluciones.

Paso 4: La solución es x  9 1  2  4  7  7  214 (mod 36) , es decir, 34.

Efectivamente, 34  4  8  2 , y 34  9  3  7 ,
9.3.4 F Problema solucionado paso a paso en vídeo.

Determina el número de parejas ordenadas ( x, y ) de enteros positivos, con y  x  100


tales que
x x 1
,
y y 1
sean enteros.

AIME 1995 #8

Solución: [Link]

9.3.5 Teorema. Teorema chino del residuo (Versión general).


Sean m1 , m2 ,..., mr enteros positivos tales que mi , m j   1 si i  j .
Sean b1 , b2 , ..., br enteros arbitrarios y a1 , a2 ,..., ar enteros tales que ak , mk   1 para todo
1 k  r .

Entonces el sistema de congruencias lineales


a1 x  b1 mod m1 
a x  b mod m 
 2 2 2

...
ar x  br mod mr 

Tiene una única solución x módulo el entero m1  m2  ...  mr .

En efecto, puesto que ak , mk   1 , existirán los inversos multiplicativos de ak , y


podemos reescribir el sistema como
 x  a11b1 mod m1 

 x  a2 b2 mod m2 
1


...
 x  a 1b mod m 
 r r r

Y aplicar el Teorema Chino del residuo en su versión del Teorema 9.3.1.


9.4 Sistemas de congruencias lineales con módulos no coprimos.
Se pueden resolver algunos sistemas de congruencias cuando sus módulos no son coprimos,
pero no siempre, como veremos en el siguiente ejemplo.

9.4.1 Ejemplo.
Resolver el sistema de congruencias
 x  1 (mod 10 )

 x  4 (mod 12 )

Solución.
 x  1 (mod 10 )  x  10 a  1
  10 a  1  12b  4  10 a  12b  3
 x  4 (mod 12 )  x  12b  4

Esta última igualdad nos lleva a contradicción, pues el valor de la derecha es impar y el
valor de la izquierda es par. Así pues, el sistema no tiene solución.

Visualmente:

9.4.2 Teorema. Teorema chino del residuo con módulos no coprimos y dos congruencias.
El sistema
 x  a1 (mod m1 )

 x  a2 (mod m2 )

Tiene solución si y solo si a1  a2 mod m1, m2  , en cuyo caso existe una única solución
mod a1, a2  .

Observamos que el Teorema chino del residuo sería un caso particular de este teorema
cuando m1, m2   1 , pues entonces garantizamos que el sistema tenga solución, y
a1, a2   m1 m2 .
9.4.3 Ejemplo.
Resolver el sistema
 x  5 (mod 12 )

 x  11 (mod 18)

Solución.
Puesto que (12 ,18 )  6 , y 6 | (11 - 5) , existirá una única solución. La vamos a obtener
con el método interactivo.

x  5 (mod 12 )  x  12 a  5
x  11 (mod 18)  12 a  5  11 (mod 18)  12 a  11  5 (mod 18)  12 a  6 (mod 18)

Esta última congruencia se puede simplificar: 6 divide a 12 y a 6, y además


mcd (6,18 )  6 , luego podemos simplificarla:

12 a  6 (mod 18)  2a  1 (mod 3)  a  2 (mod 3)  a  3k  2


x  12 a  5  12 3k  2   5  36 k  29 mod 36 

Donde hemos tenido en cuenta que 12,18  36

Efectivamente, 29  12  2  5 , y 29  18 1  11

Visualmente:

9.4.4 M
Diremos que un entero positivo n es extra-distinct si sus residuos cuando lo dividimos
entre 2, 3, 4, 5 y 6 son distintos. Determina el número de enteros positivos extra-distinct
menores que 1000.

AIME I 2023 #7

9.4.5 Teorema (versión general de 9.4.2)


Sean m1 , m2 ,..., mr enteros positivos. Entonces el sistema de congruencias lineales
 x  b1 mod m1 
 x  b mod m 
 2 2

...
 x  br mod mr 

Tiene solución si y solo si bi  b j mod  mi , m j  para todo i  j , en cuyo caso la solución


será única m1 , m2 , ..., mr .
9.5 Congruencias lineales mediante sistemas de congruencias lineales.
El Teorema chino del residuo se puede usar para convertir una congruencia lineal cuyo
módulo es un número compuesto grande en un sistema de varias congruencias con
módulos más pequeños, con los que resultará más sencillo operar.

Nos basaremos en el siguiente resultado:

a x  b (mod n)
a x  b (mod n m)  
a x  b (mod m)

En efecto: a x  b (mod n m)  a x  k nm  b  km n  b  a x  b (mod n)

Existe un recíproco: a x  b (mod n) y a x  b (mod m)  a x  b mod n, m 

9.5.1 Ejemplo.
Resolver la congruencia lineal 17 x  9 (mod 276 ) .

Solución. Puesto que 276  4  3  23 , la ecuación anterior es equivalente a resolver el


sistema de congruencias
17 x  9 (mod 3)  x  0 (mod 3)
 
17 x  9 (mod 4) o equivalentemente:  x  1 (mod 4)
17 x  9 (mod 23) 17 x  9 (mod 23)
 

x  0 (mod 3) equivale a decir que x  3a , luego sustituyendo en la segunda ecuación y


multiplicando por 3 ambos lados:
3a  1 (mod 4)  9a  3 (mod 4)  a  3 (mod 4)  a  4b  3  x  3a  12b  9

Sustituyendo en la tercera ecuación:


17 x  9 (mod 23)  17 12b  9  9 (mod 23)  204 b  153  9 (mod 23)
 204 b  144 (mod 23)  20b  17 (mod 23)  3b  6 (mod 23)
 3b  6 (mod 23)  b  2 (mod 23)  b  23k  2
 x  12 (23k  2)  9  276 k  24  9  276 k  33  x  33 (mod 276 )

Efectivamente, 17  33  561  2  276  9 .


9.6 Congruencias y sistemas de congruencias lineales con varias variables.

9.6.1 Ejemplo resuelto.


Determina todas las soluciones de 3x  7 y  11 mod 13
Solución.
3x  7 y  11 mod 13  3x  7 y  11 mod 13
En primer lugar, observamos que, fijado un 7 y  11 , la congruencia tiene solución en x
y es única, puesto que 3,13  1 y 1 | 7y  11 para cualquier valor de y (ver Teorema
9.1.3).

Vamos a determinar las soluciones en función del valor de y:


y  0  3x  11 mod 13  x  31  11  4  11  44  5  8 mod 13
En donde hemos utilizado que 31  4 mod 13 , puesto que 13  3  4  1 .
y  1  3x  7  1  11  18 mod 13  x  31  18  4  18  6 mod 13
Y de la misma manera vamos obteniendo el resto de soluciones posibles:
y  2  x  4 mod 13 , y  3  x  2 mod 13
y  4  x  0 mod 13 , y  5  x  11 mod 13
y  6  x  9 mod 13 , y  7  x  7 mod 13
y  8  x  5 mod 13 , y  9  x  3 mod 13
y  10  x  1 mod 13 , y  11  x  12 mod 13
y  12  x  10 mod 13

9.6.2
Sea N el mayor entero de cuatro dígitos con la propiedad de que si cualquiera de sus
dígitos se reemplaza por un 1, el resultado es divisible entre 7. Sean Q y R el cociente y
residuo, respectivamente, de N dividido entre 1000. Determina Q+R.
AIME II 2024 #7
10 Congruencias cuadráticas. Residuos cuadráticos.
10.1 Congruencias cuadráticas con módulos primos.
10.1.1 Definición. Congruencias cuadráticas módulo n.
Una congruencia cuadrática es una congruencia de la forma

x 2  a mod n

donde ( a , n )  1 . Si la congruencia tiene solución, diremos que a es un residuo


cuadrático módulo n.

En general, llamaremos congruencia cuadrática a la congruencia

Ax 2  Bx  C  0 mod n , con ( A, n)  1

Este primer apartado lo dedicaremos a estudiar las congruencias cuadráticas cuando el


módulo es un número primo. El método general para resolver estas congruencias es
completar cuadrados, tal y como veremos en los siguientes ejemplos resueltos.

10.1.2 Ejemplo resuelto.


Resuelve la congruencia cuadrática x 2  6 x  5  0 mod 17 

En primer lugar completamos el cuadrado, y necesitamos sumar 32  9 .


x 2  6 x  5  0 mod 17  
x 2  2  3 x  32  5  32 mod 17  
x  32  5  9 mod 17  
x  32  4 mod 17 

Tenemos que resolver la congruencia u 2  4 mod 17  , por tanteo:


u  2 mod 17  (a)
u 2  4 mod 17   
u  2  15 mod 17  (b)

(a) x  3  2 mod 17  x  2  3  1  16 mod 17


(b) x  3  2 mod 17  x  2  3  5  12 mod 17
10.1.3 Ejemplo resuelto.
Resuelve la congruencia cuadrática x 2  3x  11  0 mod 13

Completamos cuadrados:
x 2  3 x  11  0 mod 13  
4 x 2  12 x  44  0 mod 13  
(2 x) 2  2  6 x  44  0 mod 13  
(2 x) 2  2  3  2 x  32  44  32 mod 13  
2 x  32  44  32 mod 13  
2 x  3  2 mod 13 
2 x  32  35  4 mod 13   
(a )
2 x  3  2 mod 13  (b)

(a) 2x  3  2 mod 13  2x  1 mod 13  x  6 mod 13


(b) 2x  3  2 mod 13  2x  5 mod 13  x  4 mod 13

10.1.4 Ejemplo resuelto.


Resuelve la congruencia cuadrática 2 x 2  7 x  4  0 mod 19 

Utilizando que 7  12 mod 19 :


2 x 2  7 x  4  0 mod 19  
2 x 2  12 x  4  0 mod 19  
4 x 2  24 x  8  0 mod 19  
(2 x) 2  2  2 x  6  6 2  6 2  8  0 mod 19  
2 x  62  6 2  8  0 mod 19  
2 x  62  6 2  8  0 mod 19  
2 x  62  28  0 mod 19  
2 x  6  3 mod 19  (a )
2 x  62  28  9 mod 19   
2 x  6  3 mod 19  (b)

(a) 2x  6  3 mod 19  2x  9 mod 19  x  14 mod 19


(b) 2x  6  3 mod 19  2x  3 mod 19  x  11 mod 19
10.1.5 Ejercicio resuelto.
Resolver la congruencia
x 2  3x  5  0 mod 7

Solución.
Siguiendo los pasos anteriores llegamos a la congruencia equivalente
2 x  32  1 mod 7 
Y vemos que 1 es un residuo cuadrático módulo 7. En efecto: 12  1 mod 7  y
62  36  1 mod 7  . Luego nuestro problema se reduce a resolver las siguientes
congruencias lineales:
2x  3  1 mod 7 y 2x  3  6 mod 7

2x  3  1 mod 7  2x  2 mod 7  x  1  6 mod 7


2 x  3  6 mod 7   2 x  3 mod 7   x  21  3 mod 7  
x  4  3 mod 7   x  12 mod 7   x  5 mod 7 
En donde hemos utilizado que 2  4  8  1 mod 7  21  4 mod 7

Fuente: “Michael Penn Math” [Link]

10.1.6 Ejercicio resuelto.


Resuelve 5x 2  6 x  2  0 mod 13

Solución.
Siguiendo la transformación anterior, esta congruencia es equivalente a
10 x  62  9 mod 13  y 2  9 mod 13 , tomando y  10 x  6 .
Vemos que 9 es un residuo cuadrático módulo 13, en efecto:
 y  3 mod 13 
y 2  9 mod 13   
 y  13  3  10 mod 13 
Ahora deshacemos el cambio de variable:
a) y  3 mod 13  10x  6  3 mod 13  10x  9 mod 13  (1)
Calculamos el inverso multiplicativo de 10 módulo 13 mediante el ADE (ver Apartado
8.3):
13  10  1  3  3  13  10 
  1  10  3  3  10  313  10  
10  3  3  1  1  10  3  3
 10  3  13  3  10  4  10  3  13  10 1  4 mod 13 

Y por tanto:
(1)  x  4  9  36  10 mod 13

b)
y  10 mod 13   10 x  6  10 mod 13   10 x  16  3 mod 13  
 x  4  3  12 mod 13 

Así pues, las soluciones son x  10 , 12 mod 13


10.1.7 D Problema solucionado paso a paso en vídeo

Sea un número primo positivo dado. Demostrar que existe un entero α tal que
  1  3 es divisible por p si y sólo si existe un entero β tal que    1  25 es
divisible por p.

OME 2016 #2

Solución: [Link]
10.2 Congruencias cuadráticas con módulos compuestos.

10.2.1 Ejemplo resuelto.


Resuelve la congruencia x2  26 mod 55 

Solución.
Observamos que 55  5 11 .
Resolvemos las congruencias x 2  26 mod 5 y x 2  26 mod 11 por tanteo:
 x  1 mod 5
x 2  26 mod 5  x 2  1 mod 5  
 x  4 mod 5
 x  2 mod 11
x 2  26 mod 11  x 2  4 mod 11  
 x  9 mod 11

Así pues, tomando todas las parejas de soluciones posibles, tenemos cuatro sistemas de
congruencias lineales, que resolveremos directamente, sin usar el TCR:

a)
 x  1 mod 5  x  5a  1 
   11b  2  5a  1  11b  1  5a  5a  1 mod 11
 x  2 mod 11  x  11b  2 
5a  1 mod 11  a  9 mod 11  a  11c  9 
x  5a  1  511c  9  1  55c  45  1  55c  46  x  46 mod 55 

b)
 x  1 mod 5  x  5a  1 
   11b  9  5a  1  11b  8  5a  5a  8 mod 11
 x  9 mod 11  x  11b  9
5a  8 mod 11  a  6 mod 11  a  11c  6 
x  5a  1  511c  6   1  55c  30  1  55c  31  x  31 mod 55 

c)
 x  4 mod 5  x  5a  4 
   11b  2  5a  4  11b  2  5a  5a  2 mod 11
 x  2 mod 11  x  11b  2
5a  2  9 mod 11  a  4 mod 11  a  11c  4 
x  5a  4  511c  4  4  55c  20  4  55c  24  x  24 mod 55 

d)
 x  4 mod 5  x  5a  4 
   11b  9  5a  4  11b  5  5a  5a  5 mod 11
 x  9 mod 11  x  11b  9
5a  5 mod 11  a  1 mod 11  a  11c  1 
x  5a  4  511c  1  4  55c  5  4  55c  9  x  9 mod 55 

Así pues, esta congruencia tiene cuatro soluciones: x  9, 24, 31, 46 mod 55
10.2.2 Ejemplo resuelto.
Resuelve la congruencia x 2  1 (mod 144 )

Solución.
Puesto que 144  16  9 , y (16 ,9)  1 , podemos descomponer la ecuación anterior en el
sistema no lineal

 x  1 (mod 16 )
2

 2

 x  1 (mod 9)
x 2  1 (mod 16) tiene 4 soluciones: x  1 o  7 (mod 16)
x 2  1 (mod 9) tiene 2 soluciones: x  1 (mod 9)
Luego tenemos ocho alternativas:
i) x  1 (mod 16 ) y x  1 (mod 9)
ii) x  1 (mod 16 ) y x  1 (mod 9)
iii) x  1 (mod 16 ) y x  1 (mod 9)
iv) x  1 (mod 16 ) y x  1 (mod 9)
v) x  7 (mod 16 ) y x  1 (mod 9)
vi) x  7 (mod 16 ) y x  1 (mod 9)
vii) x  7 (mod 16 ) y x  1 (mod 9)
viii) x  7 (mod 16 ) y x  1 (mod 9)

Podemos ir resolviendo cada caso mediante el TCR.


Independientemente del caso, (16 ,9)  1 , luego todos los ocho sistemas tienen solución.
Además: N1  9 , N2  16 , 9 y1  1 (mod16)  y1  9 , 16 y2  1 (mod 9)  y2  4
i) x  9  9  1  16  4  1  145 (mod 144 )  1 (mod 144 )
ii) x  9  9  1  16  4  (1)  17 (mod 144 )
iii) x  9  9  (1)  16  4  1  17 (mod 144 )
iv) x  9  9  (1)  16  4  (1)  145 (mod 144 )  1 (mod 144 )
v) x  9  9  7  16  4  1  631 (mod 144 )  55 (mod 144 )
vi) x  9  9  7  16  4  (1)  503 (mod 144 )  71 (mod 144 )
vii) x  9  9  (7)  16  4  1  503 (mod 144 )  73 (mod 144 )  71 (mod 144 )
viii) x  9  9  (7)  16  4  (1)  631 (mod 144 )  55 (mod 144 )

10.2.3 F
Calcula los tres últimos dígitos de 2005 11  2005 12  ...  2005 2006

Senior Hanoi Open MO 2006

10.2.4 F
Demostrar que si x es un número impar no divisible entre tres, entonces x 2  1 mod 24 .
10.2.5 Ejemplo resuelto.
Resuelva la congruencia x 2  4 (mod 77 )

Solución:


 x  4 (mod 7)  x  2 (mod 7)
2

x 2  4 (mod 77 )   2

 x  4 (mod 11)  x  2 (mod 11)

Luego tenemos cuatro casos:


a) x  2 (mod 7) , x  2 (mod 11)
b) x  2 (mod 7) , x  2 (mod 11)
c) x  2 (mod 7) , x  2 (mod 11)
d) x  2 (mod 7) , x  2 (mod 11)

Vamos a resolver estos casos mediante el TCR:

a)
N  77
N1  11  11 y1  1 (mod 7)  y1  2
N 2  7  7 y2  1 (mod 11)  y2  8
x  11  2  2  7  8  2  156  77  2  2  2 (mod 77 )
Y por tanto la primera solución es x  2 (mod 77 )

Para ahorrarnos trabajo, podemos deducir el resto de los casos directamente de la última
igualdad:

b) x  11  2  (2)  7  8  2  68 (mod 77 )
c) x  11  2  2  7  8  (2)  68  68  77  9 (mod 77 )
d) x  11  2  (2)  7  8  (2)  156  156  77  3  75 (mod 77 )

Así pues, las soluciones son x  2 , 9 , 68 , 75 (mod 77 )

Fuente: “PROFMAT MA14 12.2” en Youtube.


10.2.6 Ejemplo resuelto.
Resuelve la congruencia x 2  x  1  0 (mod 49)

Solución.

Puesto que 49  7 2 , en primer lugar resolveremos la congruencia


x 2  x  1  0 (mod 7) .

x 2  x  1  0 (mod 7) 
x 2  x  6  0 (mod 7) 
 x  3  0 (mod 7)  x  3  4 (mod 7) (a)
( x  3)( x  2)  0 (mod 7)  
 x  2  0 (mod 7)  x  2 (mod 7) (b)

(b) x  2 (mod 7)  x  7 k  2 . Ahora substituimos en la congruencia inicial:


x 2  x  1  0 (mod 49 ) 
(7k  2) 2  7k  2  1  0 (mod 49 ) 
49 k 2  35 k  7  0 (mod 49 ) 
35 k  7  0 (mod 49 )
En donde hemos aplicado que 49  0 (mod 49 ) .
35 k  7  0 (mod 49 ) 
75k  1  0 (mod 49 ) 
5k  1  0 (mod 7) 
5k  1  6 (mod 7) 
k  4 (mod 7)  k  7u  4
Luego x  7k  2  77u  4  2  49u  28  2  49u  30  x  30 (mod 49) .

(a) x  4 (mod 7)  x  7 k  4 . Ahora substituimos en la congruencia inicial:


x 2  x  1  0 (mod 49 ) 
(7k  4) 2  7k  4  1  0 (mod 49 ) 
49 k 2  63k  21  0 (mod 49 ) 
14 k  21  0 (mod 49 )
En donde hemos aplicado que 49  0 (mod 49 ) y 63  14 (mod 49 ) .
14 k  21  0 (mod 49 ) 
72k  3  0 (mod 49 ) 
2k  3  0 (mod 7) 
2k  3  4 (mod 7) 
k  2 (mod 7)  k  7v  2
Luego x  7k  4  77v  2  4  49v  18  x  18 (mod 49) .

Así pues, hay dos soluciones para la congruencia del enunciado: x  18 , 30 (mod 49 ) .
10.2.7 F
Determina el menor entero positivo m tal que m 2  7 m  89 sea un múltiplo de 77.

Mandelbrot 2009
10.3 Congruencias con potencias y polinomios.

Proposición. Congruencias con potencias.


Las propiedades estudiadas en el Tema 4 tienen unas aplicaciones muy importantes en
el estudio de las congruencias:

Dados dos enteros positivos d , k , con d | k , entonces:


a) a  b (mod n)  a k  b k (mod n)
b) a d  b d (mod n)  a k  b k (mod n)
c) a d  b d (mod n) y k / d es impar,  a k  b k (mod n)

Demostración. a) Basta aplicar a  b | a k  b k para todo k.


b) Basta aplicar d | k  a d  b d | a k  b k .
c) a d  b d (mod n)  a d  b d  0 (mod n)  n | a d  b d (*)
Pero si k / d es impar tenemos a d  b d | a k  b k , luego
(*)  n | a k  b k  a k  b k  0 (mod n)  a k  b k (mod n)

Problema resuelto.
Aprovechando que 641  2 4  54  5  2 7  1 , demostrar que 641 | 232  1 .

641  2 4  54  2 4  54  0 (mod 641)  54  2 4 (mod 641)


 54 2 28  2 4 2 28 (mod 641)  54 2 28  232 (mod 641)

 5  27 
4
 232 (mod 641)  641  1  232 (mod 641)
4

  1  232 (mod 641)  1  232 (mod 641)  232  1  0 (mod 641)


4

 641 | 232  1

Teorema.
Dado un polinomio con coeficientes enteros p( x)  cm x m  cm1 x  ...  c1 x  c0 ,
entonces:
a  b (mod n)  p (a )  p (b) (mod n)

Demostración. a  b (mod n)  a k  b k (mod n)  ck a k  ck b k (mod n) 


m m
p(a)   ck a k  p(b)   ck b k (mod n)
k 0 k 0

10.3.1 F
Dado cualquier número positivo n, y sea S la suma de sus cifras, demuestra:
a) n  S es divisible entre 9.
b) n es divisible entre 9 si y solo si S es divisible entre 9.

En este problema justificamos el “criterio de divisibilidad del nueve”: Un número es


divisible entre nueve si y solo si las suma de sus cifras es divisible entre 9.
10.3.2 F
Demuestra el “criterio de divisibilidad del once”: Un número es divisible entre 11 si y
solo si la suma alternada de sus cifras es múltiplo de 11.

Proposición.
Dado un polinomio con coeficientes enteros p( x)  cm x m  cm1 x  ...  c1 x  c0 , diremos
que a es una solución de la congruencia p( x)  0 (mod n) si p(a)  0 (mod n) .
Si a es una solución de la congruencia p( x)  0 (mod n) y b  a (mod n) entonces b
también es una solución de la congruencia p( x)  0 (mod n) .

Demostración. a  b (mod n)  0  p (a )  p(b) (mod n)  0  p (b) (mod n)


10.4 El Teorema de Wilson.

10.4.1 Lema.
Si p es un número primo, x 2  1 mod p   x  1 mod p  .

Demostración. x  1 mod p   x 2  1 mod p  por las propiedades básicas de la


aritmética modular.
Veamos el recíproco. Supongamos que x 2  1 mod p   p | x 2  1  ( x  1)( x  1) .
Luego, por ser p un número primo, se cumple p | x  1  x  1 mod p o se cumple
p | x  1  x  1 mod p , tal y como queríamos ver.

10.4.2 Teorema. Teorema de Wilson.


Si p es un número primo,  p  1 ! 1 mod p 

Demostración. Los casos p  2 y p  3 se pueden demostrar directamente.


Supongamos que p  2 .
Vemos que todo número a   2 , 3, 4 , ... , p  2  tiene inverso a 1 mod p  , y que se
cumple a  a 1 mod p  , porque en ese caso tendríamos, aplicando el lema anterior,
a 2  a  a 1  1 mod p   a  1 mod p 
llegando a contradicción.
Así pues, podemos agrupar los elementos de  2 , 3, 4 , ... , p  2  por parejas, siendo el
producto de cada pareja congruente con 1, en lenguaje modular:
2  3  ...  ( p  2)  1 mod p
Luego, finalmente, ( p  1)! 1 2  3...   p  2 p  1  11 ( p  1)  p  1  1 mod p
tal y como queríamos ver.

Observación: El recíproco también es cierto: Si n  1 ! 1 mod n, entonces n es


primo, lo que da un nuevo método para determinar si un número es primo o no, aunque
no es nada práctico, pues n  1 ! es enorme.

10.4.3 F
Determina el residuo al dividir 14 ! entre 17.

10.4.4 F
Determina el residuo al dividir 2016 !2015 ! entre 2017.

10.4.5 F
1 1 1 1 a
Sea a un entero tal que    ...  
1 2 3 23 23!

Determina el residuo que obtenemos al dividir a entre 13.

ARML 2002
10.4.6 D
Dado un primo impar p, demuestra que
12  32  52  ...  ( p  2) 2   1 mod p 
( p 1) / 2

10.4.7 M
Diremos que un entero x 1, ...,102  es square-ish si existe un entero n tal que
x  n2  n mod 103  . Calcula el producto de todos los enteros square-ish módulo 103.

SMT 2023 Discrete #6


10.5 Residuos cuadráticos. Ley de reciprocidad.

Definición. Residuo cuadrático modular.


Sea p un número primo. Diremos que un entero a no divisible entre p es un residuo
cuadrático módulo p cuando sea un cuadrado perfecto módulo p.

Por ejemplo, en ℤ5: 1² ≡ 1, 2² ≡ 4, 3² ≡ 4, 4² ≡ 1.

Luego 1 y 4 son residuos cuadráticos en ℤ5, mientras que 2 y 3 no lo son.

Residuos cuadráticos y representación simétrica.


Si en vez de utilizar la representación estándar:
ℤp = { 0, 1, 2, … , p-2, p-1 }

utilizamos la llamada representación simétrica


ℤp = { -(p-1)/2, …,-2, -1, 0, 1, 2, … , (p-1)/2 }

entonces veremos más claro como son los residuos cuadráticos. Por ejemplo, en ℤ7:

n -3 -2 -1 0 1 2 3
n2 2 -3 1 0 1 -3 2

Hay tres residuos cuadráticos: 1, 2 y -3.

Listado de los residuos cuadráticos asociados a los primeros números primos:


3→ 1
5 → 1,-1
7 → 1,2,-3
11 → 1,3,4,5,-2
13 → 1,3,4,-4,-3,-1
17 → 1,2,4,8,-8,-4,-2,-1
19 → 1,4,5,6,7,9,-8,-3,-2
23 → 1,2,3,4,6,8,9,-11,-10,-7,-5
29 → 1,4,5,6,7,9,13,-13,-9,-7,-6,-5,-4,-1
31 → 1,2,4,5,7,8,9,10,14,-15,-13,-12,-11,-6,-3
37 → 1,3,4,7,9,10,11,12,16,-16,-12,-11,-10,-9,-7,-4,-3,-1
41 → 1,2,4,5,8,9,10,16,18,20,-20,-18,-16,-10,-9,-8,-5,-4,-2,-1
43 → 1,4,6,9,10,11,13,14,15,16,17,21,-20,-19,-18,-12,-8,-7,-5,-3,-2
47 → 1,2,3,4,6,7,8,9,12,14,16,17,18,21,-23,-22,-20,-19,-15,-13,-11,-10,-5
53 → 1,4,6,7,9,10,11,13,15,16,17,24,25,-25,-24,-17,-16,-15,-13,-11,-10,-9,-7,-6,-4,-1
59 → 1,3,4,5,7,9,12,15,16,17,19,20,21,22,25,26,27,28,29,-24,-23,-18,-14,-13,-11,-10,-8,-6,-2
61 → 1,3,4,5,9,12,13,14,15,16,19,20,22,25,27,-27,-25,-22,-20,-19,-16,-15,-14,-13,-12,-9,-5,-4,-3,-1
67 → 1,4,6,9,10,14,15,16,17,19,21,22,23,24,25,26,29,33,-32,-31,-30,-28,-27,-20,-18,-13,-12,-11,-8,-7,-5,-3,-2
71 → 1,2,3,4,5,6,8,9,10,12,15,16,18,19,20,24,25,27,29,30,32,-35,-34,-33,-31,-28,-26,-23,-22,-21,-17,-14,-13,-11,-7
73 → 1,2,3,4,6,8,9,12,16,18,19,23,24,25,27,32,35,36,-36,-35,-32,-27,-25,-24,-23,-19,-18,-16,-12,-9,-8,-6,-4,-3,-2,-1
79 → 1,2,4,5,8,9,10,11,13,16,18,19,20,21,22,23,25,26,31,32,36,38,-39,-37,-35,-34,-33,-30,-29,-28,-27,-24,-17,-15,-14,-12,-7,-6,-3
83 → 1,3,4,7,9,10,11,12,16,17,21,23,25,26,27,28,29,30,31,33,36,37,38,40,41,-39,-35,-34,-32,-24,-22,-20,-19,-18,-15,-14,-13,-8,-6,
-5,-2
89 → 1,2,4,5,8,9,10,11,16,17,18,20,21,22,25,32,34,36,39,40,42,44,-44,-42,-40,-39,-36,-34,-32,-25,-22,-21,-20,-18,-17,-16,-11,-10,
-9,-8,-5,-4,-2,-1
Teorema.
Si p es un número primo impar (es decir, excluímos p  2 ), en ℤp se cumple:
a) n es el cuadrado de a si y solo si es el cuadrado de  a .
p 1 p 1
b) En hay exactamente residuos cuadráticos y valores que no lo son.
2 2

Demostración.
a)
 p | x  y  x  y mod p 
x 2  a mod p  
  p | x  y  ( x  y )( x  y )  o
2 2

y  a mod p 
2
 p | x  y  x   y mod p 

Definición. Símbolo de Legendre.


Sea p un número primo impar y sea a un entero. Definimos
0 si (a, p )  1
a 
    1 si a es un residuo cuadrático módulo p
 q   1 si a no es un residuo cuadrático módulo p

Siguiendo el ejemplo anterior,


1 2 3 4 5
   1 ,    1 ,     1 ,    1 ,    0
5 5 5 5 5

Ley de reciprocidad cuadrática de Gauss.


Sean p y q dos números primos impares. Se cumple
 p p 1 q 1  q 

   (1) 2 2  
q  p

 p
La ley de reciprocidad cuadrática establece una sorprendente relación entre   y
q
q
  . Esta ley fue conjeturada, basándose en evidencia numérica, por Euler en 1783 y
 p
Lagrange en 1785. Legendre le dio la forma actual a esta ley, pero no pudo dar una
prueba completa. La primera prueba rigurosa fue dada por Gauss en a la edad de 18
años. Hasta el 2004 se conocían 190 pruebas diferentes. Gauss llamó a este teorema
“Aureum Theorema”. Su importancia en la teoría de números no tienen discusión. Al
respecto, Hecke afirmó al respecto: “La teoría de números moderna comenzó con el
descubrimiento de la Ley de Reciprocidad Cuadrática”.

Ejemplo 1.
Queremos determinar cuando 5 es un cuadrado perfecto en ℤp. Observemos la siguiente
tabla:
5
p   p mod 5
 p
7 -1 2
11 1 1
13 -1 3
17 -1 2
19 1 4
23 -1 3
29 1 4
31 1 1
37 -1 2
41 1 1
43 -1 3
47 -1 2

Observamos que, para determinar si 5 es o no un cuadrado perfecto en ℤp basta


comprobar si p mod 5 es 1 o 4, es decir, cuando p mod 5 sea un cuadrado perfecto.

Ejemplo 2.
Determina si 69 es o no un cuadrado perfecto módulo 389.

Aplicando la Ley de reciprocidad cuadrática de Gauss, y teniendo en cuenta que es una


función multiplicativa:
 69   3  23   3   23 
    
 389   389   389   389 

Por un lado,
 3  ( 31) / 2  ( 3891) / 2  389 
    1    1  (1)  1
 389   3 
en donde hemos aplicado:
 1(31) / 2(3891) / 2   1388/ 2   1388/ 2   1194  1
389  2 mod 3 no es ningún cuadrado perfecto.

y por otro lado,


 23  ( 231) / 2  ( 3891) / 2  389 
    1  
 389   23 
en donde hemos aplicado:
 1( 231) / 2(3891) / 2   111194  1
389  21 mod 23 no es ningún cuadrado perfecto.

luego
 69   3  23   3   23 
      (1)(1)  1
 389   389   389   389 

y por tanto sí es un cuadrado perfecto.

Observación.
Para saber si 21 mod 23 es o no un cuadrado perfecto se podría haber aplicado de
nuevo la Ley de reciprocidad cuadrática de Gauss:
 21    2    1   2 
          (1)  1  1
 23   389   23   23 

en donde hemos utilizado:


 1
   1
 23 
 2
   1 porque 5  25  2 mod 23
2

 23 

Criterio de Euler.
a
   1  a ( p 1) / 2  1 mod p 
 p

Corolario.
La ecuación x 2  a mod p  no tiene solución si y solo si a( p 1) / 2  1 mod p  , o dicho
a
de otra manera,    a ( p 1) / 2  1 mod p  .
 p

Ejemplo 1.
Supongamos que p  11 .

Por un lado, tenemos


0² ≡ 0, 1² ≡ 1 , 2² ≡ 4 , 3² ≡ 9 , 4² ≡ 5 , 5² ≡ 3 , 6² ≡ 3, 7² ≡ 5 , 8² ≡ 9 , 9² ≡ 4 , 10² ≡ 1

Así pues, los cuadrados perfectos en ℤ11 son 0, 1, 3, 4, 5, 9.

Por otro lado, tenemos a( p 1) / 2  a(111) / 2  a5 , y las potencias a5 mod 5 son
05 ≡ 0, 15 ≡ 1 , 25 ≡ -1 , 35 ≡ 1 , 45 ≡ 1 , 55 ≡ 1 , 65 ≡ -1, 75 ≡ -1 , 85 ≡ -1 , 95 ≡ 1 , 105 ≡ -
1
a
Y verificamos que se cumple el Criterio de Euler:    1  a (111) / 2  1 mod 11
 11 

Ejemplo 2.
¿ 3 es un cuadrado perfecto en ℤp, con p=726377359 ?

Mediante el Criterio de Euler, y usando Mathematica:

Luego 3( p 1) / 2  1 mod p , y por tanto 3 no es un cuadrado perfecto en este cuerpo.


Mediante la ley de reciprocidad cuadrática de Gauss:
3   p
p 1 31
 p
   (1) 2 2     
 p 3 3
Aquí hemos aplicado que
p  1 q  1 p  1 3  1 726377359  1 726377358
      363188679 es impar
2 2 2 2 2 2
y por tanto
p 1 31
 2
(1) 2 1

 p
Para calcular   vemos que p  1 (mod 3) y 1 sí es un cuadrado perfecto en ℤ3, es
3
decir,
3   p
p 1 31
 p
   (1) 2 2       1
 p 3 3

Y por tanto 3 sí es un cuadrado perfecto en ℤp.

Observamos que el segundo método, mediante la ley de reciprocidad cuadrática de


Gauss, es mucho más efectivo, pues nos ha evitado calcular una potencia modular.
11 Problemas de la tercera parte.
11.1 MF
Sea n un número natural. Probar que si la última cifra de 7 n es 3, la penúltima es 4.

OMEFL 2018 #5

11.2 MF
Demuestra que 2222 5555  5555 2222 es múltiplo de 7.

OMEFL #4

11.3 M
Determina el número de valores distintos de la expresión
n2  2
n2  n  2

Donde n  1, 2 , 3,...,100 

OME 2017 #1

11.4 F
Probar que para todo entero positivo n, n19  n 7 es divisible por 30.

OMEFL 2009 #4

11.5 D
Probar que hay infinitos números primos cuyo resto al dividirlos entre 3 es 2.

OMEFL 2017 #5

11.6 M
Determinar razonadamente si el número
n  3n 2  2n  2

es irracional para todo entero no negativo n .

OME 2012 #1

11.7 M
Los números naturales 22, 23, y 24 tienen la siguiente propiedad: los exponentes de los
factores primos de su descomposición son todos impares:
22  21  111 , 23  231 , 24  23  31

¿Cuál es el mayor número de naturales consecutivos que pueden tener esa propiedad?
Razónese la contestación.
OMEFL 2006 #5
11.8 F
a) Demostrar que
x2 y2 z2
  1
( x  1)2 ( y  1)2 ( z  1)2

Para todos los números reales x, y, z , todos diferentes de 1 y cumpliendo xyz  1 .

b) Demostrar que la igualdad acontece para infinitas ternas de números racionales


x, y, z , todos diferentes de 1 y cumpliendo xyz  1 .

IMO 2008 #2

Nota: Solo el apartado b es un problema de Teoría de Números. El apartado a es un problema propio de


Desigualdades.

11.9 MD
Demuestra que para cada entero n existe un entero de n dígitos, todos impares, divisible
por 5 n .

USAMO 2003 #1

11.10 D
Demostrar que para cualquier par de enteros positivos k y n , existen k enteros
positivos m1 , m2 , ..., mk (no necesariamente distintos) tales que

2k  1  1  1   1 
1  1   1   ... 1  
n  m1   m2   mk 

IMO 2013 #1

11.11 D
Sea n un entero positivo y sean a1 ,..., ak ( k  2 ) enteros distintos del conjunto {1, ...,n},
tales que n divide a ai ai 1  1 , para i  1,..., k  1 . Demostrar que n no divide a
ak a1  1 .

IMO 2009 #1
11.12 D
Determinar un número de cinco cifras tal que su cuadrado termine en las mismas cinco
cifras colocadas en el mismo orden.

OME 1984 #2

11.13 D
Determina la suma de todos los enteros positivos n tales que, cuando
13  23  33  ...  n 3 se divide entre n  5 , el residuo es 17.

AIME II 2020 #10


11.14 M
Determinar todos los números de cuatro cifras n  abcd tales que al insertar un dígito 0
en cualquier posición se obtiene un múltiplo de 7.

OMEFL Aragón 2021 #3

11.15 F
a) Demuestra que 2n  1 y 2n  1 son coprimos.
b) Demuestra que, si n es un número par, entonces n 2  1 y 3n  1 son primos entre sí.

11.16 F
Demuestra que 2 2001  3 2001 es divisible entre 7.

11.17 F
Determina el mayor entero positivo n para el que n3  100 es divisible entre n  10 .

AIME 1986

11.18 D
Determina el número de enteros n  600 cuyos valores pueden ser determinados
n n n
unívocamente si nos dan los valores de   ,   y   , donde x  denota el menor
4 5 6
entero menor o igual que el número real x .

AIME II 2022 #8

11.19 F
Determina el máximo común divisor de los números An  23n  36 n2  56 n2
para todo n  0 ,1, ... ,1999 .

Junior Balkan Mathematical Olympiad 2001

11.20 MD
Determina todos los enteros n  1 para los cuales si p es un divisor primo de n 6  1 ,
 
también será un divisor de n3  1 n2  1 . 
Baltic Mathematics Competition 2002

11.21 F
Consideremos los siguientes 100 conjuntos de 10 elementos:
{1, 2, 3, ... , 10},
{11, 21, 31, ... , 20},
...
{991, 991, 991, ... , 1000}.

¿Cuántos de estos conjuntos contienen exactamente dos múltiplos de 7?

(A) 40 (B) 42 (C) 43 (D) 49 (E) 50


AMC 12B 2022 #6, AMC 10B #8
11.22 D Problema solucionado paso a paso en vídeo .

Determina, de entre los siguientes números, el único que no es divisible por ningún
número primo menor que 10.

(A) 2606  1 (B) 2606  1 (C) 2607  1 (D) 2607  1 (E) 2607  3607

AMC 12B 2022 #15, AMC 10B 2022 #17

Solución: [Link]

11.23 M
Los enteros positivos x e y son tales que 3 x  4 y y 4 x  3 y son cuadrados perfectos.
Demuestra que x e y son múltiplos de 7.

OMEFL Castilla y León 2023 #4

11.24 M
Encontrar todos los enteros positivos a, b, c  1 que satisfacen
2a  7b  c 2  4

OMEFL Aragón 2023 #6

11.25 D
Demuestra que, para cada entero positivo n, existe un número entero de n dígitos
divisible entre 5 n cuyas cifras son todas impares.

USAMO 2003 #1

11.26 D
Determina el menor entero positivo n para el cual 2 n  5 n  n sea un múltiplo de 1000.

AIME II 2021 #13


12 Exponenciación modular. El problema del logaritmo discreto.
12.1 Exponenciación modular.
Definimos la exponenciación modular de la forma convencional:

a0  1
a1  a
a n  a a
...
n veces

Ejemplo.
Determina el último dígito de 7 100

Solución:
Debemos determinar 7100 mod 10 

Primera versión:
Vamos calculando potencias sucesivas:
71  7 mod 10 
7 2  49  9 mod 10 
7 3  9  7  63  3 mod 10 
7 4  3  7  21  1 mod 10 

 
Luego 7 4  1 mod 10   7100  7 425  7 4
25
 125  1 mod 10 

Segunda versión:
 
7 2  49  1 mod 10   7100  7 250  7 2
50
  1  1 mod 10 
50
12.2 Exponenciación modular optimizada (EMO).
Supongamos que queremos calcular 537 , realizando el mínimo número de operaciones
posible.

En primer lugar, escribimos 37 en base 2:

37  100101 ( 2  37  1  25  0  2 4  0  23  1  22  0  21  1  20

Luego
537  512  0  2 4  0  2 3 1 2 2  0  21 1 2 0

5

 512  502  502  512  502  512 


5 4 3 2 1 0

   5   5   5   5   5  
 52
5 1
24
0
23
0
22
1
21
0
20
1

 5   5   5   5   5   5 
1 0 0 1 0 1
25 24 23 22 21 20

Y ahora observamos que las sucesivas potencias 52  se pueden calcular


k

recursivamente:
52   51  5  52   52  52   522  52  52   52 2  52  
0 1 2 2 3 2 2 2
   
 52   52
4 3
2
 
 52   ...  52   52
3 2 k k 1
2
 
 52 
k 1 2

Así pues, hemos reducido el cálculo de 537 a realizar 5 cuadrados y 2 multiplicaciones.

Este mismo método se puede generalizar para cualquier base y cualquier exponente,
obteniendo un método para calcular potencias x n que requiere un esfuerzo de cálculo
del orden O (log n) .
Además, este método es fácilmente codificable:

# Iterative Function to calculate


# (x^y)%p in O(log y)
def power(x, y, p) :
res = 1 # Initialize result
# Update x if it is more
# than or equal to p
x = x % p

if (x == 0) :
return 0
while (y > 0) :
# If y is odd, multiply
# x with result
if ((y & 1) == 1) :
res = (res * x) % p
# y must be even now
y = y >> 1 # y = y/2
x = (x * x) % p

return res
# Driver Code
x = 2; y = 3; p = 10
print("Power is ", power(x, y, p))
# This code is contributed by Nikita Tiwari.
Ejemplo 1.
Calcular 311 (mod 500 ) .

Solución.
11  1011 ( 2  23  21  1
32  31  3 mod 500 
0

32  32  9 mod 500 
1

32  32
2
  2
 81 mod 500 
32  32
3
   81
2 2
2
 6561  61 mod 500 

Finalmente, 311  32  32  32  61  9  3  1647  147 (mod 500 )


3 1 0

Ejemplo 2.
Calcula 394 mod 17 

94  1011110 ( 2  94  0  20  1  21  1  22  1  23  1  24  0  25  1  26  21  22  23  24  26

394  32  2  2  2  2  32   32   32   32   32 


1 2 3 4 6 1 2 3 4 6

32  9 mod 17 
32   92  81  13  4 mod 17 
2

32   (4) 2  16  1 mod 17 


3

32   (1) 2  1 mod 17 


4

32   12  1 mod 17 
5

32   12  1 mod 17 
6

Luego, finalmente: 394  9  (4)  (1)  1  1  36  2 mod 17 

Ejemplo 3.
Calcula 31000 mod 26 

1000  1111101000 (2  1000  23  25  26  27  28  29  8  32  64  128  256  512

31000  38 32 64128 256 512  38  332  364  3128  3256  3512  (*)
31  3 mod 26 
32  9 mod 26 
34  92  81  3 mod 26 
Vemos que entramos en un bucle:
38  32  9 mod 26  , 316  3 mod 26  , 332  9 mod 26  , 364  3 mod 26 
3128  9 mod 26  , 3256  3 mod 26  , 3512  9 mod 26 

Por tanto: (*)  9  9  3  9  3  9  3  3  3  3  9  9  3 mod 26


12.2.1
Calcular 3172 (mod 191) .

Observación.
A partir de la versión 3.8, Python incorpora el comando

pow(a,n,m)

para determinar a n mod m de forma optimizada para aritmética modular, aplicando


las técnicas del apartado anterior.
12.3 El problema del logaritmo discreto (PLD).
Acabamos de ver que, dados dos números a, n , calcular b  a n es un problema que se
resuelve realizando una cantidad relativamente pequeña de operaciones, porque no
necesitamos realizar toda la cadena de n  1 operaciones

a  a 2  a  a  a 3  a 2  a  a 4  a 3  a  ...  a n  a n 1  a

Sin embargo, el problema inverso, es decir, dados dos números a, b , encontrar


(suponiendo que existe) el valor de n tal que b  a n , al que llamaremos "logaritmo
discreto de b en la base a", es terriblemente difícil, pues, al no poder aplicar el truco
explicado en el apartado anterior, debemos ir probando, uno por uno, los sucesivos
valores

a  a 2  a  a  a 3  a 2  a  a 4  a 3  a  ...  a n  a n 1  a  ...

hasta encontrar (si es que existe), el valor b.

Esto, cuando trabajamos con números muy grandes, es un problema que se considera
actualmente irresoluble, y es el fundamento de la “Criptografía basada en el Logaritmo
Discreto”.
12.4 Aplicación a la Criptografía: El sistema Diffie-Hellman (DH).
En el año 1976, Whitfield Diffie y Martin Hellman propusieron el primer protocolo de
intercambio de clave basado en la exponenciación en cuerpos finitos y el PLD.

Supongamos que dos personas, Alicia y Benito, quieren acordar una clave secreta
común.

Alicia Espacio público Bob

En primer lugar pactan un


¡Todo lo que pase por aquí será visto por
cuerpo finito IFq y un el Enemigo!
elemento generador g del
mismo.
 IFq , g  son la clave
pública. Bob recibe  IFq , g 

Bob elige un entero


Alicia elige un entero 2bq2
2aq2

Alicia envía g a Bob recibe g a

Alicia recibe g b Bob envía g b

 
Alicia calcula g ab  g b
a
El Enemigo puede ver g a y  
Bob calcula g ab  g a
b

g b , pero con ellos no es


capaz de deducir g a b .

Está claro que si el “Enemigo” hubiera solucionado el PLD, de g a y g b hubiera


deducido a y b y con ellos hubiera calculado g ab , haciéndose con la clave secreta.
Este método nos lleva a plantear el siguiente problema específico, llamado Problema de
Diffie-Hellman (PDH):
Dados g a y g b y un elemento s  IFq , deducir si s  g ab .

A pesar de que no se ha demostrado que PDH y PLD son equivalentes en el caso


general, existen resultados que prueban la equivalencia de ambos problemas bajo ciertas
condiciones.
12.5 Aplicación a la Criptografía: El Criptosistema de ElGamal.
Uno de los métodos criptográficos basados en Logaritmo discreto es “ElGamal”,
propuesto por Taher ElGamal en 1985.

Emisor: Alicia Espacio público Receptor: Bob

Emisor y receptor pactan


¡Todo lo que pase por aquí será visto por
un cuerpo finito IFq y un el Enemigo!
elemento generador g del
mismo.
 IFq , g  son la clave
pública. Bob recibe  IFq , g 
El Enemigo puede ver la clave Bob elige un entero a tal
pública, pero no le servirá de nada que 2  a  q  2 y
(por eso se llama pública)
calcula g a .
El entero a es su clave
privada.
Envía a Alicia el valor g a ,
Alicia recibe g a que es su clave pública.

Supongamos que Alicia


quiere enviar el mensaje
secreto m .
Elige un entero k con
2  k  q  2.
  k
Calcula g a k  g a .

Envía g k , m g a k  El receptor recibe
 
g k , m g a k , y con estos
datos es fácil deducir m :
Calcula g ak  g k a
y
entonces:
m  m gak / gak

Observación 1.
La fórmula que utiliza Bob: m  m g a k / g a k , se puede reemplazar por el siguiente
método, que evita calcular el inverso modular de g a k :
 
1. Calcula g k
q 1 a
.
2. Determina g  k q 1 a
  k
m g a k  mg k ( q 1)  ka  ak  m g q 1  m
Observación 2.
Es importante reseñar que el valor de k debe cambiar en cada envío. De lo contrario, el
Enemigo podría deducir la clave pública.

Ejemplo.
Supongamos que Alicia y Benito quieren intercambiar mensajes utilizando el
criptosistema de ElGamal.
Pactan trabajar en el cuerpo IF157 con generador g  5 .
Bob escoge su clave privada a  25 y envía a Alica su clave pública g a  34 .
Supongamos que Alicia quiere enviar el mensaje m  19 a Bob.
 
Entonces elige un entero k  89 y envía a Bob el par 589 , 19  52589   131 , 45  .
Para obtener el mensaje, Bob calcula 52589  85 mod 157  y calcula
m  45 / 85  19 mod 157

Alternativamente, Bob también podría recuperar el mensaje a partir de


589(1571 25)  133 mod 157  y por tanto m  133 45  19 mod 157 .
13 El pequeño Teorema de Fermat. El Teorema de Euler.
13.1 El Pequeño Teorema de Fermat (PTF).
Lema. "Freshman’s Dream” ("El sueño de todo bachiller").
Para todo p primo y a, b enteros se cumple:
a  b ` p  a p  b p mod p 

Demostración.
Basta desarrollar a  b  aplicando el Teorema del Binomio. Sus términos son de la
`p

forma
 p  k pk  p p!
  a b con    0k  p
k  k  k! p  k !

Los términos "internos" 0  k  p contienen el factor primo p que no se cancela, puesto


que no aparece como factor en el denominador, luego todos se cancelan al hacer módulo
p. Es decir, todos los coeficientes son divisibles entre p excepto el primero y el último.

Teorema. Pequeño Teorema de Fermat (PTF).


Si p es primo,
a) a p  a (mod p) para cualquier entero a .
b) Si p | a , entonces a p1  1 (mod p)

Nota: El recíproco no es cierto: a n1  1 (mod n) para cierto entero a 


 n primo. Esto
se estudiará detenidamente en una observación más adelante.

Demostración. a) Aunque en el próximo tema veremos que este teorema es un caso


particular de aplicación de la función Phi de Euler, vamos a presentar aquí una
demostración directa.
Caso 1: Si a  0 . a  0  a p  0  a y está claro que entonces a p  a (mod p) .
Caso 2: Si a  0 . Vamos a demostrarlo por inducción en a :
Si a  1 , a p  1p  1 y está claro que entonces a p  a (mod p) .
Supongamos cierto a p  a (mod p) , queremos ver que entonces es cierto para a  1 .
Aplicando el lema anterior y la hipótesis de inducción:
a  1p  a p  1 (mod p)  a  1 (mod p)
tal y como queríamos ver.
Caso 3: Si a  0 . Si p  2 , entonces
a 2  (a)2  a (mod 2)  2 | a 2  a  2 | a 2  a  2a  a 2  a  a 2  a (mod 2)
Si p  2 , entonces p es impar, y por tanto: a p  (a) p  (a) (mod p)  a (mod p)
En donde hemos aplicado el “Caso 1” pues  a es positivo.
 
b) Aplicando el apartado anterior, a p  a (mod p)  p | a p  a  a a p 1  1 .
Aplicando el Lema de Euclides, puesto que, por hipótesis, p | a , deducimos que
p | a p 1  1 , o equivalentemente, a p 1  1  0 mod p   a p 1  1 mod p 
Nota histórica.
El 18 de octubre de 1640, Fermat escribió una carta a Bernhard Frenicle de Bessy
(1605-1675), un funcionario de la Casa de la Moneda francesa, excelente alumno en
teoría de los números. En su carta, Fermat comunica el resultado siguiente: Si p es
primo y p no divide a entonces p|ap−1 − 1. Fermat no presentó una prueba de este
resultado, pero una nota adjunta prometía enviar una demostración, siempre que no
resultara demasiado extensa. Sin embargo, la primera prueba conocida la dio Euler un
siglo después. Este resultado es conocido como el “pequeño” teorema de Fermat para
diferenciarlo del “último teorema de Fermat” (1637): La ecuación xn + yn = zn no tiene
soluciones enteras positivas si n > 2 (demostrado por [Link] en 1995.)

Fuente: Introducción a la Teoría de Números (Walter Mora F.)

El PTF se puede aplicar al cálculo de congruencias con potencias de números, como en


los siguientes ejemplos:

Ejemplo resuelto.
Demostrar que 538  4 (mod 11) .

Solución.
Aplicamos el PTF para garantizar que 510  1 (mod 11) , luego:
  3
538  53108  510 58  1358 (mod 11)  58 (mod 11)  (*)

Por otro lado 52  25  3 (mod 11) , y por tanto:


 
(*)  52
4
(mod 11)  34 (mod 11)  81 (mod 11)  4 (mod 11)

Donde hemos aplicado que 81  7  11  4

Ejemplo resuelto.
Calcular 7121 (mod 13) .

Solución.
Puesto que 13 | 7 podemos aplicar el PTF para garantizar que 712  1 (mod 13) .
Puesto que 121  12  10  1 7121  712101  712  10
 7  110  7  7 (mod 13)

Ejemplo resuelto. Aplicación del PTF a la determinación del orden.


Determina los posibles periodos de las secuencias x , x 2 , x3 , ... mod 13 para los
diferentes valores de x. Encuentra valores de x para los que se cumplen dichos periodos
(el concepto de orden de un entero será desarrollado en el apartado 13.3).

Aplicando el PTF, sabemos que x12  1 mod 13 , luego las longitudes de los ciclos
serán un factor de 12, porque después de 12 iteraciones siempre se llegará al mismo
valor. Así pues, las longitudes de los ciclos pueden ser: 1, 2, 3, 4, 6 y 12.

Elemento con ciclo de longitud 1: x  1  1 


Elemento con ciclo de longitud 12: x  2   1 , 2 , 4 , 8 , 3 , 6 , 12 , 11 , 9 , 5 , 10 , 7
Puesto que x  2 tiene asociado un ciclo de longitud máxima, podemos ir tomando
potencias de 2 para obtener los otros ciclos:
Elemento con ciclo de longitud 2: x  212 / 2  26  64  12   1 , 12 
Elemento con ciclo de longitud 3: x  212 / 3  24  16  3   1 , 3 , 9 
Elemento con ciclo de longitud 4: x  212 / 4  23  8   1 , 8 , 12 , 5 
Elemento con ciclo de longitud 6: x  212 / 6  22  4   1 , 4 , 3 , 12 , 9 , 10 

Proposición. Descomposición de módulos.


Si p y q son primos diferentes tales que a p  a (mod q) y a q  a (mod p) , entonces:
a p q  a (mod p q)
a 
q p
 a q (mod p) por el Corolario al PTF, y a q  a (mod p) por hipótesis, luego:
 
p
Luego a p q  a q  a q (mod p)  a (mod p) , es decir: p | a p q  a
Con un razonamiento similar llegamos a q | a p q  a , y por tanto:
p q | a p q  a , o equivalentemente: a p q  a  0 (mod p q)

Ejemplo resuelto.
Demostrar que 2340  1 (mod 341) .

Solución.
Aquí 341  11  31 , y por otra parte, 210  1024  31  33  1 , luego 210  1 mod( 31) , y por
tanto:
211  2  210  2  1  2 (mod 31)
Por otro lado, 210  1024  11  93  1  210  1 (mod 11) , y por tanto:
  3
  3
231  2 210  2 210  2  13  2 (mod 11)
Y aplicando la proposición anterior: 2341  21131  2 (mod 341) , y cancelando un factor 2
llegamos al resultado deseado: 2340  1 (mod 341)

Observación. Los números pseudoprimos y el recíproco del PTF.


El PTF dice que n primo  2n  2 (mod n) , y acabamos de ver que 2341  2 (mod 341)
y sin embargo 341  11  31 no es primo, es decir un contraejemplo para el recíproco del
PTF.
Este dato es curioso porque n  341 es el primer compuesto tal que 2n  2 (mod n) , es
decir, sirve como contraejemplo del recíproco del PTF, es decir:
n primo  2n  2 (mod n) para 2  n  340
Esta es la razón por la que los matemáticos chinos antiguos creyeron, equivocadamente,
que 2n  2 (mod n) caracterizaba los números primos.
Los números n tales que 2n  2 (mod n) , es decir n | 2n  2 se denominan
pseudoprimos. Hay infinitos pseudoprimos y los más pequeños son 341, 561, 645 y
1105.
Ejemplo. Como calcular congruencias cuando el módulo no es primo.
Calcular 2018 2018 (mod 26) .

Solución:
Puesto que 26 no es primo, no podemos aplicar directamente el PTF. Puesto que
26  2  13 ,
vamos a calcular por separado 2018 2018 (mod 2) y 2018 2018 (mod 13) , y después
aplicaremos el Teorema Chino del Residuo para determinar el resultado del enunciado.
Está claro que 2018  0 (mod 2) y por tanto 2018 2018  02018  0 (mod 2) .
2018  13  155  3 , luego 13 | 2018 , y por tanto podemos aplicar el PTF:
2018 12  1 (mod 13) .
Por otro lado, 2018  12  168  2 , luego:
 
2018 2018  2018 12168 2  2018 12 2018 2  1 2018 2  2018 2 (mod 13)
168 168

2018  13  155  3  2018  3 (mod 13)  2018 2  32  9 (mod 13)


De todo lo anterior tenemos:

2018
2018
 0 (mod 2)


2018
2018
 9 (mod 13)

Y aplicamos el Teorema Chino del Residuo:


N  2  13  26
N1  13
N2  2
No hace falta resolver la congruencia 13y1  1(mod 2) pues b1  0 .
Resolvemos la congruencia 2 y2  1(mod13)  y2  7 ,
Luego 2018 2018  13  y1  0  2  7  9  126 (mod 26)  22 (mod 26)

Observación.
Para calcular potencias elevadas con módulos no primos disponemos de dos técnicas:
La exponenciación modular optimizada (EMO) (ver 12.2) y el método que acabamos de
ver: descomponer el módulo y aplicar el Teorema Chino del Residuo. Es importante
dominar estas dos técnicas, pues son la clave para resolver muchísimos problemas de
Aritmética. Se propone resolver el siguiente problema mediante las dos técnicas
anteriores:

13.1.1 F
Determina los dos últimos dígitos de 1032 1032 .

HMMT 2009

Y en el siguiente interesante problema podemos observar como aplicar el Binomio de


Newton para calcular potencias elevadas:

13.1.2 D
2001
Calcula las tres últimas cifras de 2003 2002 .

CMO 2003 #2
Ejemplo resuelto. Aplicación del PTF a la resolución de congruencias.
Determina una solución de la congruencia x103  4 (mod 11)

Solución. Puesto que, aplicando el PTF, x10  1 (mod 11)


  3
x103  x10103  x10 x3  x3 (mod 11)
Luego hemos reducido nuestro problema a resolver la congruencia x3  x (mod 11)
Probando valores x  1 , x  2 , x  3 ,... llegamos a 53  4 (mod 11) , y por tanto:
x  5 (mod 11)

Ejemplo resuelto.
Determina una solución de la congruencia x86  6 (mod 29) .

Solución.
Aplicando el PTF, sabemos que x 28  1 (mod 29) para todo x .
 
3
Luego x86  x328 2  x 28 x 2  x 2 (mod 29) , luego hemos reducido nuestro problema a
resolver la congruencia x 2  6 (mod 29) .
Vamos probando valores x  1 , x  2 , x  3 ,... hasta llegar a
x  8  x  64  29  2  6  6 (mod 29 )
Luego una solución es x  8 (mod 29 )

Nota: Existe otra solución: x  21  x 2  441  29  15  6  6 (mod 29)


Para encontrar estas soluciones se puede hacer el siguiente planteamiento:
6  64 (mod 29 ) , y por tanto, la ecuación x 2  6 (mod 29) es equivalente a
x 2  64 (mod 29) ,
y ahora:
x  8
x 2  64 (mod 29 )  x 2  64  0 (mod 29 )  x  8x  8  0 (mod 29 )  
 x  8
y finalmente:  8  21 (mod 29 )

13.1.3 F
Aplicando el PTF, determina:
a) 331 (mod 7) b) 235 (mod 7) c) 128129 (mod 17 )

13.1.4 F
Dividimos el número 21000 entre 13. ¿Cuál es el residuo?

AHSME 1972 #31

13.1.5 F
Utilizando el PTF, demuestra que 17 divide a 11104  1

13.1.6 F
Demuestra que si (a,35)  1 , entonces a12  1 (mod 35)
13.1.7 F
Sea a1  4 , an  4a n1 , n  1 . Determina el residuo cuando a100 se divide entre 7.

13.1.8 F
Demuestra que, si ( a,42 )  1 , entonces 168  3  7  8 | a6  1 .

13.1.9 F
Determina 220  330  440  550  660 (mod 7)

13.1.10 D
En los años 60, tres matemáticos americanos demostraron que una de las conjeturas de
Euler era falsa al encontrar un entero positivo n tal que

133 5  110 5  84 5  27 5  n5

Determina n.
AIME 1989 #9

Nota: Se presentan dos soluciones, pero ninguna de las dos es completa: Son argumentos que justifican que un cierto n es el mejor
candidato posible.

13.1.11 F
Determina los números primos p para los cuales 29 p  1 es múltiplo de p.

13.1.12 D
Determine todos los enteros positivos coprimos con todos los términos de la siguiente
sucesión:
an  2n  3n  6n  1 , n  1
IMO 2005 #4

13.1.13 MF
Resuelve la congruencia x103  4 mod 11

13.1.14 F
Aplicando el PTF, demuestra que 385 | n 60  1 si 5 , 7 , 11 | n .

13.1.15 F
Determina el número de números primos p para los cuales 29 p  1 es un múltiplo de p.

[Link]

13.1.16 F
Sea p  7 un número primo. Demuestra que el número 11
 ...1 es divisible entre p.
p 1
13.2 La función Phi de Euler. El Teorema de Euler.

Definición. La función Phi de Euler.


Dado un número natural n  1 , la función Phi de Euler, que denotaremos por  (n) ,
indica el número de números naturales menores que n y coprimos con n. Definimos
 (1)  1 .
Por ejemplo,  (30 )  8 porque el número de naturales coprimos con 30 son 1, 7, 11, 13,
17, 19, 23 y 29. De la misma manera vemos que

 (2)  1,  (3)  2,  (4)  2,  (5)  4,  (6)  2,  (7)  6...

Observamos p es primo si y solo si  ( p )  n  1 .

Con Mathematica: "EulerPhi[n]"

Proposición. La función Phi de Euler mediante el TFA.


Si la descomposición en factores primos de n es n  p1k1 p2k 2 ... prk r , entonces:
 1 
 (n)   p1k  p1k 1  p 2k  p 2k ... p 
r
1 1 2 2 1 kr
r  p rk r 1  n   1  
i 1  pi 

Ejemplos:
a)  (360 )  96 porque
 1  1  1 
360  23325   (360 )  360 1  1  1    96
 2  3  5 
b)  (1001 )  720 porque
 1  1  1
1001  7  11  13   (1001 )  1001 1  1  1    720
 7  11  13 
Propiedad de la función Phi de Euler.
Fijemos un número n y un divisor suyo: Por ejemplo, n  30 y d  5 .
Estudiemos el conjunto Sd   m  n , (m, n)  d . En la siguiente tabla vemos que
S5   5 , 20 

1 m  n (m,30 ) (m,30 )  5  ( 6)  2
1 1
2 2
3 3
4 2
5 5 5  5 1 1
6 6
7 1
8 2
9 3
10 10 10  5  2 2
11 1
12 6
13 1
14 2
15 15 15  5  3 3
16 2
17 1
18 6
19 1
20 10 20  5  4 4
21 3
22 2
23 1
24 6
25 5 5  55 5
26 2
27 3
28 2
29 1
30 30 30  5  6 6

Consideremos ahora n / d  6 y  (6) . El conjunto de números menores que 6 y


coprimos con 6 es  1 , 5 , luego  (6)  2 .
Vemos que existe una biyección perfecta entre S5 y los coprimos de 6, y por tanto
S d   (n / d )
Por otro lado, todo número m  n pertenece a algún S d con d | n , y por tanto:
n   Sd   (n / d )
d |n d |n

d
Existe también una biyección d | n  | n y por tanto llegamos a la siguiente
n
propiedad:
n   ( d )
d |n
Teorema. El Teorema de Euler.

a ( n )  1 (mod n) si (a, n)  1

Observación: La condición (a, n)  1 es necesaria, pues si (a, n)  1 , ya vimos en el


capítulo 8 que la congruencia a x  1 (mod n) no tiene solución, y por tanto no puede
existir ningún k tal que a k  1 (mod n) , pues en ese caso x  a k 1 sería solución de la
congruencia a x  1 (mod n) .

Nota histórica. La primera demostración del PTF fue dada por Euler en 1736. El propio
Euler presentó en 1760 este teorema, que es una generalización del PTF porque si n es
primo entonces  ( n)  p  1 , y (a, p)  1  p | a .

Ejemplo.
Tomando n  30 y a  11 ,  (30 )  8 y 118  1 (mod 30)
En efecto, 112  121  1 (mod 30)  118  112   4
 14  1 (mod 30)

Ejemplo 1.
Determina el residuo de 71000 entre 24 aplicando el Teorema de Euler.

En primer lugar vemos que los divisores de 24 son 1, 2, 3, 4, 6, 8, 12 y 24, luego


 ( 24 )  8 .
Luego, aplicando el Teorema de Euler, puesto que 7,24  1 , 78  1 mod 24 
Ahora, puesto que 1000  125  8 , 71000  78  125
 1125  1 mod 24 

Ejemplo 2.
Determina los dos últimos dígitos de la expresión decimal de 3256 .

Solución.
Está claro que este problema implica estudiar 3256 (mod 100 ) , y aquí nos puede ayudar
el Teorema de Euler:
 (100 )   2252   100 1  1    40 , y puesto que (3,100 )  1 , podemos aplicar el
 1  1 
 2  5 
Teorema de Euler: 340  1 (mod 100 )
  6
Luego 3256  340616 (mod 100 )  340 316 (mod 100 )  316 (mod 100 ) .
38  6561  61 mod( 100 )  316  3838  61  61  3721  21 mod( 100 )
Por lo tanto, el número 3256 acaba en "21".
Ejemplo 3.
Determina 298 mod 33 , mediante dos técnicas: a) PTF & TCR y b) Teorema de Euler.

Solución:
a) Aplicando el PTF:
22  1 mod 3  298  22   49
 149  1 mod 3
 
210  1 mod 11  298  290  28  210  28  28  256  3 mod 11
9

Así pues, debemos resolver el sistema


x  1 mod 3
x  3 mod 11
Este sistema lo resolveremos aplicando el TCR:
11 y1  1 mod 3  2 y1  1 mod 3  y1  5
  x  11  5  1  3  4  3  91  25 mod 33 
3 y2  1 mod 11  y2  4 

b) Aplicando Teorema de Euler.


 1  1
En primer lugar,  33   3  111  1    2  10  20 .
 3  11 
Luego, aplicando el Teorema de Euler, 220  1 mod 33 , y por tanto:
   2   2 
298  2100  22
1 20 5 2 1
 41 mod 33

Mediante cualquiera de las técnicas ya estudiadas determinamos el inverso modular de 4


y llegamos a 298  41  25 mod 33 .

13.2.1 F
¿Para cuántos enteros i , cumpliendo 1  i  1000 , existe un entero j , cumpliendo
1  j  1000 , tal que i es un divisor de 2 j  1 ?

13.2.2 M
Determina los ocho últimos dígitos de la expansión binaria de 271986 .

13.2.3 D
21
2006...
Determina los tres últimos dígitos de 20082007
1
...2
20072006
Nota: 2008 se define recursivamente:

a1  1 , a2  2a1 , a3  3a2 , ... , a2008  2008 a 2007

PuMAC (Princeton University Mathematics Competition)


13.2.4 D
xx
Definimos f ( x)  x x . Determina los dos últimos dígitos de

f (17 )  f (18)  f (19 )  f (20 )

PuMAC 2008

Nota: f (x) se define recursivamente: f1 ( x)  x , f 2 ( x)  x f1 ( x ) ,..., f ( x)  f 4 ( x)  x f3 ( x ) .

Indicación: Si (a, n)  1 , entonces a b  a b mod  ( n ) (mod n) y podemos reducir el cálculo


a determinar b mod  (n).

13.2.5 F
Encontrar las tres últimas cifras del número 7 2014

OMEFL 2014 #2

13.2.6 F
Determina el número de enteros 1  i  1000 para los cuales existe un entero
1  j  1000 tal que i es divisor de 2 j  1 .

[Link]

13.2.7 F
Sea p  5 un número primo. Demuestra que p 8  1 mod 240  .
13.3 Orden de un entero.

Definición. Orden de un entero.


Observando las tablas del apartado 13.1 vemos que si a, n  1, entonces siempre existe
un 1  k  n tal que a k  1 (mod n) .

En efecto, consideremos la secuencia



a0 , a1 , a 2 , a3 , ... , a n 
Esta secuencia consta de n  1 valores, y solo hay n valores diferentes en Z n , luego
forzosamente dos de ellos deberán ser iguales:
ai  a j mod n  para ciertos 0  i  j  n

Consideremos el valor q  j  i  j  q  i y por tanto


ai  a j  a q i  a q  ai mod n

 
Puesto que a, n  1  ai , n  1 , podemos cancelar el factor a i en la igualdad anterior
(es decir, multiplicaremos ambos lados por el inverso de a i ) para deducir que
1  a q mod n

Definimos el orden de un entero a módulo n , y escribiremos ordn (a) , como el


menor valor k tal que a k  1 (mod n) . Acabamos de ver que si a, n  1 este valor
siempre existe.

Con Mathematica:

 
El Teorema de Euler indica que si (a, n)  1 , la secuencia a , a 2 , a3 , a 4 ,...
siempre alcanza el 1 (y por lo tanto se vuelve periódica), y lo alcanza en a (n ) .
Naturalmente,  (n) no es necesariamente el primer número k para el cual a k  1 .

Ejemplos.
31  3 , 32  2 , 33  6 , 34  4 , 35  5 , 36  1 (mod 7)  ord7 (3)  6 .
ord5 (2)  4 pues 21  2 , 22  4 , 23  3 , 24  16  1 (mod 5) .
ord12 (5)  2 pues 51  5 , 52  25  2  12  1  1 (mod 12) .

Observación.
No todos los números tienen un definido un orden. Por ejemplo, si n | a , entonces
a k  0 (mod n) para todo k .
El siguiente corolario nos indica cuando un entero tendrá un orden asociado.
Corolario.
Dado un entero n  1 , a  Z tendrá orden módulo n si y solo si (a, n)  1 .

Demostración. Si (a, n)  1 , por la proposición anterior existirá un 1  k  n tal que


a k  1 (mod n) , luego por el Principio de la buena ordenación, existirá un mínimo k
cumpliendo tal condición, es decir, el número a tendrá un orden asociado.

Supongamos ahora que existe un entero positivo m tal que a m  1 (mod n) . Entonces
existirá un entero s tal que
a m  s n  1  a  a m1  s n  1

Esta última expresión es una combinación lineal de a y n , luego


(a, n) | a  a m 1  s n  1  (a, n)  1

Teorema. Teorema fundamental del orden.


Supongamos que (a, n)  1 , y sea d  ordn (a) .
Entonces, para cualquier entero k , a k  1 (mod n)  d | k .

Demostración.
Supongamos que a k  1 (mod n) . Entonces k  d por definición de orden, y por tanto
k  dq  r para cierto 0  r  d . Luego
  q
1  a k  adq r  adq  ar  ad  ar  1q  ar  ar (mod n)
Pero 0  r  d y la única posibilidad que no contradiga la definición de orden es r  0 ,
y por tanto k  dq  d | k , tal y como queríamos ver.

Supongamos ahora que d | k . Entonces k  dq y por tanto


  q
ak  adq  ad  ar  1q  1 (mod n)

Corolario.
Supongamos que (a, n)  1 , y sea d  ordn (a) . Entonces
ai  a j (mod n)  i  j (mod d )

Demostración. Podemos suponer, sin pérdida de generalidad, que i  j .


Supongamos que i  j (mod d )  i  qd  j , y por tanto
  q
ai  a qd  j  a d a j  1q a j  a j (mod n)
Supongamos ahora que ai  a j (mod n) . Entonces tomando q  i  j  0 y cancelando
términos llegamos a 1  a q  ai  j (mod n)  d | i  j por el Teorema anterior, luego
d | i  j  i  j (mod d ) .

Observación.
Este corolario nos dice que la sucesión de potencias a k se va repitiendo en ciclos de
longitud d  ordn (a) . En particular, podemos aplicar este hecho a la función Phi de
Euler: ordn (a) |  (n)
Ejercicio resuelto.
Determina ord33 (2)

 
Solución: Vemos que 25  32  1 ord 33  210  25   1  1 ord 33 . Luego,
2 2

por el corolario anterior, ord33 (2) debe ser un divisor de 10. Vamos probando, uno por
uno, todos los divisores:
21  2  1 mod 33 , 22  4  1 mod 33 , 25  32  1 mod 33 , así que solo puede ser
210  1 mod 33 , y la solución es 10.

Problema resuelto.
Determina todos los enteros positivos n tales que 2n  1 sea divisible entre 7.

IMO 1967 #1 (apartado a)

Solución: 7 | 2n  1  2n  1  0 (mod 7)  2n  1 (mod 7)


Puesto que ( 2,7)  1 , podemos aplicar el Teorema anterior:
2n  1 (mod 7)  ord7 (2) | n , 21  2 , 22  4 , 23  8  1 (mod 7)  ord7 (2)  3
Luego, finalmente, 7 | 2n  1  3 | n , es decir, para todos los múltiplos de 3.

Corolario.
Dados p primo y a entero con p | a , entonces ordn (a) | p  1

Demostración. Puesto que p es primo y p | a , podemos aplicar el PTF para garantizar


que a p 1  1 (mod p) , y aplicar ahora el teorema anterior.

Ejemplo.
Determinar ord11(8) .

Solución.
Puesto que 11 | 8 , aplicar el corolario anterior para asegurar que ord11(8) | 10 .
ord11(8)  2 no puede ser pues 82  64  9  1 (mod 11) .
ord11(8)  5 tampoco puede ser:
82  9 (mod 11) y 83  512  6 (mod 11) , luego
85  8283  9  6  54 (mod 11)  10 (mod 11) . Luego solo nos queda ord11(5)  10 .

Ejemplo resuelto.
Determina ord13 (2) .

Solución.  (13)  12  ord13 (2)  1, 2 , 3, 4 , 6 ,12 


21  2 , 22  4 , 23  8 , 24  16  3 (mod 13) , 26  32  12 (mod 13) , luego solo puede
ser ord13 (2)  12 .
Problema resuelto.
Determina la última cifra de 7 83 .

Solución. Está claro que debemos determinar 783 mod 10  .


70  1  1 mod 10  , 7 2  49  1 mod 10   7 4   1  1 mod 10 
2

Y puesto que el orden tiene que ser divisor de 10, solo puede ser ord10 7  4 .
Por otro lado, 83  3 mod 4 , luego 783  73  (1)  7  3 mod 10  , y por tanto la
última cifra es 3.

Problema resuelto.
Determina ord29 (10) .

Solución. Puesto que 29 es primo, aplicando el PTF, 10 28  1mod 29  , luego


ord29 (10) | 28 .

Vamos probando, uno por uno, todos los divisores de 28:


101  10  1mod 29  , 10 2  100  13  1mod 29  , 10 4  24  1mod 29  ...
Finalmente 10 28  1mod 29  , y por tanto ord29 (10)  28 .

Así pues:
10 28  1 mod 29   10 28  1  99
 99  0 mod 29   29 | 99
...  ...
99
28 nueves 28 nueves

Pero 29 | 99
 99 si n  28 .
...
n nueves

Ejercicio resuelto.
a) Determina, de forma directa, ord17 2 .
b) Aplicando el apartado anterior, calcula 2 20 mod 17  y 21024 mod 17  .

Solución:
a) en módulo 17, 21  2 , 22  4 , 2 3  8 , 2 4  16  1 , 25  2 , 26  4 , 2 7  8 ,
28  (1)  1 . Así pues, ord17 2  8 .
b) 220  2884  28  28  24  24  1 mod 17  , 21024  28128  28   128
 1128  1 mod 17 
Problema resuelto.
Determina el menor factor primo impar de 2019 8  1

AIME I 2019 #14

Solución.
Buscamos el menor número primo p tal que p | 2019 8  1
p | 2019 8  1  2019 8  1  0 (mod p)  2019 8  1 (mod p)
Entonces, elevando al cuadrado ambos lados, 2019 16  1 (mod p)
Pero 2019 16  1 (mod p )  ord p (2019 )  1, 2 , 4 , 8 ,16 
Sin embargo, ord p (2019 )  1, 2 , 4 , 8   2019 8  1 (mod p ) y no -1, como queríamos,
luego deducimos que ord p (2019 )  16 .
Puesto que ord p (2019 ) |  (p) ,  (p) será un múltiplo de 16.
 1
Puesto que por hipótesis p es primo,  ( p)  p  1    p  1
 p
Y por tanto p  1 (mod 16 ) . Los dos primeros primos que cumplen p  1 (mod 16 ) son
17 y 97.
Sin embargo, 2019 8  1 (mod 17 ) , pero 2019 8  1 (mod 97 ) , luego la solución es 97.

Fuente de esta solución: [Link]

Problema resuelto.
Determina el menor entero positivo n tal que, cuando ese escribe 3 n en base 143, sus
últimas dos cifras en dicha base son 01.

AIME I 2018 #11

Solución:
Sabemos que decir que la expresión de 3 n en base 143 es ...d 3 d 2 d1 01143 es equivalente a

3n  1  0  143  d1  143 2  d2  143 3  ...  1  143 2 d1  d2  143  ...  3n  1 mod 143 2 
Así pues, en este problema nos piden resolver la congruencia 3n  1 mod 143 2 con  
solución n mínima, es decir, determinar el orden de 3 módulo 143 2 .

Es importante remarcar que nos piden n mínimo, pues el Teorema de Euler nos
 
garantiza que 3 (143 )  1 mod 143 2  puesto que 3,143 2  1 , pero en este caso
2

 
 143 2  17160 y veremos que no es el valor mínimo posible.

 
143  11  13  143 2  112  13 2 , y puesto que 112 ,132  1 , para resolver la congruencia
 
3  1 mod 11  13 será suficiente resolver las congruencias 3a  1 mod 112 y
n 2 2
 
 
3b  1 mod 132 por separado.
Paso 1. Resolvemos la congruencia: 3a  1 mod 112  
Esta es sencilla. Se puede encontrar por tanteo:
31  3  3 mod 121 32  9  9 mod 121
33  27  27 mod 121 34  81  81 mod 121
35  243  2  121  1  1 mod 121

El valor mínimo es a  5 .

 
Paso 2. Resolvemos la congruencia: 3b  1 mod 132 . Esta es mucho más complicada.

Paso 2.1. Por tanteo ("bash"). Si lo hacemos por tanteo nos vamos a pasar un buen rato
calculando potencias hasta llegar al exponente 39 para encontrar 339  1 mod 132 :  
1→3 2→9 3→27 4→81 5→74
6→53 7→159 8→139 9→79 10→68
11→35 12→105 13→146 14→100 15→131
16→55 17→165 18→157 19→133 20→61
21→14 22→42 23→126 24→40 25→120
26→22 27→66 28→29 29→87 30→92
31→107 32→152 33→118 34→16 35→48
36→144 37→94 38→113 39→1

Paso 2.2. Aplicando el Teorema de Euler. Una indicación nos la puede dar la función
 
Phi de Euler:  132  2  3  13 , luego el orden de 3 módulo 156 será un divisor de
156  2  3  13 . Probando divisores encontramos la solución 39  3  13 .

Paso 2.3. Aplicando el Teorema del binomio.


 
Observamos que 3b  1 mod 132  3b  1 mod 13 .
b
 2

En efecto: 3  1 mod 13  3  132 k  1  1313k   1  3b  1 mod 13
b

Y la congruencia 3b  1 mod 13 tiene fácil solución: 33  27  2  13  1  1 mod 13 .


Así pues, cualquier solución de 3b  1 mod 13 será múltiple de 3.
Llegados a este punto podríamos aplicar la estrategia 2.2 anterior para llegar a la
solución, pero en su lugar vamos a aplicar el desarrollo binomial.

El número b buscado será múltiplo de 3: b  3c para cierto entero c , con lo que la


congruencia se transforma en:

    c
 
33c  1 mod 132  33  1 mod 132  27 c  1 mod 132  
Ahora aplicamos el desarrollo binomial:
27  2  13  1  27 c  2  13  1 
c

c c  c  c


  2  13  10   2  13  11  ...   2  13 11c 1   2  13 01c
c c 1

 0 1  c  1 c
Y observamos que, trabajando módulo 13 2 , cualquier potencia 13 c con c  2 será
(congruente con) cero.

Así pues, a todos los efectos prácticos:


 c  c
27 c   2  13  1c 1   2  13  1c  c  26  1
1 0

 c  1 c


Y por tanto, la congruencia exponencial 27 c  1 mod 132 se convierte en la 
congruencia lineal
26c  1  1 mod 132  
Que se resuelve fácilmente:

   
26c  1  1 mod 132  26c  0 mod 132  2  13c  0 mod 132  
Para que esta última congruencia se cumpla, es necesario y suficiente que c sea
múltiplo de 13.

Así pues, finalmente llegamos a un resultado múltiplo de 3 y múltiplo de 13, y el valor


mínimo posible es c  3  13  39 .

Paso 3. Juntamos las dos congruencias:

   
Hemos obtenido 35  1 mod 112 y 339  1 mod 132 , luego tomando el mínimo común
múltiplo de ambos exponentes: 5,39  5  39  195 , tendremos, aplicando 7.5d:
 
3195  3539  35  139  1 mod 112 
39


 
 3     
39 5 
3195  3395  15  1 mod 13 2   3195  1 mod 112  13 2
 
112 ,13 2  1 

Que será el valor mínimo posible pues en todos los pasos hemos obtenido los valores
mínimos posibles. La solución del problema es 159.

Fuentes:
[Link]
[Link] Analyzing the Expression in Mod 143² (2018 AIME I Prob 11) "LetsSolveMathProblems"
[Link] 2018 AIME I Problem #11 ("Osman Nal")

13.3.1 M
Determina el número de enteros positivos múltiples de 1001 que se pueden expresar de
la forma
10 j  10 i

con i , j enteros cumpliendo 0  i  j  99 .

AIME 2001

13.3.2 F
Demuestra que, si p es un número primo, todo divisor primo de 2 p  1 es mayor que p.
14 El problema de la primalidad. El método RSA.
14.1 El test de primalidad de Fermat.
Al problema de como determinar si un número n es primo se lo conoce como problema
de la primalidad. Un test de primalidad es un algoritmo que permite comprobar si un
número n es primo o es compuesto. Existen dos tipos de test de primalidad:
- Determinista: determina si el número es primo o no lo es.
- Probabilístico: determina si un número es compuesto o si es "probablemente primo".
Determinar que el número es “probablemente” primo no te asegura que sea primo, pero
existe una elevada probabilidad de que lo sea, y esta puede ser tan grande con se desee.
O dicho de otro modo, la probabilidad de que un número "probablemente primo" sea
compuesto es insignificante. Una característica de los test probabilísticos es que son
mucho más rápidos que los deterministas.

Recordemos que el PTF afirma que, si p es primo, para todo número a coprimo con p
se cumplirá a p 1  1 mod p  .
Por lo tanto, para demostrar que un número determinado n no es primo, bastará
encontrar un número a , coprimo con n , para el cual a p 1  1 mod p  .

Naturalmente, no encontrar dicho número a no garantiza que n sea primo. Por


ejemplo:
n  3215031751 pasa el test para los valores a  2 , 3 , 5 , 7 :
23215031750 1 mod 3215031751 
33215031750 1 mod 3215031751 
53215031750 1 mod 3215031751 
73215031750 1 mod 3215031751 

Y, sin embargo 3215031751 no es un número primo:


3215031751  151  751  28351

Esto no quiere decir que este método no sea efectivo. Entre todos los números n
menores que 250 billones, solo este número pasa el test para los números primos 2, 3, 5
y 7 siendo un número compuesto.

Ejemplo resuelto.
Demostrar que 117 no es primo.

Solución.
Tomamos a  2 . Sabemos que 2 7  128  11 (mod 117 ) , y que 121  4 (mod 117 ) ,
luego:
2 
7 16
   121  4  2   2 (mod 117 )
 1116  112
8 8 8 2 8 16

2117  27165  2  2  2 2 (mod 117 )  2 (mod 117 )


7 16 5 16 5 21

 2   11  121  11  4  11  44 (mod 117 )


3
Pero 2 21 7 3

Con lo que, finalmente llegamos a 2117  44  2 (mod 117 ) , y por tanto 117 debe ser un
número compuesto (de hecho: 117  13  9 ).
Ejemplo resuelto.
Demuestra, aplicando el PTF, que 232  1  4294967297 no es un número primo.

Solución: Sea p un factor primo de k  232  1 .


 
Este factor primo p estará acotado superiormente: p  232  1  216  65536 .
Por otro lado:
p | k  k  0 mod p   232  1 mod p   264   1  1 mod p 
2

Sea d  ord p 2 . Está claro que 264  1 mod p   d | 64  26  d  2r para cierto
r 6.
Si r  6  d | 32  232  1 mod p  , llegando a contradicción. Por lo tanto r  6 y
d  64 .
Ahora, aplicando el Corolario XXXX (página ????),
d  64  p  1 mod 64  p  64s  1
Con todo esto los candidatos a factor primo de k se han reducido a los primos de la
forma p  64 s  1 . Vamos dando valores a s para ir obteniendo los diferentes valores de
p asociados:
s=1 → p=65 no es primo.
s=2 → p=129 no es primo
s=3 → p=193 es primo pero no divide a k
s=4 → p=257 es primo pero no divide a k
s=5 → p=321 no es primo
s=6 → p=385 no es primo
s=7 → p=449 es primo pero no divide a k
s=8 → p=513 no es primo
s=9 → p=577 es primo pero no divide a k
s=10 → p=641, es primo y sí divide a k . En efecto: k  p  6700417

Así pues, hemos encontrado un factor de k, y por tanto hemos demostrado que no es un
número primo.

Problema.
Aplicando el método anterior, comprueba si 211  1  2047 es un número primo o
compuesto.

Solución: Supongamos que p es un factor primo de k  211  1 . Luego:


p | k  k  0 mod p   211  1 mod p 
Sea d  ord p 2 . Está claro que d | 11 y por tanto solo puede ser d  11 o d  1 .
Supongamos que d  11 . Luego, por el Corolario XXXX, p  1 mod 11 , es decir, p es
de la forma p  11s  1 para cierto s. Vamos dando valores a s para ir obteniendo
valores de p y comprobando si son primos, en cuyo caso comprobamos si son divisores
de k:
s=1 → p=12 no es primo.
s=2 → p=23 es primo y divisor de k. En efecto, k  211  1  89  23
Así pues, hemos encontrado una factorización de k, y por tanto hemos demostrado que
no es primo.
Algoritmo del Test de Primalidad de Fermat.
Input: número n .
Paso 1: Determinamos, al azar, un número 1  a  n
Paso 2: Determinamos ( a, n) mediante el Algoritmo de Euclides.
Paso 3: Si (a, n)  1 , el número n no es primo, retornamos a .
( es muy poco probable que esto suceda )
Paso 4: Calculamos r  a n 1 mod n mediante el EMO.
Paso 5: Si r  1, el número n no es primo (aunque no sabemos su factorización)
Paso 6: Este test no ha sido decisivo. Se vuelve al Paso 1 para probar con otro número.

Ejemplo.
Vamos a demostrar que el número 527 es compuesto aplicando el método anterior.

Después de mucho calcular llegamos al resultado siguiente:


240  1 mod 527 

Por otro lado, 526  40  13  6 , y por tanto


 
2526  240
13
 26  113  26  26  64  1 mod 527 

Por lo tanto podemos asegurar que el número 527 es compuesto (de hecho,
527  17  31 ).

Los números de Carmichael.


En ningún momento se ha dicho que, para todo número n compuesto, exista un a
coprimo con n para el que a n 1  1 mod n  .
Existen números n para los cuales la identidad a n 1  1 mod n  es cierta para todas las
bases a coprimas con n y sin embargo son números compuestos.
Estos números son excepcionales (pero existen infinitos), y se llaman números
Carmichael. El menor número Carmichael es 561.
14.2 Aplicación a la criptografía: El método RSA.
El método RSA fue descubierto/inventado en 1977 como el primer método de
encriptación de clave pública de la historia. Su nombre deriva de las iniciales de los tres
investigadores americanos que lo descubrieron: Rivest, Shamir y Adleman.

Este método se basa en la dificultad de factorizar números muy grandes. Los números
utilizados en este sistema suelen tener más de 300 dígitos decimales, y intentar su
factorización ocuparía miles de años para las más avanzadas computadoras que existen
en la actualidad.

Descripción del método RSA.

Emisor: Alicia Espacio público Receptor: Bob

1. El receptor selecciona dos


números primos p y q muy
grandes (de centenares de
cifras)
¡Todo lo que pase por aquí será visto por
el Enemigo!
2. Calcula n  p  q
3. Calcula
 (n)   p  1q  1
4. Determina, al azar, un
número aleatorio 1  e   ( n)
coprimo con  (n)
5. La pareja ( n, e) es la clave
pública, que se envía al
7. El emisor recibe la clave Emisor.
pública ( n, e) . Con ella El Enemigo puede ver la clave 6. Determina
encriptará el número secreto pública, pero no le servirá de nada d  e 1 mod  (n)
k de la siguiente forma: (por eso se llama pública)
La pareja (n, d ) es la clave
privada de desencriptación.

¡Los números p, q y d se
deben mantener secretos, por
8. Calcula eso no pueden salir de aquí!
f (k )  k e mod n 
y lo envía al Receptor. 9. El receptor recibe f (k ) y
¡Aunque el Enemigo vea el lo desencripta de la siguiente
número encriptado f (k ) y manera:
disponga de la clave pública ( n, e)
no podrá desencriptar el mensaje! 10. Calcula
g  f (k )  f (k ) d mod n  k
Ejemplo.
Por su valor puramente didáctivo exponemos aquí dos ejemplos prácticos con números
primos p y q muy pequeños.

Supongamos que el Emisor quiere enviar al Receptor el mensaje "16346", y el Enemigo


puede escuchar cualquier información entre Emisor y Receptor.

1. El receptor selecciona dos números primos p y q muy grandes (de centenares de


cifras), nosotros ahora, a efectos didácticos, tomaremos números no tan grandes:

p  281 y q  167

2. Calcula n  p  q  46927

3. Calcula  (n)   p  1q  1  46480

4. Determina, al azar, un número aleatorio 1  e   ( n) coprimo con  (n) :

e  39423

5. La pareja (n, e)   46927 , 39423  es la clave pública, que se envía al Emisor.

6. Determina d  e 1 mod  (n)  26767

La pareja (n, d )   46927 , 26767  es la clave privada de desencriptación. Los


números p , q , d se deben mantener secretos en todo momento.

7. Supongamos que el Emisor quiere enviar al Receptor el número secreto k  16346 .

8. El Emisor calcula
f (k )  k e mod n  16346 39423  mod 46927   21166
Envía al Receptor el número 21166

9. El receptor recibe 21166 y lo desencripta de la siguiente manera:

10. Calcula 21166 d mod n  21166 26767 mod 46927   16346

Obteniendo el número secreto desencriptado 16346.


Comentario.
El punto fundamental de la encriptación RSA es que el “Enemigo”, aunque disponga de
la clave pública ( n, e) y del número encriptado k , no será capaz de desencriptarlo. Y
esto es debido a que necesitaría determinar d  e 1 mod  (n) , y desconoce  (n) .

En efecto,  (n) es muy fácil de calcular si se conoce la factorización de n , recordemos


que el receptor pudo calcular  (n) porque conocía desde un principio:

n  p  q   (n)  ( p  1)( p  1)  pq  p  q  1  n  p  q  1

y la factorización de un número muy grande como es n es terriblemente complicada.

La seguridad del sistema RSA radica en la imposibilidad computacional de factorizar un


número de 200 cifras, ya que, con los algoritmos actuales y las mejores computadoras
requeriría siglos.
15 Raíces primitivas. Índices modulares.
15.1 Raíces primitivas.
Definición. Raíz primitiva.
Diremos que una unidad a de Z n es una raíz primitiva módulo n si ordn a    (n) .
Equivalentemente, cuando
a   a, a 2 , a3 ,..., a ( n )   Z n

O de otra forma, si para cualquier entero x coprimo con n, siempre existe un entero j tal
que x  a j mod n .

Lema.
Sea m un entero positivo y sea x un entero coprimo con m . Para todo par j, k de
enteros positivos, se cumple:

x j  x k mod m  j  k mod d  , donde d  ord m x 

Ejercicio resuelto.
Determina ord257 (5) , y comprueba que 5 es una raíz primitiva módulo 257.

Solución. Puesto que 257 es primo,  (257 )  256  28 . Así pues, ord 257 (5) | 28 , es
decir, será una potencia de 2. Vamos calculándolas una por una:
5 2  25 , 54  52  52  625  111 , 58  54  54  111  111  242 , 516  225 , 532  253 ,
564  16 , 5128  256 , 5256  1 .
Así pues, ord257 (5)  256   (257 ) , y por tanto es una raíz primitiva módulo 257.

Ejercicio resuelto.
Resuelve la congruencia x3  5 mod 17  , sabiendo que 3 es una raíz primitiva módulo
17.

Solución. Por ser 3 una raíz modular módulo 17, todo elemento de Z17 se podrá escribir
como x  3a para cierto a que vamos a determinar.
 
x3  5 mod 17   3a  5 mod 17   33a  5 mod 17 
3

Ahora escribimos 5 en forma de potencia de 3:


31  3 mod 17  , 32  9 mod 17  , 33  10 mod 17  , 34  13 mod 17  ,
35  5 mod 17 

Y por tanto nuestra congruencia queda planteada de manera que podemos aplicar el
lema anterior:
33a  35 mod 17   3a  5 mod  17   3a  5 mod 16 

Probando con posibles candidatos vemos que


3 11  33  1 mod 17   11  31 mod 17 

Y por tanto
3a  5 mod 16   a  31  5  11  5  55  7 mod 16   a  16b  7

En particular, tomando b  0  a  7  x  37 mod 17 


Siguiendo con los cálculos de arriba:
36  15 mod 17  , 37  11 mod 17 
Así pues, x  11 mod 17 .

Fuente: Michael Penn en YouTube.

Ejercicio resuelto.
Resuelve la congruencia 5x  17 mod 19  , sabiendo que 2 es una raíz primitiva módulo
19.

Solución.
Generando la tabla 2k mod 19  vemos que 17  210 mod 19  y 5  216 mod 19  , por
tanto
 
5x  17 mod 19   216  210 mod 19   216x  210 mod 19 
x

 16 x  10 mod  (19 )   16 x  10 mod 18   16 x  10 mod 18  


8 x  5 mod 9
Por tanteo vemos que 8  8  64  1 mod 9  81  8 mod 9 y por tanto
8x  5 mod 9  x  81  5  8  5  40  4 mod 9  x  9k  4
Tomando k  0  x  4 , y tomando k  1  x  13 , es decir, aparecen dos soluciones
módulo 19.

Fuente: Michael Penn en YouTube.

Ejercicio resuelto.
Resuelve la siguiente congruencia:
17 x  10 mod 27 

Solución.
En primer lugar, vemos que, puesto que 27  33 , existirá una raíz primitiva módulo 27.
De hecho, se demuestra que 2 es una raíz primitiva módulo 27.
Para aplicar el lema anterior, debemos escribir 17 y 10 como potencias de la misma
base.
21  2 mod 27  2 2  4 mod 27  23  8 mod 27 
24  16 mod 27  25  32  5 mod 27  26  10 mod 27 
27  20 mod 27  28  13 mod 27  29  26  1 mod 27 
210  2  25 mod 27  211  4  23 mod 27  212  8  19 mod 27 
213  11 mod 27  214  22 mod 27  215  44  17 mod 27 

 
Luego 17 x  10 mod 27   215  26 mod 27   215x  26 mod 27 
x
Y ahora aplicamos el lema anterior:
 
215x  26 mod 27   15 x  6 mod 18  , pues  (27 )   33  18

Con lo que hemos reducido una congruencia exponencial a una congruencia lineal.
15x  6 mod 18  5x  2 mod 6 dividiendo entre tres toda la congruencia.
Esta última congruencia la podemos resolver viendo que 51  5 mod 6 y por tanto
5x  2 mod 6  x  51  2  5  2  10  4 mod 6
Puesto que nosotros buscamos soluciones módulo 16, el conjunto total de soluciones
será
4 , 4  6 , 4  12 mod 18  4 , 10 , 16 mod 18
Ejercicio resuelto.
Resuelve la siguiente congruencia:
9x  10 mod 13

Solución.
En primer lugar, vemos que 2 es una raíz primitiva módulo 13.

Para poder aplicar el lema anterior, debemos escribir 9 y 10 como potencias de 2:


21  2 mod 13 2 2  4 mod 13 23  8 mod 13
24  16  3 mod 13 25  6 mod 13 26  12  1 mod 13
27  2  11 mod 13 28  4  9 mod 13 29  18  5 mod 13
210  10 mod 13

 
Luego 9x  10 mod 13  28  210 mod 13  28 x  210 mod 13
x

y aplicando el lema anterior:


28 x  210 mod 13  8x  10 mod 12  , pues  (13)  12

Pero vemos que esta congruencia lineal no tiene solución (8,12 )  4 y 4 | 10 .

Así pues, la congruencia del enunciado no tiene solución.

Ejercicio resuelto.
Resuelve la siguiente congruencia:
11x  17 mod 18 

Solución.
Sabemos que 5 es una raíz primitiva módulo 18.
Para poder aplicar el lema anterior, debemos escribir 11 y 17 como potencias de 5:
51  5 mod 18  52  25  7 mod 18  53  35  1  17 mod 18 
54  5  13 mod 18  55  25  11 mod 18 

Luego
 
11x  17 mod 18   55  53 mod 18   55 x  53 mod 18 
x
y aplicando el lema anterior:
55 x  53 mod 18   5x  3 mod 6 , pues  (18)  6

Para resolver esta congruencia lineal aprovechamos que 5 y 6 son coprimos, luego 5
tendrá inverso módulo 6, en efecto, es el propio 5, luego

5x  3 mod 6  x  51  3  5  3  15  3 mod 6

15.1.1 F
Determina el menor entero n  2021 para el cual 30 n 3  143 n 2  117 n  56 sea divisible
entre 13.

SMT 2021 Number Theory #2


15.2 Determinación de raíces primitivas.

Teorema.
Supongamos que a es una raíz primitiva módulo n.
 ( n)
a) ord n a k  
 k ,  ( n) 
b) Si d |  n , entonces existen  d  elementos de Z n de orden d, y a k tiene orden d si
 (n)
y solo si  k ,  (n)  
d
Corolario.
Si Z n tiene una raíz primitiva a, entonces tiene   (n) raíces primitivas: a k es una raíz
primitiva si y solo si  k ,  (n)   1 .

Ejercicio resuelto.
Determina las raíces primitivas módulo 17 sabiendo que 3 es raíz primitiva módulo 17.

Solución.
 
Por el corolario anterior, sabemos que existen   (17 )    16    24  8 raíces
primitivas. Serán todas aquellas que se puedan escribir como 3 k con  k , 16   1 , es
decir,
k  1,3,5,7,9,11,13,15 , que dan lugar a los valores
31  3 , 33  10 , 35  5 , 37  11 , 39  14 , 311  7 , 313  12 , 315  6 .

Ejercicio resuelto.
Determina las   29   28  12 raíces primitivas módulo 29, sabiendo que 2 es una
raíz primitiva módulo 29.

Solución.
Aplicando el corolario anterior, las raíces primitivas de Z 29 serán todos los elementos
de la forma 2 k , con  k , 28   1 , es decir, k  1, 3, 5, 9, 11, 13, 15, 17, 19, 23, 25, 27 .
Para estos índices k tenemos los valores 2, 8, 3, 19, 18, 14, 27, 21, 26, 10, 11, 15 , que serán
las 12 raíces primitivas buscadas.

El siguiente teorema es una generalización del teorema anterior.

Teorema. Teorema de Gauss.


Existen raíces primitivas módulo n  1 si y solo si n  2 , 4 , p , 2 p donde p es un
primo impar y α es un entero positivo.
15.3 Índices modulares.

Definición. Índice modular.


Sea b una raíz primitiva módulo n. Si (a, n)  1 , entonces el más pequeño entero
positivo k tal que a  b k mod n  se denota por ind b (a) y se llama índice de a respecto
a la base b módulo n.

Al índice se le suele llamar también logaritmo discreto, pues sus propiedades y sus
aplicaciones son similares a las del logaritmo.

Ejercicio resuelto.
a) Demuestra, mediante cálculos directos, que 3 es una raíz primitiva módulo 17, y
construye la tabla de índices de base 3 módulo 17.
b) Aplicando la tabla de índices anterior, resuelve la congruencia 7 x  5 mod 17.
c) Aplicando la tabla de índices anterior, resuelve la congruencia x 7  5 mod 17  .
d) Aplicando la tabla de índices anterior, resuelve la congruencia x8  8 mod 17  .

Solución.
En Z17 tenemos: 31  3 , 32  9 , 33  27  10 , 34  30  13 , 35  5 , 36  15 , 37  11 ,
38  16 , 39  14 , 310  8 , 311  7 , 312  4 , 313  12 , 314  2 , 315  6 , 316  1 .
Así pues, 3 tiene orden 16 módulo 17, y por tanto es una raíz primitiva módulo 17.

a 1 2 3 4 5 6 7 8 9 10 11 12 13 14 15 16
ind3a 0 14 1 12 5 15 11 10 2 3 7 13 4 9 6 8

b)
7 x  5 mod 17  
ind 3 7 x   ind 3 5 mod 16  
ind 3 7   ind 3 x   ind 3 5 mod 16  
ind 3 x   ind 3 5  ind 3 7  mod 16 

Determinamos los índices de la derecha en la tabla del apartado (a):


ind 3 7   ind 3 x   ind 3 5 mod 16  
ind 3 x   5  11  6  10 mod 16  
x 8 mod 17 
c)
 
x 7  5 mod 17   ind 3 x 7  ind 3 5 mod 16  
7ind 3  x   ind 3 5 mod 16   7ind 3  x   5 mod 16  
ind 3  x   7 1  5 mod 16   ind 3  x   7  5  35  3 mod 16  
x  10 mod 17 

d)
x 8  8 mod 17  
 
ind 3 x 8  ind 3 8mod 16  
8  ind 3 x   10mod 16 
Pero esta última congruencia no tiene solución, puesto que  8 , 16   8 , y 8 | 10 .
16 Problemas de la cuarta parte.

16.1 MD
Demuestra que existen infinitas parejas a, b de números enteros positivos distintos y
relativamente coprimos a  1 y b  1 tales que a b  b a es divisible entre a  b .

USAMO 2017 #1

16.2 MD
Sea S el conjunto de todos los números racionales que se pueden escribir como
decimales periódicos de la forma [Link] , donde al menos uno de los dígitos a, b, c, d
no es cero. Sea N el número de numeradores distintos que se encuentran en S cuando
estos decimales se escriben en forma de fracción irreducible. Por ejemplo, 4 y 410
4 410
aparecen en S porque 0.3636  y 0.1230  . Determina el residuo cuando N se
11 3333
divide entre 1000.

AIME I 2022 #13

16.3 MD
Dado que 20 22  1 tiene exactamente 4 divisores primos p1  p2  p3  p4 , determina
p1  p2 .

SMT 2022 Discrete #8

16.4 MF
Seleccionamos al azar un entero N en el rango 1  N  2020 . Determina la probabilidad
que el residuo al dividir N 16 entre 5 sea 1.

(A) 1/5 (B) 2/5 (C) 3/5 (D) 4/5 (E) 1

AMC 10B 2017 #14


17 Números factoriales.
Definición. Función suelo, o parte entera.
Definimos la parte entera de un número real x , y la denotaremos por x  , como el
mayor entero n menor o igual que x . Por ejemplo, 2.4  2 , 1.99   1 ,  3.15   4

El Tema 9 de PA está dedicado íntegramente a la función parte entera. Se


recomienda practicar con ella realizando los problemas que allí se proponen.

17.2 M
Calcula la suma
 20   21   22   21000 

3 3 3   ...   3 
       

All Russian MO 2000

17.3 F
¿Cuál es la probabilidad de que un entero del conjunto  1 , 2 , 3 , ... , 100  pueda ser
divisible entre 2 y no divisible entre 3?

AMC 10A 2003 #15

Orden p-ádico de un número. La fórmula de Polignac.


17.4 Definición. Orden p-ádico de un número.
Dado un número entero n , su valuación p-ádica o orden p-ádico es la potencia
máxima de p que divide a n :
v p (n)  k  p k | n y p k 1 | n
o equivalentemente:
v p (n)  max  k , p k | n 

o equivalentemente:
Si n  p1a1 p2a 2 ... piai ... pna n es la descomposición canónica de n , entonces
v pi (n)  ai .

En Mathematica: IntegerExponent[n,p]

Antes de presentar la fórmula de Polignac, propondremos un par de ejemplos para


entender por qué aparecen los elementos que la constituyen.
17.5 Problema resuelto.
¿Cuál será el exponente del factor 5 en la factorización en números primos de 1005 ! ?

Solución.
1005 !  1  2  3...  1004  1005

De todos los factores de la derecha, uno de cada cinco es múltiplo de 5, luego habrán
1005 5
0 201
201 múltiplos de 5, que contribuirán en una unidad al exponente de 5.

De los 201 anteriores, uno de cada cinco será múltiplo de 25, luego habrán
201 5
1 40
40 múltiplos de 25, que contribuirán con una unidad más al exponente de 5.

De los 40 anteriores, uno de cada cinco será múltiplo de 5 3 , luego habrán


40 5
0 8
3
8 múltiplos de 5 , que contribuirán con una unidad más al exponente de 5.

De los 8 anteriores, uno de cada cinco será múltiplo de 5 4 , luego habrá


8 5
3 1
4
un único múltiplo de 5 , que contribuirá con una unidad más al exponente de 5.

Está claro que no existirán múltiplos de 55 ,56 ,...


Luego el exponente de 5 será 201  40  8  1  250

17.6 Teorema. La fórmula de Polignac.


Estos mismos cálculos los podríamos haber realizado con cualquier número primo y con
cualquier factorial. Para expresarlos en una única fórmula con el lenguaje algebraico
observamos lo siguiente:
1005 / 5  201 , 1005 / 5   40 , 1005 / 5   8 , 1005 / 5   1
2 3 4

Luego la suma anterior se puede expresar mediante la función “parte entera”,


justificando la denominada fórmula de Polignac:

La mayor potencia de un número primo p que divide a n! , es decir, el orden p-adico de


n! viene dado por

n 
v p n!    k 
k 1  p 

Esta fórmula también es conocida como "fórmula de Legendre" (1806).


Observación: Esta serie es en realidad una suma finita, pues para k suficientemente
n
altos se cumple n  p k y por tanto  i   0 para todo i  k .
p 
17.7 Ejemplo resuelto.
¿Cuál será el exponente del factor 2 en la factorización en números primos de 1005 ! ?

Solución.
En 1005 ! uno de cada dos factores es par, luego
1005 2
1 502

hay 502 números pares, de los cuales, uno de cada dos será múltiplo de 2 2 :
502 2
0 251

De estos 251, la mitad serán múltiplos de 2 3 :


251 2
1 125

De estos 125, la mitad serán múltiplos de 2 4 :


125 2
1 62

De estos 62, la mitad serán múltiplos de 2 5 :


62 2
0 31

De estos 31, la mitad serán múltiplos de 2 6 :


31 2
1 15

De estos 15, la mitad serán múltiplos de 2 7 :


15 2
1 7

De estos 7, la mitad serán múltiplos de 2 8 :


7 2
1 3

De estos 3, la mitad serán múltiplos de 29 :


3 2
1 1

Y ya no hay múltiplos de 210 , 211 , 212...

El exponente de 2 en la factorización será 502+251+125+62+31+15+7+3+1=997


17.8 Problema resuelto.
Determina el mayor n tal que 10 n divide a 1005 !

AHSME 1977

Solución.
Puesto que 10  2  5  10 n  2n 5n , el número n quedará determinado por el número de
parejas 2  5 que podemos hacer en la descomposición factorial de 1005 !, es decir, el
mínimo entre a y c
donde 2a  3b  5c...  1005! es la descomposición factorial de 1005 !

Acabamos de ver que a  997 y c  250 , luego el resultado será min 250,997  250 .

O, escrito en la forma de la fórmula de Polignac:



1005 
El exponente de 2 será   k   502  251  125  62  31  15  7  3  1  997
k 1  2 

1005 
El exponente de 5 será   k   201  40  8  1  250
k 1  5 

17.9 Problema resuelto.


¿Cuántos ceros hay a la derecha de la expresión decimal de 300 ! ?

Solución.
El número de ceros vendrá dado por el número de veces que 10 divide a 300! .
Puesto que hay más factores de 2 que de 5 en 300! , el número de ceros vendrá dado la
mayor potencia de 5 en 300! , y aquí aplicamos la fórmula de Polignac:

 300   300   300   300 


 51   60 ,  52   12 ,  53   2 , y si k  4  5k   0 .


 300 
Por lo tanto,   5
k 1
k 

 60  12  2  74 .

17.10 M
1000 
Demostrar que 7 no divide a   .
 500 

17.11 F
 200 
¿Cuál es el mayor factor primo de dos dígitos del entero n    ?
 100 
AIME 1983 #8
17.12 MD
Determina todos los pares ( k , n) de enteros positivos tales que

   
k! 2n  1 2n  2 2n  4 ... 2n  2n 1 
IMO 2019 #4

17.13 MF
Determina el mayor entero positivo x tal que 23 6  x divide 2000 !

17.14 Corolario de 17.6.


n 1
a) Si n  p k , con p primo, v p n ! 
p 1

En particular:
p 1
b) Si n  p , con p primo, v p  p !  1.
p 1
p2 1
 
c) Si n  p 2 , con p primo, v p p 2 ! 
p 1
 p 1.

Demostración.
n  n   n   n  pk 1 n 1
v p n !      2   ...   k 1    k   p k 1  p k 2  ...  p  1  
 p  p  p  p  p 1 p 1

17.14 MD
Determina el número de enteros n entre 1 y 50, inclusive, tales que
n 2  1!
n!n
es un entero (Recuerda que 0! 1 ).

(A) 31 (B) 32 (C) 33 (D) 34 (E) 35

AMC 12A 2019 #24


18 Números combinatorios.

El Tema 4 de PC está dedicado íntegramente a los números combinatorios. Se


recomienda practicar realizando los problemas que allí se proponen.

18.1
Demuestra que la expresión
(m, n)  n 
 
n  m 
es un entero para todo par de números enteros n  m  1 .

Putnam 2000

18.2 Proposición.
 p
a) Si p es primo, p divide   para todo 0  k  p .
k
b) Si p es primo, p | 2  2 .
p

Demostración.
 p  p  1  p
a) De la identidad k    p  se deduce que p divide k   .
k   k 1 k
 p
Puesto que 0  k  p , tenemos  k , p   1 , y por tanto p divide   por 4.5c.
k
 p  p  p
b) 2 p        ...   
0 1  p
 p  p  p  p 
Y puesto que       1 , tenemos 2 p  2     ...   
 0   p 1  p  1
 p
Aplicando el apartado anterior, p |   para todo 0  k  p , luego p | 2 p  2 .
k
Números de Catalan.

18.3 Definición. Número de Catalan de orden n.


Definimos el número de Catalan de orden n como
1  2n 
Cn   
n  1  n 

Los primeros diez números de Catalan son 1, 2, 5, 14, 42, 132, 429, 1430, 4862, 16796.

18.4 Proposición.
Todos los números de Catalan son enteros.

Demostración.
2n  1  2n   2n  1
Vamos a aplicar la identidad    
n  1  n   n  1 
Puesto que 2n  1 y n  1 son coprimos, y la parte de la derecha de la identidad anterior
 2n 
es un entero, está claro que n  1 divide a   .
n
19 Números primos de Fermat y de Mersenne.
19.1 Números primos de Fermat.
19.1.1 Definición. Números de Fermat. Primos de Fermat.
Queremos estudiar los números primos de la forma 2n  1 . Ya vimos en el Tema 4 que
si es primo entonces n debe ser una potencia de 2.

Definimos el n-ésimo número de Fermat como Fn  22  1 , n  0 .


n

F0 = 3, F1 = 5, F2 = 17, F3 = 257, F4 = 65537, F5 = 4294967297

El propio Fermat conjeturó que Fn era primo para todo n, sin embargo Euler demostró
que 641 | F5 . El razonamiento se encuentra en el Tema 4.

Así pues, no todo número de Fermat es primo. Llamaremos “primos de Fermat” a


aquellos números de Fermat que sean primos. Es un problema abierto la existencia o no
de infinitos primos de Fermat. Se conjetura que Fn es compuesto para n  5 .

19.1.2 Proposición.
Los números de Fermat son todos coprimos entre ellos:  Fn , Fm   1 si n  m .

Demostración.
Supongamos que m  n , y sea d un divisor común de Fn y Fm . En primer lugar vemos
que d debe ser impar, pues lo son todos los números de Fermat.
d | Fn  22  1  22  1  0 (mod d )  22  1 (mod d )
n n n

Pero entonces
22  22
m n m n
2
 22 n 2 m n
  1
2 m n
 1 (mod d )

Pero, por otro lado, d | Fm  22  1  22  1  0 (mod d )  22  1 (mod d ) , con lo


m m m

cual llegamos a 1  1 (mod d )  2  0 (mod d )  d | 2 , y puesto que d es impar, solo


puede ser d  1 .

Nota: Un argumento alternativo podría ser aprovechar la identidad


Fn  2  F0 F2 ...Fn 1

Que se deduce de
2 2  1  2  12  12 2  1... 2 2
m
 n 1

1

Y por tanto, si d divide Fn y Fm , con m  n , también dividirá a 2  Fn  F0 ...Fn 1 , y


puesto que d es impar, llegamos a d  1
19.2 Números primos de Mersenne.

19.2.1 Números de Mersenne. Primos de Mersenne.


Definimos los números de Mersenne como aquellos de la forma M n  2n  1 , n  1.

Está claro que si n es compuesto, también lo será M n :


Si n  ab  M n  2ab  1  2a   1b  2a  12a 1  2a  2  ...  2  1  2a  1 | M n
b

Por lo tanto, si M n es primo, también lo será n.

Sin embargo, existen primos p para los cuales M p es compuesto. Por ejemplo:
47 | M23, 167 | M83, 263 | M13

Llamaremos “primos de Mersenne” a los números de Mersenne que sean primos. Es


un problema abierto la existencia o no de infinitos primos de Mersenne.

19.2.2 Los mayores primos de Mersenne descubiertos hasta ahora.


El mayor número primo de Mersenne que se conoce es 282589933 1 , descubierto en
diciembre del 2018.

Los primeros 38 primos de Mersenne son los siguientes:

22−1 , 23−1 , 25−1 , 27−1 , 213 – 1 , 217−1 , 219−1 , 231−1 , 261−1 , 289 −1 , 2107−1 ,
2127−1 , 2521−1 , 2607−1 , 21279−1 , 22203−1 , 22281−1 , 23217−1 , 24253−1 , 24423−1 ,
29689−1 , 29941−1 , 211213−1 , 219937−1 , 221701−1 , 223209−1 , 244497−1 , 286243−1 ,
2110503−1 , 2132049−1 , 2216091−1 , 2756839−1 , 2859433−1 , 21257787−1 , 21398269−1 ,
22976221−1 , 23021377−1 , 26972593 −1.
20 Número y suma de los divisores de un entero.
20.1 Número de divisores de un entero.
20.1.1 Definición. Número de divisores de un entero.
Dado un número entero n, definimos  (n ) como el número de enteros positivos
divisores de n.

Ejemplos: Los divisores positivos de 6 son 1 , 2 , 3 , 6 , luego  (6)  4 .


Los divisores positivos de 20 son 1 , 2 , 4 , 5 , 10 , 20 , luego  (20 )  6 .
 (1)  1,  (2) = 2,  (3) = 2,  (4) = 3,  (5) = 2,  (6) = 4,  (7) = 2,  (8) = 4,  (9) = 3,
 (10) = 4
Problema motivador: #1.2.1.
Con Mathematica:

20.1.2 Proposición.
Para cualquier entero positivo n,
a)  (n)  1 y  (n)  1  n  1 .
b) Para todo n  2 ,  ( n)  2 y  (n)  2  n es primo.
c)  (n)  2 n

Demostración. a) y b) se deducen de la propia definición de  (n ) .


c) Cada divisor positivo a de n tiene asociado su divisor complementario b  n / a .
Puesto que a  b  an / a  n , se tiene que cumplir a  n o b  n . Así pues,
existirán como mucho 2 n divisores de n.

20.1.3 Teorema.
La función d (n) es multiplicativa, es decir, si n  a b , con (a, b)  1 , entonces
 ( n)   (a ) (b)

20.1.4 Corolario.
a) Si n  p a para cierto p primo, entonces los divisores de n son

1 , p , p 2 , p 3 , ... , p a 
y por tanto  ( n)  a  1

b) Si n  p1a1 p2a2 ... prar es la descomposición en factores primos de n, entonces los


divisores d de n son todos los números que se pueden escribir como
d  p1b1 p2b2 ... prbr con 0  bi  ai ,
y
 (n)  a1  1a2  1...ar  1

Por ejemplo:  (2904 )  d (23  3 112 )  (3  1)(1  1)(2  1)  24


20.1.5 F
Demostrar que  (n ) es impar si y solo si n es un cuadrado perfecto.

20.1.6 F
Demostrar que  (n) es primo si y solo si n  p q1 con p, q primos.

20.1.7 MF
Si tomamos aleatoriamente un divisor de 10 99 , ¿Cuál es la probabilidad de que sea
múltiplo de 10 88 ?

AIME 1998 #5

20.1.8 F
Determina todos los enteros positivos n tales que  ( n)  6 .

20.1.9 F

Determina el número de enteros positivos que son divisores de al menos uno de los
siguientes números: 10 10 , 15 7 , 1811 .

AIME II 2005 #4

20.1.10 D
Sea  (n ) el número de enteros positivos de n. Determina la suma de los seis enteros
positivos n más pequeños tales que
 (n)   (n  1)  7

AIME 1 2019 #9

20.1.11 F
¿Cuántos divisores positivos de 2004 2004 son divisibles por exactamente 2004 enteros
positivos?

AIME II 2004 #8

20.1.12 F
Joey, Chloe y su hija Zoe cumplen años el mismo día. Joey es un año mayor que Chloe
y Zoe cumple hoy su primer año. Hoy es el primero de los nueve aniversarios en los que
la edad de Chloe será múltiple de la edad de Zoe. Determina la suma de los dos dígitos
de la edad de Joey que tendrá la próxima vez que sea múltiple de la edad de Zoe.

(A) 7 (B) 8 (C) 9 (D) 10 (E) 11

AMC 12B 2018 #14


20.1.13 F
Sea N un entero positivo. Supongamos que existen exactamente 2005 pares ordenados
x, y  de enteros positivos tales que
1 1 1
 
x y N

Demuestra que N es un cuadrado perfecto.

BMO 2005 Round 2 #1

20.1.14 Proposición. Producto de los divisores de un entero.


Una propiedad interesante de  (n ) es que el producto de todos los divisores de n es
igual a la raíz cuadrada de n ( n ) :
d 
d |n
n ( n )

Demostración.
La clave está en observar que los divisores de un número van por parejas: Dado un
divisor d , tenemos su divisor complementario d ' n / d , de forma que d  d '  n .
Sean d1 , d 2 , ..., d ( n ) todos los divisores de n. Si multiplicamos dos veces dichos
divisores, los factores se pueden reorganizar agrupando  (n ) parejas cuyo resultado sea
n, luego:
2 2
   
  d    d   d  d1.d 2 ...d ( n )  d1.d 2 ...d ( n )  n ( n )    d   n ( n )
   
 d |n  d |n d |n  d |n 

20.1.15 Corolario.
De la proposición anterior se deduce que n ( n ) es un cuadrado perfecto. Esto es obvio si
 (n ) es par, pero no estaba tan claro si  (n ) es impar. Pero en el problema 20.1 anterior
ya demostramos que si  (n ) es impar, n es un cuadrado perfecto, y por tanto también lo
es n ( n ) .

20.1.16 Ejemplo.
El producto de todos los divisores de 16 es
 d | 16 
d |n
16 ( n)  16 5  45  1024

En efecto, 1 2  4  8 16  1024 .


20.1.17 MD
Denotamos por d (n) el número de enteros positivos que dividen a n, incluyendo 1 y n.
Por ejemplo: d (1)  1 , d (2)  2 , d (12 )  6 (esta función es conocida como la función
divisor). Sea
d ( n)
f ( n)  3
n

Existe un único entero positivo N tal que f ( N )  f (n) para todo entero positivo
n  N . Determina la suma de los dígitos de N.

(A) 5 (B) 6 (C) 7 (D) 8 (E) 9

AMC 12A 2021 #25


20.2 Suma de los divisores de un entero.

20.2.1 Definición. Suma de los divisores de un entero.


Dado un número entero n, definimos  (n) como la suma de todos los enteros positivos
divisores de n, incluyendo 1 y el propio n.

 ( n)   d
d |n

Está claro que  (n)  n  1 , y  ( n)  n  1 si y solo si n es primo.

Ejemplos.
Los divisores positivos de 6 son 1 , 2 , 3 , 6 , luego  (6)  1  2  3  6  12 .
Los divisores positivos de 20 son 1 , 2 , 4 , 5 , 10 , 20 , luego
 (20 )  1  2  4  5  10  20  42 .

20.2.2 Teorema.
La función  (n) es multiplicativa.

20.2.3 Proposición.
a) Si n  p a para cierto primo p , entonces  (n)  1  p  p 2  ...  p a .
b) Si n  p1a1 p2a2 ... prar es la descomposición en factores primos de n, entonces

 (n)  1  p1  p12  ...  p1a 1  p2  p22  ...  p2a ...1  pr  pr2  ...  pra  
1 2 r

p1a1 1  1 p2a 2 1  1 pra r 1  1


  ...
p1  1 p2  1 pr  1

Demostración.
a) Aplicando la propia definición de  (n) .
b) Basta tener en cuenta que  (n) es multiplicativa y aplicar la serie geométrica:
n1 n 2 x n1  1
x x
n
x  ...  x  1 
x 1

Por ejemplo:
23  1 33  1 52  1 7 26 24
180  22  32  5   (180 )        7 13  6  546
2 1 3 1 5 1 1 2 4
20.2.4 Problema resuelto.
Sea n un entero positivo tal que 24 | n  1 . Demuestra que entonces se cumple
24 |  (n) .
Putnam 1969

Solución.
n  1  0 (mod 3)  n  -1 (mod 3)
24 | n  1  
n  1  0 (mod 8)  n  -1 (mod 8)

Observamos que los divisores de un número van en parejas: Si d1 es divisor entonces su


n
complementario d 2  también es divisor.
d1
Por un lado, haciendo módulo 3:
n
d1  d 2  d1  n  1 (mod 3) , y las únicas opciones son:
d1
d1  1 (mod 3) y d2  2 (mod 3) o viceversa.
En todo caso se cumple d1  d2  0 (mod 3) .

Y por otro lado, haciendo módulo 8:


n
d1  d 2  d1  n  1 (mod 8) , y las únicas opciones son:
d1
d1  1 (mod 8) y d2  7 (mod 8) o viceversa.
d1  3 (mod 8) y d2  5 (mod 8) o viceversa.
En todo caso se cumple d1  d2  0 (mod 8) .

Así pues, d1  d2  0 (mod 3) y d1  d2  0 (mod 8) , luego d1  d2  0 (mod 24) , y por


tanto el sumatorio  (n) se puede reordenar por parejas divisibles entre 24, y por tanto:
 (n)   d será divisible entre 24.
d |n

20.2.5 F
Los únicos factores primos de un número n son 2 y 3. Si la suma de todos los divisores
de n (incluyendo el propio n) es 1815, determina n.

PUMaC 2011/NT #A1


20.3 Números perfectos.

20.3.1 Definición de número perfecto.


Los antiguos griegos clasificaban los enteros positivos en tres clases, en función de la
suma de sus divisores:
“Números perfectos” si  (n)  2n
“Números abundantes” si  (n)  2n
“Números deficientes” si  (n)  2n

Los primeros números perfectos son: 6 , 28 , 496 , 8128 .

Si utilizamos la notación clásica en la que se cuentan solo los divisores propios, es


decir, todos menos el propio n, tiene más sentido la idea de “número perfecto” como
aquel que es igual a la suma de todos sus divisores (propios).

20.3.2 Teorema. Caracterización de los números pares perfectos.


Un número par n es perfecto si y solo si existen números primos p, q tales que
n  2 p1 q con q  2 p  1

Así pues, cada número par perfecto está asociado a un primo de Mersenne, que fueron
introducidos en el Tema 19.

Demostración.
 Supongamos que n se puede escribir de esta forma.
Está claro que q es impar , 2n  2 p q y que q  1  2 p .
2 p11  1
 2 p1    2 p  1 ,  q  q  1
2 1
   
Luego  (n)   2 p 1  q   2 p  1 q  1  2 p q  2 p  (q  1)  2n
Y por tanto n es perfecto.

 Supongamos ahora que n es perfecto, es decir, que  (n)  2n . Puesto que además
estamos suponiendo que n es par, podemos escribir n  2 k 1 m con m impar y k  2 .

   
Luego 2k m  2n   (n)   2k 1 m   2k 1  m  2k  1  m  
2k 11  1
Donde hemos utilizado que  2 k 1
   2k  1 .
2 1

 
De la igualdad 2k m  2k  1  m se deduce que 2k  1 | 2k m , y puesto que
 
2k  1 , 2k  1 , el Lema de Euclides implica que 2k  1 | m , y por tanto

 
m  2k  1 l para cierto entero l impar.
2k 2k  1l  2k m  2n   (n)   2k 1 (2k  1)l    2k 1  (2k  1)l  
   
 2k  1  (2k  1)l  2k l   (2k  1)l   (*)
 
Si l  1 entonces 1, l , (2k  1)l son factores diferentes de 2k  1 l , y por tanto, por la
 
definición de  , tenemos que 1  l  2k  1 l   2k  1 l 
   
Luego 2k l   2k  1 l  1  l  2k  1 l  1  l  2k l  l  1  2k l absurdo.
 
Así pues, l  1, y n  2k 1 2k  1 . Ahora solo falta demostrar que k es primo.

De (*) tenemos que 2k   2k  1  1  2k  1  d  d  0


d |( 2 k 1) d | ( 2 k 1)
1 d  2 k 1 1 d  2 k 1

Es decir, 2k  1 no tiene divisores propios, es decir, es un número primo. Ya vimos en el


Tema 15 que si un número de Mersenne es primo entonces k es primo, y con esto
acabamos la demostración.

20.3.3 Observación.
Acabamos de ver que todo número perfecto par está asociado a un primo de Mersenne.
Es un problema abierto la existencia o no de infinitos números perfectos pares, y apenas
sabemos nada de los números perfectos impares, ni tan solo si existe alguno.
21 Encriptación mediante Curvas Elípticas. Encriptación bitcoin.

En 1985, Neal Koblitz y Victor Miller propusieron independientemente la criptografía


basada en curvas elípticas. Es la técnica utilizada para el bitcoin entre otras
criptomonedas.

21.1 Curvas elípticas sobre cuerpos en general.


Trabajar con números reales, es decir, en el plano IR IR , nos permite visualizar las
operaciones que realizamos, con lo que “vemos qué estamos haciendo”, pero es un
plano teórico, es un primer paso para tomar confianza con los métodos y fórmulas
utilizadas para pasar después a trabajar sobre cuerpos finitos, que es como trabajan las
computadoras.

Una curva elíptica es una curva en el plano con ecuación


y 2  x3  a x  b

La curva elíptica utilizada por Bitcoin, Ethereum y muchas otras criptomonedas es la


curva secp256k1, cuya ecuación es:
y 2  x3  7

Es decir, es un caso particular de curva elíptica tomando a  0 y b  7 . Su


representación gràfica sobre los reales es la siguiente:

Hay que dejar muy claro que esta representación de la curva es sobre el cuerpo IR , y
por lo tanto se presenta como una colección de puntos ordenados siguiendo una
trayectoria suave y contínua, pero en criptografía se utilizan cuerpos finitos, y en estos
cuerpos esta misma curva se presenta como una nube de puntos con poca o ninguna
continuidad visible.

Curvas elípticas no singulares.


Diremos que una curva elíptica y 2  x3  a x  b es no singular cuando su discriminante
sea distinto de cero:
4a 2  27 b 2  0
Suma de puntos en una curva elíptica.
Sean P   x1 , y1  y Q   x2 , y2  con P  Q dos puntos de una curva elíptica.
Definimos el punto R  P  Q , al que llamaremos suma de P y Q, de la siguiente
forma:
1) Trazamos la recta r que pasa por P y Q.
2) Determinamos el tercer punto S de corte de esta recta con la curva elíptica.
3) Trazamos el simétrico de S respecto del eje X.

Con coordenadas:
Si P   x1 , y1  y Q   x2 , y2  con P  Q , las coordenadas de R   x3 , y3  son:
2
y y   y  y1 
x3   2 1   x1  x2 , y3   2  x1  x3   y1
x 
 2 1
x x 
 2 1x
Ejemplo.
Dada la curva elíptica y 2  x3  9 x  9

P   0 ,  3  , Q   1,  1   R  P  Q   3 ,  3 

P    3 ,  3  , Q   1,  1   R  P  Q   9 / 4 , 3 / 8    2.25 , 0.375 
El punto del infinito.
Si P   x1 , y1  y Q   x2 , y2  con x1  x2 , la recta r  PQ es vertical y no tiene ningún
otro punto de corte con la curva. Para evitar esta excepción, añadiremos a nuestro
conjunto de puntos un punto al que denotaremos por O y que llamaremos “elemento
cero” o “punto del infinito”, que será, por definición, el punto de intersección entre una
recta vertical y la curva.

Multiplicación escalar de puntos en una curva elíptica.


Cuando Q  P , la recta r  PQ se convierte en la recta tangente por el punto P a la
curva elíptica, y esto nos justifica definir el punto R  2P  P  P de la siguiente forma:
1) Trazamos la recta r tangente a la curva elíptica por el punto P .
2) Determinamos el tercer punto S de corte de esta recta con la curva elíptica.
3) Trazamos el simétrico de S respecto del eje X.

Con coordenadas:
Si P   x1 , y1  y la curva elíptica es y 2  x3  a x  b , las coordenadas de
2P   x3 , y3  son:
2
 3x 2  a   3x 2  a 
x3   1   2 x1 , y3   1  x1  x3   y1
 2 y1   2 y1 
Ejemplo.
Sea P   0 , 1  de la curva elíptica y 2  x3  2 x  1 , entonces 2P   1 , 0 

Múltiplos de un punto. Orden de un punto.


Una vez que hemos definido 2P  P  P , podemos definir 3P  P  2P ,
4P  P  3P ,... a los que llamaremos múltiplos del punto P.

Dado un punto P, definimos su orden como el menor natural n tal que nP  O , si es


que existe.
Método efectivo para calcular múltiplos de un punto. El método del “dobla y suma”.
La misma técnica para optimizar la exponenciación que explicamos en el apartado 12.2
se puede aplicar perfectamente al problema de optimizar la multiplicación de puntos.

Supongamos que queremos calcular 227 P para cierto punto P dado. No es nada
efectivo ir calculado, una tras otra, las 226 multiplicaciones
2P  P  P  3P  P  2P  4P  P  3P  ...  227 P  P  226 P

Es mucho más efectivo aprovechar los cálculos anteriores para ir calculando las
potencias de dos:
2P  P  P
4 P  2 P  2 P  22 P 
8P  4 P  4 P  24 P 
16 P  8P  8P  28P 
...
Y ahora, puesto que la descomposición binaria de 227 es 11100011, tenemos
227 P  27 P  26 P  25 P  21 P  20 P
En total, entre potencias 2k P y las sumas anteriores hemos reducido el cálculo a 8
operaciones.
Este método se llama “dobla y suma” y permite calcular puntos n P con n enormes en
un número reducido de operaciones. Por ejemplo, aunque el número n fuera tan grande
como el número de átomos del universo, del orden de 10 82  2275 , este método
permitiría determinar n P en apenas 275 operaciones.

El problema del logaritmo discreto en curvas elípticas (ECDLP).


Planteamos ahora el problema inverso:
Dados dos puntos P y Q, determinar el valor de n tal que Q  n P

Llamaremos a resolver este problema el “logarítmo discreto de P”.

Acabamos de ver que, dado un punto P y un número natural n, es fácil y rápido


determinar el punto n P mediante la técnica de “dobla y suma”. Pero el problema
inverso, dado un punto P y el punto n P , es terriblemente lento determinar el valor de
n , pues no podemos aplicar el truco anterior y debemos tenemos que ir probando
múltiplo a múltiplo:
P  2P  3P  4P  ...

La terrible dificultad de encontrar el logarítmo discreto se llama “Elliptic Curve


Discrete Logarithm Problem” (ECDLP), y es el fundamento de la encriptación
mediante curvas elípticas.
Este problema es similar a otros logaritmos discretos estudiamos anteriormente, y la
ventaja del ECDLP respecto de los anteriores es que parece ser mucho más duro, y por
tanto mucho más seguro.
Una vez introducidas estas ideas, pasaremos a desarrollarlas en planos sobre cuerpos
finitos.
21.2 Curvas elípticas sobre cuerpos finitos.
La curva secp256k1, sobre F17 consta de los siguientes 17 puntos:

(1,5) ; (1,12) ; (2,7) ; (2,10) ; (3,0) ; (5,8) ; (5,9) ; (6,6) ; (6,11) ; (8,3) ; (8,14) ;(10,2); (10,15) ; (12,1) ; (12,16) ; (15,4) ; (15,13)

Y su representación gráfica es:

Esta misma curva, sobre F257 consta de 257 puntos y su representación en un sistema
X-Y es una nube de puntos:

El bitcoin trabaja sobre F2 28 1 , y sobre este cuerpo la curva secp256k1 es ya una nube
de
115792089237316195423570985008687907852837564279074904382605163141518161494337

puntos totalmente dispersos, imposible de representar visualmente. Una muestra sería la


siguiente:
Práctica de Python.
Crea un programa de Python que genere todos los puntos de la curva secp256k1 módulo
un cuerpo Fp dado:

#
# Generador de puntos de la curva secp256k1 módulo p por Brute-Force
#
p=257
Total=0
for x in range (0,p):
for y in range (0,p):
v=y**2-x**3-7
if v%p==0:
print(x,",",y)
Total=Total+1
print("Total de puntos:",Total)

Exporta estos puntos generados por Python a una hoja de Excel mediante las opciones
de "importar dados" de la hoja de cálculo, para representarlos mediante una nube de
puntos.

Comprueba, mediante este programa, los resultados obtenidos en los ejemplos


anteriores, y genera la nube de puntos asociada a la curva secp256k1 para el caso del
cuerpo F23 .

La encriptación bitcoin.
Para encriptar bitcoin se utiliza la curva elíptica secp256k1, cuya ecuación es:
y 2  x3  7

Sobre el campo finito Z p , con p  2256  232  977 .

Aquí se eligió p para estar relativamente cerca de 2256, aunque no es el primo más
grande menor de 2256; hay muchos primos entre p y 2256 . Otros factores también
entraron en la elección de p.

El punto base elegido para el bitcoin es, en notación hexadecimal:


x = 79BE667EF9DCBBAC55A06295CE870B07029BFCDB2DCE28D959F2815B16F81798
y = 483ADA7726A3C4655DA4FBFC0E1108A8FD17B448A68554199C47D08FFB10D4B8

Este punto base se eligió para tener un módulo grande, y de hecho su orden es
aproximadamente 2256. Específicamente, su orden, escrito en hexadecimal, es

n = FFFFFFFFFFFFFFFFFFFFFFFFFFFFFFFEBAAEDCE6AF48A03BBFD25E8CD0364141

El siguiente programa Python nos permite comprobar que, efectivamente, este punto
pertenece a la curva dada:

x = 0x79BE667EF9DCBBAC55A06295CE870B07029BFCDB2DCE28D959F2815B16F81798
y = 0x483ADA7726A3C4655DA4FBFC0E1108A8FD17B448A68554199C47D08FFB10D4B8
p = 0xFFFFFFFFFFFFFFFFFFFFFFFFFFFFFFFFFFFFFFFFFFFFFFFFFFFFFFFEFFFFFC2F
print( (y*y-x*x*x-7)%p == 0 )

Cuyo output es, obviamente, “True”.


Otro ejemplo real de los parámetros utilizados en ECDH por [Link] son los de la
curva P-384: x2=y3+ax+b mod(p) , con:

a=115792089210356248762697446949407573530086143415290314195533631308867097853948
b=1058363725152142129326129780047268409114441015993725554835256314039467401291
p=11579208921035624876269744694940757353008614341529031419553363130886709785395

Múltiples de un punto.
Tomemos la curva y 2  x 3  2 x  3 mod 97  y el punto P   3, 6  . Los múltiplos n P
generan una sucesión cíclica de orden 5, es decir, el orden del punto P es 5:

1P  P   3, 6 , 2P   80,10 , 3P   80, 87 , 4P  3, 91 , 5P  O , 6P  P , 7P  2P ,...

Además, está claro que estos cinco puntos forman un grupo cerrado con la suma: La
suma de dos múltiplos de P es también un múltiplo de P. Es lo que se llama un
“subgrupo” del grupo general de la suma de puntos.

Un resultado conocido de la Teoría de Grupos es que el orden de cualquier subgrupo de


un grupo dado es un divisor del orden de dicho grupo.

Por ejemplo, la curva y 2  x 3  x  3 mod 37  tiene orden 42. Por lo tanto, todos sus
subgrupos tendrán orden 1, 2, 3, 6, 7, 14, 21 o 42. En particular:

P   2 ,3 , 2P   23,14 , 3P   21,17 , 4P  21, 20 , 5P  23, 23 , 6P  2 , 34 , 7P  O

Y por tanto el orden de ( 2 , 3 ) es 7.

Para los propósitos de la criptografía ECC, interesa encontrar puntos que tengan el
mayor orden posible.
21.3 Protocolo de intercambio de claves Diffie-Hellmann en Curvas Elipticas
(ECDH).
Supongamos que Alicia y Bob desean generar una clave secreta común R que solo ellos deben
conocer, pero se tienen que comunicar a través de un espacio público no seguro.

Alicia Espacio público Bob

¡Todo lo que pase por aquí será


visto por el Enemigo!

Alicia y Bob pactan los


siguientes datos públicos:
1. La curva elíptica
y 2  x3  ax  b mod p 
usada , es decir, los
parámetros a , b , p
2. Un punto generador G de
esta curva.
Alicia envía a Bob estos datos
a ,b , p , G Bob recibe a , b , p , G

Determina, al azar, un Determina, al azar, un


número aleatorio  número aleatorio 
Es su clave privada. Es su clave privada.

Recibe Q   G Computa Q   G
y lo envía a Alicia

Computa P   G
Recibe P   G
Y lo envía a Bob

Computa R    G Computa R    G

  
Puesto que   G    G ,  El enemigo solo puede ver
Alicia y Bob pueden compartir esta Q   G y P  G
clave secreta R, que en ningún pero con ellos no es capaz de
momento será transmitida por el determinar ni  ni  , y por tanto
espacio público ni podrá ser
calculada por nadie más. no es capaz de determinar R,
gracias al “Problema del Logaritmo
Discreto en ECDH”.
22 Ecuaciones diofánticas.
Las ecuaciones diofánticas son aquellas ecuaciones en las que solo son admitidas
soluciones enteras, y son un problema transversal en la teoría de números. Estas
ecuaciones ya han aparecido a lo largo de todo este libro como aplicación de las
diversas técnicas presentadas, en sus respectivos apartados. Aquí se ofrece una
presentación algo más sistemática de algunos modelos concretos de ecuaciones
diofánticas, así como de algunas técnicas que pueden ser útiles para su resolución y un
recopilatorio de algunas ecuaciones diofánticas aparecidas en el contexto de pruebas
olímpicas.

22.1 Ecuaciones diofánticas lineales.


Definición. Ecuaciones diofánticas lineales.
Son las ecuaciones diofánticas que tienen la forma a1 x1  a2 x2  ...  an xn  b , con
ai  enteros.
Ejemplo 1.
Por ejemplo, la ecuación 3 x  6 y  18 tiene infinitas soluciones:
x  4 , y  1  3  4  6  1  18
x  6 , y  6  3  (6)  6  6  18
x  10 , y  2  3  10  6  (2)  18
En general, cualquier valor x  2k , y  3  k  3(2k )  6(3  k )  6k  18  3k  18
Sin embargo, la ecuación 2 x  10 y  17 no tiene solución: Para cualquier x, y , la parte
izquierda de la ecuación será par, mientras que la parte derecha es un impar.

Ejemplo 2.
Consideremos las siguientes ecuaciones diofánticas: 2 x  3 y  2 y 6 x  3 y  1 .
Observamos que la primera tiene infinitas soluciones (-5,4), (-2,2), (1,0), (4,-2), (7,-4)…
mientras que no parece existir ninguna solución para la segunda:

Esto se puede formalizar en el siguiente Teorema:


Teorema fundamental de las ecuaciones diofánticas lineales.
Una ecuación diofántica lineal a x  b y  c tendrá solución si y solo si (a, b) | c .

Si  x0 , y0  es una solución particular de esta ecuación, entonces todas las soluciones


son de la forma
b a
x  x0    k , y  y0    k para cualquier k entero.
d  d 

Ejemplo resuelto.
Resuelve la ecuación diofántica 172 x  20 y  1000

Solución. d  (172 ,20 )  4 , y 4 | 1000 , luego esta ecuación tiene solución.


Probando números vemos que 172  5  20  7  1000 , luego x0  5 , y0  7 es una
solución particular de la ecuación.
Por el Teorema anterior, las soluciones de esta ecuación son todas las parejas de la
forma
 20   172 
x  5    k  5  5 k  5(1  k ) , y  7    k  7  43 k , con k  Z
 4   4 
o equivalentemente, tomando q  1  k ,
x  5q , y  7  43(q  1)  7  43q  43  50  43q , con q  Z .

22.1.1 F
Resuelve la ecuación diofántica 7 x  9 y  3

22.1.2 F
Un cliente compra una docena de piezas de fruta, manzanas y naranjas, por 1.32€. Si
una manzana cuesta 3 céntimos más que una naranja, y se compraron más manzanas
que naranjas, cuantas piezas de cada fueron compradas?
Resolución de ecuaciones diofánticas lineales mediante el Algoritmo de Euclides.
El Algoritmo de Euclides para el cálculo del máximo común divisor de dos números
mediante sucesivas divisiones nos permite resolver ecuaciones diofánticas lineales. Lo
veremos con el siguiente ejemplo:

Resuelve la ecuación diofántica 2173 x  2491 y  53

Solución. Aplicamos el algoritmo de Euclides:


2491  1  2173  318 
2173  6  318  265 
  (2173 ,2491 )  53 , y claramente 53 | 53 , luego hay solución.
318  1  265  53 
265  5  53  0 

Deshaciendo los pasos del algoritmo de Euclides:


53  318  1  265 

265  2173  6  318   53  318  1  265  2491  1  2173  1  2173  6  318  
318  2491  1  2173 
 2491  1  2173  2173  6  2491  1  2173  
 2491  1  2173  2173  6  2491  6  2173 
 7  2491  8  2173

Luego x  8 , y  7 es una solución de la ecuación diofántica del enunciado.


El conjunto de soluciones de la ecuación serán las parejas de la forma:
2491 2173
x  8  k  8  47 k , y  7  k  7  41k
53 53

22.1.3 F
Resuelve la ecuación diofántica 858 x  253 y  33 mediante el algoritmo de Euclides.

22.1.4 F
Resuelve la ecuación diofántica 258 x  147 y  369

22.1.5 F
Resuelve la ecuación diofántica 60 x  33 y  9

22.1.6 F
Las medidas (en grados) de los ángulos interiores de un hexágono convexo forman una
sucesión aritmética de enteros positivos. Sea m º la medida del mayor de los ángulos
interiores de este hexágono. El mayor valor posible de m º es

(A) 165º (B) 167º (C) 170º (D) 175º (E) 179º
AHSME 1991 #12
22.1.7 F
Determina un número que, cuando se divide entre 10, deja un residuo de 9, cuando se
divide entre 9 deja un residuo de 8, entre 8 el residuo es 7, y así sucesivamente, hasta
que, finalmente, cuando se divide entre 2, deja un residuo de 1.
ASHME 1951 #37

22.1.8 F
Resuelve la ecuación diofántica lineal 19 x  17 y  1 .
22.2 Ternas pitagóricas.

Definición. Terna Pitagórica.


Definimos por Terna Pitagórica las ternas ( x, y , z ) de números enteros solución de la
ecuación diofántica
x2  y 2  z 2

Las ternas pitagóricas son un caso particular de la llamada Ecuación de Fermat:


xn  y n  z n

Que fue resuelta por Wiles en 1994, más de 350 años después de que este problema
fuera propuesto, demostrando que para n  2 no existen soluciones que no sean la
trivial x  y  z  0 .

Lema.
Si ( x, y , z ) es una terna pitagórica y d  mcd ( x, y, z ) , entonces:
a) d  mcd ( x, y )  mcd ( y, z )  mcd ( x, z )
b) Si escribimos x  d x ' , y  d y ' , z  d z ' , entonces ( x ' , y ' , z ' ) también es una terna
pitagórica.

Demostración.
a) Demostraremos la primera igualdad. Está claro que
d|x,d| y,d|zd|x,d| y
Por otro lado:

d | x  d 2 | x2 
2
 d 2 | x2  y 2  z 2  d 2 | z 2  d | z
d| yd | y 
2

b) Es trivial.

Definición. Terna pitagórica primitiva.


Una terna pitagórica ( x, y , z ) se dice que es primitiva si mcd ( x, y, z )  1 , o
equivalentemente mcd ( x, y )  mcd ( y, z )  mcd ( x, z )  1

Lema.
Sea ( x, y , z ) una terna pitagórica primitiva. Entonces uno y solo uno de los tres números
es par, y siempre es x o y .

Demostración.
x par  x 2 par  x imp  x 2 imp 

  z  x  y imp ,   z  x  y imp
2 2 2 2 2 2

y imp  y imp 
2
 y par  y par 
2

Recordemos que todo cuadrado perfecto es 0 o 1 módulo 4. Luego
x imp  x 2 imp  x 2  1 (mod 4) 

  z  x  y  2 (mod 4) absurdo.
2 2 2

y imp  y imp  y  1 (mod 4)


2 2

Puesto que estamos suponiendo ternas pitagóricas primitivas, el caso x,y ambos pares
no se puede dar.
Teorema. Caracterización de las ternas pitagóricas.
Las ternas pitagóricas primitivas son todas las ternas de enteros que se pueden escribir
de la forma

 x  m2  n2  x  2mn
 
 y  2mn  y  m  n (con x par)
2 2
(con y par) o bien
 z  m2  n 2  z  m2  n2
 

Donde m  n  0 son números coprimos con paridad diferente.

Demostración.
Que las ternas de enteros anteriores son pitagóricas es fácil de comprobar:
 
x2  y 2  z 2  m2  n2  2mn   m2  n2
2 2
  2

Simplificando:
m      
 n2  2mn   m2  n2  m2  n2  4m2n2  m2  n2 , que es una
2 2 2 2 2

2

aplicación directa de la igualdad (a  b)2  4ab  (a  b)2 .

Veamos que si una terna es pitagórica, necesariamente debe ser como las anteriores.
Vamos a demostrar la columna de la izquierda, es decir, cuando x es impar y y es par.

x2  y 2  z 2 se puede escribir como


y 2  z 2  x 2  z  x z  x 
Y por tanto:
2
 y y2 z  x z  x
    
2 4 2 2
zx zx zx zx
Teniendo en cuenta que   z,   x , y mcd ( x, z )  1 ,
2 2 2 2
llegamos a
 zx zx 
mcd  ,  1
 2 2 

zx zx
Y por tanto, aplicando 5.2.26b, y son cuadrados perfectos, es decir:
2 2
zx zx
 n2 y  m2 para ciertos enteros n y m.
2 2
Se cumple claramente que m  n y son enteros coprimos.
También se cumple que m y n tienen diferente paridad, puesto que z  m2  n 2 es impar.
Y se cumple x  m2  n 2 , y  2mn , z  m2  n 2 , tal y como queríamos ver.

22.2.1 D
Calcula todos los números enteros a, b y c tales que a 2  2b 2  3c 2
OMEFL 2011 #4
22.3 La ecuación diofántica x2  y 2  k .

Lema.
Un elemento importante para resolver la ecuación x2  y 2  k es observar que, por cada
posible factorización a  b  k ,
x  y  n
x 2  y 2  k  ( x  y )( x  y )  n  m  k  
x  y  m

Y el sistema de ecuaciones de la derecha tiene como única solución


nm
Sumando las dos ecuaciones: 2 x  n  m  x 
2
nm
Restando las dos ecuaciones: 2 y  n  m  y 
2

Aunque no todas las soluciones en x, y serán aceptables por no ser enteras: Está claro
que n  m y n  m deben ser ambos pares, luego n, m deben tener la misma paridad.

22.3.1 Ejemplo 1.
Determina todas las soluciones enteras de la ecuación
x2  y 2  108

Solución.
x2  y 2  108  ( x  y)( x  y)  22 33
Y por tanto
 x  y  2 3
a b

 para ciertos 0  a  2 , 0  b  3 .
 x  y  22  a33 b
Luego
 2a3b  22  a33 b
 x   2a 13b  21 a33 b
 2
 2  a 3b
 y  2 3  2 3  2a 13b  21 a33 b
a b


 2
El único valor de a para el que x, y son enteros es a  1 , y en este caso:

x  y  2  3
b

x  3  3
b 3 b

 

x  y  2  3
3 b

y  3  3
b 3 b

Dando valores a n aparecen las cuatro soluciones posibles con x  0 :


b  0  x  28 , y  26
b  1  x  12 , y  6
b  2  x  12 , y  6
b  3  x  28 , y  26
Las soluciones son ocho:
( x, y )  (  28 ,  26 ) , ( x, y )  (  12 ,  6 )
22.3.2 Ejemplo 2.
Determina todas las soluciones enteras de la ecuación
x 2  y 2  600

Solución.
x 2  y 2  600  ( x  y)( x  y)  23  3  52
Y por tanto
 x  y  2a  3b  5c

 para ciertos 0  a  3 , 0  b  1 , 0  c  2

 x  y  23 a
 31 b
 52c

Luego
 2a  3b  5c  23 a  31b  52  c
 x   2a 1  3b  5c  22  a  31b  52  c
 2
 3 a 1 b 2c
 y  2  3  5  2  3  5  2a 1  3b  5c  22  a  31b  52  c
a b c


 2

Luego a  1 o a  2 . En total tenemos las siguientes combinaciones:


a=1 , b=0 ,c=0, x= 151, y= -149 ; a=1 , b=0 ,c=1, x= 35, y= -25
a=1 , b=0 ,c=2, x= 31, y= 19 ; a=1 , b=1 ,c=0, x= 53, y= -47
a=1 , b=1 ,c=1, x= 25, y= 5 ; a=1 , b=1 ,c=2, x= 77, y= 73
a=2 , b=0 ,c=0, x= 77, y= -73 ; a=2 , b=0 ,c=1, x= 25, y= -5
a=2 , b=0 ,c=2, x= 53, y= 47 ; a=2 , b=1 ,c=0, x= 31, y= -19
a=2 , b=1 ,c=1, x= 35, y= 25 ; a=2 , b=1 ,c=2, x= 151, y= 149

Que dan lugar a las siguientes 24 soluciones:


( x, y )  (151,149 ), (35,25), (31,19 ), (53,47 ), (25,5), (77 ,73)

22.3.3 Ejemplo 3.
Determina todas las soluciones enteras de la ecuación
x 2  y 2  294

Solución.
x 2  y 2  294  ( x  y)( x  y)  2  3  72
Y por tanto

x  y  2  3  7
a b c

 para ciertos 0  a  1 , 0  b  1, 0  c  2

 x  y  21 a
 3 1 b
 7 2c

Luego
 2a  3b  7c  21 a  31b  7 2  c
 x  2a 1  3b  7c  2 a  31b  7 2  c
 2
 1 a 1 b 2c
 y  2  3  7  2  3  7  2a 1  3b  7c  2 a  31b  72  c
a b c


 2

Y vemos que no existe ningún valor 0  a  1 para el cual x, y sean enteros. Por lo
tanto, esta ecuación no tiene ninguna solución entera.
22.3.4 Ejemplo 4.
Determina todas las soluciones enteras de la ecuación
x 2  y 2  189

Solución.
x 2  y 2  189  ( x  y)( x  y)  33  7

Y por tanto

x  y  3  7
a b

 para ciertos 0  a  3 , 0  b  1
 3 a
x  y  3  7
1 b

Luego
 3a  7b  33 a  71b
 x 
2
 3 a 1 b
y  3 7  3 7
a b

 2

Las soluciones son:


a=0 , b=0, x= 95, y= -94 ; a=0 , b=1, x= 17, y= -10
a=1 , b=0, x= 33, y= -30 ; a=1 , b=1, x= 15, y= 6
a=2 , b=0, x= 15, y= -6 ; a=2 , b=1, x= 33, y= 30
a=3 , b=0, x= 17, y= 10 ; a=3 , b=1, x= 95, y= 94

Que dan lugar a las siguientes 16 soluciones:


( x, y )  (95,94 ), (17 ,10 ), (33,30 ), (15,6)

22.3.5 Ejemplo 5.
Determina todas las soluciones enteras de la ecuación
x2  y 2  441

Solución.
x2  y 2  189  ( x  y)( x  y)  32  72

Y por tanto

x  y  3  7
a b

 para ciertos 0  a  2 , 0  b  2
 3 a
x  y  3  7
1 b

Luego
 3a  7b  32  a  7 2 b
 x 
2
 2a 2b
y  3 7  3 7
a b

 2
Las soluciones son:
a=0 , b=0, x= 221, y= -220 ; a=0 , b=1, x= 35, y= -28
a=0 , b=2, x= 29, y= 20 ; a=1 , b=0, x= 75, y= -72
a=1 , b=1, x= 21, y= 0 ; a=1 , b=2, x= 75, y= 72
a=2 , b=0, x= 29, y= -20 ; a=2 , b=1, x= 35, y= 28
a=2 , b=2, x= 221, y= 220

En este caso observamos que el número de soluciones con x  0 es impar.


Las soluciones a la ecuación son las siguientes 18:
( x, y )  (221,220 ), (35,28), (29,20 ), (75,72 ), (21,0)
De las cuales son positivas las cinco siguientes:
( x, y )  (221,220 ), (35,28), (29,20 ), (75,72 ), (21,0)

Teorema.
El número r de soluciones enteras ( x, y ) de la ecuación x2  y 2  k queda
determinado por su descomposición en factores primos k  p11 p2 2 ... pn n .
a) Si p1  2 y 1  1 , es decir, el número k es de la forma k  2q con q impar, no hay
solución posible: r  0 .

b) Si con p1  2 , es decir, el número k es impar, entonces la fórmula


n

 
i 1
i  1

genera todas las soluciones de la forma ( x, y ) , ( x, y ) , con x  0 , luego


n
r  2  i  1
i 1

c) Si p1  2 y 1  1 , es decir, el número k es de la forma k  4q , entonces la fórmula


n
(1  1)  i  1
i2
genera todas las soluciones de la forma ( x, y ) , ( x, y ) , con x  0 , luego
n
r  2(1  1)  i  1
i2

Demostración.
a) x 2  y 2  2q  ( x  y)( x  y)  2q
Vemos que 2 y q son coprimos, y es imposible encontrar factorizaciones 2q  n  m con
la misma paridad.

b) Si k  p11 p2 2 ... pn n con p1  2 , entonces


 p11 p2 2 ... pn n  p11  1 p2 2   2 ... pn n   n
 x 
2
 n
con 0  i   i
1  2
 y  p1 p2 ... pn  p1
 1  1  2   2
p2 ... pn n   n
 2
Y todas las combinaciones posibles son válidas pues los numeradores son siempre
pares, pues son sumas y restas de números impares.

c) Si k  21 p2 2 ... pn n , entonces


 2 1 p2 2 ... pn n  21  1 p2 2   2 ... pn n   n
 x   2 1 1 p2 2 ... pn n  21  1 1 p2 2   2 ... pn n   n
2
 n
1  2
 y  2 p2 ... pn  2
 1  1  2   2
p2 ... pn n   n
 2 1 1 p2 2 ... pn n  21  1 1 p2 2   2 ... pn n   n
 2

con 0  i   i

Pero para 1  0 y 1  1 los valores obtenidos de x, y no serán enteros, luego hay


que descartarlos. Por lo tanto el exponente 1 debe ser tratado de forma diferente.

Observación.
Si queremos determinar únicamente las soluciones no negativas, debemos diferenciar
los siguientes casos:
a) Si k no es un cuadrado perfecto, es decir, algún  i es impar y por tanto algún  i  1
es par, el número de soluciones enteras será un número par (la mitad positivas y la
mitad negativas). En particular:
n
a1) Si p1  2 , es decir, k es impar, la fórmula  
i 1
i  1 genera todas las soluciones

( x, y ) , ( x, y ) con x  0 , luego el número de soluciones positivas será


1 n
s    i  1
2 i 1
a2) Si p1  2 y 1  1 , entonces el número de soluciones positivas será
n
s  (1  1)  i  1
1
2 i2

b) Si k es un cuadrado perfecto, es decir, todos los  i son pares y por tanto todos los
i  1 son impares, aparecerá la solución x  k , y  0 :
k  k 0  k 0 
Esta solución hará que el número total de soluciones sea impar.

Por ejemplo, en el Ejemplo 4,


1 n
x 2  y 2  189  33  7    i  1  1 (3  1)(1  1)  8  4
2 i 1 2 2
Que son las cuatro soluciones ya encontradas anteriormente:
( x, y )  (95,94 ), (17 ,10 ), (33,30 ), (15,6)
22.3.6 Ejemplo.
Determina el número de soluciones enteras de la ecuación x2  y 2  9 . ¿Cuántas de
ellas no son negativas?

Solución.
n
Aplicando la fórmula del teorema anterior, 9  32    i  1  2  1  3
i 1

Luego hay 3  2  6 soluciones enteras, de las cuales 3 / 2  1.5  2 no son negativas.


Concretamente son: x  3, y  0 ; x  5, y  4

22.3.7 Ejemplo.
Determina el número de soluciones enteras de la ecuación x 2  y 2  400 . ¿Cuántas de
ellas no son negativas?

Solución.
n
400  24  52  (1  1)  i  1  3  3  9 .
i2

Luego hay 9  2  18 soluciones enteras, de las cuales 9 / 2  4.5  5 no son


negativas.

Observación 2.
En el apartado 20.1 se introduce el número de divisores positivos de k ,  (k ) , y
aplicando la fórmula de 20.4 se deduce que, si k es impar, se cumple:
1
 (k )  r
2
Fuente: “On the Number of Solutions of the Diophantine Equation x2 − y2 = N” , J.F.T. Rabago
INTERNATIONAL JOURNAL OF MATHEMATICS AND SCIENTIFIC COMPUTING
(ISSN: 2231-5330), VOL. 2, NO. 2, 2012 13

22.3.8 D
Dado un número entero positivo n , definimos λ(n) como el número de soluciones
enteras positivas de la ecuación x 2  y 2  n . Diremos que el número n es “olímpico”
si λ(n) = 2021. ¿Cuál es el menor entero positivo que es olímpico? ¿Y cuál es el menor
entero positivo impar que es olímpico?

OME 2021 #2

22.3.9 M
Suponiendo que las raíces de la función f ( x)  x 2  a x  2a son valores enteros,
determina la suma de todos los posibles valores de a .

(A) 7 (B) 8 (C) 16 (D) 17 (E) 18

AMC 10A 2015 #23


22.4 La técnica del descenso infinito de Fermat.
Supongamos que queremos resolver una ecuación diofántica, y encontramos que, dada
una solución x1  0 , entonces también será solución x2  0 , con x1  x2  0 .

Obtenemos así una sucesión infinita y decreciente de soluciones x1  x2  ...  xn  0 , lo


cual es imposible pues trabajamos con números naturales, llegando a contradicción.

Así pues, esta técnica nos permite demostrar que una determinada ecuación diofántica
no tiene solución posible, o solo tiene la solución trivial x  0 .

Una definición más formal de esta técnica sería la siguiente: Supongamos que queremos
demostrar una propiedad P para los enteros. Si la hipótesis de que se cumple para un
cierto número entero positivo n0 implica que también se cumple para otro entero
positivo más pequeño n1  n0 , entonces ningún entero positivo satisface P.

Esta técnica se justifica formalmente en el principio de que todo conjunto finito de


números naturales tiene un mínimo. Sea S el conjunto de números naturales que
satisfacen P. Entonces S tiene un mínimo n , y la existencia de m  n satisfaciendo P,
es decir, m  S nos lleva a contradicción.

Este método se asocia a Fermat (1601-1665) porque este matemático fue el primero en
utilizarlo explícitamente en sus razonamientos. Es intersante reseñar que Fermat, pese a
ser proclamado padre de la Teoría de Números moderna, siempre se consideró a sí
mismo un matemático amateur, y nunca se preocupó de publicar ninguno de sus
resultados matemáticos, contentándose con enviarlos a su amigo el matemático Marin
Mersenne (1588-1648).

Podemos leer en Cut the Knot que este método fue el utilizado por Euclides en Los
Elementos VII.31 para demostrar que todo número compuesto tiene un divisor primo.

22.4.1 F
Determine todas las soluciones enteras de la ecuación
x 2  y 2  3z 2

22.4.2 M
Determine todas las soluciones enteras de la ecuación
a 2  b 2  c 2  a 2b 2

USAMO 1976 #3

22.4.3 M
Resuelve la siguiente ecuación diofántica:
x3  2 y 3  4 z 3
22.5 El método de la contradicción modular.
Una técnica muy potente para resolver ecuaciones diofánticas es pasar la ecuación a un
módulo adecuado en el que los candidatos aceptables se reduzcan significativamente.
También se puede demostrar que una ecuación no tiene solución si no tiene solución
para algún módulo conveniente.

22.5.1 D
Determina todas las parejas de enteros  x, y  que satisfacen la ecuación
x 2  y! 2001

Nota: Cuando trabajamos con números factoriales es interesante tener en cuenta que
n! 0 mod m para todo n  m , y así reducir los valores aceptables significativamente.

22.5.2 F
Demuestra que para todo m, n enteros positivos, 3m  3n  1 no puede ser un cuadrado
perfecto.
USAMTS

22.5.3 D
Demuestra que 19 19 no se puede escribir como x 3  y 4 , con x , y enteros.

22.5.4 F
Determina todas las soluciones de la ecuación x5  y 2  4 en enteros positivos.

22.5.5 F
¿Existe alguna pareja de enteros a, b para los cuales a 5b  3 y ab5  3 son cubos
perfectos?

USAJMO 2013 #1

22.5.6 M
Demuestra que la ecuación
x  12  x  22  ...  x  2001 2  y 2
no tiene soluciones enteras.

22.5.7 D
Determina todas los pares  p, q  de números primos tales que
p3  q5   p  q
2

Russian Mathematical Olympiad


22.6 Resolución de ecuaciones diofánticas mediante factorización.
Fuente: An Introduction to Diophantine Equations A Problem-Based Approach (Andreescu, Cucurezeanu, Andrica) , página 4 en
adelante.

El método más común para resolver una ecuación diofántica es escribirla como
producto de expresiones más sencillas igual a un número:

f1 x1 , x2 , ..., xn  f 2 x1 , x2 , ..., xn  ... f m x1 , x2 , ..., xn   a

Ahora, factorizando el número a , obtenemos todas las posibles expresiones


a  a1  ... am , convirtiendo la ecuación diofántica original en un conjunto de sistema de
ecuaciones:
 f1  x1 , x2 , ..., xn   a1
 f  x , x , ..., x   a
 2 1 2 n 2

...
 f m  x1 , x2 , ..., xn   am

Ejemplo resuelto 1.
Determina todas las soluciones enteras de la ecuación
  
x 2  1 y 2  1  2x  y 1  xy   41  xy 

Solución:
Reescribimos la ecuación de la siguiente forma:
x 2  1y 2  1  2x  y 1  xy   41  xy  
x 2 y 2  2 xy  1  x 2  y 2  2 xy  2( x  y )(1  xy)  4 
xy  12  x  y 2  2x  y xy  1  4 
xy  1  x  y 2  4 
xy  1   x  y   2 
( x  1)( y  1)  2

Veamos los casos posibles:


x  1  1  x  1  1 x  1  1
  x  0, y  3   x  2, y  3   x  0, y  1
 y 1  2  y 1  2  y  1  2
 x  1  1 x  1  2  x  1  2
  x  2, y  1   x  1, y  2   x  3, y  2
 y  1  2  y 1  1  y 1  1
x  1  2  x  1  2
  x  1, y  0   x  3, y  0
 y  1  1  y  1  1

Es fácil comprobar que las ocho soluciones encontradas satisfacen la ecuación del
enunciado.
Ejemplo resuelto 2.
Determina todas las soluciones enteras positivas de la ecuación
1 1 1
 
x y pq

Solución.
1 1 1
   pq( x  y )  xy 
x y pq
pqx  pqy  xy  0 
( x  pq)( y  pq)  p 2 q 2  0 
( x  pq)( y  pq)  p 2 q 2

Y probamos con todas las combinaciones posibles:


 x  pq  1  x  pq  p  x  pq  p 2
  
 y  pq  p q  y  pq  pq  y  pq  q 2
2 2 2

 x  pq  q 
 x  pq  q
2
 x  pq  pq
  
 y  pq  p q 
 y  pq  p  y  pq  pq
2 2

 x  pq  p 2 q  x  pq  pq 2  x  pq  p 2 q 2
  
 y  pq  q  y  pq  p  y  pq  1

Que llevan a las nueve soluciones posibles:


1  pq, pq(1  pq) ,  p(1  q), pq(1  q) , q(1  p), pq(1  p)
 p( p  q), q( p  q) , 2 pq,2 pq ,  pq(1  q), p(1  q) ,
 pq(1  p), q(1  p) , q( p  q), p( p  q) ,  pq(1  pq),1  pq
22.6.1 M
Determina todas los pares ( x, y ) de enteros no negativos para los cuales
xy  7 2  x 2  y 2
Indian Mathematical Olympiad

22.6.2 M
Resuelve la siguiente ecuación para soluciones enteras x, y :
x 2  y  1  y 2 x  1  1

Polish Mathematical Olympiad

22.6.3 F
Determina todas las parejas de enteros positivos no nulos x, y  que satisfacen la
ecuación
xy 2  2 y 2  x  107  0

Kangourou S 2021 #26


22.6.4 F
Las butacas de un teatro forman un rectángulo, ordenadamente por filas (butacas una al
lado de la otra) y en columnas (una detrás de otra). Sabemos que la capacidad del teatro
es la mínima que permite situar en cada fila exactamente 12 hombres, en cada columna
exactamente 15 mujeres y dejar exactamente 7 butacas libres. Determina la capacidad
del teatro.

(A) 635 (B) 754 (C) 736 (D) 650 (E) 680

Cangur B2 2022 #30

22.6.5 F Problema solucionado paso a paso en vídeo .

Un partido de fútbol entre equipos de North Berracan y South Berracan se juega en un


estadio que tiene una serie rectangular de asientos para los espectadores. Hay 11
partidarios de North Berracan en cada fila y 14 partidarios de South Berracan en cada
columna. Esto deja 17 asientos vacíos. ¿Cuál es el menor número posible de asientos en
el estadio?

(A) 500 (B) 660 (C) 690 (D) 840 (E) 994

Kangaroo Student 2022 #30

Solución: [Link]

22.6.6 MD
Determina todas las ternas de enteros positivos ( x, y , z ) que satisfacen la ecuación
2x  y  z  2 xyz   2 xy  2 yz  2 zx  1  2023
2 2

USAJMO 2023 #1
Nota: Se presenta una solución incompleta.
22.7 Resolución de ecuaciones diofánticas aplicando desigualdades.
Este método consiste en aplicar algún tipo de desigualdad para reducir el número de
casos aceptables. Para ello aplicaremos las técnicas estudiadas en el libro
Desigualdades.

22.7.1 D
Determina todos los pares x, y  de enteros para los que se verifica
x3  y 3  x  y 
2

22.7.2 M
Encontrar todas las soluciones enteras positivas de

1 1 1 1
   1
ab bc ca abc2
OMEFL 2017 #4

22.7.3 D
Sean a, b, n enteros positivos tales que a  b y ab  1  n 2 . Prueba que
a  b  4n  3

Indica justificadamente cuando se alcanza la igualdad.

OME 2013 #1

22.7.4 D
Determina todas las parejas de enteros positivos x, y  que son solución de la ecuación
x4  y3
 x y
x2  y

OMEC 2021 #3

22.7.5 M
Resuelve la siguiente ecuación para valores enteros positivos de x, y, z :
1 1 1 3
  
x y z 5

Romanian Mathematical Olympiad

22.7.6 D
Determina las soluciones enteras y positivas a, b, c de la ecuación
 1  1  1 
1  1  1    2
 a  b  c 

BMO 1995 Round 2 #1


22.8 Ecuaciones diofánticas en competiciones olímpicas.
22.8.1 M
Determina todos los enteros no negativos a, b tales que a  b  2009

BMO 2009 Round 2 #1

22.8.3 F
Resuelve la siguiente ecuación para x, y, z  IN
1 2 3
  1
x y z

BMO 1988 #4

22.8.4 F
Determina todas las soluciones enteras positivas n, m , con n impar, de la ecuación
1 4 1
 
m n 12

BMO 2001 Round 1 #1

22.8.5 F
Determina todos los pares a, b de enteros positivos tales que
1 1 3
 
a b 2018
Putnam 2018 A1

22.8.6 F
Determina justificadamente todos los pares de números enteros ( x, y ) que verifican la
ecuación x 2  y 4  2009 .

OME 2009 #4

22.8.7 M
Determina 3x 2 y 2 si x, y son enteros que satisfacen y 2  3x 2 y 2  30 x 2  517 .

AIME 1987 #5

22.8.8 F
Encuentra todos los primos p y q que satisfacen la ecuación p  q  ( p  q)3

Rusia, 2001

22.8.9 M
Determina todas las ternas de enteros positivos a , b , c tales que abc  a  b  c  1 .

México 2010
22.8.10 F
Describir todas las soluciones enteras positivas m, n de la ecuación
8m  7  n 2

y dar el primer valor de m (si existe) mayor que 1959.

OMEFL 2017 #1

22.8.11 D
 
Calcular todos los pares de enteros x, y  tales que 3423 x 2  y 2  x3 y 3 .

OME 2019 #4

22.8.12 M
Determina todos las parejas de enteros positivos  x, y  tales que x 3  y 3  xy  61

Russian Math Olympiad

22.8.13 M
Resuelve la siguiente ecuación para valores enteros x, y  0 :
x 2
 
 y x  y 2  x  y 
3

USAMO 1987 #1

22.8.14 M
Determina todos los pares de enteros ( x, y ) tales que x 6  3x3  1  y 4

Romanian Math Olympiad

22.8.15 F
Encontrar todas las soluciones enteras posibles, x e y, de la ecuación
px  y   x y

siendo p un cierto número primo.

OMEFL 2007 #2

22.8.16 MD
Determine todas las parejas de enteros ( x, y ) tales que 1  2 x  22 x 1  y 2

IMO 2006 #4

22.8.17 D
Determina todos los x, y, z  IN tales que 3x  4 y  5 z .

IMO 1991 Shortlist

22.8.18 D
Determina todas las soluciones enteras de la ecuación 3m  2 n  1
22.8.19 D
Determina todos los pares de enteros  x, y  que satisfacen la ecuación
1  x 2 y  x 2  2 xy  2 x  y

BMO 2001 Round 2 #2

22.8.20 F
En una calle hay una fila de casas numeradas del 1 al n, donde n es un número de tres
cifras. Exactamente 1/k de estos números empiezan con el dígito “2”, donde k es un
entero positivo. Determina los posibles valores de n.

BMO 2022 Round 1 #1


22.9 Cuadrados perfectos.
22.9.1 M
Determina todos los números primos p, q para los cuales p  q y p  7 q son ambos
cuadrados perfectos.

22.9.2 M
Determina todos los primos p y q para los que p 2  7 pq  q 2 es un cuadrado perfecto.

22.9.3 MD
 
Determinar todos los números primos p  3 tales que  p  1 / 2 y p 2  1 / 2 son
cuadrados perfectos.

OMEC 2022 #5

22.9.4 F
Sea n un entero positivo y sea d un divisor positivo de 2n 2 . Demuestra que n 2  d no
es un cuadrado perfecto.

Kürschák Math Competition 1953

22.9.5 D
Determina todos los números primos p tales que 2 p 4  p 2  16 sea un cuadrado
perfecto.

Leningrad Math Olympiad 1980

22.9.6 MD
Demuestra que la ecuación 4 xy  x  y  z 2 no tiene soluciones en los enteros
positivos.
(Euler)

22.9.7 M Problema solucionado paso a paso en vídeo.

Un número triangular es cualquier entero positivo que se puede expresar de la forma


tn  1  2  3  ...  n ,
para cierto entero positivo n. Los primeros tres números triangulares que son también
cuadrados perfectos son: t1  1  12 , t8  36  62 y t49  1225  35 2
Determina la suma de los dígitos del cuarto número triangular más pequeño que es
también un cuadrado perfecto.

(A) 6 (B) 9 (C) 12 (D) 18 (E) 27

AMC 12A 2022 #16

Solución: [Link]
22.10 Ecuaciones de Frobenius. Problema de las monedas.

Definición. Ecuación de Frobenius. Número de Frobenius.


Llamaremos ecuación de Frobenius a toda ecuación diofántica de la forma
a1x1  a2 x2  ...  an xn  b

con ai , xi , b enteros , ai  0 y xi  0 .

Llamaremos “Número de Frobenius” al mayor b para el cual la ecuación no tiene


solución. Por ejemplo, la ecuación de Frobenius 4 x  9 y  b tiene asociado el número
de Frobenius 23.

En Mathematica:

El problema de las monedas.


Dados dos números positivos m, n , queremos estudiar las combinaciones lineales
a m  bn

con coeficientes enteros no negativos a, b  0 .

Este problema se llama problema de las monedas de Frobenius o simplemente problema


de Frobenius, pues este matemático alemán estudió las posibles sumas de dinero que se
podían obtener acumulando ciertos tipos de moneda.

El teorema “Chicken McNugget”.


Dados dos números positivos coprimos m, n , el mayor entero que no puede
representarse como combinación lineal a m  b n con a, b enteros no negativos es

mn  m  n

Una consecuencia de este teorema es que existen exactamente


m  1 n  1
2
enteros positivos que no se pueden representar de dicha forma.

Se dice que el origen de este teorema y de su curioso nombre viene de cuando


McDonalds ofrecía sus nuggets en paquetes de 9 y de 20, y algunos aficionados a las
matemáticas se preguntaron por la cantidad máxima que no se podían comprar en cajas
completas (son 151).
Teorema de Frobenius.
Supongamos que a y b son enteros positivos.
a) Si a, b  1 , el número de enteros positivos n que no se pueden escribir como
(a  1)(b  1)
ax  by  n , con x, y  0 , es exactamente .
2

b) Si a, b  1 , todo número entero positivo de la forma ax  by  n será múltiple de


a, b , luego existirán infinitos números que no se puedan escribir como ax  by  n .
c) Si a, b  1 y n  ab  a  b , la ecuación ax  by  n no tiene solución con x, y  0 ,
pero sí la tiene para todo n  ab  a  b (aunque puede tener solución para algunos
n  ab  a  b concretos)

22.10.1 MF
En un pueblo llamado Hamlet hay 3 personas por cada caballo, 4 ovejas por cada vaca,
y 3 patos por cada persona. ¿Cuál de los siguientes valores no puede ser la suma de
personas, caballos, ovejas, vacas y patos en Hamlet?

(A) 41 (B) 47 (C) 59 (D) 61 (E) 66

AMC 10B 2015 #15

22.10.2 D
Determina todos los posibles valores de enteros positivos n para los cuales 91 céntimos
es el mayor valor que no se puede formar disponiendo de una infinita cantidad de sellos
de 5, n y n  1 céntimos.

AIME II 2019 #14

22.10.3 F
Se propone un juego en el cual el jugador gana a puntos si gana o b puntos si pierde
( a, b  IN , a  b ) , y los puntos se van acumulando partida a partida. Se sabe que
existen exactamente 35 puntuaciones no alcanzables y que una de ellas es 58. Determina
a y b.

Putnam 1971 #A5

22.10.4 D Problema solucionado paso a paso en vídeo

Tomamos 94 ladrillos de medidas 4’’×10’’×19’’ y los apilamos uno encima del otro
hasta formar una torre de 94 ladrillos. Cada ladrillo puede estar orientado de cualquier
forma, por lo que contribuye en 4’’, 10’’ o 19’’ en la altura total de la torre. Determina
el número de alturas diferentes de las torres que podemos formar con estos 94 ladrillos.

AIME 1994 #11

Solución: [Link]
22.10.5 M
En un país llamado Coinland hay tres tipos de moneda, de 6, de 10 y de 15. Determina
la suma de los dígitos del mayor valor que es imposible obtener con estas monedas.

(A) 8 (B) 10 (C) 7 (D) 11 (E) 9

AMC 12B 2023 #16


Soluciones.
1.1.1
Supongamos que N  n  0  n  1  n  2  n  3  ...  n  99  100 n  (1  2  3  ...  99 )
99(99  1)
Sabemos que 1  2  3  ..  99   4950
2
Luego N  100 n  4950 , luego será un número que acabe en "50", y el único candidato
aceptable es (A).

1.1.2
N  9  99  999  9999  ..  99
 99 
...
321cifras

      
 10  1  10  1  10  1  ...  10 321  1
1 2 3

 
 10  10  10  ...  10   1
1 2 3

321
 ...
1  1  10
1

 10 2  10 3  ...  10 321  321  (*)
 321 
10  10  10  ...  10  10  100  1000  ..  100
1 2 3 321
...   111
000
  ...111
0
321 321
(*)  111
  0  321  111
...111  ....
111
 0789
321 321 3  318
Las cifras de este número suman 1 318  0  7  8  9  342

1.1.3
Vamos a ir observando la pauta sn  1  2  3  ...  n
s1  1  d1  1
s2  1  2  3  d 2  3
s3  1  2  3  6  d3  3
s4  1  2  3  4  10  d 4  0
s5  1  2  3  4  5  15  d5  5
s6  1  2  3  4  5  6  21  d 6  1
s7  1  2  3  4  5  6  7  28  d 7  8
Las sucesivas sn siguen de la siguiente manera:36, 45, 55, 66, 78, 91, 105, 120, 136, 153,171,
190,210,231,253,276,300,325,351,378,406,435,465,496,528,561,595,630,666,703,741,
780,820,861,903...
Vemos que los primeros d1  d2  ...  d19  70 .
y vemos que, a partir de la posición d19 sigue una pauta repetitiva de 20 números:
0, 0, 1, 3, 6, 0, 5, 1, 8, 6, 5, 5, 6, 8, 1, 5, 0, 6, 3, 1
que suman 70.

Entre d19 y d 2017 hay 20 bloques completos y 19 elementos "sueltos" finales:


2017  18  1999  99  20  19
Los 19 primeros elementos de la pauta anterior suman 69, luego
2017

d
n 1
n  70  99  70  69  7069 . El residuo de 7069 entre 1000 es 69.
1.1.4
Vamos calculando pacientemente la sucesión:
a3  4(8  5  2)  60  5 , a4  4(5  8  5)  76  6 ,
a5  10 , a6  7 , a7  4 , a8  7 , a9  6 , a10  2 , a11  5 , a12  8

Vemos que los tres últimos elementos coinciden con los primeros, por lo que podemos asegurar
que los elementos se van repitiendo en grupos de diez.

2018  201  10  8  a2018  a8  7 



2020  202  10  0  a2020  a0  2  a2018  a2020  a2022  7  2  8  112
2022  202  10  2  a2022  a2  8 

1.1.5
2  4  6  8  384 , luego la multiplicación de cuatro números, acabados en 2, 4, 6 y 8 acabará en
4.
La multiplicación de números acabados en 4 va alternando: Acabados en 6 y acabados en 4:
4 2  4  4  16, 43  16  4  64, 4 4  64  4  256 ,... luego 410 acabará en 6.
2  4  6  8 12 14 16 18  22  24  26  28  ... 92  94  96  98 la podemos entender como la
multiplicación de diez grupos, cada uno de ellos acabado en 4, luego el resultado será el mismo
que 410 , es decir, 6.

1.1.6
Supongamos que nuestra sucesión es de la forma xn  a n  b para ciertos enteros a, b .
Supongamos que para cierto n se cumple xn1  a n1  b  m 2 .
Entonces
m  a 2  m2  a 2  2am  a n1  b  a 2  2am  an1  a  2m  b
Y por tanto el término a n2 con n2  n1  a  2m  n1 es también un cuadrado perfecto. Esta
sucesión de cuadrados perfectos la podemos hacer tan grande como queramos.

1.1.7
N  a  112  b  11  c  1  83  b  82  c  8  a , con 0  a, b, c  7 .

0  1  83  b  82  c  8  a  a 112  b 11  c 
   
0  1  83  82  11 b  8  1c  1  112 a 
0  512  53b  7c  120 a 
120 a  53b  7c  512

Luego
120 a  53b  7c  512  512  a  512 / 120   4

Con a  4 nos quedamos cortos, luego a  5 .

Supongamos que a  5 . La ecuación queda de la forma:


600  53b  7c  512 
600  53b  7c  512 
88  53b  7c

Con b  0 la ecuación resultante 88  53  0  7c  7c no tiene solución entera.


Con b  1 la ecuación resultante 88  53 1  7c tiene solución entera:
88  53 1  7c  88  53  7c  35  7c  c  5

Luego hemos encontrado la solución a  4, b  1, c  5 , y como en todo momento hemos


trabajado con valores mínimos, esta será la solución mínima que nos pide el enunciado.

1.1.8
a  x  10  y 
  a  b  10 x  y  10 y  x  11( x  y )
b  y  10  x 
Puesto que estamos tratando con números de dos dígitos, se debe cumplir x, y  0 , y por lo
tanto la suma es un múltiple de 11.

1.1.9
n  a b c  c b a  99  a  100  b  10  c  c  100  b  10  a  99 
100 a  10b  c  100 c  10b  a  99  99 a  99 c  99  0 
99a  c  1  0  a  c  1

c b a  a b c  99  c  100  b  10  a  a  100  b  10  c  99 
 99 c  99 a  99 
99c  a   99  c  a  1  c  a  1

El valor de a puede estar entre 1 y 9, pero el 9 lo debemos descartar porque entonces c sería
igual a 10, lo cual no es posible. Por lo tanto hay 8 valores posibles de a, y puesto que el valor
de b es libre (y hay 10 posibles valores de b, entre 0 y 9), y el valor de c está determinado por el
valor de a, tendremos un total de 8  10  80 posibles valores. (A)

1.1.10
Vemos que todas las potencias de 5 acaban en cifra 5. Luego, si sumamos uno, acabarán en
cifra 6. Multiplicando números que acaban en 6 siempre obtenemos resultados que acaban en 6,
luego la solución es 6 (A).

1.1.11
Sean a y b las dos cifras del número n, es decir: n  a b . Entonces:
a b  b a  132  a  10  b  b  10  a  132  10 a  b  10b  a  132  11a  11b  132 
11(a  b)  132  a  b  132 / 11  12  b  12  a

El número a puede tomar todos los valores enteros entre 1 y 9, pero de estos valores debemos
descartar los que generen valores de b inaceptables:
0  b  9  0  12  a  9  12  a  9 12  12  a  3  3  a  12 , así pues,
debemos eliminar los casos a  1 y a  2 , luego hay 7 casos aceptables para a, que determinan
el valor de b, la respuesta correcta es (B).
1.1.12
La ecuación que se plantea es
a  a  a  b  b  b  b  10a  b  3a  4b  10a  b  0  7a  3b  3b  7a

Así pues, b es un múltiplo de 7, y sabemos que 0  b  9 . Si b  0 entonces a  0 y no sería un


número de dos cifras, por lo tanto la única posibilidad es b  7 , que implica a  3 , y por tanto
a  b  10 (A).

1.1.13
Veamos los palíndromos en base 8 y seleccionemos aquellos que también sean palíndromos en
base 10.
n  a  b  8  c  8 2  d  83  a  8b  64 c  512 d , con 0  a, b, c, d  7 , pero puesto que n  1000
podemos suponer que d  1 .
Puesto que buscamos el valor máximo, podemos suponer que d  1 . Por ser palíndromo, a  d  1 y
c  b , y por tanto
n  1  8b  64b  512  513  72b .
b  7  n  513  72  7  1017  1000
b  6  n  513  72  6  945 no es palíndromo en base 10.
b  5  n  513  72  5  873 no es palíndromo en base 10.
b  4  n  513  72  4  801 no es palíndromo en base 10.
b  3  n  513  72  4  729 no es palíndromo en base 10.
b  2  n  513  72  2  657 no es palíndromo en base 10.
b  1  n  513  72 1  585 sí es palíndromo en base 10.
El número buscado es 585.

1.1.14
Sean los dígitos de N an , an1 , an3 ,..., a1 , a0 , donde a0 es el dígito de las unidades. Nos vamos
a concentrar en los últimos dos dígitos, así que llamaremos A  an  an1  an3  a2 .

Supongamos en primer lugar que a0  9


Queremos resolver la ecuación A  a1  a0  3 A  a1  a0  3 . Esta ecuación no tiene una
solución aceptable en nuestro contexto:
A  a1  a0  3 A  a1  a0   9  2 A  a1  a0   9  A  a1  a0 
9
, absurdo, pues los
2
valores de an , an1 , an3 ,..., a1 , a0 son todos enteros positivos entre 0 y 9.

Supongamos en primer lugar que a0  9


Queremos resolver la ecuación A  a1  9  3 A  a1  1  0.
A  a1  9  3 A  a1  1  A  a1  9  3 A  3a1  3  2 A  a1   6
 A  a1  3
Aquí podríamos encontrar una solución mínima en A  0, a1  3 , obteniendo el número
N  39 . En efecto,
N  39  3  9  12 
  12  3  4
N  1  40  4  0  4
La solución es N  39  3  9  12 (C)
Un razonamiento alternativo sería el siguiente:
Si no se produce acarreo, es decir, los dígitos son todos ai  9 , el pasar de N a N+1 hace que la
suma de dígitos pase de S a S+1, y las condiciones del enunciado no se pueden cumplir.
Si se produce acarreo en la última cifra, es decir, a0  9 , pasar de N a N+1 hace que la suma de
dígitos pase de S a S-8, con lo que sí puede haber solución.

1.2.1
Vemos que la puerta k será abierta por el estudiante 1, pasará por ella el estudiante k, y quedará
abierta si han pasado por ella un número impar de estudiantes.

Por otro lado, sobre la puerta k actuará el estudiante j si y solo si k es un divisor de j.

Observamos como son los divisores de cada número:


3  1, 3
4  1 , 2, 4
5 1, 5
6 1, 2 , 3 , 6

9 1, 3 , 9
16  1 , 2 , 2 2 , 2 3 , 2 4

Vemos que los únicos números que tienen una cantidad par de divisores son los cuadrados
perfectos. Son éstos, y solo éstos, los que quedarán abiertos: 1, 4, 9 y el 16.

1.3.1
a2  b2
Supongamos, por el contrario, que existe un k  , con a, b enteros positivos que no es
1  ab
un cuadrado perfecto.
Por ser a, b simétricos en la expresión, podemos suponer que a  b .
Podemos suponer, además, que b es el entero positivo más pequeño posible.

En primer lugar, vemos que a  b , pues


a 2  b2 2a 2
abk    2  k  1 , pero 1 es un cuadrado perfecto: 1  12 , llegando a
1  ab 1  a 2

contradicción.

Así pues, a  b .
a 2  b2
k  1  abk  a 2  b 2  k  abk  a 2  b 2
1  ab
 a 2  b 2  k  abk  0  b 2  akb  a 2  k  0

Esta última expresión la podemos considerar como una ecuación de segundo grado en b, con lo
que existirán dos soluciones, b,b1 , tales que:
ak  b  b1
a 2  k  b b1
Puesto que a, b  0 , la hipótesis b1  0 contradice la condición
1  ab1 k  a 2  b12
Puesto que k no es un cuadrado perfecto, la hipótesis b1  0 también contradice la condición
anterior:
1  a  0k  a 2  02 . Así pues, b1  0 .
Por otro lado:
b1b  b1b  b1 ak  b1   a 2  k  akb1  b12  a 2  k  akb1  k  a 2  b12
a 2  b12
 k ab1  1  a 2  b12  k 
ab1  1

Y también:
a 2  k b2  k
b1   b
b b

a 2  b12
Luego hemos encontrado un nuevo número b1  b que también satisface k  , cuando
ab1  1
habíamos supuesto que b era el menor número posible, llegando a contradicción. Por lo tanto,
k tiene que ser un cuadrado perfecto.

Fuente de la solución: Elementary Number Theory Notes (David A. Santos, 2004) , pág. 3.

3.1
Basta realizar la suma “en columnas”:

1 5 7 2 8
2 2 A 0 4
 C D 3 3 1
5 7 2 6 3

De izquierda a derecha vamos deduciendo A=2, D=9 y C=1 (B)

3.2
b  2a  1
  c  2(2a  1)  1  4a  2  1  4a  3
c  2b  1
Se cumple c  1000 , luego a  250 , luego a  2 N 2 con N  5 o bien a  1M 1 .

Probamos las diferentes posibilidades:


a  242  b  2a  1  485 no cumple las condiciones del enunciado.
a  232  b  2a  1  465 no cumple.
a  222  b  2a  1  445 no cumple.
a  212  b  2a  1  425 no cumple.
a  202  b  2a  1  405 no cumple.
a  101  b  2a  1  203 no cumple.
a  111  b  2a  1  223 no cumple.
a  121  b  2a  1  243 no cumple.
a  131  b  2a  1  263 no cumple.
a  141  b  2a  1  283 no cumple.
a  151  b  2a  1  303 sí cumple, pero c  2b  1  607 no cumple.
a  161  b  2a  1  323 sí cumple, pero c  2b  1  607 no cumple.
a  171  b  2a  1  343 sí cumple, pero c  2b  1  687 no cumple.
a  181  b  2a  1  363 sí cumple, y c  2b  1  727 también cumple.
a  191  b  2a  1  383 sí cumple, y c  2b  1  767 también cumple.

Luego cumplen dos números (C).

3.3
Basta realizar la suma en columnas:
7 2 4 3
2 1 A 7
 C D 2 6
1 1 1 2 6

Para deducir que A=5, D=7 y C=1 (B)

3.4
2019  224  9  3 , luego con 224 dígitos seguro que nos quedamos cortos. Con 225 dígitos, el
más pequeño será 39999  ...
999
 cuya primera cifra es 3 (B)
223

3.5
Para llegar a T , R o P o S necesita realizar un número par de pasos. Para llegar a Q necesita un
número impar de pasos. El número 2019 es impar, luego solo a Q. (C)

3.6
Analizando detenidamente la larga cadena de operaciones involucradas en este problema,
podemos especular que g (n)  10 , el primer valor que no se puede representar en base 16 con
los dígitos  0 ,1, 2 , ..., 9 .

En base 8 disponemos de los dígitos del 0 al 7, y por lo tanto, el valor mínimo para que la suma
sea 10 será 3+7, es decir, el número 37 8  3  8  7  31 .

Finalmente, buscamos un número cuya suma de dígitos en su representación en base 4 sea 31.
En base 4 tenemos los dígitos  0 ,1, 2 , 3 . 31  3 10  1 , luego el número buscado será
 
n  1333333333 34  1 411  3  410  3  49  ...  3  4  3  40  411  3 410  49  ...  4  1

Aplicando la fórmula de la serie geométrica:


1  411 411  1
410  49  ...  4  1  
1 4 3
Para calcular 411  222 a mano vamos calculando potencias de 2:
210  1024
220  1024 1024  1048576
411  222  220  4  4194304

Con lo que, finalmente:


 411  1 
n  411  3   2097151
 3 
3.7
 10    9   8   7    6   5   4   3   2   1  0 
1  2  3  4  5  6  7  8  9  10  11  12  13  14 
 11  12  13  14  50
Luego en cada fila, columna y diagonal la suma será 10 (C)
3.8
Interpretando esta ecuación como una ecuación de segundo grado en y, tenemos
2  4  4 1  x 2020 2  4(1  x ) 2  2 1  x 2020
2020
y  2y  x
2 2020
0 y   
2 2 2
 1  1  x 2020
x  1
Que tendrá solución siempre que 1  x 2020  0  
x  0
x  1  y  1  0  1 una solución
x  0  y  1  1  0,1 dos soluciones
En total, hay 3 soluciones (C)

3.9
Vemos que log a es un entero positivo si y solo si  
a  10 c  a  10 c
2
 10 2c para cierto
entero positivo c .
Luego  
log a  log 10 2c  2c y debe ser un entero, luego c  2e 2 para cierto entero e , y
 
log a  log 10 2c  2c  2e
Por otro lado, log a  c  2e2 .

De la misma forma vemos que b  10 d  b  10 d 2


 10 2d para cierto entero positivo d.
Luego  
log b  log 10 2 d  2d debe ser un entero, luego d  2 f 2 para cierto entero f , y
log b  2d  2 f
Por otro lado, log b  d  2 f 2 .

Así pues, la ecuación del enunciado se transforma en


100  log a  log b  log a  log b  2e  2 f  2e 2  2 f 2 
50  e  f  e2  f 2  e(e  1)  f ( f  1)
Buscamos posibilidades: 1  2  2 , 2  3  6 , 3  4  12 , 4  5  20 , 5  6  30 , 6  7  42
50  2  48 , 50  6  44 , 50  12  38 , 50  20  30
50  32  20 , 50  42  8

Las únicas combinaciones aceptables son e  4, f  5 y e  5, f  4 .

Luego
 
a  10 c
2
 10 2c  10 22e  10 4 e 
2 2

  4e 2
 4f2
 4e 2  4 f 2
 4 e 2  f 2 

 ab 10 10 10 10
b  10   10 2 d  10 22 f  10 4 f 
d 2 2 2

 10 4 4 52   10164 ( D)
2

Observación: En las soluciones oficiales


[Link] se presentan otros desarrollos
alternativos.

3.10
Haciendo el listado exhaustivo de todos los valores que vamos obteniendo para valores
pequeños de n , vemos que los conjuntos


Sn  an  3n  an 1  3n 1  ...  a1  31  a0  30 ,  1  ai  1 
Constan de 3 n enteros, sin que se produzcan repeticiones, y se cumple 0  Sn , y hay simetría:
x  Sn   x  Sn .
Así pues, en particular, para el caso n  7 , tendremos 38  34  34  81  81  6561 números
diferentes: 3280 negativos, 3280 positivos y el cero. El total de elementos no negativos es, por
tanto, 3281 (D).

3.11
Fijando un k  1,2,..., 2450 , los triangulos ABC y APk Qk son semejantes por Tales, ya que
Pk Qk // BC .

Las regiones separadas por segmentos tienen la misma área


ABC  .
2450
Los trapecios se pueden unir de forma que APk Qk   k
ABC 
2450
Es decir, la razón de proporcionalidad entre las áreas de los triángulos es:
APk Qk   k  k
ABC  2450 2  52  72
Sabemos que las áreas están en proporción el cuadrado de la razón de semejanza de las
longitudes de los lados, luego la razón de proporcionalidad de las longitudes es:

k k /2
 
25 7
2 2
57

Y por tanto
k /2 1 3k 1 3  3k 3 k
Pk Qk  BC    5 3  5 3    que será un número racional
57 7 2 7 3 2 7 6
k
si y solo si es racional, es decir, para todos los k tales que k / 6 sea un cuadrado perfecto:
6
k / 6  p2  k  6 p2

p  1  k  6  12  6 , p  2  k  6  22  24 , ... , p  20  k  6  20 2  2400 ,
Con p  21  k  6  212  2646 ya nos pasamos luego hay 20 valores de k aceptables.

3.12
PR #1.16

3.13
Para resolver este problema vamos a aplicar la siguiente identidad notable:
a 2  b2 c2  d 2   ac  bd 2  ad  bc2
Esta identidad se demuestra fácilmente mediante números complejos, tomando z1  a  bi y
z2  c  di , vemos que se reduce a la conocida identidad
2 2 2
z1 z2  z1 z2  z1 z2  z1 z2

Volviendo a nuestro problema, multiplicamos por 5  12  22 ambos lados de la igualdad y


aplicamos la identidad anterior:

73 2  34 2  5  1297  73 2  34 2 12  22  52  1297   
73  1  34  2  73  2  34  1
2 2
 52  1297 
52  180 2  52  1297 
  2
52  22  32  5  52  1297 
5  5 2  3   5  1297 
2 2 2 2 2 2

1  2  3   1297 
2 2 2

12  36 2  1297

3.14
Basta con multiplicar esta expresión por 1  (3  2) y aplicar la identidad “suma por
diferencia”:
(a  b)(a  b)  a 2  b2
En nuestro caso:
(3  2)2  3  32  22
32
      3 2
 22 22  32  32  22
2 2 4 4

34
 2 2
4 4
 3  3   2   3  2
4 4 2 4 2 8 8

Para ir cancelando, uno por uno, todos sus términos hasta quedar 3128  2128 (C)
3.15
Analizando la tercera columna vemos que acaba en 4, y analizando la primera columna vemos
que el resultado es 26 contando el acarreo, luego la suma será 24. (C)

3.16
En primer lugar podemos suponer a  1 es decir mínimo, para buscar el máximo de b.
Vemos que para b  c  1000 se cumple a  b  c  1000000 , y que para cualquier número
mayor, por ejemplo b  c  1001 entonces a  b  c  1000000 . (D)

3.17
El 0 se puede obtener tomando a  0 , b  0 , c  0 .
El 2 se puede obtener tomando a  1 , b  0 , c  0 .
El 8 se puede obtener tomando a  2 , b  0 , c  0 .
El 6 se puede obtener tomando a  1 , b  1 , c  0 .
Luego solo nos queda la solución 1. (B)

3.18
(B)

3.19
Puesto que Par+Par=Par, Impar+Par=Impar, Impar+Impar=Par, la sucesión sigue la siguiente
pauta: I I P I I P I I P I I P...
2020  3  673  1 , luego habrán 673 números pares. (A)

3.20
Basta aplicar la propiedad conmutativa en la columna de la derecha: 79  2  158 (B)

3.21
Por tanteo se llega a  1  0  1  2  2 (C)

3.22
El próximo será 2121, y transcurrirán 101 años (B)

3.23
Se deduce que es 2 (B)

3.24
Sea x el valor central. Sumando los elementos de dos formas diferentes llegamos a la ecuación
40  4 x  13  4 cuya solución es x  3 (A)

3.25
95  55  150 km y no llegaría con los 14 litros de combustible. La anterior está en el kilómetro
75  55  130 y le faltarían recorrer 520  130  390 kilómetros con lo que necesitaría 39 de
los 40 litros de gasolina disponibles (D).

3.26
Empezamos por el número 7 que solo puede ser 1  2  4 . Luego, por tanteo, llegamos a la
siguiente combinación aceptable:
Cuyas caras suman 7, 8, 10, 9 y 13. Luego la solución es 13 (C)

3.27
Observando las sumas parciales que aparecen y por tanteo, llegamos a esta solución aceptable:

Con lo que la respuesta correcta es 7 (C)


3.28
En primer lugar vemos que
1 1 1 1 1 1 3 3
5abc       
a b c 5 5 5 5 4
Y no se podrá cumplir la igualdad del enunciado. Luego nos podemos reducir a los casos
1 a  4

Primer caso: a  1 .
1 1 3 1 1 3 1
1      1  lo cual es imposible con b, c positivos.
b c 4 b c 4 4

Segundo caso: a  2
1 1 1 3 1 1 3 1 1
       
2 b c 4 b c 4 2 4

1 1 2 1
Si 8  b  c     , y la igualdad requerida no se podrá cumplir. Luego nos
b c 8 4
reducidos a 2  b  8 .
1 1 1 bc 1
     4(b  c)  bc  bc  4c  4b  c(b  4)  4b
b c 4 bc 4
4b
c
b4
4b
De aquí se deduce claramente que c  0b  4
b4
Vamos probando casos:
45
b 5c   20
54
46
b6c   12
64
4  7 28
b7c  no aceptable
74 3
48
b 8c  8
84

Segundo caso: a  3
1 1 1 3 1 1 3 1 5
       
3 b c 4 b c 4 3 12
1 1 2 5
Si b, c  5     pues 24  2  12  5  5  25 . Luego nos podemos reducir a los
b c 5 12
casos 1  b  4
1 1 5 bc 5
     12 (b  c)  5bc  12b  12 c  5bc  12 c  5bc  12b
b c 12 bc 12
12b
 c(12  5b)  12b  c 
5b  12
Los únicos valores aceptables son
12  3 12  4
b 3c  3 , b4c  6
15  12 20  12

Tercer caso: a  4
1 1 1 3 1 1 3 1 1
       
4 b c 4 b c 4 4 2

1 1 2 1
Si b, c  4     y no se puede dar la igualdad, luego la única opción posible es
b c 4 2
1 1 1
b, c  4   
b c 2
Que efectivamente cumple la condición requerida.

Luego las soluciones son a  2, b  5, c  20 ; a  2, b  6, c  12 ; a  2, b  8, c  8 ;


a  3, b  3, c  12 ; a  3, b  4, c  6 ; a  4, b  4, c  4 .

3.29
Para resolver este problema es suficiente garantizar que existen tres enteros 2n  a  b  c  n
tales que
a  b  r2
b  c  p2
c  a  q2
para ciertos enteros r, p, q .
De esta manera, puesto que hay tres números y dos pilas, por el Principio del palomar
forzosamente siempre tendremos al menos dos de estos números en la misma pila.

Sumando las tres ecuaciones llegamos a 2a  b  c   r 2  p 2  q 2


luego r 2  p 2  q 2 debe ser par.
Por otro lado,
r 2  p2  q2 r 2  p2  q2
2a  b  c   r 2  p 2  q 2  a  b  c   a  p2  
2 2
r 2  p2  q2
a  p2
2
Y de la misma forma:
r 2  p2  q2 r 2  p2  q2
b  q2 , c   r2
2 2

Una hipótesis natural :-) es suponer que existe un entero k tal que
p  2k  1
q  2k
r  2k  1

Es fácil comprobar que entonces r 2  p 2  q 2  12 k 2  2 es un número par.

La condición 2n  a se puede resolver completando cuadrados:


r 2  p2  q2
a  p 2  6k 2  1  2k  1  2k 2  4k  2n  k 2  2k  n
2

2
 k  2k  1  n  1  k  1  n  1  k  n  1  1
2 2

Y la condición c  n se resuelve también completando cuadrados:


r 2  p2  q2
 r 2  6k 2  1  2k  1  2k 2  4k  n  k 2  2k 
n
c
2

2 2

 k 2  2k  1   1  k  1   1  k 
n n n
1 1
2

2 2 2

Así pues, todo se reduce a demostrar que, para cualquier n  100 , existe un k tal que
n
 1  1  k  n  1 1
2

Puesto que
 n  n n
n  1  1    1  1  n  1  1  1 1  n 1  1  2
 2  2 2

n
es suficiente demostrar que n 1   1  3 para todo n  100 .
2
2
n  n  n n
n 1   1  3   n  1   1   9  n  1   1  2 n  1 1  9 
2  2  2 2

n  n  3n  14 n 
 2 n  1  1  7   7  2 n  1  1   2 n  1  1 
3n 3n
2 2  2 2  2 2 
n  n 
3n  14  4 n  1  1  3n  14   16 n  1  1
2

2  2 
 9n 2  84 n  196  8n 2  24 n  16  n 2  108 n  180  0

Esta última desigualdad se soluciona de la forma habitual, encontrando las raíces de la ecuación
n 2  108 n  180  0

Pero sin tener que hacer cálculos, está claro que n 2  108 n  180  nn  108   180  0 si
n  108 .

(determinando las raíces podríamos haber ajustado un poco más y garantizar que se cumple
para todo n  107 )

Los casos 100  n  107 se tienen que observar por separado y se comprueba que, para todos
ellos, k  9 es un valor aceptable.

Fuente de esta solución: Múltiples fuentes en Internet.


3.30
Vamos haciendo las multiplicaciones, y solo nos fijamos en la última cifra:
131 acaba en 3
132 acaba en 9
133 acaba en 7
134 acaba en 1
135 acaba en 3
Luego nos encontramos con un ciclo de longitud 4, y por tanto, puesto que 2003  500  4  3 ,
acabará en 7, (C).

3.31
Observamos que las potencias de los números que acaban en 5 siempre acaban en 5, luego el
resultado será 5  1  6 (C).

3.32
p  1 , 3 , 5 , 7 , 11 , 13 , 17 , 19 ...  p 2  1 , 9 , 25 , 49 , 121 , 169 , 261 , 361 ...
y vemos que la única posibilidad aceptable (aquí podemos aprovechar que las respuestas
posibles son todas valores únicos) es
p 2  q 2  361  4  19 2  22  p  q  19  2  21 (B)

3.33
Trabajando con números pequeños nos pone sobre la pista del resultado:
4489  67
444889  667
Observando la pauta de multiplicar el número 666 ...67 por sí mismo nos lleva a la solución:
La solución es 666 6 7 .
...
2021dígitos

3.34
abc  ab 2   c 2 b 2 c  a 3   b 2 ca 2 c  b   a a 2b  c 3  
    
0  ab c  ab 2  c 2 b 2 c  a 3  b 2 c a 2 c  b  a a 2b  c 3    
 abc  a b  b c  a c  a b c  b c  a b  ac 
2 3 2 3 3 2 2 2 2 3 3 3

 abc  ac 3  b 3c  b 2 c 3  a 3b  a 3c 2  a 2b 3  a 2b 2 c 2 

 abc  ac 3  b 3c  b 2 c 3  a 3b  a 3c 2  a 2b 3  a 2b 2 c 2  
  
 c ab  ac 2  b 3  b 2 c 2  a 2 ab  ac 2  b 3  b 2 c 2  
  
 c  a 2 ab  ac 2  b 3  b 2 c 2 
 c  a a b  c   b b  c  
2 2 2 2

c  a 2  0  c  a 2


 c  a2   
a  b 2 b  c 2  a  b 2  0  a  b 2
b  c 2  0  b  c 2

Fuente de esta solución: Antonius Benedictus en Facebook
3.35
Solo de 2:
3  5$  1  2$  17 $
1  5$  6  2$  17 $
La respuesta correcta es (A).

3.36

Haciendo la división larga poco a poco vemos como se produce la secuencia de dígitos, y
observamos que se genera un cero por cada tres dígitos sin contar el primero. Luego la solución
es 2004 1  2003 , 2003 / 3  667 (D)
3.37
Por las condiciones del enunciado debemos encontrar tres números enteros 0  a, b, c  9 ,
a  0 , de forma que
a 10 2  b 10  c  b  9 2  c  9  a 
a 10 2  b 10  c  b  9 2  c  9  a  0 
a(100  1)  b(10  81)  c(1  9)  0 
99 a  71b  8c  0 
99 a  71b  8c

Vamos probando en búsqueda de una solución aceptable:


Si a  1 , b  1, 99  71  8c  c  7 / 2 no es aceptable
Si a  2 , 198  71b  8c , tomando b  2 y c  7 se cumple la ecuación.
Puesto que se nos pide una solución que se supone única, no hace falta seguir, la solución al
problema es 227 .

Observación.
Una manera más elegante de resolver la ecuación 99 a  71b  8c sería pasar a módulo 71, en
donde esta ecuación se convierte en
99a  8c mod 71  28a  8c mod 71  7a  2c mod 71
En donde hemos podido simplificar porque (4,71)  1 .

Y esta última congruencia lineal solo tiene solución a  2 , b  7 .

3.38
Para encontrar el mínimo, debemos minimizar el numerador y maximalizar el denominador.
Nuestro objetivo es intentar encontrar un numerador igual a 1, que sería el valor mínimo
posible. Entre todos los candidatos, encontrar el que deja tres números no utilizados mayores,
que irán en el denominador.
El 1 nos sirve de comodín, pues puede ir en cualquier lado. Probando probando encontramos la
combinación
2  3  6  1 5  7 1
2  3  6 1 5  7  1 que da lugar a 
489 288

En el denominador hemos colocado el 8 y el 9, que son dos de los números más altos posibles,
luego podemos esperar que esta combinación sea la más pequeña.
La respuesta correcta es, por tanto, 1  288  289 .

Observación: En [Link] podemos encontrar algunos razonamientos


que justifiquen que esta combinación es realmente la mínima posible.

3.39
Empezaremos por la columna de la izquierda. Solo hay tres combinaciones posibles que sumen
25: 10+8+7 y 10+9+6.
Probando con la segunda, tampoco hay muchas combinaciones de tres números que sumen 24,
así que ponemos el 6 en la fila central y probamos combinaciones hasta encontrar la adecuada:
El número que tenemos que poner en la posición del interrogante es el 1, y la respuesta correcta
es (E).

3.40
x 2  bx  c  0 no tendrá dos soluciones reales distintas si y solo si su discriminante no es
positivo:
1  b 2  4 1 c  0
Y de la misma manera, la segunda ecuación no tendrá dos soluciones reales distintas si y solo si
2  c2  4 1 b  0

Así pues, nuestro problema se reduce a determinar las soluciones enteras positivas b, c  del
sistema de inecuaciones:
b 2  4c  0
 b 2  4c 

 2 
c  4b  0
2
 c  4b 

c 2  4b  c 2  4b  c  2 b , donde estamos teniendo en cuenta que c  0 . Luego:


b 2  4c 

  b  42 b  8 b
2

c  2 b 
Comparando las funciones f1 (b)  b 2 , y f 2 (b)  b , vemos que la desigualdad b 2  8 b solo
se puede cumplir para los primeros valores de b, exactamente hasta
b  0
b 2  8 b  b 4  8 2 b  b 4  64 b  0  bb 3  64   0   3
b  64  0  b  4

El sistema es simétrico en la incógnita c, así pues, hemos reducido los posibles candidatos a
1  b, c  4 .

Si b  c está claro que se satisface el sistema, que se reduce a la desigualdad b 2  4b :


b  1  12  4 , b  2  2 2  8 , b  3  32  12 , b  4  4 2  16

Para los otros casos:


4  4 93 
b  2, c  1   se cumple b  3, c  1   no se cumple
1 8  1  12 
16  4 4  12 
b  4, c  1   no se cumple b  2, c  3   no se cumple
1  16  98 
4  16  9  16 
b  2, c  4   no se cumple b  3, c  4   no se cumple
16  8  16  12 

Así pues, se cumple para las parejas (1,1), (1,2),(2,1),(2,2),(3,3), (4,4) (B)

3.41
Vamos a calcular dicho número:
2021  224  9  5
Luego N es el número 5 99 ...
9
224 nueves

Luego N  2021 será:


5 9 ... 9 9 9 9 9
+ 2 0 2 1
6 0 ... 0 2 0 2 0

Y la suma de sus dígitos será 6  0  0  ...  0  2  0  2  0  10 (C)

3.42
Sabemos que un número será divisible entre 3 cuando la suma de sus cifras sea divisible entre
tres. Veamos todas las combinaciones posibles:
1 1 1 (1) 1 1 3 (0) 1 1 5 (0) 3 3 1 (0) 3 3 3 (1)
3 3 5 (0) 5 5 1 (0) 5 5 3 (0) 5 5 5 (1) 1 3 5 (6)

En total hay 1+1+1+6=9 (E)

3.43
Podemos escribir la lista directamente:
0 → 0002 7 → 2002
1 → 0020 8 → 2020
2 → 0022 9 → 2022
3 → 0200
4 → 0202
5 → 0220
6 → 2000
Hay nueve números, y por tanto el del medio es el quinto: 220 (B).

3.44
1  2  3  4  5  6  7  8  9  10  11  12  78
78  41  26  11
La única forma de sumar 11 con cuatro números entre 1 y 12 es
11  1  2  3  5
El primer grupo, para alcanzar 41, necesitará, seguro, la pesa 12. Porque sin la pesa 12 lo
máximo que se alcanza es 11  10  9  8  38 .
Nos quedan los números 4, 6, 7, 8, 9, 10, 11, vamos probando...
Para llegar a 26 con cuatro de estos números tomamos 26  4  6  7  9 , y efectivamente,
41  8  10  11  12 , luego ya hemos resuelto una combinación aceptable. Vemos que con el 9
va el 7 (B).

3.45
Sea S  0  1  2  ...  9  45 . Entonces vemos que el truco está en agrupar por decenas:
1  0  1  1  1  2  ...  1  9 
 2  0  2  1  2  2  ...  2  9 
...
 9  0  9 1  9  2  ...  9  9 
 1  0  0  1  0  1  1  0  2  ...  1  0  9 
 1S  2S  3S  ...  9S  S  S  S 2  45 2  2025

3.46
Vamos probando diversas combinaciones hasta encontrar que
19  10  7  21  2  15 y queda el número 9 (C)

3.47
Probamos distintas combinaciones hasta encontrar la adecuada:

La solución es (B).

3.48
Denotamos por a,b,c,d y x las casillas señaladas en la imagen siguiente:

Entonces:
20  a  21  b  2  24  c  5  d  6  16  a  25  c  4  22  b  1  d  x 
43  a  b  35  c  d  45  a  c  23  b  d  x 
43  a  b  35  c  d  (45  a  c)  (23  b  d  x)  0 
43  a  b  35  c  d  45  a  c  23  b  d  x  0 
43  a  b  35  c  d  45  a  c  23  b  d  x  0 
10  x  0 
x  10 (C)

3.49
27  128

Si 100  a  b   2 n  999  2 n  28  256
c

29  512

Además, a  b también tiene que ser una potencia de dos. Veamos todos los casos posibles:
a  2, b  0
ab  2   (a  b) c  2c  6,7,8  6 casos
a  b  1
a  4, b  0
a  b  2

ab  4   c  (a  b) c  2 2c  c  3  4 casos
 a  3, b  1
a  1, b  3
a  8, b  0
a  b  4

a  7, b  1

a  1, b  7
ab 8   c  (a  b) c  8c  23c  c  2  8 casos
 a  6, b  2
a  2, b  6

a  3, b  5
a  5, b  3

a  9, b  7

a  b  16  a  8, b  8  c  (a  b) c  16 c  2 4 c  c  2  3 casos.
a  7, b  9

En total hay 6  4  8  3  21 casos (E).

3.50
Se van a jugar un total de 28 partidos. Sea E el número de partidos que acaban en empate y sea
G el número de partidos que no acaban en empate, es decir, que gana uno de los equipos.
Las ecuaciones que obtenemos son:
G  E  28

3G  2 E  61
Resolviendo el sistema anterior llegamos a G=5, E=23,
Por lo tanto el número máximo de partidos ganados por un mismo equipo será 5, y empatando
los otros dos tenemos un total de 3+3+3+3+3+1+1=17 puntos (D).
Se puede comprobar que esta configuración es realmente válida:

A B C D E F G H
A 3 3 3 3 3 1 1
B 0 1 1 1 1 1 1
C 0 1 1 1 1 1 1
D 0 1 1 1 1 1 1
E 0 1 1 1 1 1 1
F 0 1 1 1 1 1 1
G 1 1 1 1 1 1 1
H 1 1 1 1 1 1 1

3.51
1 · 2 · 3 · 4 · 5 · 4 · 3 · 2 · 1 = 120 · 24, y sin tener que completar esta última multiplicación
vemos que acabará en “80”, y por tanto la respuesta correcta es A.

3.52
Primera versión.
Sea k un entero positivo tal que n 2  5n  6  k 2  n 2  5n  6  k 2  0
Interpretando esta ecuación como una ecuación de segundo grado en n, sus posibles soluciones
son
 5  52  4 1  (6  k 2 )  5  25  24  4k 2  5  1  4k 2
n  
2 2 2

Así pues, si n es un entero, 1  4k 2  1  2k  será un cuadrado perfecto. Pero entonces


2

tendremos dos cuadrados perfectos consecutivos, lo cual es imposible.


En efecto, si b  a  1  b 2  a  1  a 2  2a  1  a 2  2  1  a 2  3 , en donde hemos
2

aplicado que a  1  2a  2 .

Segunda versión.
Observamos que n 2  5n  6  (n  2)(n  3) , y por tanto
(n  2) 2  (n  2)(n  3)  n 2  5n  6  (n  3) 2 , y por tanto este valor está entre los cuadrados
de dos números enteros positivos consecutivos, y por tanto no puede ser ningún cuadrado
perfecto.

Fuente de la segunda versión: Soluciones oficiales (Compendium OMEC, pág 439)

3.53
Observamos que 19  6  2  7  6  6  2  2  2  1 y por lo tanto se puede conseguir 19 puntos
con dos seises, pero es imposible obtener 19 con tres seises en cinco tiradas, luego la respuesta
correcta es (C)

3.54
 
x  2 y  210  x  210  2 y  2 29  y , luego x es par, es decir, x  2x' para cierto entero
positivo x' . Volviendo a la ecuación original,
2 x'2 y  210  x' y  29  y  29  x' , luego x' recorre todos los números del 1 al 29  1 , y
por tanto la solución es (A).

3.55
(A) El 2 o el 6

3.56
Si m y n son impares, también lo será su producto m·n, y por tanto también lo será m+n+2
(D).

3.57
Era 80-7-7-7-7-6=46 años, porque la más joven no había nacido y por tanto solo podemos
restarle 6 años. (D).
3.58
Ha ganado un 49% de 200 partidas = 98 partidas de un total de 200.
Si juega dos más y las gana, tendrá 100 partidas ganadas sobre 202 , y no llega al 50%.
Necesita jugar 6 partidas más y ganarlas para obtener 104 partidas ganadas sobre 206 jugadas,
que sí está por encima del 50%. (D).

3.59
Vemos que la cantidad de tablones siempre es de la forma 1 5k para un cierto k entero, y la
opción (B) es la única que se adapta a este formato: 96  1  5 19 .

3.60
Queremos determinar todos los números primos p que se pueden escribir como
p  a  b

p  c  d
con a, b, c, d números primos.

El único primo par es 2.


Tomamos la igualdad p  c  d y supongamos que d  2 . Puesto que c  d  2 se trata de la
diferencia de dos números impares, y la diferencia de dos impares es un número par, luego p
será un número par, luego solo puede ser 2.
Así pues, p  2 , pero p  a  b , suma de dos enteros positivos mayores o iguales a 2, luego es
imposible, no hay solución.

Así pues, d  2 , y por tanto 2 p  a  b  c  2  2 p  2  a  b  c .


La suma de tres impares es impar, y claramente 2 p  2 es par, luego debe ser par ( y por tanto
2) alguno de los tres sumandos a, b, c . El número c no puede ser 2, pues c  d  2 .
Supongamos que a  2 (el caso b  2 es idéntico).
Entonces p  c  2  c  p  2 , y por lo tanto b  p  2 , p , y c  p  2 son primos. Pero al
menos uno de los tres debe ser múltiplo de 3, y la única opción posible aceptable es que el
menor de los tres sea igual a 3:
b  p  2  3  p  5.

Fuente de esta solución: MichaelPennMath

3.61
De la segunda condición deducimos que al menos contiene dos nueves.
De la tercera condición se deduce que el número de elementos tiene que ser par.
No puede contener tres nueves porque entonces para que la suma fuera 30 el resto de elementos
tendría que sumar 3. La única combinación aceptable sería {9,9,9,3} y su mediana es 9.
Así pues, contiene exactamente dos nueves.
Con cuatro elementos {9,9,a,b} tenemos a+b=12. La pareja 9, 9 no puede estar en el centro,
pues entonces la mediana sería 9, luego tiene que ser de la forma a<b<9<9 y el único candidato
aceptable es 5<7<9<9, y la respuesta correcta es 52  7 2  92  92  236 .
Se puede comprobar que con seis números no hay ninguna combinación aceptable. En primer
lugar, los dos nueves tienen que ser los valores más altos. Es decir, buscamos combinaciones de
la forma a<b<c<d<9.
El valor menor a no puede ser 2, pues entonces la menor combinación aceptable: 2<3<4<5 ya
se pasa de 12.
Luego buscamos combinaciones de la forma 1<b<c<d<9.
b no puede ser 3, pues entonces la mínima combinación 1<3<4<5 ya se pasa de 12.
Luego buscamos combinaciones de la forma 1<2<c<d<9 con c+d=9.
Pero esto es imposible, porque para que la mediana del conjunto sea un valor entero, de debe
cd
cumplir entero, es decir, que c  d sea par, llegando a contradicción.
2

3.62
Observemos los primeros valores de n n .
6 6  10000 y no hace falta tenerlo en cuenta, luego podemos utilizar los siguientes cinco
números:
11  1 , 21  4 , 33  27 , 4 4  256 , 55  3125
La única manera de obtener un número a b c d  1000 sin utilizar 55  3125 sería
4 4  4 4  4 4  4 4  1024 llegando a contradicción.
Luego para obtener un número entre 1000 y 9999 sumando cuatro de estos cinco números
vemos que la clave es usar forzosamente al menos una vez 55  3125 , pero solo una vez, pues
de lo contrario obtendríamos un número con a  6 , y ya hemos dicho que 6 6  10000 .
Y utilizando solo una vez 55  3125 vemos que forzosamente este número empezará por 3. Así
que la respuesta correcta es B.

Observación: Se puede comprobar (mediante fuerza bruta por ordenador o por algún método
más elegante, que la única solución aceptable es 3435).

3.63
Supongamos que ha salido el 6. Puesto que 6  2  3 , el 6 no puede volver a salir. La
combinación más alta posible en este caso es
6  5  5  5  5  5  18750 , que cumple todas las condiciones del enunciado (puesto que en la
competición Canguro solo hay una solución válida, ya hemos resuelto el problema y no hace
falta continuar).
El 4 no puede aparecer, luego la siguiente combinación aceptable sería
6  3  5  5  5  5  11250  15000 , con la que ya no llegamos a 15000.
Supongamos que no ha salido el 6. Puesto que el 4 no puede aparecer, y no se pueden repetir el
mismo número seis veces, la máxima combinación posible es
5  5  5  5  5  3  9375 con la que ya no llegamos a 15000.
Así pues, la única combinación posible es 6  5  5  5  5  5 y 6  5  5  5  5  5  31 (C)

3.64
Escribimos la condición del enunciado como
r s 10 p  q  10 r  s
10 p  q    
10 100 2
r s
20 p  2q    10 p  q  10 r  s 
5 50
r s
20 p  2q  10 p  q  10 r  s   
5 50
49 49 49  s 
10 p  q  r s r   
5 50 5  10 
 s 
510 p  q   49 r  
 10 
s
De aquí deducimos que debe ser un entero. Y puesto que 0  s  9 , forzosamente debe ser
10
s  0 , y la ecuación anterior queda
510 p  q  49r
De donde se deduce que r debe ser un múltiplo de 5. No puede ser 0, pues entonces
510 p  q  0  10 p  q  0  p  q  0
Luego r  5 , y la ecuación queda
10 p  q  49
de donde, finalmente, deducimos que p  4 y q  9 .
La única solución de este problema es N  4950  4  9  5  0  18 (B)

3.65
Denotamos por a, b, c, d, e, f, g, h, i, j, k, l los círculos del esquema, tal y como se muestra en la
imagen siguiente:

10 4 6 24
abde  10  ab  , cdgh  4  cg  , efij  6  fj  , hikl  24  kl 
de dh ei hi
Luego
10 4 6 24 10  4  6  24 10  4  6  24 10  4  6  24
abcgfjkl     10  4  40 (B)
de dh ei hi dedheihi dehi  dhei 12 12

3.66
Observamos que la opción E son todos los múltiplos de 3. Puede ser una pista.
Vemos que una combinación aceptable es 1 - 2 - 4 - 5 - 7 - 8 - 10 - 11 , eliminado los múltiplos
de 3, que coincide con la opción (E).

3.67
Vemos que la única manera de que tres números consecutivos acaben con la cifra de las
decenas igual a la cifra de las unidades es que sean de la forma X99 → Y00→ Y01
Luego ♢=9,△=0, y por tanto los números son 199, 200, 201, luego vendrá el 202 que se
representa por ♡ △ ♡ (E).
3.68
Observamos que 2023 acaba en “3”, y que en la tabla del 7 solo acaba en 3 el 7x9=63, luego
podemos esperar una suma de 9 filas.
La segunda columna tiene que sumar un número acabado en “2”, y tengo en cuenta que debo
sumar los 6 del acarreo de la columna de las unidades, luego tiene que acabar en “6”, y el único
producto que acaba en 6 es 7x8=56, luego podemos esperar 8 sietes en la columna de las
decenas.
Finalmente necesito un “20” teniendo en cuenta que sumo “6” de la columna anterior, luego
necesitaré un 14 que se puede obtener con 2 sietes. Y en total he utilizado los 19 sietes:
777+777+77+77+77+77+77+77+7=2023
y en la suma aparece seis veces el 77 (E).

3.69
Tenemos la ecuación 18  P  7  G  R con números diferentes, enteros y en orden
decreciente. Por tanteo llegamos a una solución posible: 18  8  7  2  1 , y por tanto el gato
pesa 2 kilos (A).

3.70
Probando diferentes combinaciones encontramos una combinación compatible con las
condiciones del enunciado:
2+1=3, 4+3=7, 6+5=11
y la única opción que no se cumple es la (A).

3.71
Los múltiples de 8 serán números que sean pares y sigan siendo pares después de dividirlos
entre 2 dos veces. Veamos las posibilidades tomando los últimos cuatro dígitos:
4330/2=2215 y ya no es par.
4332/2=2166, 2166/2=1083 ya no es par.
4334/2=2167 ya no es par.
4336/2=2168, 2168/2=1084 es aceptable.
4338/2=2169 ya no es par.
hay un número aceptable (B).

3.72
Sea n  1 la posición que ocupa Martín. Luego tiene delante n personas y detrás otras n . En
total hay 2n  1 personas en la fila, que sabemos que es un múltiplo de 3.
Puesto que las posiciones 19 y 28 están detrás de él, se deberá cumplir
n  1 19 y 2n  1  28 y deber ser un múltiplo de 3.
n  1  19  n  18 .
Vamos probando casos:
n  17  2n  1  35 no es múltiplo de 3
n  16  2n  1  33 sí es múltiplo de 3
n  15  2n  1  31 no es múltiplo de 3
n  14  2n  1  29 no es múltiplo de 3
n  13  2n  1  27  28 ya no se cumplen las condiciones del enunciado.
Luego la única solución aceptable es n  1  16  1  17 (D).

3.73
 
85  510 155  23  510  3  5  215  510  35  55  215  515  35  1015  35  243 1015
5 5

Y este número tiene 18 dígitos (E).

4.1.1
  16
416525  22 525  2216525  232525  27 225525  128  (2  5)25  128  10 25  1.28  10 27
Un número que tiene 28 cifras decimales.

4.1.2
Si n es impar, n  2k  1 para cierto k entero, luego
n 2  2k  1  4k 2  4k  1  4k (k  1)  1
2

Ahora bien, el producto de dos números consecutivos es siempre par, pues o bien k es par o
bien k  1 es par, y por tanto 4k ( k  1) es múltiplo de 8, tal y como queríamos ver.

4.1.3
Sea x la longitud del lado del cuadrado.
Entonces x  d es la longitud del lado del triángulo equilátero y
3( x  d )  4 x  1989 
3x  3d  4 x  1989 
3d  x  1989 
3d  1989  x

Puesto que suponemos x  0 , se tiene que cumplir 3d  1989  0 y esto solo pasa si d  663 .
Luego existen 663 casos (contando d  0 ) que no son válidos para d.

4.1.4
La condición (i) equivale a a  4,5
La condición (ii) equivale a d  0,5
La condición (iii) equivale a a b 34,35,36,45,46,56
Luego el número posible de enteros es 2  6  2  24

4.1.5
Una vez introducida la cuarta puntuación, el resultado tiene que ser múltiplo de 4.
El 80 es solución al problema. En efecto, 71  76  82  91  320 , que es múltiplo de 4.
Cualquier otra posibilidad no es aceptable:

71  400  71  329 , 76  400  76  374 , 82  400  82  378 , 91  400  91  389

Ninguno de estos números es múltiplo de 4.

4.1.6
Tenemos que resolver la ecuación x 2  6 x  n 2 con x, n enteros.
x2  6 x  n2  x2  6 x  n2  0 

x  

 6  62  4  1  (n 2 )  6  36  4n 2  6  4 n 2  9


2 2 2
 6  2 n2  9
  3  n 2  9
2

Luego n 2  9 debe ser un cuadrado:


n2  9  m2  9  m2  n2   m  n  m  n 
Por otro lado, m  n  m  n , luego la posibilidad 9  3  3 queda descartada, y la única
m  n 1
posibilidad es  m  n  m  n   1  9  
m  n  9

Resolvemos el sistema anterior:



m  n 1 m 1 n
 1 n  n  9  1 2n  9  n  4  m 1 4  5

 m  n  9

Así pues, las soluciones son:


n  4, m  5  (m  n)(m  n)  (1)  (9)  9
n  4, m  5  (m  n)(m  n)   9 1  9
n  4, m  5  (m  n)(m  n)  (9)  (1)  9
2
En todo caso, n 2  m2  9  25  9  16  x  3  16  9  3  25  3  5  
 8
4.1.7
Primera versión.
27 b  2b  7 es un cubo perfecto, es decir, 2b  7  m3 para cierto entero m.

b  1000  m3  2b  7  2007

Puesto que hay menos cubos perfectos que cuadrados perfectos, listamos todos los cubos:

23  8, 33  27, 43  64, ..., 123  1728

Puesto que 2b  7 es siempre impar, podemos eliminar de la lista todos los cubos pares, y
quedarnos solo con
33 , 53 , 73 , 93 ,113

36 b  3b  6 es un cuadrado perfecto, es decir, 3b  6  n 2 para cierto entero n.

n2  3b  6  3b  2  3 | n2  3 | n  n  3k  n2  (3k )2  9k 2

Luego nos podemos quedar solo con aquellos cubos divisibles entre 9: 33 , 93
Ya solo nos queda comprobar si estos candidatos se adaptan a nuestras condiciones:
33  27  2b  7  b  10  3b  6  36  62 , luego la base 10 es aceptable.
93  729  2b  7  b  361  3  361  6  1089  33 2

Luego el resultado es 361  10  371 .

Segunda versión.
36 b  3b  6  3(b  2)  n 2 es un cuadrado perfecto, luego n  3k  n 2  9k 2 .
3(b  2)  9k 2  b  2  3k 2  b  3k 2  2

El valor de k está limitado superiormente: b  1000  3k 2  2  1000  k  18

Sustituyendo en la segunda ecuación:


  
m3  2b  7  2 3k 2  2  7  6k 2  4  7  6k 2  3  3 2k 2  1 
y vemos que 3 es divisor de m3 , y por tanto 3 es divisor de m .
También vemos que es impar:
 
2k 2 par  2k 2  1 impar  3 2k 2  1 impar

Y que está acotado superiormente:


b  1000  m3  2b  7  2  1000  7  2007

y 13 2  2197 , luego m  13

Los cubos que cumplen las condiciones anteriores son dos: 33 , 93 , y como en la versión
anterior, solo nos queda comprobar que, efectivamente, satisfacen las condiciones del
enunciado.

Fuente de estas versiones: [Link]

4.1.8
n 2  85 n  2017 es un cuadrado perfecto 
n 2  85 n  2017  m 2 para cierto m  Z 

n2  85n  2017  m2  0  n 

 85  85 2  4  1  2017  m2


 85  4m2  843

2 1 2 1
4m 2  843 es un cuadrado perfecto 
4m 2  843  p 2  4m 2  p 2  843   2m  p  2m  p   843  3  281

Las dos posibilidades son:


a)

 2m  p  3 
2 m  p  3
   m  71, p  139
 2m  p  281
 2 m  p  281

p  139  4m 2  843  p 2  139 2 
 85  4m 2  843  85  139 2  85  139 27
n   
2 1 2 1 2 1  112
b)

 2m  p  1 
2 m  p  1
   m  211, p  421
 2m  p  843
 2 m  p  843

p  421  4m 2  843  p 2  421 2 


 85  4m 2  843  85  421 2  85  421 168
n   
2 1 2 1 2 1  253

Comprobamos estas cuatro soluciones:


n  27  n 2  85 n  2017  5041  712
n  112  n 2  85 n  2017  5041  712
n  168  n 2  85 n  2017  44521  211 2
n  253  n 2  85 n  2017  44521  211 2

Finalmente, nos piden la suma de las soluciones positivas: 27  168  195 .

4.1.9
Seguimos la indicación propuesta.
Todo número n acabado en "08" es múltiplo de 4. En efecto, se podrá escribir como
n  m 100  8  425m  2
Luego todo número de la forma 111 ...111 se puede escribir como
111 ...111  111 ...108  3  4k  3 para cierto k natural.
Supongamos que 111 ...111  111 ...108  3  4k  3  a 2 para cierto a  IN
Está claro que a 2 es impar, luego a será impar, y por tanto a  2b  1 , para cierto b  IN .

Luego
a 2  2b  1  4b 2  4b  1  4k  3  4b 2  4b  1 
2

 
4k  4b 2  4b  2  2 2b 2  2b  1 
2k  2b(b  1)  1

Lo cual es contradictorio, pues 2k es par, y 2b(b  1)  1 es impar, llegando a contradicción.

4.1.10
Sabemos que x 2  x  n  ( x  a)( x  b)  x 2  (a  b) x  ab

Puesto que el término independiente es negativo, a o b deben ser negativos. Por lo tanto,
buscamos a, b positivos tales que:
ab  n
x 2  x  n  ( x  a)( x  b)  x 2  (a  b) x  ab    n  b(b  1)
a  b  1  a  b  1
Los números entre 1 y 100 que se pueden escribir como el producto de un número y el
siguiente son los siguientes nueve:

1  2  2 , 2  3  6 , 3  4  12 , 4  5  20 , 5  6  30 , 6  7  42 , 7  8  56 , 8  9  72 , 9 10  90

4.1.11
Nos basaremos en la igualdad
1  a  n  m  100
x 2  ax  b  x  n x  m  x 2  (n  m) x  n m  
b  n m  0
La segunda condición implica que n, m tienen el mismo signo, pero entonces la primera
condición implica que los dos no pueden ser negativos, luego n, m serán positivos o cero.

Si b  0 , el polinomio x 2  ax  b  x 2  ax  xx  a  es factorizable siempre, luego hay cien


casos: 1  a  100 .

Supongamos que 0  b  n m  n, m  0

Al no encontrar más restricción, procedemos a contar casos.

Para cada valor de 1  a  100 , las parejas no ordenadas n, m, n, m  0 , tales que n  m  a
(no importa el orden pues ( n, m ) y ( m, n ) dan lugar a la misma ecuación pues la ecuación es
simétrica en n, m)

Por ejemplo:
a 1 
a  2  1,1
a  3  1,2
a  4  1,3, 2,2
a  5  1,4, 2,3
a  6  1,5, 2,4, 3,3
a  7  1,6, 2,5, 3,4
a  7  1,6, 2,5, 3,4
a  8  1,7, 2,6, 3,5, 4,4
a  9  1,8, 2,7, 3,6, 4,5
...
a  98  1,97, 2,96, 3,95, 4,94...
a  99  1,97, 2,96, 3,95, 4,94...
a  100  1,99, 2,98, 3,97, 4,96...

Vemos que van por parejas, y que la suma de todos los elementos será
1  1  2  2  3  3  4  4  5  5  ...  49  49  50 
49
49  50
2 n  50  2  50  49  50  50  2500
n 1 2
Y sumando los 100 casos cuando b  0 dan un total de 2600 casos.
Finalmente, 2600 mod 1000  600

4.1.12
Primera versión.
Las secuencias a tratar son de la forma

 2a  1 , 2a  3 , ... , 2a  31    2a  2b  1  1  b  16

La suma será
16 16 16
s   2a  2b  1  16 2a  1   2a  16 2a  1  2 a 
b 1 b 1 b 1

16  17
 16 2a  1  2  32 a  16  16  17  32 a  16  16  25 a  8
2

Buscamos valores de a de forma que s  25 (a  8) sea un cubo perfecto.

25 (a  8)  23 22 (a  8) es un cubo perfecto  22 (a  8) es un cubo perfecto


22 (a  8)  d 3  2 | d 3  2 | d  d  2e  e3  2e   23 e3 
3


22 (a  8)  23 e3  a  8  2e3  a  2e3  8  2 e3  4 
Luego los valores de a buscados son los de la forma a  2 e3  4  
Ahora solo queda acotar el rango de valores, pues nos imponen
100  2a  1  999  99  2a  998  99 / 2  a  998 / 2  49.5  a  498 
50  a  498
Y por tanto
 
50  2 e3  4  498  50 / 2  e3  4  498 / 2  25  e3  4  249 
25  4  e  249  4  29  e3  253
3

33  27 , 53  125 , 63  216 , 73  343 , luego 4  e  6 , y hay tres secuencias.

Segunda versión. La solución oficial de este problema es mucho más expeditiva.


Queremos determinar secuencias n, n  2, n  4, n  6,..., n  30 con n impar y cuya suma sea
un cubo perfecto.
La suma es 82n  30   23 (2n  30) será un cubo perfecto si y solo si 2n  30 es un cubo
perfecto.
Luego buscamos valores n impares y 101  n  999 tales que 2n  30 sea un cubo perfecto (y
además es par).
Los cubos pares entre 232 y 1968 son 512, 1000 y 1728, que se corresponden con los valores
de n: 241, 485 y 849. Luego hay exactamente tres secuencias válidas.

Fuente de la segunda versión: Solución oficial de la OME.

4.1.13
a) Los números pares e impares van alternados, luego en la secuencia n(n  1) al menos uno de
los factores es par.
b) De la misma manera, para una sucesión de tres números consecutivos, al menos uno de ellos
ha de ser múltiplo de tres. Y también se puede aplicar el apartado a, luego la sucesión será
múltiplo de 2  3  6 .
      
c) n5  n  n n4  1  n n2  1 n2  1  nn  1n  1 n2  1 , luego podemos aplicar los
apartados anteriores para garantizar que es divisible entre 6. Veamos que también es divisible
entre 5.
Por el Teorema de la división, el número n se puede escribir como n  5k  q , 0  q  4 .
Si n  5k  0  n  5k y sería un factor de n 5  n .
Si n  5k  1  (n  1)  5k y sería un factor de n 5  n
Si n  5k  4  (n  1)  5k y sería un factor de n 5  n .
 
Si n  5k  3  (n2  1)  (5k  3)2  1  52 k 2  2  5  3k  9  1  5 5k 2  6k  2 y sería un
factor de n 5  n .
 
Si n  5k  2  (n2  1)  (5k  2)2  1  52 k 2  2  5  2k  22  1  5 5k 2  4k  1 y sería un
factor de n 5  n .
Así pues, 5 | n 5  n .
Y puesto que también son divisores 2 y 3, lo será su producto: 2  3  5  30 | n 5  n

4.1.14
Los múltiplos de 5 acaban todos en 0 o en 5, luego si es uno menor, acabará en 4 o en 9.
Los múltiplos de 4 son todos pares, luego si sumamos uno, serán impares, y por tanto no
pueden acabar en 4.
Por lo tanto, solo nos queda buscar entre los primos acabados en 9 que sean de la forma 4k  1 .
Solo hay dos: 29  4  7  1 y 89  4  22  1 .
Y su suma es 29  89  118 .

4.1.15
log b 729  a  b a  729  36  323 . Las posibilidades son las cuatro siguientes:
323  b  3, a  6
3   b  9, a  3
2 3

3   b  27, a  2
3 2

3   b  729 , a  1
2 3 1

4.1.16
a) Aplicando el algoritmo de la división, todo entero n siempre será de la forma
3a  0 , 3a  1 , 3a  2
 
n  3a  0  n 2  3a   9a 2  3  3a 2  3k
2

 
n  3a  1  n 2  3a  1  9a 2  6a  1  3 3a 2  2a  1  3k  1
2

n  3a  2  3(a  1)  1  3b  1 , tomando b  a  1
y por tanto n 2  3b  1  9b 2  3b  1  33b 2  b   1  3k  1
2

b) Es una aplicación directa del apartado anterior, pues


 
n 2  3a 2  1  3k  3a 2  3k  1  3 a 2  k  1  3 | 1 absurdo.
 
n 2  3a 2  1  3k  1  3a 2  3k  2  3 a 2  k  2  3 | 2 absurdo.

4.1.17
En primer lugar dividimos numerador entre denominador:

N2 +7 N+4
-4N +7 N-4
23
N2  7 23
Luego  N 4
N 4 N 4
23
Y por tanto nuestro problema se reduce a determinar las fracciones no irreducibles,
N 4
cuando 23 y N  4 tengan algún divisor común. Puesto que 23 es primo, esto solo pasará
cuando N  4 sea múltiplo de 23.

23 | N  4  N  4  23k  N  23k - 4
5 1994
1  23k - 4  1990  1  4  23k  1990  4  5  23k  1994  k
23 23
 1  k  86

La solución es 86.

4.1.18
El enunciado nos está diciendo que
1059  k1 d  r

1417  k 2 d  r
2312  k d  r
 3

Restando la segunda ecuación a la primera obtenemos:


358  k2  k1 d  d | 358  2 179

Restando la tercera a la segunda obtenemos:


895  k3  k2 d  d | 895  5 179

Y restando la tercera a la primera obtenemos:


1253  k3  k1 d  d | 1253  7 179

Puesto que d  1 , la única posibilidad es d  179 , y entonces, realizando por ejemplo la


primera división tenemos
1059  5 179  164  r  164

Finalmente, d  r  179  164  15

4.1.19
n 2  23  n2  1  24  (n  1)(n  1)  24
Luego basta demostrar que existen infinitos n tales que 24 | (n  1)(n  1) .

Tomando n  12 k  1, por un lado tenemos n  1  12 k  12 | n  1

Pero también vemos que n es impar, y por tanto n  1 es par, luego 2 | n  1

12 | n  1
  24 | (n  1)(n  1)  24 | (n  1)(n  1)  24  n  23
2
Finalmente,
2 | n 1 

Nota: Más fácilmente, bastaba tomar elementos de la forma n  24k  1 .

4.1.20
d | n2  1 

 
  d | n  1  1  n  1  2n  1
2 2

d | n  1  1
2

d | 2n  1  d | 2n  1  4n 2  4n  1
2

d | 4n 2  4n  1

   
  d | 4 n  1  1  4n  4n  1  4n  7
2 2

d | n  1  1 
2

d | 2n  1 
  d | 4n  7  22n  1  5
d | 4n  7 

d | 5  d  1, 5

 1 , 5 | 2 1
2

Las dos soluciones son válidas, basta tomar n  2  



 1 , 5 | (2  1)  1
2

4.1.21
El número n se puede escribir como n  10 a  b para ciertos a  0 y 0  b  9 .

Luego la condición del enunciado es a | 10 a  b , y puesto que a | 10 a se deduce que a | b

Recíprocamente, si a | b tendremos seguro que a | 10 a  b . Luego la solución es todo 0  b  9


y todo a | b .

4.1.22
Primera versión. (Versión propia)
Mediante el cambio de variable k  n  1 ,
n  7 k  8 k  8 k k  8 k
  
n 1 k k k k

Que esta división sea entera equivale a demostrar que k | k  8 k .


k | k  8 k  k 2 | k  8  k  k | k  8  k 2  16 k  64  k | 64 .
2 2

Probamos con los divisores de 64 que sean cuadrados perfectos:


k 1
k  8 k
9 1
9n2
k 1

k 4
k  8 k  12 4  6  n  5
k 4

k  16 
k  8 k  24 16  6  n  17
k 16

k  64 
k  8 k  72 64  9  n  65
k 64

La respuesta es 65  17  5  2  89 .

Segunda versión. (Solución oficial)


n  7 n 1 8
 
n 1

8

n  1 n  1  8  n  1  8
n 1 n 1 n 1 n 1 n 1 n 1 n 1 n 1
Que será entero si y solo si n  1 es un divisor de 8.
Como en la primera versión, probamos los divisores de 8 y llegamos al mismo resultado.

4.2.1
En primer lugar, vemos que d2  2 , pues si d2  2 entonces n es impar y el resto de divisores
también son números impares, pero entonces d 22 , d33 también son impares, y n  d 22  d 33 es la
suma de dos impares, luego par, llegando a contradicción.

Así pues, 1  2  d3  ...  dk  n , y n  22  d33  4  d33

En segundo lugar, d 3  3 , pues si d 3  3  n  22  33  31 que no es aceptable.

En tercer lugar, vemos que d 3 tiene que ser 4, pues


d3 | d33 
  d3 | 4  n  d3  d3  4 , y como d3  1,2,3 , necesariamente d 3  4 .
3

d3 | n 

Y, finalmente, n  d 22  d33  22  43  68 , y efectivamente, los divisores de 68 son:


1  2  4  17  34  68 , y 68  d 22  d33  22  43

4.2.2
Sabemos que el número de divisores de n 2  262 338 es 62  138  1  2457 .
Todos estos divisores se pueden agrupar por parejas a , b de forma que a  b  n 2
De forma que, o bien a  b  n o bien a  n o b  n .
Así pues, hay
62  138  1  1  1228 factores de n 2 menores que n.
2
De entre estos 1228 factores, hay 31  119  1  1  639 que son divisores de n, luego el
número buscado es 1228  639  589 .

4.2.3
ab
 a  b   nab  a 2  b 2  2ab  nab
1 1 n n
   
2

a b ab ab ab
 a  (n  2)ab  b  0
2 2

Interpretando esta última ecuación como una ecuación de segundo grado en la incógnita a ,
tenemos:
(n  2)b  (n  2)b2  4  1  b2 (n  2)b  b (n  2) 2  4
a  
2 2
b

 n  2  n 2  4n
2

De aquí se desprende que n 2  4n debe ser un cuadrado. Esto es cierto para n  4 .

Si n  4 , n 2  4n no puede ser un cuadrado. En efecto, por un lado:


n2  4n  n2  4n  2  (n  2)2

Y por otro lado:


9
n2  4n  (n  3)2  n2  6n  9  4n  6n  9  2n  9  n 
2

Así pues, para todo n  4 , n 2  4n está siempre entre (n  3) 2 y (n  2) 2 , y por tanto no puede
ser un cuadrado.

Para n  3  n 2  4n  3 , n  2  n 2  4n  4 , n  1  n 2  4n  3 , son valores


negativos, y por tanto no son cuadrados.

Así pues, la única solución es n  4 , y para esta solución tenemos


a  4  2  0  b
b
2

Efectivamente, para b  a y n  4 se cumple la igualdad del enunciado:


1 1 2 4
  
a a a aa
Fuente de esta solución: Problemas y Soluciones Volumen 1 (Francisco Javier García Capitán) , pág. 156

4.2.4
Sea x  BC . Aplicamos el Teorema de Stewart (ver GA/9.1.6) :
n 2 (n  2)  n  2  (n  2)  (n  1) 2 n  x 2  2 
(n  2)(n 2  2n)  n(n  1) 2  2 x 2 
2 x 2  (n  2)(n 2  2n)  n(n  1) 2  n(2n  3) (*)

Aquí vemos que n debe ser par, pues 2n  3 es impar. Sea n  2k . Luego
(*)  2 x 2  2k (4k  3)  x 2  k (4k  3)
De aquí se deduce que k (4k  3) debe ser un cuadrado perfecto, pero esto es imposible.
En efecto:
k (4k  3)  k  4k  (2k )2
pero por otro lado, 4k  1  3k y por tanto : (2k  1)2  4k 2  4k  1  4k 2  3k  k (4k  3)
Así pues k (4k  3) se encuentra entre dos enteros consecutivos: (2k ) 2 y (2k  1)2 , y por tanto
no puede ser entero.

4.2.5
Si escribimos
 los dígitos de n ,
n  abcd e

Entonces q  a b c y r  d e , y por tanto q  r es el entero que se forma
 

n  a b  d c  e  , y por el criterio de divisibilidad del 11, será divisible entre 11 si y solo si
 

a  c  e   b  d
es divisible entre 11, que es precisamente el criterio de divisibilidad del 11 aplicado al número
n.
Así pues, serán todos los números entre 10000 y 99999 divisibles entre 11:
99999 / 11  9090   9090  909  8181 (B)

9999 / 11  909 

Observación: Una manera más elegante de llegar a este resultado es observar que
n  100 q  r  99 q  q  r , luego q  r será divisible entre 11 si y solo si lo es n .

4.2.6
Utilizamos la desigualdad 2a 2  b 2   a  b  (ver DE Tema 1) para obtener 2n 3  n 4 , es
2

decir, n  2 .
Para n  0 , tenemos la solución a  b  0 .
Para n  1 , tenemos la solución a  1 , b  0 .
Para n  2 , tenemos la solución a  b  2 .
Así pues, las soluciones son tres: n  0 , n  1 y n  2 .

Fuente de la solución: Number Theory Structures, Examples, and Problems (Titu Andreescu, 2009) , pág 47.

4.2.7
n(n  1)  n(n  1) 
Sabemos que 1  2  3  ...  n  . Sea m   
2  2 
Entonces
n(n  1) n(n  1)
m  m  1  m2   m  1
2

2 2

Y por tanto
n(n  1)
 m2  m  1  m2  2m  1
2

2
Luego
n(n  1) n(n  1)
 m2  2m  2  2n(n  1)  2n2  2n  2n  1
2 2
En donde hemos tenido en cuenta que n  3  2n2  2n  4n2  4n  1  (2n  1)2

Y puesto que cualquier número k  2n  1 se puede escribir como suma de dos números de
1, 2 ,..., n , hemos acabado el problema.
Fuente de esta solución: Number Theory Structures, Examples, and Problems (Titu Andreescu, 2009) pág. 48

4.2.8
Supongamos que a  b  c  0 . Entonces:
abc  ab  bc  ac  ab  ab  ab  3ab  abc  3ab  c  3

Si c  1 tenemos la desigualdad
ab  b  a  ab  b  a  0
que es siempre cierta.

Si c  2 tenemos la desigualdad
ab  2b  2a  2ab  2b  2a  ab  2(b  a)  ab

Luego, puesto que a  b


4a  2(b  a )  ab  4a  ab  4  b

Si b  3 queda la desigualdad
c  2, b  3  3a  6  2a  6a  5a  6  6a  6  a

Si b  2 queda la desigualdad
c  2, b  2  2a  4  2a  4a  4a  4  4a  4  0

Luego las soluciones son:


c  1, a, b  IN , c  2, b  3, 3  a  5  , c  2, b  2, a  IN
y, puesto que la expresión es simétrica, todas las permutaciones posibles de estos elementos.

4.2.9
La cadena más larga de divisores encadenados que podemos encontrar es
1|2|4|8|16|32|64|128|256|512|1024

Que consta de 11 números. Estos 11 colores diferentes son suficientes. En efecto, es la cadena
que empieza en el número más pequeño posible y va saltando siempre mediante un factor 2,
que es el factor más pequeño posible. Cualquier otra cadena de divisores encadenados, o bien
empezará con un número inicial mayor o bien pasará de un número a su múltiplo siguiente con
un factor mayor de 2, y por tanto será más corta.

Otra forma de argumentar lo mismo la encontramos en las soluciones oficiales. Si marcamos de


un mismo color todos los números de 2k a 2k+1-1, vemos que los 11 colores utilizados son
suficientes. En efecto, si 2k ≤ a < 2k+1-1, y b=ka, puesto que k≥2,
2k  a k 1
 b  k a  2 2  2
k
y pertenecerá a una categoría con otro color diferente.
2k 
4.3.1
Serán los mismos puntos que en el extremo entre ( 0 , 0 ) y ( 48  3 , 281  17 )  ( 45 , 264 ) y ya
sabemos que son en total: mcd ( 45 , 274 )  1  3  1  4 .
Concretamente:
45  ( 3 ,17 )
 15  
3  ( 3  15 ,17  88 )  (18 ,105 )

264 (18  15 ,105  88 )  ( 33 ,193 )
 88  
3 
 ( 33  15 ,193  88 )  ( 48 ,281 )

4.3.2
( n  k , n) | n  k 
  ( n  k , n) | n  k  n  k
( n  k , n) | n 

4.3.3
Reordenando los números si es necesario, podemos suponer que
Mcd (a, b)  Mcd (b, c)  Mcd (c, a)
Luego las combinaciones posibles son:
1) Mcd (a, b)  1 , Mcd (b, c)  1 , Mcd (c, a)  7
a y c son múltiples de 7:
a  7, b  7  c  9 es una combinación aceptable.
a  7, b  14  c  23  7  14  2  Mcd (c, b)  1 no es aceptable.
2) Mcd (a, b)  1 , Mcd (b, c)  2 , Mcd (c, a)  6
a y c son múltiples de 6, b y c son múltiples de 2, luego no puede ser Mcd ( a, b)  1
3) Mcd (a, b)  1 , Mcd (b, c)  3 , Mcd (c, a)  5
a y c son múltiples de 5, b y c son múltiples de 3, luego c es múltiple de 15.
a  5, c  15  b  3 es aceptable.
4) Mcd (a, b)  1 , Mcd (b, c)  4 , Mcd (c, a)  4
a y c son múltiples de 4, b y c son múltiples de 4. Luego no es posible que
Mcd ( a, b)  1
5) Mcd (a, b)  2 , Mcd (b, c)  2 , Mcd (c, a)  5
a y c son múltiples de 5, b y c son múltiples de 2, luego c es múltiple de 10.
a  5, c  10  b  8  Mcd (a, b)  1  2
6) Mcd (a, b)  2 , Mcd (b, c)  3 , Mcd (c, a )  4
a y c es múltiple de 4, b y c es múltiple de 3, luego c es múltiple de 12.
a  4, c  12  b  7  Mcd (a, b)  1  2
a  8, c  12  b  3  Mcd (a, b)  1  2
7) Mcd (a, b)  3 , Mcd (b, c)  3 , Mcd (c, a)  3
a  3, c  3  b  17  Mcd (a, b)  1  3
a  3, c  6  b  14  Mcd (a, b)  1  3
a  3, c  9  b  11  Mcd (a, b)  1  3
a  3, c  12  b  8  Mcd (a, b)  1  3
a  3, c  15  b  5  Mcd (a, b)  1  3
a  3, c  18  b  2  Mcd (a, b)  1  3
Luego las únicas combinaciones aceptables son
a  7 , b  7 , c  9  a 2  b 2  c 2  49  49  81  179
a  5 , b  3 , c  15  a 2  b 2  c 2  25  9  225  259
Y por lo tanto la solución es 179  259  438 (B)

4.3.4
Veamos como son estos números para los primeros valores de n:
n  1  13  23  33  43  53  23  33  26  53  29  33  53
n  2  23  33  43  53  63  23  33  26  53  (2  3)3  212  36  53
n  3  33  43  53  63  73  33  (22 )3  53  (2  3)3  73  29  36  53  73
Está claro que k debe ser un divisor de 29  33  53 .
En cualquier número de la forma n3 (n  1)3 (n  2)3 (n  3)3 (n  4)3 , aparecerá un único múltiplo
de 5, luego el factor 5 3 siempre estará.
En cualquier número de la forma n3 (n  1)3 (n  2)3 (n  3)3 (n  4)3 , con n  2 , aparecerán dos
múltiples de 3, luego podemos garantizar que el factor 3 3 siempre estará.
En cualquier número de la forma n3 (n  1)3 (n  2)3 (n  3)3 (n  4)3 , con n  2 , aparecerán como
mínimo, un múltiple de 2 y un múltiple de 4, diferentes, luego podemos garantizar que el factor
  3
23  43  23  22  29 siempre estará.
Así pues, el mínimo común múltiplo es 29  33  53 (E).

4.3.5
mcm ( 50 , 6 )  150 esto quiere decir que, desde la primera vez que Fernanda atrapa la pelota
hasta la segunda vez que la atrapa, la pelota habrá pasado por encima de 150 estudiantes. De
estas 150, la habrán atrapado 150  6  25 personas. Y la pelota da 150  50  3 vueltas.
Desde la segunda vez que Fernanda atrapa la pelota hasta la tercera vez, la atraparán estas
mismas 25 personas, y así siempre.
Luego el número de personas que no atrapa la pelota será 50-25=25 personas.

4.4.1
El problema equivale a demostrar que 21n  4 , 14n  3  1 , y esto, por el TDB, es una
consecuencia directa de encontrar una combinación lineal de ambas expresiones igual a 1:

 2(21n  4)  3(14 n  3)  42 n  8  42 n  9  1

4.4.2
an  100  n 2 , an 1  100  (n  1) 2  100  n 2  2n  1  an  2n  1
Luego dn  an , an 1   an , an  2n  1   an , 2n  1 
Y también d n | (100  n 2 )  n(2n  1)  200  n
y por tanto dn | 2200  n  (2n  1)  401 .
Así pues, d n | 401 . Y este valor se alcanza en algún n. Por ejemplo, para n  200 ,
a200  100  n 2  100  200 2  100  4  100 2  100 1  4  100   100  401
a201  a200  2  200  1  100  401  401  401  101
y por tanto d 200  401 .
4.5.1
En primer lugar, podemos suponer m  n  k con k  1 . Debemos demostrar que

(n  k , n)  (n  1  k , n  1)  (n  2  k , n  2)  2k  1

Vamos a utilizar la propiedad (n  k , n) | k , independientemente del valor de n , demostrada en


el problema 4.2.

Luego
( n  k , n) | k  k  a  ( n  k , n)
(n  1  k , n  1) | k  k  b  (n  1  k , n  1)
(n  2  k , n  2) | k  k  c  (n  2  k , n  2)

k  a  ( n  k , n) 
  k | an
( n  k , n) | n 

k  b  (n  1  k , n  1)
  k | b(n  1)
(n  1  k , n  1) | n  1 

k
Puesto que ( n, n  1)  1 , de aquí deducimos que k | ab , y por tanto k  ab  b
a

k
De la misma forma deducimos que k | bc , y por tanto k  bc  b
c

Así pues,
k k k k k
(n  k , n)  (n  1  k , n  1)  (n  2  k , n  2)     b   b  2b 
a b c b b

k
Queda por demostrar que 2b   2k  1
b
k
2b   2k  1  2b2  k  2kb  b  2b2  k  2kb  b  0  (b  k )(2b  1)  0
b

Pero b | k  b  k  0 , y por otro lado b  1  2b  1  0 , con lo que acabamos la


demostración.

Fuente de la solución: Solución oficial.

4.9.1
B
4.9.2
C
4.9.3
E
4.9.4
B
4.9.5
A
4.9.6
A
4.9.7
B
4.9.8
A
4.9.9
C
4.9.10
B
4.9.11
864  25  33 

5
 864 ,1944   25  35  9  864
1944  2  3 
3

Luego los múltiples comunes de 864 y 1944 son uno de cada nueve: P  1/ 9 .

4.9.12
18  2  9
Un número será divisible entre 18 cuando sea divisible entre 2 y entre 9.
Un número es divisible entre 9 cuando la suma de sus dígitos es divisible entre 9. En este caso:
1  2  3  4  5  6  7  8  9  45 , que es claramente divisible entre 9.
Luego nuestro problema se reduce a comprobar cuando es divisible entre 2, es decir, cuando
acaba en cifra par. De las nueve cifras que tenemos, cuatro son pares, luego la probabilidad es
4/9 (B)

4.9.13
22021  22022  220211  2  22021  3
32021  32022  320211  3  32021  4  32021  22
mcd ( 22021  3 , 32021  22 )  22  3  12

4.9.14
Probando por tanteo vemos que el primero de ellos será 13  8  104 , y el último será
13  76  988 , luego hay 76  8  1  69 (B)

4.9.15
Sean las edades x, x  1, x  2, x  3, x  4, x  5 .
Todos los hermanos responderán x  5 , excepto el mayor que responderá x  4 . Luego la suma
será S  5x  5  x  4  5x  25  x  4  6x  29
Luego S  29 será un múltiple de 6. Veamos qué candidatos cumplen o no esta condición:
A : 233  29  204 sí
B : 205  29  176 no
C : 167  29  138 sí
D : 125  29  96 sí
E : 95  29  66 sí
Luego solo la opción B no cumple la condición.

4.9.16
Sean α y β los ángulos que se indican en la figura, en un polígono regular de n lados:

  360 º / n
360 º 360 º
El ángulo buscado es 2  180 º  , que será entero si y solo si es entero y positivo,
n n
es decir, cuando n sea divisor de 360 menor de 180.
360  23  32  5 . Los contamos uno a uno:
1, 2 , 3 , 5 , 2  2 , 2 3 , 2 5 , 35 , 33
2 2 2 , 2  23 , 2 25 , 235 , 233 , 335
3  2  2  2 , 2  2  25 , 3  2  25 , 3  235 , 2  233
33 2 22 , 33 225
22 en total, (C).

4.9.17
a 7
  ab  5  7
5 b
Las posibilidades son a  1, b  35 ; a  35, b  1 ; a  5, b  7 ; a  7, b  5 , cuatro en total (E).

4.9.18
Está claro que pisará un peldaño negro cada 6 peldaños, si empieza por el pie derecho serán los
peldaños 3, 9, 15... y si empieza por el pie izquierdo serán los peldaños 6, 12, 18...
Puesto que 2023=337×6+1, pisará 337 peldaños negros y puesto que el residuo es 1, menor que
3, no nos importa el pie con el que empiece. (C)

4.9.19
Está claro que el mínimo sp es 11 y el máximo sp es 77, y por tanto 11es el único divisor
común. (D)

4.9.20
Un número es múltiplo de 6 cuando lo es múltiplo de 2 y de 3. Si es múltiplo de 2, debe acabar
en cifra par. La mayor cifra par es 8, luego será de la forma “8a8”.
Si es múltiple de 3 la suma de sus cifras será múltiplo de 3. Luego
8  a  8  16  a debe ser múltiple de 3. El mayor valor de a para que esto suceda es a  8 ,
luego el número es 888 y la suma de sus dígitos es 24 (E).

4.9.21
4  6  12  13  22  29  86 huevos en total antes de la venta.
Sea x la cantidad de huevos en la cesta que ha comprado el primer cliente.
86  x  g  p  2 p  p  3 p
Luego 86  x es múltiple de 3.
86  4  82 descartado
86  6  80 descartado
86  12  74 descartado
86  22  64 descartado
86  29  57 aceptable.
La única opción válida es 29 (E).

5.1.3

17 | 2 x  3 y 
  17 | 15 x  3 y  35 x  y  , puesto que 17 | 3 , necesariamente 17 | 5 x  y
17 | 17 x 
17 | 2 x  3 y 
  17 | 2 x  14 y  2x  7 y  , y puesto que 17 | 2 , necesariamente 17 | x  7 y
17 | 17y 

17 | 5 x  y  17 | 25 x  y   10 x  2 y 
  17 | 10 x  2 y  ( x  7 y)  9 x  5 y
17 | x  7 y 

tal y como queríamos ver.



17 | 9 x  5 y 
  17 | 8 x  5 y
17 | 17 x 
17 | 9 x  5 y 
  17 | 12 y - 9 x  3(4 y  3x)  17 | 4 y - 3x  17 | 24 y - 3x   8 y  6 x
17 | 17 y 

17 | 8 x  5 y 
  17 | 8 y  6 x  8 x  5 y   8 y  6 x  8 x  5 y  3 y  2 x
17 | 8 y  6 x
tal y como queríamos ver.

5.1.4
Está claro que
121  1110  11  1110  1
11211 111100  111  111100  1
1112111 11111000  1111  11111000  1
Sea 1...1 el número formado por n unos.
n

...1  10 n1  1
Vemos que, en general, 1...121 ...1  1 ...1  10 n1  1
...1  1
n n n
Luego todos los números de esta sucesión son compuestos. (A)

5.1.5
p1  p2  p3  p4  p5
Un número de la forma n  es entero si y solo si 5 divide a la suma de
5
estos cinco primos.
Haciendo pruebas con los números primos más pequeños llegamos al resultado
30
2  3  5  7  13  30 y la media aritmética de esos cinco primos es  6 (A).
5
5.1.6
Tenemos la igualdad 2 p  2q  40  p  q  20 . Veamos los posibles casos:
19  1  A  19 , 17  3  A  51 , 13  7  A  91
El área máxima es 91 m² (C)

5.2.1
20 20  240520 , y por tanto los enteros m y n deben ser de la forma m  2a5b y n  2c5d

Luego
 2
m 2 n  20 20  2a5b 2c5d  240520  22 a52b 2c5d  240520 
2a  c  40
22 a  c52b  d  240520  
2b  d  20

La ecuación 2a  c  40 tiene las siguientes soluciones:


a  0, c  40, a  1, c  38 , a  2, c  36,..., a  20, c  0, hay 21 en total.
La ecuación 2b  d  20 tiene las siguientes soluciones:
b  0, d  20, b  1, d  18, b  2, d  16,..., b  10, d  0, hay 11 en total.
Luego el total de parejas será 21 11  231 .

5.2.2
Aplicamos las propiedades básicas de los logaritmos.
C  A log 200 5  B log 200 2  log 200 5 A  log 200 2 B  log 200 5 A  2 B 
200 C  5 A  2 B
Y ahora aplicamos el TFA:
2C  A

200 C  52  23 
C
 52C  23C  5 A  2 B  
3C  B
Por hipótesis, 1  ( A, B, C )  ( 2C , 3C , C )  C ( 2 , 3 ,1)  C  C  1
Y por tanto: A  2C  2 , B  3C  3 , y C  1 , y finalmente: A  B  C  2  3  1  6 .

5.2.3
28  211  2n  2304  2n  48 2  2n  k 2  2n  k 2  48 2  k  48 k  48  .
Luego, aplicando el TFA, k  48  2a y k  48  2b , con a  b  k .
Pero entonces, restando las dos igualdades anteriores:

25  3  96  2a  2b  2b 2a b  1 
Y , de nuevo por el TFA, b  5 , y 3  2a  b  1  4  22  2a  b  a  b  2
Con lo que llegamos a  2  b  2  5  7 , es decir, k  7  5  12 .

5.2.4
1998  2  3  3  3  37
Tenemos 1998  2  3  3  3 37  54  37 y 54  37  17
y es fácil ir comprobando, una a una, que cualquier otra combinación posible da como
resultado una diferencia mayor. Por ejemplo:
1998  2  3  3 3  37  27 104 y 104  27  77

5.2.5
Buscamos el menor número n tal que (n  1)  n  (n  1)  3n sea un cuadrado y sea un cubo
perfecto, es decir:

3n  a 2 , 3n  b3 para cierto ciertos a, b  IN


3n  a 2  3 | a 2  3 | a . Así pues, la descomposición factorial de a y a 2 :
a  p1a1 3a 2 p3a3 ... pka k  a 2  p12 a1 32 a 2 p32 a3 ... pk2 a k

También 3n  b3  3 | b3  3 | b . Así pues, la descomposición factorial de b y b 3 :


b  p1b1 3b2 p3b3 ... pkbk  b3  p13b1 33b2 p33b3 ... pk3bk

Y, puesto que a 2  b3 ,
p12 a1 32 a 2 p32 a3 ... pk2 a k  p13b1 33b2 p33b3 ... pk3bk

Y por tanto 32a2  33b2  2a2  3b2 . El número 6  mcm ( 2,3) es el exponente más pequeño
que sea múltiplo de 2 y de 3, luego, el candidato mínimo que cumpla las condiciones
anteriores será cuando tomemos solamente el factor 3 y con su exponente más pequeño:
3n  36  n  35  243 .
Así pues, los números buscados son 242, 243 y 244.

242  243  244  729  36  32  33    


3 2

5.2.6
Primera versión.
Queremos determinar números a  0 y n  3 tales que

2009  a1  a2 ,...  an  a  1  a  2  a  3  ...  a  n  n  a  (1  2  3  ...  n) 


n(n  1)
 na 
2

n(n  1)  n  1   2a  n  1  n
Es decir: 2009  7 2  41  n  a   n a    n   2a  n  1
2  2   2  2
Estudiemos todas las posibilidades:
n 1 7 1
n7,a  7  41  a  7  41   283
2 2
n 1 49  1
n  7 2  49 , a   41  a  41   16
2 2
n 1 41  1
n  41 , a   7 2  49  a  49   28
2 2
n 1 287  1
n  41  7  287 , a  7a 7  137 (*)
2 2
n 1 11
n 1, a   2009  a  2009   2008 (*)
2 2
n 1 2009  1
n  2009 , a  1 a 1  1004 (*)
2 2
n 7  41  1  14
 7  n  14 , 2a  n  1  2a  14  1  7  41  a   136
2 2
n 49  1  82
 41  n  82 , 2a  n  1  2a  82  1  49  a   17 (*)
2 2
n 41  1  98
 7 2  49  n  98 , 2a  n  1  2a  98  1  41  a   29 (*)
2 2
n 2009  1  2
 1  n  2 , 2a  n  1  2a  2  1  2009  a   1003 (*)
2 2
n 1  1  4018
 2009  n  4018 , 2a  n  1  2a  4018  1  1  a   2009 (*)
2 2

Las soluciones marcadas con (*) quedan descartadas por no cumplir las condiciones del
enunciado, luego las soluciones del problema son cuatro:

a  283 , n  7 ; a  16 , n  49 ; a  28 , n  41 ; a  136 , n  14

Segunda versión.
En primer lugar, acotamos los posibles valores de n:
 n 1  n 1
2009  n a    n 
 2   2 
 n 1
La ecuación 2009  n  tiene soluciones n  62.8 , luego n  62 .
 2 

 n 1  2a  n  1  n
2009  7  7  41  n a    n   2a  n  1
 2   2  2

Si n es impar, n  7 , n  41 o n  7 2  49 , pues cualquier otro divisor impar de 2009 es mayor


que 7  41  287 , y se obtienen las sucesiones con 283, 28 y 16.

Si n es par, entonces n / 2 es un divisor de 2009, y por tanto n  14 y a  136 , porque


cualquier otra combinación daría un n  62 .

Fuente de la segunda versión: Solución oficial.

5.2.7
2001  3  23  29  1 3  23  29 , luego el mayor valor posible aparecerá multiplicando los dos
factores más grandes: 23  29  667
Luego será tomando I  1, M  3, N  667  I  M  O  667  3  1  671

5.2.8
En primer lugar, aplicamos las propiedades fundamentales de los logaritmos:
A log 200 5  B log 200 2  C 
log 200 5 A  log 200 2 B  C 
 
log 200 5 A 2 B  C 
5 A  52C  A  2C
5 2  200  2 5
A B C
 
3 2 C
2 5
3C 2 C
 B
2  23C  B  3C

En el último paso hemos aplicado el TFA, es decir, la unicidad de la factorización de todo


entero.
Está claro que C | A y C | B , luego para que A y B sean coprimos es necesario que C  1 , y
por tanto A  2, B  3 , y A  B  C  2  3  1  6

5.2.9
Por el TFA, puesto que 2310  2  3  5  7 11 , cinco números primos, el número de formas de
escribir 2310  2  3  5  7 11 como producto de tres números será el número de formas que
tenemos de repartir los números 2 , 3, 5, 7 ,11 en tres grupos, sin repeticiones y sin importar el
orden.
Puesto que uno de los tres elementos puede ser 1 (como mucho uno de ellos, pues suponemos
que a, b, c son distintos), también debemos considerar el caso que uno de los tres grupos esté
vacío, por ejemplo a  1 , b  2  3 , c  5  7 11 .

Organizamos los casos en función del número de elementos de cada grupo:


5!
i) (3,1,1)  C53   10 posibilidades.
3!(5  3)!
5!
ii) (0,1,4)  C54   5 posibilidades.
4!(5  4)!
5!
iii) (0,2,3)  C53   10 posibilidades.
3!(5  3)!
iv) (1,2,2)  5 posibilidades para el primer grupo, y el número de posibilidades de repartir 4
1  4
elementos en dos grupos: 5     5  3  15 posibilidades.
2  2

Luego, en total, hay 10  5  10  15  40 posibilidades.

5.2.10
Primera versión.
   
16 p  1  n3  16 p  n3  1  n  1 n2  n  1 luego 24 p  n  1 n2  n  1
De todas las posibilidades que podemos plantear, si n  1 es 1 , 2 , 4 , 8 entonces
n2  n  1  n(n  1)  1 será múltiplo de 2, lo cual es absurdo pues es impar (el producto de dos
números consecutivos es par, y si le sumamos 1 será impar).

Veamos la opción n  1  24 , p  n2  n  1
n  1  24  n  17  p  17 2  17  1  307 , que es primo, y 16 p  1  4913  17 3
Segunda versión.
De 16 p  1  n3 se deduce que n tiene que ser impar. Luego n  2a  1
 
16 p  1  (2a  1)3  8a 3  12 a 2  6a  1  16 p  2a 4a 2  6a  3 

8 p  a 4a 2  6a  3 
4a 2  6a  3 es impar, luego en su factorización no puede tener ninguna potencia de 2.
La única opción válida es que a  8 , y por tanto p  4a 2  6a  3  4  82  6  8  3  307 .

Tercera versión.(Utilizando congruencias, que se estudiarán en el Tema 6)


De 16 p  1  n3 se deduce que n  1 (mod 4) , y por tanto n  4a  1 , luego
16 p  1  64 a 3  48 a 2  12 a  1 
16 p  64 a 3  48 a 2  12 a 
 
4 p  a 16 a 2  12 a  3  4 | a  a  4b 

4 p  4b 16  16 b  12  4b  3 
2


p  b 256 b 2  48b  3 
Y, puesto que p es primo, b  1  p  256  12  48  1  3  307

5.2.11
702  am  r 
 787  bm  702  am 85  5  17  m(b  a) 
787  bm  r r  702  am   
 855  cm  702  am  153  32  17  m(c  a)
855  cm  r 
De donde deducimos que m  17 , y por tanto:
702  41  17  5 

787  46  17  5  r  5
855  50  17  5 
412  dn  s 
 722  en  412  dn 310  2  5  31  n(e  d )
722  en  s   s  412  dn   
 815  fn  412  dn  403  13  31  n( f  d ) 
815  fn  s 
De donde deducimos que n  31 , y por tanto:
412  13  31  9 

722  23  31  9  s  9
815  26  31  9 

Y, finalmente, m  n  r  s  17  31  5  9  62

5.2.12
El polinomio se podrá expresar de la forma f ( x)  a ( x  b)( x  c) , con a, b, c enteros.
Sabemos que 2  3  5  67  2010  f (0)  a (0  b)(0  c)  a b c

Primera parte: a, b, c son positivos.


Nuestro problema se reduce a contar el número de combinaciones posibles de 4 elementos
agrupados en tres cajas a, b, c , teniendo en cuenta que alguna (pero no las tres a la vez) puede
estar vacía (es decir, conteniendo el factor 1). Las cajas b y c son indistinguibles.
a) 4,0,0 a  2010 , b  1, c  1 1 caso
b) 3,1,0 a  p1 p2 p3 , b  p4 , c  1 : 4 casos.
 4
c) 2,1,1 a  p1 p2 , b  p3 , c  p4  : C42    
4!
 6 casos
 2  2!(4  2)!
 4
d) 2,2,0 a  p1 p2 , b  p3 p4 , c  1 : C42    
4!
 6 casos
 2  2!(4  2)!
e) 1,0,3 a  p4 , b  1 , c  p1 p2 p3  : 4 casos.

 4
f) 1,2,1 a  p1 , b  p2 p3 , c  p4  : C42    
4!
 6  6  2  12
 2  2!(4  2)!

g) 0,4,0 a  1 , b  2010 , c  1 : 1 caso


h) 0,3,1 a  1 , b  p1 p2 p3 , c  p4  : 4 casos.
 4
i) 0,2,2 a  1 , b  p1 p2 , c  p3 p4  : C42    
4!
 6 casos/2 =3
 2  2!(4  2)!

Total: 1+4+6+6+4+12+1+4+3 = 41 casos.

Segunda parte: a, b, c con signo.


Dos de estos números pueden ser negativos, no los tres, luego el signo multiplica por cuatro las
posibilidades:
a  0, b  0, c  0 2( x  3)( x  5  67 )  2 x 2  676 x  2010
a  0, b  0, c  0 2( x  3)( x  5  67 )  2 x 2  676 x  2010
por ejemplo:
a  0, b  0, c  0 (2)( x  3)( x  5  67 )  2 x 2  664 x  2010
a  0, b  0, c  0 (2)( x  3)( x  5  67 )  2 x 2  664 x  2010

Excepto en el caso a) que solo tiene tres:


2010 ( x  1)( x  1)  2010 x 2  4020 x  2010
2010 ( x  1)( x  1)  2010 x 2  4020 x  2010
(2010 )( x  1)( x  1)  2010 x 2  2010
(2010 )( x  1)( x  1)  2010 x 2  2010

luego el total es 4  40  3  163 polinomios diferentes.

5.2.13
 p p 2  4  444 p
x  px  444 p  0  x 
2

2
Las soluciones serán enteras si y solo si p 2  4  444 p es entero, es decir, si p 2  4  444 p es
un cuadrado perfecto.
 
Si p 2  4  444 p  p p  24  3  37 es un cuadrado perfecto entonces
 
p | p  24  3  37  p | 24  3  37  p  2 , 3, 37 .
Comprobemos estos posibles valores:
p  2  p 2  4  444 p  3556 no es un cuadrado.
p  3  p 2  4  444 p  5335 no es un cuadrado.
p  37  p 2  4  444 p  67081  259 2 es un cuadrado, y las soluciones de la ecuación son 111
y -148.

5.2.14
Observamos que todo número natural se puede escribir de forma única de la forma
a  2 k b , con b impar y 0  k .
Observamos también que si el número impar b es el mismo, digamos a1  2k1 b y a2  2k2 b ,
Entonces
k1  k2  a1  a2 , k1  k2  a1 | a2 , k1  k2  a2 | a1

y en todo caso uno divide al otro.

Entre 1 y 2n hay n números impares, por lo tanto, si asignamos a cada número xi  2 ki bi de


nuestro conjunto su valor impar asociado bi , al haber n  1 elementos en nuestro conjunto,
aplicando el Principio del casillero, al menos dos elementos tendrán asociados el mismo impar,
y por tanto, como hemos visto al principio, uno será divisor del otro.

5.2.15
k será divisor de 1212=224·312, luego será de la forma k=2a3b, con 0  a  24 y 0  b  12 .
1212=224·312 es el menor múltiplo de 66=2636 y 88=224 y k=2a3b, luego 0  b  6 .
12=max{6,b} y de esto deducimos que b=12.
Hay 25 valores diferentes para a, luego hay un total de 25 valores diferentes para k.

5.2.16
[a,b]=1000=2353
[b,c]=2000=2453
[c,a]=2000=2453
Luego a  2i1 3 j1 , b  2i2 3 j 2 , c  2i3 3 j3 .
Vamos a estudiar, en primer lugar, los exponentes del 3:
Está claro que j1 , j2 , j3  3 , y que al menos dos elementos de j1, j2 y j3 tienen que ser iguales a
3. No hay más limitaciones.
Las opciones son (0,3,3) , (1,3,3) , (2,3,3) , (3,3,3) , (3,0,3) , (3,1,3) , (3,2,3) , (3,3,0) , (3,3,1) ,
(3,3,2) . Hay 10 en total.
Veamos ahora los exponentes del 2:
i1, i2  3 y entre i1, i2 uno de los dos tiene que ser igual a 3
i2 , i3  4 y entre i2, i3 uno de los dos tiene que ser igual a 4.
i1 , i3  4 y entre i1, i3 uno de los dos tiene que ser igual a 4.
Si i1=3, entonces i3=4, y la única condición para i2 es i2  3 .
Si i2=3, entonces i3=4, y la única condición sobre i1 es i1<=3.
Los casos posibles son 16:
(i1,i2)=(3,0), (3,1), (3,2), (3,3), (0,3), (1,3), (2,3)
Hay 7 casos diferentes.
Luego el total es 7·10=70 triplas (a,b,c) ordenadas diferentes.

5.2.17
y 2  3 x 2 y 2  30 x 2  517  y 2  3 x 2 y 2  30 x 2  517 
y 2
    
 10 3 x 2  1  10  517  y 2  10 3x 2  1  507  3  13 2

Casos:

 y  10  3  y  13
2 2

 2 imposible, pues 13 no es un cuadrado.



3x  1  13
2


 y  10  13  y  23
2 2

 2 imposible, pues 23 no es un cuadrado.



3x  1  3 13

 y  10  13  y  13  10  179
2 2 2 2

 2 imposible, pues 179 no es un cuadrado.



3x  1  3 13

 y  10  3 13  y  39  10  49  y  7
2 2

 2 , en todo caso, 3x 2 y 2  12  49  588



3x  1  13  3x  12  x  4  x  2
2 2


 y  10  1  y  11
2 2

 2 , imposible, pues 11 no es un cuadrado.



3x  1  517

 y  10  507  y  517
2 2

 2 , imposible, pues 517 no es un cuadrado.



3x  1  517

La única solución posible es 588

5.2.18
Primera versión.
16 p  1 es impar. El cubo de un par es par, luego a es impar: a  2b  1
16 p  1  a 3  (2b  1)3  8b3  12 b 2  6b  1 
  
16 p  2b 4b 2  6b  3  8 p  b 4b 2  6b  3 
Por un lado, p es primo, luego impar, y por otro, 4b 2  6b  3 es impar.
Luego b  8 y p  4b2  6b  3  307 .

Segunda versión.
Observamos que el cubo de un par es par, luego a debe ser impar, pues 16 p  1 es impar.

16 p  1  a3  24 p  a3  1  (a  1) a 2  a  1 
a  1 es par, a 2  a  1  a(a  1)  1 es impar, luego a  1 debe ser múltiple de 16.
a3  1
Pero p  y por tanto, cualquier otro múltiplo que no sea 16 hará que p no sea primo.
16
Así pues, a  1  16  a  17  p  307 .
Fuente de estas soluciones: [Link]

5.2.19
m  ( a, b)
a  k1m
m | a, b  
b  k2 m
(a, b)a, b  ab  k1 m k2 m  k1 k2 m 2  ma, b  k1 k2 m 2  a, b  k1 k2 m

Y la ecuación queda
m  k1 k2 m  k1 m  k2 m  6  m  k1 k2 m  k1 m  k2 m  6 
m1  k1 k2  k1  k2   6  mk1  1k2  1  6  1  2  3

Las opciones son:


m  1, k1  1  2, k2  1  3  a  3, b  4  (a, b)  1 , a, b  12
m  2, k1  1  1, k2  1  3  a  4, b  8  (a, b)  4 , a, b  8 
m  3, k1  1  2, k2  1  1  a  9, b  6  (a, b)  3 , a, b  18
m  6, k1  1  1, k2  1  1  a  12, b  12  (a, b)  12 , a, b  12 
m  1, k1  1  6, k2  1  1  a  7, b  2  (a, b)  1 , a, b  14

Las opciones marcadas con  no cumplen la ecuación del enunciado, luego las soluciones son
a, b  3,4,2,7,6,9.
5.2.20
Sabemos que a, b(a, b)  a  b , luego en el caso de ser coprimos a, b  a  b , y por tanto
a, b serán las soluciones de la ecuación de segundo grado
x 2  (a  b) x  a, b  0

Si no son coprimos, realizamos el cambio de variable


a  ( a, b) a '
para ciertos a ' , b' tales que (a, b)  1
b  (a, b)b'

Y podemos aplicar el método anterior, para finalmente deshacer el cambio de variable.

Luego necesitamos conocer el máximo común múltiplo de a, b . Para ello basta tener en cuenta
que (a, b)   a  b , a, b  .

En el caso concreto del enunciado determinaremos el mínimo común múltiplo mediante el


algoritmo de Euclides:

985928  3972  248  872


985928,3972  3972 248  872,3972  872,3972
3972  4  872  484
3972,872  4  872  484,872  484,872
872  1 484  388
872,484  1 484  388,484  388,484
484  1 388  96
484,388  1 388  96,388  96,388
388  4  96  4
388,96  4  96  4,96  4,96  4
Así pues, 985928,3972  4
985928  246482  4
3972  993  4

Y tenemos que resolver la ecuación x 2  993 x  246482  0


  993 2  4  246482  986049  985928  121  112
993  11 502
x 
2 491
Y por tanto los números buscados son 502  4  2008 y 491  4  1964

5.2.21
n  50  a 2 

2
 a 2  100  b 2  100  b 2  a 2  (a  b)(a  b)
n  50  b  
2  5  100  (a  b)(a  b)
2 2

Veamos todas las posibilidades:


a  b 1 
  2a  101 no es entero
a  b  100 
ab  2 
  2a  52  a  26, b  2  26  0 negativo
a  b  50 
ab  4 
  2a  29 no es entero
a  b  25 
ab 5 
  2a  25 no es entero
a  b  20 
a  b  10 
  2a  20  a  10, b  10  a  0
a  b  10 
a  b  20 
  2a  25 no es entero
a b 5 
a  b  25 
  2a  29 no es entero
a b  4 
a  b  50 
  2a  52  a  26  b  50  26  24
a b  2 
a  b  100 
  2a  101 no es entero
a b 1 

Así pues, las dos únicas posibilidades son:


a  10, b  0, n  10 2  50  50 , en efecto, 10 2  50  50 , 02  50  50 .
a  26, b  24, n  26 2  50  626 , en efecto, 24 2  50  626 , 26 2  50  626 .

Las soluciones son 626 y 50.

5.2.22
Son todos aquellos números que se pueden escribir de la forma a  2 a1 5a211a3 y b  2b1 5b211b3
0  a1 , b1  3 , 0  a2 , b2  7 y 0  c1 , c2  13
cumpliendo, además max( a1 , b1 )  3 , max( a2 , b2 )  7 , max( a3 , b3 )  13
Las posibilidades para a1 ,b1 son las siguientes 7:
(0,3),(1,3),(2,3),(3,3),(3,2),(3,1),(3,0) .
De la misma forma, las posibilidades para a2 ,b2 son 15, y las posibilidades para a3 ,b3 son 27.
Finalmente, llegamos a un total de 7 15  27  2835 .

5.2.23
13 ! 13  11  7  52  35  210

m m
Este número será un entero cuadrado perfecto si y sólo si m es un divisor del numerador de
forma que genere un cociente con potencias pares, es decir:
m  13  11  7  5a  3b  2 c con a  0,2 , b  1, 3, 5 y c  0, 2, 4, 6, 8, 10
Para sumar todos estos 2  3  6  36 términos, podemos sacar factor común:
S1  1  2 2  2 4  26  28  210  1365  3  5  7  13
S 2  S1  32  S1  34  S1  S1 1  9  81  S1  91  3  5  7  13  7  13  3  5  7 2  13 2
S3  S 2  52 S 2  S 2 1  25   S 2  26  3  5  7 2  13 2  2  13  2  3  5  7 2  13 3
S3  13  11  7  50  31  20 S3  13  11  7  50  31  20  2  3  5  7 2  133 
 2  32  5  73  11  13 4
Así pues, el resultado es 1  2  1  3  1  4  12 .

5.2.24
Está claro que la resolución de este problema se basa en el estudio de la factorización de todos
los números hasta el 25. Una de las claves para resolver este problema es ver que forzosamente
deberemos eliminar todos los primos superiores a 7: 13, 17, 19 y 23, pues no van a encontrar
“pareja”.

2 2
3 3
4 2²
5 5
6 2 3
7 7
8 2³
9 3²
10 2 5
11 11
12 2² 3
14 2 7
15 3 5
16 24
18 2 3²
20 2² 5
21 3 7
22 2 11
24 2³ 3
25 5²
Total 22 12 6 3 2

Vemos que el único factor problemático es el 7, que encuentra pareja con 14 y 21, luego hay
tres “7”. Será imposible separalos en dos grupos, luego hay que eliminar uno de ellos. Vemos
que el factor 2 y el factor 3 quedan pares, luego lo más sencillo es eliminar el 7.

Hemos eliminado 5 números, que es una de las opciones del enunciado (B). ¿Tendremos que
eliminar más? En el contexto de una prueba con tiempo limitado, tal vez la mejor estrategia sea
arriesgarnos y escoger (B). Si queremos o podemos dedicarle más tiempo al problema, se puede
comprobar que es la opción correcta, pues con el resto de números se pueden hacer dos grupos
con el mismo producto:
3×5×8×14×15×18×20×22×24 = 2×4×6×9×10×11×12×16×21×25 = 211 ×35 ×53 ×7×11

Fuente de este ejemplo: Compendium Canguro UK pág. 431

5.2.25
Está claro que nuestros números serán de la forma
a  2 a 2  3a 3  5a 5 , b  2 b2  3b3  5b5 , c  2c2  3c3  5c5 y d  2d 2  3d3  5d5
cumpliendo a2  b2  c2  d2  6 , a3  b3  c3  d3  9 y a5  b5  c5  d5  7 .
Sabemos que el mínimo común múltiplo de dos números se obtiene mediante el máximo
exponente, luego tenemos que ir imponiendo las condiciones. Los exponentes de las potencias
de 2, 3 y 5 son independientes, luego las iremos determinando por orden.

En primer lugar, la potencia de 2.


a2  b2  c2  d2  6 , max( a2 , b2 )  3 , max( a2 , c2 )  3 . max( a2 , d2 )  3 . max( b2 , c2 )  1
max( b2 , d2 )  2 , max( c2 , d2 )  2
max( b2 , c2 )  1  b2  1
  a2  3
max( b2 , a2 )  3 
Nos quedan las condiciones siguientes;
b2  c2  d2  3 , max( b2 , c2 )  1 , max( b2 , d2 )  2 , max( c2 , d2 )  2
La única combinación aceptable es b2  1 , c2  0 , d2  2 o b2  0 , c2  1, d2  2 ,Y el mínimo
será 0.

Ahora la potencia de 3.
a3  b3  c3  d3  9 , max( a3 , b3 )  2 , max( a3 , c3 )  3 . max( a3 , d3 )  3 . max( b3 , c3 )  3
max( b3 , d3 )  3 , max( c3 , d3 )  3
max( a3 , b3 )  2  b3  2
  c3  3
max( b3 , c3 )  3 
max( a3 , b3 )  2  a3  2
  d3  3
max( b3 , d 3 )  3 
Nos quedan las condiciones siguientes: a3  b3  3 y max( a3 , b3 )  2
Por tanteo la única condición aceptable es que uno de ellos sea 1 y el otro 2. En todo caso el
mínimo será 1.

Finalmente, la potencia de 5.
a5  b5  c5  d5  7 , max( a5 , b5 )  3 , max( a5 , c5 )  3 . max( a5 , d5 )  3 . max( b5 , c5 )  2
max( b5 , d5 )  2 , max( c5 , d5 )  2 .
max( b5 , c5 )  2  b5  2
  a5  3
max( a5 , b5 )  3 
Nos quedan las condiciones siguientes:
b5  c5  d5  4 , max( b5 , c5 )  2 , max( b5 , d5 )  2 , max( c5 , d5 )  2 .
La única opción aceptable es que dos de ellos sean 2 y el otro 0. Luego el mínimo es 0.

Puesto que el máximo común divisor de a,b,c y d se toma con los exponentes mínimos,
tenemos que Mcd (a, b, c, d )  20  31  50  3

5.3.1
 
Puesto que n3  1  n  1 n2  n  1 , y teniendo en cuenta que n 2  n  1  1 ,
n3  1 primo implica que n  1  1  n  2  n 2  1  7 . El único primo de esta forma es el 7.

5.3.2

N  2903 n  803 n  464 n  261n  2903 n  803 n  464 n  261n 
Aplicando la proposición anterior:
7  300  2100  2903  803 divide a 2903 n  803 n
7  29  203  464  261 divide a 464 n  261 n
Y por tanto 7 divide a N.


Por otro lado, 2903 n  803 n  464 n  261n  2903 n  464 n  803 n  261n 
271  9  2439  2903  464 divide a 2903 n  464 n
271  2  542  803  261 divide a 803 n  261 n
Y por tanto 271 divide a N.

Finalmente, puesto que (7,271)  1 , tendremos que 1897  7  271 dividirá a N.

5.3.3
Primera versión.
Está claro que para n  1 , n 4  4  5 que es primo.
n 4  4  n 4  4n 2  4  4n 2  (n 2  2) 2  (2n) 2  (n 2  2  2n)(n 2  2  2n) 
 
 (n  1) 2  1 (n  1) 2  1
Y si n  1 cada factor es mayor que 1, luego n 4  4 no puede ser primo.

Segunda versión.
Está claro que para n  1 , n 4  4  5 que es primo.

Aplicamos la identidad algebraica x 4  4 y 4  x 2  2 xy  2 y 2 x 2  2 xy  2 y 2 .  
En nuestro caso:

n4  4  n4  4  14  n2  2n  2 n2  2n  2  
Puesto que n 2  2n  2  1 , debemos suponer que
n2  2n  2  1  0  n2  2n  1  (n  1)2  n  1

5.3.4
Si n  1 , n 4  4 n  1  4  5 que es primo. Supongamos que n  1 .
Está claro que si n es par no será primo:
n  2k , k  1  n4  4n  (2k )4  42 k  16 k 4  16 k es un múltiplo de 16.

Supongamos que n es impar.


    2 2

n 4  4 n  n 2  2 n  2n 2 2 n  2n 2 2 n  n 2  2 n  2n 2 2 n  
2

 n  2   n 2  n  2   2
n 2
n n 2
n 1 ( n 1) / 2 2
2 2 2

 n  2  2
2 n
n n  2  2
( n 1) / 2
n 2 n ( n 1) / 2

n 1
Esta última igualdad está bien construida pues estamos suponiendo que n es par, luego es
2
entero.

Está claro que n 2  2n  2( n 1) / 2 n  1 .


(Faltaría demostrar que n 2  2n  2( n 1) / 2 n  1 si n  3 ).

5.3.5
Aplicamos el Teorema del Binomio:
n  1n  n n    n n1    n n2  ...    n1  1  n n    n n1    n n2  ...  n 2  1
n n n n n
1  2 1 1  2
n n  n n 
Luego n  1  1  n n    n n 1    n n  2  ...  n 2  n 2  n n  2    n n  3    n n  4  ...  1 ,
n

1  2  1  2 

5.3.6
Teniendo en cuenta que 1993 es impar, basta agrupar por parejas:
1001  1  1000 | 11993  1000 1993
1001  2  999 | 21993  999 1993
1001  3  998 | 31993  9981993

Por lo tanto, 1001 dividirá al total.

5.3.7

De la igualdad x 2n 1  y 2n 1  ( x  y) x 2n  x 2n 1 y  x 2n  2 y 2  ...  y 2n 
deducimos que si n es impar, a  b | a n  b n . En nuestro caso n  47 impar, luego:
Luego 7  1  6 | 147  647 , 7  2  5 | 247  547 y 7  3  4 | 347  447 , y por tanto
7 dividirá a la suma de todos, tal y como queríamos ver.

5.3.8
Supongamos que n  ab , es decir, n  10 a  b . Entonces
10 a  b2  10b  a 2  100 a 2  20 ab  b2  100 b2  20 ab  a 2  
 
 100 (a 2  b 2 )  b 2  a 2  100 (a 2  b 2 )  a 2  b 2  99 (a 2  b 2 ) 
99 (a  b)(a`b)
Que es un número divisible entre 9, entre 11, entre la suma de los dígitos y entre la diferencia
de los dígitos, luego la respuesta es (B).

5.3.9
Observamos que
n5  n 4  1  n5  n 4  n3  n3  n 2  n  n 2  n  1  n3 n 2  n  1  nn 2  n  1  n 2  n  1 
 
 n 2  n  1 n3  n  1 
Y por lo tanto se puede siempre escribir como producto de dos enteros mayores que 1. Luego
no es primo.

5.3.10
Observamos que
 
2
a 4  4b 4  a 4  4b 4  4a 2b 2  4a 2b 2  a 4  2b 2  4a 2b 2 
 
 a 2  2b 2  2ab a 2  2b 2  2ab 
y que a 2  2ab  2b 2  a 2  2ab  2b 2 , luego será primo si y solo si

a  2ab  2b  1
2 2

 4

a  4b  a  2ab  2b
4 2 2

1  a 2  2ab  2b2  (a  b)2  b2  2b2  (a  b)2  b2

Las posibilidades son:


( a  b) 2  1 
  a  1  a  4b  1 que no se considera primo.
4 4

b  0  b  0
2

( a  b) 2  0 

  (a  1)  0  a  1  0  a  1  a  4b  1  4  5
2 4 4

b  1  b  1
2

5.3.11
a  2  b  88 y no es primo.
a  3  b  87 y no es primo.
a  5  b  85 y no es primo.
a  7  b  83 y sí es primo . (D)

5.3.12
Sean a, b  0 estas dos soluciones. Por las fórmulas de Vieta (ver PA 4.5), sabemos que se
cumplirá
 p  ab

q  a  b

Luego, o bien se cumple


a  1, b  p  q  1  p  p  2, q  3
o se cumple
a  p, b  1  q  1  p  p  2, q  3

Puesto que 2 y 3 son los dos únicos primos cuya diferencia es 1.

5.3.13

Aplicando la identidad x3  1  x  1 x 2  x  1 

27000001  300 3  1  300  1 300 2  300  1 
301 es divisible entre 7, en efecto: 301  7  43 , y ya hemos encontrado dos de los cuatro
factores.

300 2  300  1  300 2  2  300  1  900  300  1  30 2  301  30 301  30  


2

 271  331

Estos dos últimos factores son primos, por lo que hemos resuelto el problema:
27000001  7  43  271  331

5.3.14
Aplicando la identidad “Suma por diferencia”:
x 2  y 2  ( x  y)( x  y)  23

Y, puesto que 23 es primo, sus únicos divisores son 1 y 23. Las posibilidades son, por tanto:
x  y  1  x  y  23
  x  12, y  11   x  12, y  11
 x  y  23 x  y  1
 x  y  1  x  y  23
  x  12, y  11   x  12, y  11
 x  y  23  x  y  1

La única solución aceptable con enteros positivos es x  12, y  11 .

5.3.15
Nos encontramos ante la ecuación diofántica
pq  ( p  q)  a

Donde a es uno de los cinco posibles candidatos del enunciado.

Aplicando el SFFT,
a  pq  ( p  q)  pq  p  q  ( p  1)(q  1)  1  ( p  1)(q  1)  a  1

Los números primos entre 4 y 18 son: 5, 7, 11, 13, 17


y probando distintas combinaciones vemos que la única combinación aceptable es
(13  1)(11  1)  12  10  120  119  1
Y la respuesta correcta es (C)

5.3.16
Realizando el cambio de variable a  m2 , b  n2 convertimos la ecuación del enunciado en
a  3ab  30b  517

para ciertos cuadrados a, b que queremos determinar.


a  3ab  30b  517  517  a  3ab  30b  (a  10 )(3b  1)  10 
(a  10 )(3b  1)  507  3  13 2

a  10  1  a  11
a  10  3  a  13
a  10  13  a  23
a  10  3  13  39  a  49  7 2
a  10  13  13  169  a  169  10  179
a  10  507  a  517

La única opción con la que obtenemos un cuadrado es a  49 , y por tanto


3b  1  13  b  4  22 .

Así pues, 3m 2 n 2  3ab  3  49  4  588

5.3.17
Completamos cuadrados:
y 2  x 2  84 x  2008  x 2  2  42 x  42 2  244  ( x  42 ) 2  244 
y 2  ( x  42 ) 2  244 
( y  x  42 )( y  x  42 )  244  22  61

Vamos probando las posibles factorizaciones de 244:


y  x  42  4 
  x  27 / 2, y  65 / 2
y  x  42  61
y  x  42  2 
  x  18, y  62
y  x  42  2  61

Puesto que el enunciado dice que solo hay una solución con enteros positivos, debe ser esta, y
no hace falta seguir probando. La respuesta correcta es 18  62  80 .

5.3.18
181 2   x  1  x 3  x 3  3 x 2  3 x  1  x 3  3 x 2  3 x  1 
3

181 2  1  3 x 2  3 x  3 x x  1  32760  3 x x  1  10920  x x  1


Puesto que 100 2  10000 , vamos probando con números consecutivos superiores a 100 hasta
encontrar la solución 10920  104 105 , así pues, la solución es 181 2  105 3  104 3

Observación: Una manera mucho más elegante de acabar este problema sería
3 x x  1  181 2  1  (181  1)(181  1)  182  180  2  7  13  2 2  32  5 
 
x x  1  23  3  5  7  13  23  13  3  5  7   104  105

5.3.19
2 1 1 1 x  y 1
     5( x  y  1)  2 xy  5 x  5 y  2 xy  5 
5 x y xy xy
 5  5 25  15
 x   2 y  5   5   x   2 y  5  5 
25
 
 2 2  2 2 2
 5
2 x   2 y  5  15  5  2 x  2 y  5  15  3  5
 2

Vamos probando posibilidades:


5  2x  1  5  2x  3 
  x  2, y  5   x  1, y  0
 2 y  5  15   2 y  5  5
5  2x  5  5  2 x  15 
  x  0, y  1   x  5, y  2
 2 y  5  3  2 y  5  1
5  2 x  1  5  2 x  3 
  x  3, y  10   x  4, y  5
 2 y  5  15   2 y  5  5
5  2 x  5  5  2 x  15 
  x  5, y  4   x  10, y  3
 2 y  5  3  2 y  5  1

Las únicas soluciones aceptables son: x  3, y  10 ; x  4, y  5 ; x  5, y  4 ; x  10 , y  3

5.3.20
      
36  26  33  23  33  23 33  23  27  827  8  19  35  19  5  7
2 2

Y la suma es 19  5  7  31 (B)

5.3.21
x  1 y  1  77  7 11, y puesto que ambos factores son positivos, las únicas posibilidades
son:
x  1  1
  x  0 , no aceptable.
 y  1  77
x  1  7
  x  6, y  10  x  y , no aceptable.
 y  1  11
 x  1  11
  x  10, y  6 , aceptable.
y 1  7
 x  1  77
  x  76, y  0 , no aceptable.
y 1  1
Así pues, la única solución que cumple las condiciones del enunciado es (D).

5.3.22
   
16383  16384  1  214  1  27  12  27  1 27  1  128  1128  1  127  129 
2

 3  43  127

y ya tenemos la descomposición factorial de este número porque sabemos que 3, 43 y 127 son
primos. Luego la solución es 1+2+7=10 (D)

5.3.23
Sean a  x 2 , b  y 2 para ciertos enteros x, y , que vamos a suponer positivos.
Todo número primo se entiende positivo, luego a  b  0  x 2  y 2  0  x 2  y 2  x  y
a  x2 
2
 a  b  x 2  y 2  ( x  y )( x  y )  p
b y  
x  y siempre será menor que x  y , luego la única opción aceptable es
x  y  p

x  y 1 
Restando las dos ecuaciones tenemos:
2y  p 1  p  2y 1
Vamos a estudiar las cinco candidaturas que presenta el enunciado:
(A) b  10000  y 2  y  100  p  2  100  1  201 no es primo.
(B) b  256  y 2  y  16  p  2  16  1  33 no es primo.
(C) b  144  y 2  y  12  p  2  12  1  25 no es primo.
(D) b  100  y 2  y  10  p  2  10  1  21 no es primo.
(E) b  900  y 2  y  30  p  2  30  1  61 sí es primo.
Luego la única opción aceptable es (E).

5.3.24
Sea n  a1 a2 a3 ...a10
Si a1  a2  ...  a10  15  3  5 , la única posibilidad es que 8 de estos dígitos sean “1” y los otros
dos sean “3” y “5”, y su suma será 1  1  1  1  1  1  1  1  1  1  3  5  16 (D)

5.3.25
  2

2 24  1  2 212  1  212  12  212  1 212  1   
 2  12  1  2 
26 12 6 2
 1 2
2 12
 1  2  12
6 6
 
 1 212  1
 2  12  12  12
3 3 6 12
 1
23  1  7
2 3  1  9  32
Los otros dos factores los descomponemos aplicando la identidad de la suma de cubos:
     
26  1  22  13  22  1 22  22 1  12  5  16  4  1  5 13
3 2

212 3
2 
 1  2   1  2  1
4 3 4 4 2
 2 1  1   17  256  16  1  17  241
4 4
Sabemos que 241 es un número primo, o lo comprobamos directamente por tanteo.
Así pues, 2 24  1  7  32  5  13  17  241

Ahora vamos encontrando los factores de dos dígitos de este número uno a uno:
7  3 , 7  32 , 7  5 , 7 13 , 3  5 , 7  3  5 , 3 13 , 3 17 , 32  5 , 5 13 , 13 , 17
un total de 12 factores (D)

5.3.26
Factorizamos parcialmente 332  1 aplicando sucesivamente la identidad “diferencia de
cuadrados”:
    
332  1  3216  12  316  12  316  1 316  1 
2

 3  1 3  1  3   1 3  1 


2 8 2 16 8 2 2 16

 3  13  13  1  3  1 3  13  1 


8 8 16 2 4 2 8 16

 3   1 3  13  13  1  3  13  13


4 2 2 4 8 16 2 2 4
  
 1 38  1 316  1 
 8  10  17  82  3  1  8  10  17  2  41  3  1
16 16

Estudiando los factores que hemos encontrado en esta factorización parcial podemos encontrar
uno de los divisores que buscamos, uno es
8 10  80 , puesto que 75  80  85 .
Y el otro es 41  2  82 , puesto que 75  82  85 .
La solución es, por tanto, 82  80  6560 (B)

6.1
Basta descomponer en factores:
n5  5n3  4n  n(n4  5n2  4)  n(n2  4)(n2  1)  (n  2)(n  1)n(n  1)(n  2) ,
y por tanto es el producto de 5 enteros consecutivos.
En cinco enteros consecutivos siempre encontraremos al menos un múltiplo de 2, un múltiplo
de 3, un múltiplo de 4 (diferente al primero) y un múltiplo de 5, por lo que su producto será
divisible entre 2  3  4  5  120 .

6.2
Sea k  n 2  3n  5 . Supongamos que 121  112 | k  11 | k .
Observamos que k  (n  7)(n  4)  33 , y que 11 | 33 , luego 11 | (n  7)(n  4) y por tanto
11 | (n  7) o 11 | (n  4) , por ser 11 primo.
Pero 11  n  7  (n  4) , luego si 11 divide a uno, también dividirá al otro, y por tanto
121 | (n  7)( n  4) , pero 121 | 33 , y por tanto 121 | (n  7)(n  4)  33 , llegando a
contradicción.

6.3
34  34 4  34
5 4
  4 4

45  4  45 1  4  22
6 6
 
 4  225 1  4  245 1/ 2  4  25 1/ 2
56 1 6 6
 6

4

En donde hemos aprovechado que 5 6 es impar, luego 56  1 es par, y por tanto 56  1 / 2 es  


entero.
Así pues, hemos escrito 34  45 de la forma m 4  4n 4 y hacemos la siguiente descomposición:
5 6

 
2

m4  4n4  m2  2n2  4m2n2  m2  2m n  2n2 m2  2m n  2n2 
El factor más pequeño es el de la izquierda, que cumple:
m2  2m n  2n2  (m  n) 2  n2  n2  25 1  28008  24
6
 
2002
 10 2002

6.4
Buscamos una combinación lineal  11a  2b   18a  5b que sea múltiple de 19.
11  18   19
Probamos con la combinación  11a  2b    18 a  5b   19 a  19b  
2  5  19
Y vemos que tiene solución entera:   13 , b  9
Luego  1311a  2b  918a  5b  19a  b que es divisible entre 19.
Luego 19 | 1311a  2b  19 | 918a  5b
Pero (19 ,13)  1 , y por tanto 19 | 1311a  2b  19 | 11a  2b
Y de la misma forma (19 ,9)  1 , y por tanto 19 | 918a  5b  19 | 18a  5b

6.5

Queremos ver que n2  n  1 , n2  2n  1 
n2  n  1  (1)(n2  2n)  n  1  (n  1)
a | n 2  n  1

Luego   a | (n  1)  a | n  1
a | n  2n 
2

a | n  n  1  n(n  1)  1
2
Pero   a | 1  a  1
a | n 1 
y por tanto no hay divisores comunes que no sean la unidad

6.6
Queremos determinar los enteros x para los cuales 2 x  1 | x3  3x  2

1 1 11  27
Realizando la división entera tenemos que x 3  3x  2   x 2  x   
2 4 8 8

   
Luego 8 x3  3x  2  4 x 2  2 x  11 2 x  1  27

Luego
   
2 x  1 | x 3  3 x  2  2 x  1 | 8 x 3  3x  2  4 x 2  2 x  11 2 x  1  27 
2 x  1  1  x  0
2 x  1  3  x  1, x  2

 2 x  1 | 27  3  
3

2 x  1  9  x  4, x  5
2 x  1  27  x  13, x  14

6.7
Supongamos que 16 p  1  n3 para cierto entero positivo n .
En primer lugar vemos que n debe ser impar pues lo es 16 p  1 .
16 p  1  n3  16 p  n3  1  n  1 n2  n  1 
Escribir 16 p como producto de un par a multiplicado por un impar b  1 solo es posible como
a  16 y b  p .

O si lo queremos ver de otra manera:


Puesto que n es impar, n  1 es par y n 2  n  1 es impar, luego no puede tener ningún factor
2, y por tanto n  1  16 k .
Luego
16 p  16 k n 2  n  1  p  k n 2  n  1 
k  1 , p  n 2  n  1

 n  0 absurdo
k  p , 1  n  n  1  n(n  1)  1  n  1 absurdo
2

 

En todo caso k  1 , p  n2  n  1  n  1  16  n  17 .
17 3  1 4913  1 4912
y p    307
16 16 16

6.8
Primera versión.
En este problema utilizaremos un resultado importante:
Si ab es un cuadrado perfecto, y a y b son coprimos, entonces a y b son cuadrados
perfectos.

Buscamos los k  0 para los cuales k 2  pk  k (k  p) sea un cuadrado perfecto.


a) Si k y p son coprimos, podemos aplicar el principio anterior, y deducir que
k y k  p deben ser cuadrados perfectos:
k  q2
r 2  k  p  q 2  p  p  q 2  r 2  (q  r )(q  r )

Y puesto que p es primo,


q  r 1 
    
p 1
 
 p  12
 2 q p 1 q k
q  r  p 2 4

b) Si k y p no son coprimos, es decir, p | k  k  ap


k 2  pk  a 2 p 2  pap  ap2 (a  1)

Puesto que a y a  1 son consecutivos, son coprimos, y como su producto es un cuadrado


perfecto,podemos aplicar el lema anterior para deducir que a y a  1 son cuadrados perfectos.
Pero los únicos cuadrados consecutivos son 0 y 1, luego su producto es 0, y este valor está
descartado por el enunciado.
Fuente de la versión: Number Theory: Concepts and problems (Andreescu, Dospinescu, 2017) pág. 15
Segunda versión.
p p 2  4n 2
k 2  pk  n 2  k 2  pk  n 2  0  k 
2
Luego vemos que p  4n debe ser un cuadrado perfecto.
2 2

p 2  4n2  a 2  p 2  a 2  4n2  (a  2n)(a  2n)


Y puesto que p es primo y a  2n  a  2n , solo se pueden dar dos casos:
Primer caso:
a  2n  p 2  p2  1
  2 a  p 2
 1  a 
a  2n  1  2
p 2  2 p  1  p  1
2
Tomando la suma: k  
4 4
p  ( p  1) / 2 2 p  ( p 2  1) 2 p  p 2  1  ( p  1) 2
2
Tomando la resta: k    
2 4 4 4
Segundo caso:
a  2n  p 
  2a  2 p  a  p
a  2n  p 
 2p
p p k  p
p  4n  a  p  k 
2 2
 2
2 
k  0
El caso k  0 queda descartado por el enunciado.

Fuente de la versión: Soluciones oficiales (SE pág. 762)

6.9
24  23  3 , luego todo múltiple de 24 es de la forma 23  3  a , donde a tiene que contener al
menos un factor 2 para completar el 2 3 a exponente par y un factor 3 para completar el 3 a
exponente par y finalmente un cuadrado perfecto. Los números buscados son de la forma
n  2 4  32  b  144 b con b un cuadrado perfecto, es decir, son números de la forma

n  223c 
2
con c 2  b

 2
 2
n  223c  10 6  103  223c  103  12c  1000
Puesto que 1000 / 12   83 , hay 83 números que cumplan la condición del enunciado.

6.10
Haciendo una división sintética de polinomios obtenemos
x2 36
f ( x)   x6
x6 x6

36
Que será entero si y solo si es entero, es decir, x  6 | 36  22  32
x6
x  6  1  x  5, x  6  2  x  4, x  6  4  x  2, x  6  3  x  3
x  6  9  x  3, x  6  6  x  0, x  6  12  x  6, x  6  18  x  12
x  6  36  x  30

Es fácil comprobar que, efectivamente, todos estos valores generan imágenes enteras en la
función.

6.11
a  a  a  b  b  b  b  10a  b  3a  4b  10a  b  3b  7a
Luego vemos que 7 divide a b, y puesto que b  10 , la única posibilidad es b  7 , y por tanto
3  7  7a  a  3  a  b  10 (C)

6.12
Puesto que se empaquetan igual número de manzanas, el número de cajas ha de ser un divisor
de 60.
1 manzana  60 cajas es imposible, pues tocaría a una pera por caja, y necesitamos cantidades
diferentes de peras.
2 manzanas  30 cajas es imposible, pues tendríamos, como mínimo:
30
30  31
1  2  3  ...  30   k   15  31  60 peras.
k 1 2
3 manzanas  20 cajas es imposible, pues tendríamos, como mínimo:
20
20  21
1  2  3  ...  20   k   10  21  210  60 peras.
k 1 2
4 manzanas  15 cajas es imposible, pues tendríamos, como mínimo:
15
15  16
1  2  3  ...  15   k   15  8  120  60 peras.
k 1 2
6 manzanas  10 cajas sí es posible:
10
10  11
1  2  3  ...  10   k   55  60 peras.
k 1 2
Podemos hacer la distribución 1  2  3  ...  8  9  15  60 (D)

6.13
Factorizamos el producto:
10  20  30  ...  90  216  34  510  71

Y necesitamos una factorización cuyos exponentes sean todos pares.


Está claro que debemos eliminar el 7, luego tachar el 70, con lo que nos queda:
215  34  59
Si tachamos además el 10 nos queda la factorización
214  34  58
Cuyos exponentes son todos pares, es decir, es un cuadrado perfecto.
Luego necesitamos eliminar 2 (B)

6.14
213  23  210  8  210 , luego serán 8: 1 210 , 2  210 , 3  210 , 4  210 , 5  210 , 6  210 , 7  210 , 8  210
(D)
6.15
7!8!9! 7!1  8  8  9   7!81  7  6  5  4  3  2  34  7  2  3  5  4  3  2  34 
 36 7  2  5  4  2  ( D)

6.16
n 6 6
 1 es un entero si y solo si es un entero, es decir n  6 es un divisor de 6.
n6 n6 n6
Luego

n  6 1 n  7
n6 2 n 8
n6 3 n 9
n  6  6  n  12

Para n  8 el divisor más grande es 4, y no 2. Luego el número de casos posibles es 3 (C)

6.17
b  1  21  2 2  23  ...  29  210  21 23... 910  255
a  1  21  2 2  23  ...  29  210 es la suma de una sucesión geométrica de 10 términos y razón
2, luego
a  1  21  2 2  23  ...  29  210  1  21  21  2 2  ...  28  29  
 
a  1  2 a  210  2a  211  211  1  2a  2  a
Finalmente,
5
 
a  211  1  a  1  211  a  1  211  255  b (B)
5

6.18
Denotando los diez números con a1 , a2 , a3 ,..., a10 , y teniendo en cuenta que
a1  a2  a3  a4  a5  a6  a7  a8  a9  a10  1  2  3  4  5  6  7  8  9  10  55
tenemos
S  a1  a2  a3  a4 

S  a4  a5  a6  a7   3S  a1  a2  a3  2a4  a5  a6  2a7  a8  a9  a10 
S  a7  a8  a9  a10 
 a1  a2  a3  a4  a5  a6  a7  a8  a9  a10  a4  a7  55  a4  a7

Luego 55  a4  a7 es un múltiplo de 3 mayor que 55. El primero es 57  19  3 , pero entonces


a4  a7  2 lo cual es imposible.
El siguiente es 60  20  3 , para el cual S  20 . (C)

Nota: Queda demostrar que existe al menos una configuración con S  20 . En las soluciones
oficiales Kangourou encontramos la siguiente:
6.19
Factorizamos: n 2  2n  3  (n  1)(n  3)  n 2  2n  3  n  1 n  3

Será primo si y solo si uno de estos dos factores es 1:


n 1  1  n  0,  2
n  3 1  n  4,2

Hay cuatro soluciones posibles. (D)

6.20
Sean H y M el número de chicos y de chicas, respectivamente, en el curso pasado.
1 1 1 1 H M
H  H  M  M  H  M 1  H  M  1  1 H  M  5
5 5 5 5 5
1 1
Por otro lado, implícitamente H y M son enteros, luego H y M son múltiples de 5, y por
5 5
tanto H  M es un múltiple de 5.
Con esto descartamos A, B, y E.
Veamos B:
H  M  25 
  2 H  30  H  15  M  10
H M 5 

Veamos D:
H  M  30  35
  2 H  35  H  imposible porque no es entero.
H M 5  2
La única solución válida es B.

6.21
b 
a   11 
c  b a ab  1 c 1
  25  11  14  a   b   a  b   (a  b)1    (a  b)
a
b   14 
c c c  c c
c 

Y por tanto 25c  (a  b)(c  1)  c  1 | 25  52
a
c  1  25  c  24  b  24  b   14 imposible. Además, el sistema
c
b 
a  11
24 
 no daría soluciones enteras.
a
b  14 
24 
c  1  1  c  0 imposible.
Solo nos queda la posibilidad
a  b 20
c  1  5  c  4  25  4  (a  b)  5  a  b  5  4  20    5 (E)
c 4

6.22
N  10000 a  2020 para cierto a | N
Luego N  ak para cierto entero k y por tanto
ak  10000 a  2020  ak  10000 k  2020  2 2  5 101
Los posibles valores de a son:
1 1
2 2
5 5
101 2
4 4
10 1
202 4
505 10
20 2
404 8
1010 2
2020 4
Total 45

Por otro lado, los 12 números de S acaban todos en 2020, y por lo tanto tenemos que sumar
12  (2  0  2  0)  12  4  48

Finalmente, en total, tenemos 45  48  93 .

6.23
La clave para resolver este problema es ver que la condición nn | mm implica que si p es un
factor primo de n , también lo será de m .
Por otro lado, la condición n | m implica n  1 .

Otra clave importante es ver que la condición ( m  n , 210 )  1 , con 210  2  3  5  7 implica
que si un primo p de un dígito (2,3,5 o 7) divide n, entonces también dividirá a m, y por tanto
dividirá m  n , con lo que ( m  n , 210 )  1 .
Así pues, los factores primos de n serán 11, 13, 17…

Si estos factores primos de n estuvieran elevados a 0 o 1 como máximo, por ser también
factores de m , entonces m tendría todos los factores de n elevados a sus potencias respectivas,
es decir, n | m , contradiciendo la tercera hipótesis del enunciado.

Así pues, la factorización de n debe contener algún exponente mayor o igual que 2.

El menor candidato posible de n es n  112  121 , y por tanto 112   121


 11242 .
Puesto que, por hipótesis, 11242 | mm , necesitamos un m que sea múltiplo de 11, mayor o igual
que 242, que no sea divisible por 121 y que ( m  n , 210 )  1 .

m  242 no cumple pues es divisible entre 121.


m  253 no cumple pues m  n  253  121  374 es divisible entre 2.
m  264 no cumple pues m  n  264  121  385 es divisible entre 5.
m  275 no cumple pues m  n  275  121  396 es divisible entre 2.

Finalmente, m  286 cumple las condiciones del enunciado:


m  n  286  121  407  11  37 , y por tanto ( m  n , 210 )  1
m  2 11 13 , n | m .

Luego la solución es 407.

Fuente: [Link]

6.24
Todo número que cumpla las condiciones del enunciado se podrá escribir como ABCD con
D 0, B, C  0,2,4,6,8, A 2,4,6,8
En total hay 4  5  5 1  100 (B)

6.25
(n  2)!(n  1)! (n  1)!(n  2  1) (n  1)!(n  1)
   (n  1)(n  1)  (n  1) 2
n! n! n!

6.26
Queremos determinar los enteros positivos n cumpliendo  5!, n   5  10!, n  con 5 | n
  
 5!, n   5  10!, n   5  3  23 , n  5 7  52  34  28 , n 
Aplicaremos AR/4.30:
Sea n  2a2 3a3 5a5 7 a7 p8a8 p9a9 ... pkak la descomposición factorial de n, con 7  p8 , p9 ,..., pk ,
cumpliendo a2 , a3 , a7  0 y a5  1 pues 5 | n .

 
5 7  52  34  28 , n  2b2 3b3 5b5 17b7 con b2  min( a2 ,8) , b3  min( a3 ,4) , b5  min( a5 ,2) y
b7  min( a7 ,1)

Por otro lado,


 
5  3  23 , n  2c2 3c3 5c5 7c7 p8a8 p9a9 ... pkak con c2  max( a2 ,3) , c3  max( a3 ,1) , c5  max( a5 ,1) ,
c7  max( a7 ,0)
   
Y de la igualdad 5  3  23 , n  5 7  52  34  28 , n deducimos que a8  a9  ...  ak  0
Y también se deduce que:
min( a2 ,8)  max( a2 ,3)  3  a2  8
min( a3 ,4)  max( a3 ,1)  1  a3  4
min( a5 ,2)  1  max( a5 ,1)  a5  3
min( a7 ,1)  max( a7 ,0)  0  a7  1
En total hay 6  4  1  2  48 ( D )

6.27
60  log 10 x  2 log 10 Mcd ( x, y )   log 10 x  Mcd ( x, y ) 2 

570  log 10 y  2 log 10 mcm ( x, y )   log 10 y  mcm ( x, y ) 2 
Sumando las dos igualdades llegamos a
630  log 10 x  Mcd ( x, y ) 2   log 10  y  mcm ( x, y ) 2  

 log 10 x  Mcd ( x, y ) 2 y  mcm ( x, y ) 2  
x  Mcd ( x, y ) y  mcm ( x, y )  10
2 2 630

Utilizando ahora la identidad


Mcd ( x, y )  mcm ( x, y )  x  y

Llegamos finalmente a
x  y  x 2  y 2  x3  y 3  10 630  xy  10 210  2
 2
 ...
 2  5
 5
 ...
 5  m  n  420

210 210

Sean x  2 a 5b , y  2c5d .
Entonces Mcd ( x, y)  2min(a, c )5min(b, d ) , mcm ( x, y)  2max( a , c )5max( b, d ) , y por tanto

 
60  log 10 x  Mcd ( x, y ) 2  x  Mcd ( x, y ) 2  10 60  260560 
x2 2 min( a , c ) 2 min( b , d )
5 2 5 
60 60

2 a5b  2 2 min( a , c )52 min(b , d )  260560 


a  2 min( a, c)  60
2 a  2 min( a , c )5b  2 min(b , d )  260560  
b  2 min( b, d )  60
 
570  log 10 y  mcm ( x, y ) 2  y  mcm ( x, y ) 2  10 570 
c d
25 2 max( a , c ) max( b , d )
5 2 5570 570

c  2 max( a, c)  570
2c  2 max( a , c )5d  2 max( b, d )  25705570  
d  2 max( b, d )  570
Supongamos a  c , b  d
a  2 min( a, c)  60 a  2a  60 3a  60  a  20
b  2 min( b, d )  60 b  2b  60 3b  60  b  20
  
  
c  2 max( a, c)  570 c  2c  570 3c  570  c  190
d  2 max( b, d )  570 d  2d  570 3d  570  d  190
Y por tanto x  2 20520 , y  21905190 , Mcd ( x, y )  x , mcm ( x, y )  y ,
En efecto:
  2
x  Mcd ( x, y)2  10 20  10 20  10 20  10 40  10 60
10 
y  mcm( x, y)2  10190 190  10 28510380  10570
2

Cualquier otra posibilidad nos lleva a una incompatibilidad.


Supongamos a  c , b  d
a  2 min( a, c)  60 a  2a  60 a  20
b  2 min( b, d )  60 b  2d  60 b  360
  
   absurdo.
c  2 max( a, c)  570 c  2c  570 c  190
d  2 max( b, d )  570 d  2b  570 d  150

Supongamos a  c , b  d
a  2 min( a, c)  60 a  2c  60 a  360
b  2 min( b, d )  60 b  2b  60 b  20
  
   absurdo.
c  2 max( a, c)  570 c  2a  570 c  150
d  2 max( b, d )  570 d  2d  570 d  190

Supongamos a  c , b  d
a  2 min( a, c)  60 a  a  60
b  2 min( b, d )  60 b  2b  60
 
  absurdo.
c  2 max( a, c)  570 c  a  570  a  a  570
d  2 max( b, d )  570 d  2d  570

La única solución posible es x  2 20520  m  40 , y  21905190  n  380 , y por tanto


3m  2n  3  40  2  380  880

6.28
Sea n un número que cumpla las condiciones del enunciado.
20 | n , luego n  20 k
n  2019 , luego k  100
20   n   20 k    225k  
Aplicaremos detenidamente la fórmula 19.4b.

El exponente 2 del factor 2 genera un 3 en la función  , luego será imposible obtener 20. Hay
que aumentar este exponente.
  
20   225  2k   235  k 
Ahora tenemos un 4  2  8 , que nos impide obtener el 20.

Si aumentamos el exponente del 5 tampoco nos sirve, pues entonces


20   2352  k 
Obtenemos un 4  3  12 … y no son divisores de 20.

Así pues, el factor 2 tiene que tener exponente 4, luego:


  
20   22  5  22 k   24  5  k 
Ahora tenemos un factor 5  2  10 y solo tenemos que multiplicarlo por 2, lo que sucede si y
solo si añadimos un primo p diferente de 2 y de 5. Luego serán todos los números de la forma
n  24  5  p  20  4  p , con 4  p  100  p  25 , es decir: p  3,7, 11, 13, 17, 19, 23 .

Su suma será 24  5  3  24  5 11  ...  24  5  23  24  53  7  11  13  17  19  23  24  5  93

 
Otra combinación posible es 20   24  53   2000  , luego la suma total será
2 4  5  93  2 4  53

2 4  5  93  2 4  53
Y el resultado pedido es  2 2  93  2 2  52  372  100  472
20

6.29
Claramente (E), pues 27 no es primo y 27  2  25 tampoco lo es.

6.30
Sea x la cantidad de dinero de Casper, en céntimos.
Luego x debe ser múltiple de 12, 14 y 15, y por tanto el valor mínimo es
x  mcm (12,14,15)  420 cent.
420
Para esta cantidad, puede comprar  21 caramelos púrpura. (B).
20

6.31
100000  2 5  5 5 , luego S está formado por todos los números de la forma 2 a  5 b con
0  a, b  5 , y por tanto los productos de dos números de S serán todos aquellos números de la
forma 2 c  5 d con 0  c, d  10 .
En total hay 1111  121 casos.

Pero debemos observar que algunos casos solo se pueden aparecer como el producto de dos
elementos iguales (recordemos que, por separado, el exponente máximo es 5).
 
1  20  50  20  50 20  50 
1  210  25  25
1  510  55  55
 
1  210  510  25  55 25  55 
Luego la respuesta correcta es 121  4  117 (C).

6.32
n
2 2n
Si n es positivo, 4000     2 5  53 n que será entero para n  0,1,2,3 .
5 5
5 n
n
2
Si n es negativo, 4000     2 5  53  n que será entero para n  1,2,3,4,5 .
5 2
En total hay 9 casos (E)
6.33
Está claro que 1 no puede ser mínimo de S, pues sería divisor de cualquier otro elemento
posible, llegando a contradicción.
El 2 tampoco puede ser mínimo, pues es incompatible con 4, 6, 8, 10 y 12, y con el resto
tenemos un único posible conjunto S  3,5,7,9,11 que no es aceptable porque 3 y 9 no son
compatibles.

El 3 tampoco puede ser mínimo: Los otros candidatos son: 4, 5, 7, 8, 10, 11, y en este caso el 5
es incompatible con el 10, y el 4 es incompatible con el 8 luego eliminando alguno de ellos nos
quedan solo 4 candidatos posibles, y no llegamos a los 6 necesarios.

Con el 4 de mínimo tenemos candidatos posibles 5, 6, 7, 9, 10, 11. El 5 es incompatible con el


10, y con el resto podemos hacer un posible conjunto S  4,6,7,9,10,11. Así pues, el valor
mínimo es 4 (C).

6.34
Aplicando la suma de la sucesión geométrica:
10 n  1
An  aa a  a 11
...  
...1  a 10 0  101  ...10 n  a  9
n n

10 n  1
Bn  bb b  b
...
n 9

Cn  c c  c  11
 10 2 n  1 

...1  c   
 
10 n  1 10 n  1 
c...
    c
2n 2n  9  9

Cn  Bn  A 2
 c
10 n
 
 1 10 n  1
b
10 n  1  10 n  1 
  a 
2
2  10  1 
  a  
n

2

9 
n
9 9   9 
10 n  1
puesto que n  0 podemos cancelar el factor para llegar a
9
 10 n  1 
c  10 n  1  b  a 2   
 9 

Esta ecuación es lineal en 10 n , luego si es cierta para dos valores de n concretos será cierta
para cualquier n .

 10  1 
Para n  1 c  10  1  b  a 2     11c  b  a
2

 9 
 100  1 
Para n  2 c  100  1  b  a 2     101c  b  11a
2

 9 
11c  b  a 2


2
 101c  b  11c  b  11a 2  a 2  90 c  10 a 2  9c  a 2
101c  b  11a  

11c  b  a 2  110 c  10b  10 a 2 


2
   110 c  10b  101c  b  10 a  11a
2 2

101c  b  11a  101c  b  11a 2



2a 2
 9c  9b  a  a  9b  a  9b  2a  b 
2 2 2 2

9
2a 2 a 2 a2
Queremos encontrar el valor máximo de a  b  c  a   a
9 9 3
Equivale a encontrar el valor máximo de a .

La ecuación 9c  a 2 tiene tres soluciones ( a, c ) : (3,1) , (6,4) , (9,9) .

Pero sustituyendo para encontrar b : 11c  b  a 2  b  11c  a 2

(3,1)  b  11  9  2 , (6,4)  b  44  36  8 , (9,9)  b  99  81  18

Vemos que la solución (9,9) no es aceptable pues incumple la condición b  9 .

Observamos que se cumple siempre para a  6, b  8, c  4 . Lo podemos ver con un ejemplo:


n  4  62 1111  39996  40000  4  40000  4  8  4 10 4  1  8  
Si observamos la forma de los números que se obtienen de la forma a 211
 ...1 :
n
1 – 1111 2 – 4444 3 – 9999 4 – 17776 5 – 27775
6 – 39996 7 – 54439 8 – 71104 9 – 89991

Vemos que el siguiente candidato para obtener un número “parecido” a 10 n sería


a  9  92  11 ...11  89 ...91

Pero vemos que no es aceptable:


n  4  92 1111  89991  90000  9  90000  9  18  9 10 4  1  18 
Pero b no puede ser 18.

Luego la solución es 6  8  4  18 (D).

Fuente: [Link]

6.35
Primera versión.
Sabemos que 2 | n y que 323 | n , y puesto que 2 y 323 son coprimos, se deduce que 646 | n , es
decir, n  646 k . Puesto que 1000  n  10000 , está claro que 2  k  15 .
Por otro lado, vemos que 323  17 19 , luego la lista de todos los números n posibles es:
2 17 19  2 , 2 17 19  3 , 2  17  19  2 2 , 2 17 19  5 , 2  17  19  2  3
2 17 19  7 , 2 17 19  23 , 2 17 19  32 , 2 17 19  2  5 , 2 17 19 11
2  17  19  3  2 2 , 2 17 19 13 , 2 17 19  2  7 , 2 17 19  3  5

Tomamos las posibles respuestas que nos ofrece el enunciado y vamos descartando, de menor a
mayor:
324  22  34 y no aparece como divisor de ninguno de los números anteriores.
330  2  3  5 11 y tampoco es aceptable.
340  22  5  17 y este valor sí que aparece como divisor de 2 17 19  2  5
luego 340 es la respuesta correcta (C).
Segunda versión.
Sabemos que 323  17 19 | n , luego el siguiente divisor que buscamos debe ser múltiple de 17
o de 19, porque en caso contrario obtendríamos un número demasiado grande.
Por ejemplo:
Supongamos que 324  22  34 | n . Entonces:
324  22  34 | n 
  2  3 17 19 | n  104652 | n  n  104652 absurdo.
2 4

323  17 19 | n 

El primer valor de la lista que cumple esta condición es la opción C: 340  22  5 17 , y con este
número podemos obtener un valor de n aceptable: 2 17 19  2  5  6460 , así pues, la respuesta
es (C).

6.36
Primera versión.
   35 q  63 p  36 q  0  63 p  35 q  36 q  35 q  0  79 p  5q   q
5 p 4
9 q 7

De aquí se deduce que q  7 .


Para los primeros valores de q tenemos que
1  5q
1  9 p  5q  p  , que no es entero para todos los valores q  14 .
9
2  5q
Para q  14 , también podráimos tener 9 p  5q  2 , y entonces: p  .
9
Seguimos probando: El valor q  15 no es aceptable, pero con q  16 tenemos:
1  5 16
p  9 , resultado entero. Por lo tanto la solución es 16  9  7 (A).
9

Segunda versión.
5 p 4 5 4
   q p q
9 q 7 9 7
5 4
Representando las respectivas gráficas f (q)  q , g (q)  q
9 7
Se observa que el primer punto de componentes enteras que está entre las dos gráficas es
(16,9), y por tanto la solución es 16  9  7 (A).

Fuente de estas soluciones: [Link]

6.37
Supongamos que x 2  a x  b  0 tiene dos raíces reales, p'  3 p y q '  3q , para ciertos p y q
enteros. Entonces, aplicando las fórmulas de Vieta:

a  ( p' q' )  3( p  q )


b  p' q'  3 p3q  9 pq

Y por tanto está claro que a y b también son enteros, y que a es múltiple de 3 y que b es
múltiple de 9: a  3 , b  9 

Por otro lado, puesto que x 2  bx  9a  0 tiene una única solución real, su discriminante será
cero:
0    b2  4 1 (9a)  b2  36 a  b2  36 a

Luego
9 2  363   81 2  36  3  3 2  4  22
De aquí deducimos que  es múltiple de 3, y que  es múltiple de 2. Así pues, a es múltiple
de 9 y b es múltiple de 18.

 b 2  18 k   18 2 k 2
2
Luego b  18k para cierto k entero, y a     9k 2
36 36 36

con lo que la primera ecuación del enunciado queda


x 2  9k 2 x  18k  0

Con dos soluciones reales diferentes.

Primer caso: k  0 .
La ecuación se reduce a x 2  0 , con una única solución doble x  0 , y por tanto no podemos
considerar que se cumplen las condiciones del enunciado.

Segundo caso: k  1 .
La ecuación x 2  9 x  18  0 tiene soluciones x  3 y x  6 , que satisfacen las condiciones del
enunciado, luego a  9 y b  18 es una solución aceptable para nuestro problema.

Tercer caso: k  1 .
Para que la ecuación x 2  9k 2 x  18k  0 tenga dos soluciones reales diferentes, su
discriminate debe ser positivo:
 2

0     9k 2  4  1  18k  81k 4  72 k  32 9k 4  8k 
 
Así pues, 32 9k 4  8k es un cuadrado perfecto, y por tanto 9k 4  8k es un cuadrado perfecto.

Ahora bien, puesto que k  1 , vemos que se cumplen las desigualdades


9k 4  8k  9k 4  3k 2   2

3k 1
2 2
 
 9k 4  6k 21  9k 4  6k 21  9k 4  8k , puesto que k  1  6k 21  8k

Así pues, tenemos


3k 1  9k
2 2 4
 
 8k  3k 2
2

Es decir, 9k 4  8k se encuentra estrictamente entre dos cuadrados perfectos consecutivos, y por


tanto no puede ser un cuadrado perfecto.

Cuarto caso: k  1 .
La ecuación x 2  9 x  18  0 no tiene soluciones enteras, luego no se cumplen las condiciones
del enunciado.

Quinto caso: k  1.


El valor 9k 4  8k estudiado en el tercer caso, que debe ser un cuadrado perfecto, tomando
ahora h  k , con h  1, se convierte en
9 h 4  8h

que no puede ser un cuadrado perfecto puesto que (siguiendo razonamientos similares al tercer
caso) tenemos las desigualdades
3h 
2 2

 9h4  9h4  8h  3h2  1 
2

es decir, se encuentra entre dos cuadrados perfectos consecutivos.

Así pues,la única solución aceptable es a  9 y b  18 .


Fuente de esta solución: Germán García López en Facebook.
6.38
n  a  1000  b  100  c  100  d  x 2

Primer caso.
Vamos a suponer que las cifras iguales están en la segunda y tercera posición, como en el
ejemplo del enunciado. Entonces:

m  y 2  (a  1) 1000  b 100  c 10  d  1  a 1000  1000  b 100  c 10  d  1 


 a 1000  b 100  c 10  d  1000  1  x 2  1001

Luego
7 11 13  1001  y 2  x2   y  x  y  x 
Las posibilidades son:
y  x 1 
  2 y  1002  y  501 y no es aceptable porque y  9999 .
2

y  x  7 11 13  1001 


yx7   y  75  m  y 2  5625

  2 y  11 13  7  150  
y  x  11 13 
 x  75  7  68  n  x  4624
2

y  x  11   y  51  m  y 2  2601

  2 y  102  
y  x  7  13  91 
 x  51  11  40  n  x  1600
2

y  x  13   y  45  m  y 2  2025

  2 y  90  
y  x  7 11  77  
 x  45  13  32  n  x  1024
2

y  x  7 11  77   y  45  m  y 2  2025
  2 y  90   no es aceptable.
y  x  13   x  13  45  0
y  x  7  13  91  y  51  m  y 2  2601
  2 y  102   no es aceptable.
y  x  11   x  11  51  0
y  x  11 13  143   y  75  m  y 2  5625
  2 y  150   no es aceptable.
yx7   x  7  75  0
y  x  7  11  13  1001 
  2 y  1002  y  501 no es aceptable.
y  x 1 

Segundo caso.
Vamos a suponer que las cifras iguales están en la primera y tercera posición. Entonces:
m  y 2  a 1000  (b  1) 100  c 10  d  1  a 1000  b 100  100  c 10  d  1 
 a 1000  b 100  c 10  d  100  1  x 2  101
Luego
101  y 2  x2   y  x  y  x 
Puesto que 101 es primo, solo hay dos posibilidades:
y  x  101  y  51  m  y 2  2601
  2 y  102   no es aceptable.
y  x 1   x  1  51  0
y  x 1  
 y  51  m  y  2601
2

  2 y  102  
y  x  101 
 x  101  51  50  n  x  2500
2

Tercer caso.
Vamos a suponer que las cifras iguales están en la primera y segunda posición. Entonces:
m  y 2  a 1000  b 100  (c  1) 10  d  1  a 1000  b 100  c 10  10  d  1 
 a 1000  b 100  c 100  d  10  1  x 2  11

Luego
11  y 2  x2   y  x  y  x 
De nuevo, puesto que 101 es primo, solo hay dos posibilidades:
y  x  11
  2 y  12  y  6  m  y  36  1000 no es aceptable.
2

y  x 1 
y  x 1 
  2 y  12  y  6  m  y  36  1000 no es aceptable.
2

y  x  11
Cuarto caso.
Vamos a suponer que las cifras iguales están en la primera y cuarta posición. Entonces:
m  y 2  a 1000  (b  1) 100  (c  1) 10  d  1  a 1000  b 100  100  c 10  10  d 
 a 1000  b 100  c 100  d  100  10  x 2  110

Luego
2  5 11  y 2  x 2   y  x  y  x 
Las posibilidades son:
y  x 1 
  2 y  111  y  111 / 2 no es aceptable.
y  x  2  5 11  110 
yx2 
  2 y  57  y  57 / 2 no es aceptable.
y  x  5 11  55
yx5 
  2 y  27  y  27 / 2 no es aceptable.
y  x  2  11  22 
y  x  11 
  2 y  21  y  21 / 2 no es aceptable.
y  x  2  5  10 
y  x  2  5  10 
  2 y  21  y  21 / 2 no es aceptable.
y  x  11 
y  x  2 11  22 
  2 y  27  y  27 / 2 no es aceptable.
y x5 
y  x  5 11  55
  2 y  57  y  57 / 2 no es aceptable.
yx2 
y  x  2  5 11  110 
  2 y  111  y  111 / 2 no es aceptable.
y  x 1 

No hay ningún caso aceptable.

Quinto caso.
Vamos a suponer que las cifras iguales están en la tercera y cuarta posición. Entonces:
m  y 2  (a  1) 1000  (b  1) 100  c 10  d  a 1000  1000  b 100  100  c 10  d 
 a 1000  b 100  c 100  d  1000  100  x 2  1100
Luego
22  52 11  y 2  x 2   y  x  y  x 

Estudiando las posibles combinaciones, como en los casos anteriores, llegamos a las dos únicas
soluciones:
x  14 
  n  x  196  1000 no es aceptable
2

y  36 
x  50  n  x  2500
2


  que es la única solución aceptable.
y  60  m  y 2
 3600

Sexto caso.
Vamos a suponer que las cifras iguales están en la segunda y cuarta posición. Entonces:
m  y 2  (a  1) 1000  b 100  (c  1) 10  d  a 1000  1000  b 100  c 10  10  d 
 a 1000  b 100  c 100  d  1000  10  x 2  1010

Y estudiando las posibles combinaciones observamos que no hay ninguna aceptable.

Así pues, las soluciones para este problema son las cinco parejas siguientes:
(5625,4624), (2601,1600), (2025, 1024) , (2601, 2500), (2500, 3600)

Nota: Podríamos haber desechado muchos casos directamente teniendo en cuenta que x  y y
x  y tienen la misma paridad, y que 999  x 2  y 2  10000  31  x  y  100 y por tanto
63  x  y  199 .

6.39
Vemos que
8n  3
17 n  9

es irreductible si y solo si 8n  3 y 17 n  9 son coprimos, es decir, su máximo común divisor es


1.
Podemos aplicar el Algoritmo de Euclides (ver 4.17) para encontrar el máximo común divisor:

17 n  9 8n  3
16n  6 2
n 3

Luego  17n  9 , 8n  3    n  3 , 8n  3 
De nuevo

8n  3 n 3
8n  24 8
21

Y por tanto  n  3 , 8n  3    n  3 , 21 

Así pues, buscamos todos los enteros positivos n tales que n - 3 es coprimo con 21 , es decir,
todos los números n tales que n - 3 que no sea divisibles ni entre 3 ni entre 7.
Pero n - 3 es divisible entre 3 si y solo si n es divisible entre 3, y n - 3 es divisible entre 7 si y
solo si n  3  7k  n  7k  3 .
Luego serán todos los enteros positivos n tales que no sean de la forma n  3a ni n  7b  3 .

6.40
6n sí es divisible entre 3,luego 6n  1 no será divisible entre 3, pues siempre generará residuo
2. (D)

6.41
Si es divisible entre 2 y entre 3 debe ser divisible entre 6, luego "6 y 7" queda descartada.
Si es divisible entre 10 debe ser divisible entre 5 luego "4 y 5" queda descartada.
Si es divisible entre 6 debe ser divisible entre 2 y entre 3, luego "2 y 3" queda descartada.
Si es divisible entre 2 y entre 5, lo será también entre 10, luego "10 y 11" queda descartada.
La única opción que queda es "7 y 8". (D)

6.42
Sea a el número de movimientos "sacar 30 canicas" y b el número de movimientos "poner 18
canicas".
El número de canicas que quedarán será 71  30a  18b  71  65a  3b es decir, que
quitaremos o pondremos un número de canicas múltiplo de 6.
Luego el máximo se encontrara para 5  71  6 11  11  5a  3b que se obtiene para
a  4, b  3 . (C)

6.43
Si vemos como evoluciona el número primo p2 , vemos que la pirámide tendrá 9 filas. Ahora
vamos marcando las casillas en las que aparecerá el primo p4 y vemos que hay 24 en total.

(C)

6.44
Sea 10  n  99 de forma que 20  n  5  2 2  n es un cuadrado perfecto. Esto es equivalente a
que 5n sea un cuadrado perfecto, luego n será múltiplo de 5 por un cuadrado perfecto. Son
tres: n  5  4  5  2 2 , n  5  9  5  32 , n  5  16  5  4 2 . (C)

6.45
Un número es divisible entre 18 cuando lo es entre 2 y entre 9.
Tomando todas las cifras del 1 al 9 siempre obtenemos un número divisible entre 9 porque la
suma será 45, que es divisible entre 9.
Será, además, divisible entre 2 cuando acabe en cifra par, es decir, 2, 4, 6, 8, luego son 4 casos
de los 9 posibles, la probabilidad es 4/9 (B)

6.46
Analicemos como debe ser es número:
0 | a b 0 nunca, 1 | a b 1 siempre, 2 | a b 2 siempre, 3 | a b 3 , 4 | a b 4 , 5 | a b 5 siempre,
6| a b 6 , 7| a b 7 , 8| a b 8, 9| a b 9 .

Para que sean consecutivos, al menos se debe cumplir 3 | a b 3 y 7 | a b 7 . Con estos dos datos
ya podemos deducir el número.
3| a b 3  3| a  b  3  3| a  b .
b/a 0 1 2 3 4 5 6 7 8 9
0 X X X
1 X X X
2 X X X
3 X X X X
4 X X X
5 X X X
6 X X X X
7 X X X
8 X X X
9 X X X X

Encontramos muchos candidatos posibles, para que se cumpla 7 | a b 7 , puesto que en la tabla
del 7 el único número acabado en 7 es 7  1  7 , el cociente deberá acabar en 1 y por tanto
7 | a b . Probando probando con los valores de la tabla anterior llegamos al único candidato
aceptable que es a  8 , b  4 , por lo que el número buscado es 841 y la solución es
8  4  1  13 (D)

Nota: Una solución más elegante (y desde luego mucho más rápida para un contexto de una
prueba con tiempo limitado) sería observar que
c | a b c  c | a b 0 , y que 3  5  7  8  840

6.47
Sea n el número de minutos que necesita para recorrer una milla en el primer día. Por
60 60
proporcionalidad directa, el primer día recorrerá millas, el segundo , el tercero
n n5
60 60
y el cuarto . Puesto que se nos dice que las distancias recorridas son números
n  10 n  15
enteros, deducimos que n, n  5, n  10, n  15 son divisores de 60. Los divisores de 60 mayores
de 15 son 20, 30 y el propio 60, luego probando vemos que la única opción aceptable es
n  5, n  5  10 , n  10  15, n  15  20 .
60 60 60 60
La distancia recorrida es     12  6  4  3  25 (C)
5 10 15 20

6.48
 5n 2  1  nk  5n 2  5  nk
5 n 1 n n n
     2
k n  1 n n  1 n  1n  1 n  1
 5n 2  nk  5  n5n  k   5

Las únicas combinaciones posibles con enteros son:


n  5
  25  k  1  k  24
5n  k  1
n  1
  5  k  25  k  20
5n  k  25
Esta segunda solución no es aceptable pues se piden valores positivos, luego la única solución
posible es k  24 (C)

Un segundo razonamiento alternativo sería:


5 n k n2  1 1 5
 2    n   k  5n 
k n 1 5 n n n
5
Puesto que k es entero, y 5n es entero, necesariamente debe ser entero, luego n  1 o n  5
n
Y se llega igualmente al resultado deseado.

6.49
abc  1
(a  1)(b  1)(c  1) divide abc  1 si y solo si es un entero.
(a  1)(b  1)(c  1)
Observamos que
abc  1 abc a b c
    (*)
(a  1)(b  1)(c  1) (a  1)(b  1)(c  1) a  1 b  1 c  1

Y que
a 1 1
2a 1 1 2
a 1 a 1 2 1
b 1 1 3
3b 1  1 
b 1 b 1 3 1 2
c 1 1 4
4c 1 1 
c 1 c 1 4 1 3

Luego
3 4
(*)  2    4
2 3

abc  1
Así pues, solo puede ser 1, 2 o 3.
(a  1)(b  1)(c  1)

Vemos que no puede ser 1, puesto que, expandiendo el denominador:


abc  1 abc  1
1  
(a  1)(b  1)(c  1) abc  1  a  b  c  ab  bc  ac
 a  b  c  ab  bc  ac  0  a  b  c  ab  bc  ac

Lo cual es absurdo, porque 1  a  b  c  ab  bc  ac  a  b  c .

abc  1
Luego solo puede ser igual a 2 o 3.
(a  1)(b  1)(c  1)

Con un razonamiento parecido al primero, vemos que si a  4 , entonces


abc  1 abc a b c
    (**)
(a  1)(b  1)(c  1) (a  1)(b  1)(c  1) a  1 b  1 c  1
1 1 1 a 1 1 4
a  4  a 1  3     1 1 
a 1 4 1 3 a 1 a 1 3 3
1 1 1 b 1 1 5
b  5  b 1  4     1 1 
b 1 5 1 4 b 1 b 1 4 4
1 1 1 c 1 1 6
c  6  c 1  5     1 1 
c 1 6 1 5 c 1 c 1 5 5
Y por tanto:
4 5 6 6
(**)      2
3 4 5 3

abc  1
Luego  2 y ya hemos visto que es imposible.
(a  1)(b  1)(c  1)
Así pues, a  2 o a  3 .

abc  1 2bc  1
Caso 1: a  2  
(a  1)(b  1)(c  1) (b  1)(c  1)

Caso 1.1
2bc  1
2  2(b  1)(c  1)  2bc  1  2(bc  b  c  1)  2bc  1
(b  1)(c  1)
 2b  2c  3  0  3  2(b  c)
Lo cual es imposible, pues la parte izquierda es impar y la parte de la derecha, par.

Caso 1.2
2bc  1
3  3(b  1)(c  1)  2bc  1  3(bc  b  c  1)  2bc  1
(b  1)(c  1)
 3bc  3b  3c  3  2bc  1  bc  3b  3c  4  0
 5  bc  3b  3c  9  (b  3)(c  3)
Las únicas posibilidades son:
b  3  5 b  8
  que no cumple la condición c  b
c  3  1 c  4
b  3  1 b  4
  que sí cumple todas las condiciones.
c  3  5 c  8
Comprobamos la segunda solución:
(a  1)(b  1)(c  1)  1  3  7  21
a  2, b  4, c  8  
abc  1  2  4  8  1  63
Y efectivamente, 21 | 63
Así pues, hemos encontrado la primera solución: a  2, b  4, c  8 .

abc  1 3bc  1
Caso 2: a  3  
(a  1)(b  1)(c  1) 2(b  1)(c  1)

Caso 2.1
3bc  1
 2  3bc  1  4(b  1)(c  1)  4bc  4b  4c  4
2(b  1)(c  1)
 0  bc  4b  4c  5  11  bc  4b  4c  16  (b  4)(c  4)

Las únicas posibilidades son:


b  4  11 b  15
  que no cumple la condición c  b
c  4  1 c  5
b  4  1 b  5
  que sí cumple las condiciones requeridas.
c  4  11 c  15

Comprobamos la segunda solución:


(a  1)(b  1)(c  1)  2  4 14  112
a  3, b  5, c  15  
abc  1  3  5 15  1  224
Y efectivamente, 112 | 224 .

Así pues, hemos encontrado una segunda solución: a  3, b  5, c  15 .

Caso 2.2
3bc  1
 3  3bc  1  3(b  1)(c  1)  3bc  3b  3c  3
2(b  1)(c  1)
4
 0  3b  3c  4  4  3(b  c)  b  c 
3
Y no puede cumplirse pues b y c son enteros.

Las únicas soluciones son a  2, b  4, c  8 y a  3, b  5, c  15 .

Fuente de esta solución: [Link]

6.50
Está claro que x x y  1984  x  1984  x  1984 .
1984  x  y  y  1984  x  y   1984  x   1984  x  2
2
1984 x

De aquí deducimos que 1984 x debe ser entero, luego 1984 x  26  31x tiene que ser un
cuadrado perfecto.

 2
26  31x  23 31x es un cuadrado perfecto  31x es un cuadrado perfecto.
31x  a 2  31 | a 2  31 | a  a  31b  a 2  312 b 2
31x  312 b 2  x  31b 2
x  31 b 2  1984  b  1984 / 31  8

Para cada valor 1  b  7 obtenemos un valor x  31b2  x  31b , y un valor


y  1984  x   8
2

31  31b  318  b   318  b 
2 2 2

La condición x  y se convierte en
31b 2  31(8  b) 2  b 2  (8  b) 2  b 2  64  16b  b 2 
 0  16 4  b   0  4  b  b  4
Luego son 3 los posibles valores aceptables. Aunque no es necesario para la resolución del
problema, las soluciones son:
b  1  x  31  y  318  1  31  7 2  ( 31,1519 )
2

b  2  x  124  y  318  2  1116  (124 ,1116 )


2
b  3  x  279  y  318  3  775  (279 ,775 )
2

A partir de este valor se contradice la hipótesis x  y :


b  4  x  496  y  318  3  496  (496 ,496 )
2

6.51
Las condiciones del enunciado se corresponden con el siguiente sistema de ecuaciones
diofánticas:
n  14 2 a  14b  c

n  15 a  15c  b
2

n  6 3 a  6 2 c  6 a  c

cumpliendo, además: 0  a  6 , 0  c  6 , 0  b  14

Simplificamos la tercera ecuación:


n  63 a  62 c  6a  c  a(63  6)a  c(62  1)  222 a  37 c

y la sustituimos en las otras dos ecuaciones:


222 a  37 c  14 2 a  14b  c  0  26 a  14b  36 c  0  13a  7b  18c



222 a  37 c  15 a  15c  b  0  3a  b  22 c
2

Multiplicamos la segunda ecuación por 7 y le restamos la primera:


0  13a  7b  18c
  0  34 a  136 c  34 a  4c 
0  21a  7b  154 c

Luego a  4c

Puesto que 0  a  6 , la única posibilidad es que c  1 , y por tanto a  4 , y


n  222  4  37  1  888  37  925

6.52
Puesto que los únicos factores de 23 son 23 y 1, tenemos:
c  23
ac  bc  23  c(a  b)  23  
c  1

Supongamos que c  1 . Entonces el sistema queda:


ab  b  44 b(a  1)  44 a  1  b  23  1  22
   (23  a)(a  1)  44  
a  b  23 b  23  a a  21  b  23  21  2

Las soluciones son: a  1, b  22 , c  1 y a  21, b  2, c  1 . Efectivamente, las comprobamos:


1  22  22  1  44
a  1, b  22, c  1  
1  1  22  1  23
21  2  2  1  44
a  21, b  2, c  1  
21  1  2  1  23
El caso c  23 no se puede dar puesto que a y b son positivos, y por tanto a  b  1 y
23(a  b)  23

Sin tener en cuenta este hecho, y intentando resolver el sistema, hubiéramos llegado a
soluciones negativas, y por tanto no válidas:
ab  23b  44 ab  23b  44  b(a  23)  44 a  1
   (1  a)(a  23)  44  
23a  23b  23 a  b  1  b  1  a a  21
El número de soluciones es 2.

6.53
abc  70  2  5  7

cde  71  71
efg  72  23  32

La clave está en la segunda ecuación: cde  71 que es un número primo, luego solo puede
ocurrir uno de los tres casos siguientes:
a) c  1, d  1, e  71
Pero entonces 71  fg  72 , lo cual es imposible.
b) c  71, d  1, e  1
Pero entonces ab  71  70 , lo cual es imposible.
c) c  1, d  71, e  1
Tenemos el sistema
ab  2  5  7

 fg  2  3
3 2

La primera ecuación permite los siguientes valores de a :


a  1,2,5,7,2  5,2  7,5  7,2  5  7 , ocho casos diferentes.
La segunda ecuación permite los siguientes valores de f :
f  1,2,22 ,23 ,3,32 ,2  3,2  32 ,22  3,22  32 ,23  3,23  32 , doce casos diferentes.
Luego el total de casos es 8  12  96 .

6.54
4 2 4n  2m
 1  1  4n  2m  m n  4n  2m  m n  0
m n mn

Completando cuadrados:
0  4n  2m  m n  (m  4)(n  2)  8  (m  4)(n  2)  8  23

Hay cuatro posibilidades:


m  4  1 , n  2  8  m  5 , n  10
m42,n24m6,n6
m4 4,n2 2 m 8,n  4
m  4  8 , n  2  1  m  12 , n  3
6.55
1. Caso c  0  c  c
1.1 Si a  b  0 , la ecuación queda
a  b  ab  78  ab  a  b  78 , y completando cuadrados:
ab  a  b  1  78  1  (1  a)(1  b)  79 , y 79 es primo, luego:
1  a  79  a  78 
 no cumplen las hipótesis.
1 b 1 b  0 

1.2 Si a  b  0 , la ecuación queda


 (a  b)  ab  78  78  ab  a  b , y completando cuadrados:
ab  a  b  1  78  1  (a  1)(b  1)  79
a  1  79  a  78 
 no cumple a  b  0
b 1 1 b  0 
a  1  79  a  80 
  c  19  a  b  19   80  2  19  82  0
b  1  1  b  2 

Luego no hay solución posible con c  0 .

2. Caso c  0  c  c :
 a  b  c  19
El sistema queda de la siguiente forma: 
 ab  c  97
2.1 Supongamos que a  b  0 . Entonces
a  b  c  19  c  19  a  b
  ab  (19  a  b)  97  ab  a  b  97  19  116
ab  c  97 
Completamos cuadrados:
ab  a  b  116  ab  a  b  1  116  1  (a  1)(b  1)  117  32  13
a 1  3  a  2 
  c  ab  97  2  38  97  21
b  1  3  13  39  b  38 
a 1  9  a  8 
  c  ab  97  8  12  97  1
b  1  13  b  12 
a 1 1  a  0 
  c  ab  97  0  97  97
b  1  117  b  116 

Las soluciones que aparecen son:


a  2 , b  38 , c  21 , a  38, b  2 , c  21
a  8 , b  12 , c  1 , a  12, b  8 , c  1
a  0 , b  116 , c  97 , a  116, b  0 , c  97
2.2 Supongamos que a  b  0 . Entonces
 (a  b)  c  19  c  19  a  b
  ab  (19  a  b)  97  ab  a  b  97  19  116
ab  c  97 
Completando cuadrados:
ab  a  b  116  ab  a  b  1  116  1  (1  a)(1  b)  117

1 a 1  a  0 
  c  ab  97  0  97  97
1  b  117  b  116 
1  a  3  a  2 
  c  ab  97  (2)(38)  97  21
1  b  3  13  39  b  38 
1  a  9  a  8 
  c  ab  97  (8)(12 )  97  1
1  b  13  b  12 

Las soluciones que aparecen son:


a  0 , b  116 , c  97 , a  116, b  0 , c  97
a  2 , b  38 , c  21 , a  38 , b  2 , c  21
a  8 , b  12 , c  1 , a  12 , b  8 , c  1
En total 12 soluciones posibles.

6.56
a  1 (1)

a  1  a  1 , b par (2)
b

a  0, b  0 (3)

x  2
(1) x 2  x  1  1  0  x 2  x  2  ( x  2)( x  1)  
 x  1
Las dos soluciones son aceptables.

x  0  x  2  2
(2) x 2  x  1  1  0  x 2  x  x( x  1)  
x  1  x  2  3
La solución x  1 no es aceptable pues entonces el exponente no es par.

(3) x  2  0  x  2  (2)2  (2)  1  4  2  1  5  0 luego es aceptable.

Luego la ecuación tiene cuatro soluciones: x  2 ,  1 , 0 ,  2

6.57
Si m  0 , la ecuación 1  1  2  3n no tiene solución por el TFA.
Si m  1 , la ecuación 2  1  3n tiene solución n  1 .
Supongamos que m  2 .
2 m  1  3n  2 m  3n  1 y puesto que m  2 , 4 | 3n  1  3n  1  0 (mod 4)  3n  1 (mod 4)
Pero 3  1 (mod 4)  3n  (1) n  1 (mod 4)  n es par  n  2k
 
2
  2

Así pues, 2m  1  32k  3k  2m  3k  1  3k  1 3k  1  
  
Para que 3k  1 3k  1 sea una potencia de 2 es necesario que ambos factores sean una
potencia de dos:

3  1  2
k a

 k con a  b  m

3  1  2
b

Pero entonces, sumando las dos igualdades llegamos a 2  3k  2 a  2b


Si a, b  2 , entonces 4 | 2 a  2b , pero está claro que 4 | 2  3k . Así pues, la menor potencia tiene
que ser 2, es decir, 3k  1  2  3k  3  k  1
La ecuación queda 2 m  1  32  9  2 m  8  m  3 .
Así pues, las dos únicas soluciones son m  1 , n  1 y m  3 , n  2 .

Fuente de la solución: “Teoría de números: Divisibilidad y Congruencias (Entrenamiento de Hidalgo para la Olimpiada Mexicana de
Matemáticas)”, pág. 25

6.58
La única combinación aceptable es
y7 
  x  4  7  1  x  1  28  29
x  4 y  1

6.59
Las parejas ( x, y ) de enteros positivos que satisfacen x y  24 son las siguientes:
(1,24 ), (2,12 ), (3,8), (4,6), (6,4), (8,3), (12,2), (24,1)

De la misma manera, las parejas ( x, z ) de enteros positivos que satisfacen x z  48 son las
siguientes:
(1,48), (2,24 ), (3,16 ), (4,12 ), (6,8), (8,6), (12,4), (16,3), (24,2), (48,1)

Y las parejas ( y , z ) de enteros positivos que satisfacen y z  72 son las siguientes:


(1,72 ), (2,36 ), (3,24 ), (4,18), (6,12 ), (8,9), (9,8), (12,6), (18,4), (24,3), (36,2), (72,1)

De entre todas las combinaciones x, y, z posibles, la única aceptable es


x  4, y  6, z  12 , y por tanto x  y  z  4  6  12  22 (D)

6.60
7! 7  6  5  4  3  2  1  24  7  3  5  3
Si A es el conjunto de divisores de 7  3  5  3 , el conjunto de todos los divisores de 7! se puede
dividir en la partición 1A, 2 A, 4 A, 8 A, 16 A , y por tanto solo una quinta parte son impares (A).

6.61
N  34  34  63  270  23  35  5  7  17 2
Todo divisor impar k de N será cualquier divisor de 35  5  7  17 2 , y para cada divisor impar k
fijo, obtendremos tres divisores pares de N, de la forma 2k , 4k o 8k , claramente distintos.
Luego, al sumar todos los divisores k, en el denominador aparecerá k y en el denominador
aparecerá 2k  4k  8k  14k .
Así pues, podremos sacar factor común 14 en el denominador y la razón será 1:14 (C).

6.62
Puesto que ambos son impares, su suma será par (A)
6.63
5m  n | 5n  m si y solo si existe una k entera tal que 5n  m  k (5m  n) .
5n  m  k (5m  n)  5km  kn 
5n  m  5km  kn  0 
n (5  k )  m(1  5k )  0 
n (5  k )  m(5k  1)
De aquí deducimos que 0  k  5 , pues
k  5  n(5  k )  0  m(5k  1)  0  m  0 contradiciendo las hipótesis del enunciado.
k  5  n(5  k )  0  0  m(5k  1)  m  24  m  0 , contradiciendo de nuevo las hipótesis
del enunciado.
Y finalmente, k  0 tampoco es aceptable, pues en dicho caso, volviendo a la igualdad del
inicio, 5n  m  k (5m  n)  0  5n  m  0 , absurdo si n y m son positivos.

n 5k  1 5k  1
Ahora, n (5  k )  m(5k  1)   , y solo queda comprobar que es un entero
m 5k 5k
para 0  k  5 . En efecto:

5k  1 4 5k  1 9 5k  1 14
k 1  1 , k  2    3 , k 3   7,
5k 4 5k 3 5k 2
5k  1 19
k 4   19
5k 1
n
Así pues, en todos los casos aceptables, es un entero, es decir, se cumple m | n .
m

6.64
Sean c y d las soluciones de la ecuación x 2  b x  80  0 .
Aplicando las fórmulas de Vieta tenemos que
c  d  80  24  5

c  d  b
Los candidatos para c y d enteros, positivos, distintos y pares son:
c  2 , d  23  5  b  2  23  5  42
c  22 , d  22  5  b  22  22  5  24
c  23 , d  2  5  b  23  2  5  14
y aparecen tres valores para b, por lo que la respuesta correcta es (D).

6.65
Sean c y d las raíces de la ecuación x 2  a x  2007  0 .
Aplicando las fórmulas de Vieta,
c  d   a

cd  2007  3  223
2

Puesto que c y d son enteros, los candidatos aceptables para c y d (sin importar el orden) son:
   
1, 32  223 ,  3, 3  223  , 32 , 223 , y como también aceptamos valores negativos, hay un total
de 6 combinaciones posibles (C).
6.66
A falta de un método mejor, calculamos todas las posibilidades:
6  m  1 6  m  1
  m  5, n  6   m  7, n  18
6  n  12 6  n  12
6  m  2 6  m  2
  m  4, n  0   m  8, n  12
6  n  6 6  n  6
6  m  3 6  m  3
  m  3, n  2   m  9, n  10
6  n  4 6  n  4
6  m  4 6  m  4
  m  2, n  3   m  10, n  9
6  n  3 6  n  3
6  m  6 6  m  6
  m  0, n  4   m  12, n  8
6  n  2 6  n  2
6  m  12 6  m  12
  m  6, n  5   m  18, n  7
6  n  1 6  n  1
Luego comprobamos que hay 12 valores diferentes (C).

6.67
102 2  2  3  17   22  32  17 2 . Calculamos 17 2  289 , y por tanto:
2

2  3  17 2  1734 , 22  3  17 2  3468 , 32  17 2  2601 , 2 2  17 2  1156 y 2  32  17 2  5202 .


hay un total de 5 divisores, (D).

6.68
Aplicamos la Identidad "diferencia de cuadrados":
255 2  1  255  1255  1  254  256  2  127  28  127  29 (C)

6.69
Sean a y b las dos soluciones de la ecuación. Aplicando las fórmulas de Vieta tenemos que:
a  b  63

a b  k
Para que la suma de dos números sea impar uno de ellos debe ser par y el otro impar. El único
número primo par es 2, luego el otro es 61, y el único valor de k es 2  61  122 .

6.70
2020  22  5  101
Observamos que cualquier divisor producto de dos primos (o más) distintos ya cumple la
condición: pq   1 , p , q , pq 
Luego los divisores serán 2  5 , 2  101 , 5  101 , 2  2  5 , 2  2 101 , 2  5 101 , 2 2  5  101 , un total
de 7 (B).

6.71

Sea g  2002  2 , 2002 2  2 , 2002 3  2 , ... 
En primer lugar nos centramos en los dos primeros términos de la sucesión:
2002 2  2  2000 (2002  2)  6 , y por tanto, aplicando el Algoritmo de Euclides,
 2002  2 , 2002 2

 2   2002  2 , 6   6
Así pues, g debe ser un divisor de 6.
Todos los números que aparecen en la sucesión son divisibles entre dos, luego para demostrar
que
g  6 basta ver que todos los números son divisibles entre 3, pues 2 y 3 son coprimos.
En efecto, 2002  3  667  1  2002 n  3  667  1  3  an  1 , aplicando el Binomio de
n

Newton.
Por lo tanto, 2002 n  2  3  an  1  2  3an  1 es múltiple de 3.

6.72

16 n 2  n  1 
2


4n 2  4n  4

 
2n  2  2n  1  5  2n  1  5
2 2
 
2 2

2

2n  1 2n  1 2n  1 2n  1

Mediante el cambio de variable u  2n  1 , la expresión anterior se convierte en


u 2

5
2

Está claro que u | u 2 y u | u 4 , luego u | u 4  10u 2  25  u | 25

Luego los posibles valores aceptables son:


u  2n 1  1  n  1
u  2n 1  5  n  3
u  2n 1  25  n  13
u  2n 1  1  n  0
u  2n 1  5  n  2
u  2n 1  25  n  12

Observación: Otro desarrollo alternativo sería ver que


u5 2


2
u 4  10u 2  25 u 4 10u 2 25
    u 3  10u 
25
u u u u u u
Que será entero si y solo si u | 25

6.73
p 2  2q 2  1  2q 2  p 2  1 , que es un número par, luego p 2 debe ser impar, y por tanto q
debe ser impar: p  2k  1 .
Substituimos este valor en la ecuación:
1  p 2  2q 2  (2 p  1) 2  2q 2  4k 2  4k  1  2q 2 
 
0  4k 2  4k  2q 2  2 2k 2  2k  q 2  2k 2  2k  q 2  0
 q 2  2k k  1
Así pues, q 2 es par, luego q es par. Pero el único primo par es 2. Así pues, q  2 , y por tanto
 1  p  2(1)  1  -3 absurdo
q 2  4  2k k  1  2  k (k  1)  k  
2  p  2  2  1  3
y la única solución aceptable es p  3 , q  2 .

6.74
Sea A1 el número de adultos en el concierto antes de la llegada del autobús, y sea A2 el número
de adultos que llegan en el autobús.
Sea T el total de asistentes al concierto antes de la llegada del autobús.

A1 5 
 
  A1  A2  T  50    A2  T  50 
T 12 11 5T 11
A1  A2 11  25 12 25

T  50 25  

A2 
11
T  50   5T  11T T  22  5T   11  5 T  22  7 T  22
25 12 25 12  25 12  300

7
Puesto que A2 y 22 son números enteros, T debe ser un número entero, y por tanto
300
T  300 k , con k  1 .
A 5 5
Por otro lado, 1   A1  T , luego T debe ser un múltiplo de 12.
T 12 12
300 es un múltiplo de 12, luego ya nos sirve.
5 7
Tomando k  1  T  300  A1  T  125 , A2  T  22  7  22  29
12 300
Y el total de adultos es A1  A2  125  29  154 .

Segunda versión.
La solución "oficial" que aparece en [Link] se basa en aritmética
modular:

  T  A1  T  0 mod 12 
A1 5 12
T 12 5
A1  A2 11
  T  50  0 mod 25   T  0 mod 25 
T  50 25
Puesto que buscamos el valor de T mínimo, la solución es T  300 . Así pues, hay 350 personas
11
después de que llegue el autobús, de las que 350  154 son adultos.
25

6.75
La recta será de la forma y  ax  b , cumpliendo:
0  a  p  b para cierto p primo.
a  0  b  b es un entero positivo. Luego  b  a  p  a es un entero negativo.
Pero entonces 3  a  4  b  4a  a  p  a 4  p
Posibilidades:
a  1 a  3
 
4  p  3  p  7 4  p  1  p  5
Y hay, por tanto, dos soluciones posibles: (C)

6.76
n 2  2n  3  (n  3)(n  1)  n  3  n  1
El producto de dos números positivos es primo si y solo si al menos uno de ellos es 1.
n  3  1  n  4
n 3 1 
n  3  1  n  2
n  1  1  n  0
n 1  1  
n  1  1  n  2
Luego hay cuatro casos posibles: (C)

6.77
El único primo múltiplo de 3 es el propio 3. Cualquier otro número primo no será múltiplo de
3, y por tanto será de la forma p  3k  1 y por tanto

p 2  2  3k  1  9k 2  6k  1  2  9k 2  6k  3  3 3k 2  2k  1
2

es decir, será un múltiplo de 3 mayor que 3, y por tanto no primo.
Así pues, hay un único primo p cumpliendo las condiciones del enunciado (B).

6.78
El único número primo par es el 2. Cualquier otro primo será impar, y por tanto se podrá
escribir de la forma p  2k  1 . Y por tanto:
p 4  1  (2k  1)4  1  16 k 4  32 k 3  24 k 2  8k  2
Es un número par mayor que 2, luego seguro que no es primo.
Así pues, solo hay un número cumpliendo estas condiciones, y la respuesta correcta es (B).
6.79
Está claro que si n  0 entonces 3 n no es entero, luego no existe solución.
Si n  0 , la ecuación que obtenemos es
m 3  24 m 2  2022  1 
m 3  24 m 2  2023  0 
m 3  24 m 2  2023 
m 2 m  24   2023  7  17 2
Las únicas posibilidades son m  1 o m  17 para el cuadrado, y que la única opción que
satisface la ecuación es m  17 .

Si n  1, 3 n es múltiplo de 3, como también lo son 24m 2 y 2022 , luego m 3 será múltiplo de 3,


y por tanto también lo será m . Digamos que m  3k .
m 3  24 m 2  2022  3n  (3k ) 3  24 (3k ) 2  2022  3n 
 33 k 3  8  3  32 k 2  3  2  337  3n 
32 k 3  8  32 k 2  2  337  3n1 
32 k 2 k  8  3n1  2  337

Observemos la parte de la derecha. Para n  1 , 3n1  2  337  1  2  337  675  33  52 , y la


ecuación se transforma en 32 k 2 k  8  33  52  k 2 k  8  3  52 , que, siguiendo
razonamientos similares a los anteriores, solo tendrá solución para k  5 , y por tanto
m  15 .

Si n  2 , en la expresión 33 k 3  8  3  32 k 2  3  2  337  3n , todos los términos son múltiples de


9 excepto 3  2  337 , llegando a contradicción.
Así pues, las únicas soluciones son m  17 , n  0 y m  15, n  1 .

6.80
Como es habitual en este tipo de problemas, comenzamos factorizando la parte de la derecha:
   
p x  y 4  4  y 4  4 y 2  4  4 y 2  y 2  2  2 y   y 2  2  2 y y 2  2  2 y
2 2

Luego y 2  2  2 y  p n y y 2  2  2 y  p m , con 0  n, m cumpliendo n  m  x .

Puesto que y 2  2  2 y  y 2  2  2 y se cumplirá que n  m , y por tanto p n | p m , es decir:


y2  2  2 y | y2  2  2 y  y2  2  2 y  4 y  y2  2  2 y | 4 y  y2  2  2 y | 4 y2

 
Por otro lado, 4 y 2  4 y 2  2 y  2  8 y  1 , luego
y 2  2  2 y | 4 y 2  y 2  2  2 y | 8( y  1)

  y  2  2y | 8
2

y  2  2y | 4y  y  2  2y | 8y
2 2

Así pues, y 2  2  2 y  1, 2, 4, 8

Veamos cada caso por separado:


y 2  2  2 y  1  y  1  p x  14  4  5  p  5, x  1 , llegando así a la solución
 5, 1, 1 .
y 2  2  2 y  2  y  2  p x  24  4  20  22  5 , no es una potencia de un primo.
y 2  2  2 y  4 y y 2  2  2 y  8 no tienen soluciones reales.
La única solución posible al problema es  5, 1, 1 .

Fuente de esta solución: Soluciones oficiales.

6.81
2 2
 19  35  19  35
k  n  19 n  99   n   
2 2
 k2  n    
 2 4  2 4
 19   19  35  2k  2n  19   2k  2n  19  35
  k  n   k  n      
 2  2 4  2  2  4
 2k  2n  19 2k  2n  19   35  5  7

Ahora estudiamos los diferentes casos:


2k  2n  19  5 2  3  2n  19  5
  4k  12  k  3 , y por tanto  n9
2k  2n  19  7 2  3  2n  19  7
2k  2n  19  7 2  3  2n  19  7
  4k  12  k  3 , y por tanto   n  10
2k  2n  19  5 2  3  2n  19  5
2k  2n  19  35 2  9  2n  19  35
  4k  36  k  9 , y por tanto   n  18
2k  2n  19  1 2  9  2n  19  1
2k  2n  19  1 2  9  2n  19  1
  4k  36  k  9 , y por tanto   n 1
2k  2n  19  35 2  9  2n  19  35

Así pues, la respuesta correcta es 9  10  18  1  38 .

6.82
La parte de la izquierda es siempre un entero, luego también lo debe ser la derecha,y por tanto
n  0.
Supongamos que n  0 , es decir:
m3  24 m2  2022  1  m3  24 m2  2023  m2 m  24   2023  7  17 2 .
Los candidatos para m son 1 y  17 . Probando estas opciones vemos que la única posibilidad
aceptable es m  17 .

Supongamos que n  1, es decir: m 3  24 m 2  2022  3 .


Todos los términos son múltiples de 3, luego también lo será m 3 , y por tanto también lo será
m . Supongamos que m  3k , para cierto k  0 . Entonces:
3  m 3  24 m 2  2022  27 k 3  33  23  k 2  2022 
2025  27 k 3  33  2 3  k 2 
75  k 3  8k 2 
3  5 2  k 2 k  8

Las únicas posibilidades son k  1 y k  5 . Probando estas opciones vemos que la única
aceptable es k  5 y por tanto m  15 .

Supongamos que n  1. La parte de la derecha será múltiple de 3, luego también lo será la parte
de la izquierda. Los números 24 y 2022 son ambos múltiples de 3, luego también lo será m 3 , y
por tanto también lo será m . Supongamos que m  3k , para cierto k  0 .
3n  m 3  24 m 2  2022  3k   24 3k   2022  33 k 3  32  8k 2  2  3  337 
3 2

 
 3 32 k 3  3  8k 2  2  337 
n 1
3  3 k  3  8k  2  337 
2 3 2

2  337  32 k 3  3  8k 2  3n1
Puesto que n  1, toda la parte de la derecha de esta última expresión es múltiple de 3, pero no
lo es la parte de la izquierda, llegando a contradicción.
Luego las únicas soluciones posibles son m  17 , n  0 y m  15, n  1 .

6.83
 
2
x 2  y 2  2000 2  ( x  y)( x  y)  24  53  28  56

La parte de la derecha es un número par, los dos factores de la izquierda: x  y y x  y tienen


la misma paridad, luego ambos tienen que ser pares. Esto quiere decir que:

x  y  2  5
a b



x  y  2  5
8 a 6 b

Con 1  a  7 y 0  b  6
Sumando las dos ecuaciones: 2 x  2a  5b  28  a  56  b  x  2a 1  5b  27  a  56  b
Restando las dos ecuaciones: 2 y  28 a  56 b  2a  5b  y  27  a  56 b  2a 1  5b
Es decir, en todo momento el sistema es compatible determinado.
Son 7  7  49 combinaciones diferentes.
Observamos que en la expresión x  2a 1  5b  27  a  56  b el valor es siempre positivo.
También tenemos que contar los valores de x negativos, por lo tanto tenemos un total de
2  49  98 combinaciones diferentes.

6.84
Primera versión.

n 2  97 n  85  k 2  n 2  97 n  85  k 2  0  n 
 
97  97 2  4 1  85  k 2

97  4k 2  9749
2 1 2 1

De esto deducimos que 4k 2  9749 tiene que ser un cuadrado perfecto.


4k 2  9749  q 2  9749  q 2  (2k ) 2  (q  2k )(q  2k )

El número 9749 es primo, luego solo se pueden dar los siguientes casos:
q  2k  1 
  q  4875 , k  2437
q  2k  9749 
q  2k  1 
  q  4875 , k  2437
q  2k  9749 
q  2k  9749 
  q  4875 , k  2437
q  2k  1 
q  2k  9749 
  q  4875 , k  2437
q  2k  1 

Así pues, k  2437 , y por tanto la ecuación del enunciado se reduce a


n  2389
n 2  97 n  85  2437 2  
n  2486

El razonamiento de esta primera versión implica resolver al final una ecuación de segundo
grado bastante farragosa. Un razonamiento más aceptable en el contexto de una competición
matemática sería el siguiente.

Segunda versión.
Supongamos que n 2  97 n  85  k 2 para cierto entero k. Multiplicamos por cuatro la ecuación.
4n 2  4  97 n  4  85  4  k 2 
2n 2  2  2  97 n  4  85  2k 2 
2n  97 2  9749  2k 2 
2n  97 2  2k 2  9749 
2n  97  2k 2n  97  2k   9749
El razonamiento sigue como en la versión anterior. Puesto que 9749 es primo,
2n  97  2k  1 
  n  2486 , k  2437
2n  97  2k  9749 
2n  97  2k  1 
  n  2389 , k  2437
2n  97  2k  9749 
2n  97  2k  9749 
  n  2486 , k  2437
2n  97  2k  1 
2n  97  2k  9749 
  n  2389 , k  2437
2n  97  2k  1 

6.85
2016  0  2 m  2 n  0  2 m  2 n  m  n
Luego podemos sacar factor común n:
 
2m  2n  2n 2mn  1  2016  25  32  7
Está claro que 2 mn  1 es impar, luego no puede contener ningún factor potencia de 2. Luego
la única posibilidad es que n  5 .
 
25 2m5  1  25  32  7  2m5  1  32  7  63  2m5  64  25  m  5  5  m  10 .
Así pues, la única solución posible es (m, n)  (10 ,5)

6.86
Sea n  a b c d e , de forma que abcde  1000  (2  5) 3  23  53
Observamos la factorización de los cinco dígitos:
1  1, 2  2 , 3  3, 4  2 2 , 5  5 , 6  2  3, 7  7 , 8  23 , 9  32
Está claro que los únicos dígitos que pueden aparece son 1, 2, 4, 5 y 8.
El dígito 5 tiene que aparecer tres veces, pues es la única forma de obtener aparezca 5 3 .
Quedan tres posiciones libres, que juntas deben generar 2 3 . Las posibilidades son:
1  8  32 y 2  4  23
5! 5  4
Las formas de colocar tres “5” en cinco posiciones diferentes son P52,3    10
2!3! 2
Por un lado, tomando las cifras 1 y 8 obtendremos 10  2  20 posibilidades (pues pueden
aparecer como 15855 o 85155)
Tomando las cifras 2 y 4 obtendremos otras 10  2  20 posibilidades (pues pueden aparecer
como 25455 o 45255)
En total, 20  20  40 posibilidades (D).

6.87
Vamos a resolver este problema a base de contraejemplos.
Para A, N  1  40  23  5  N  451
Para B, N  1  30  2  3  5  N  252
Para C, N  1  25  52  N  54
Para E, N  1  24  23  3  N  45
Luego solo la opción D sería una respuesta correcta.

6.88
100  4  25  2  2  5  5
La cantidad de divisores de este número es suficientemente pequeña como para poder
considerar los casos, uno por uno:
1 2  50 , 1 4  25 , 1 5  20 , 2  5 10
Son cuatro las factorizaciones que cumplen las condiciones del enunciado (E).

6.89
Observamos que 768  28  3  2  384 y 384  2 7  3
Sea d (768 ) la suma de todos los divisores de 768:
d (768 )  1  2  2 2  23  ...  28  3  3  2  3  2 2  3  23  ...  3  28

Sea d (384 ) la suma de todos los divisores de 384:


d (384 )  1  2  22  23  ...  27  3  3  2  3  22  3  23  ...  3  27
Por otro lado,
d (768 ) d (384 ) d (768 ) 2  d (384 ) d (768 )  2  d (384 )
f (768 )  f (384 )     
768 384 768 2  384 768

Y vemos que 2  d (384 )  2  2 2  23  2 4  ...  28  3  2  3  2 2  3  23  ...  3  28 , que contiene


los mismos sumandos que d (768 ) exceptuando el 1 y el 3, luego al hacer d (768 )  2  d (384 )
se cancelarán todos los términos excepto el 1 y el 3, y por tanto:
d (768 )  2  d (384 ) 1  3 22 1 1
  8  6  (B)
768 768 2  3 2  3 192

6.90
84  2 2  3  7
Luego nuestro problema se reduce a encontrar los valores de a para los cuales
3 | a(a  1)(a  2)
4 | a(a  1)(a  2)
7 | a(a  1)(a  2)

La primera de las condiciones se cumple siempre, pues en tres números seguidos seguro que
uno es múltiplo de tres.
Puesto que a  25 , la tercera condición nos permite hacer una búsqueda rápida:
a  7  7  8  9 cumple la condición
a  1  7  6  7  8 cumple la condición
a  2  7  5  6  7 no cumple la condición, pues no es divisible entre 4.
a  14  14  15  16 cumple la condición
a  1  14  13  14  15 no cumple la condición
a  2  14  12 13  14 cumple la condición
a  21  21  22  23 no cumple la condición
a  1  21  20  21  22 cumple la condición
a  2  21  19  20  21 cumple la condición
Luego hay 6 valores aceptables (E).

6.91
En primer lugar observamos que
50! 2 47  322  512  78  114  13 3  17 2  19 2  23 2  29  31  37  41  43  47
5! 23  3  5
Luego buscamos números de la forma
x  5!a
y  5!b
con (a, b)  1
y satisfaciendo a  b  2 44  321  511  78  114  13 3  17 2  19 2  23 2  29  31  37  41  43  47

Para este problema no hacía falta saber las potencias de cada uno de estos primos, solo saber el
número de primos menores de 50, pues si uno de estos números a o b toma una potencia de, por
ejemplo, 3, debe tomarla con el exponente máximo, 321 , porque de lo contrario a y b no serían
coprimos.
Así pues, a y b toman todos los subconjuntos posibles de estas 15 potencias de primos, pero
como además x  y , es decir, a  b , tomaremos solo la mitad. Así pues, el número de
posibilidades es 214 .

6.92
Decir que x 2  81 es múltiple de 100 es equivalente a decir que x 2 acaba en “81”.
x 2  81  100 y  ( x  9)( x  9)  100 y  5  20 y
Luego ( x  9)( x  9) es múltiple de 5
Primera opción: x  9 es múltiple de 5. Entonces
x  9  5k  x  5k  9 
 x 2  81  5k  9  81  25 k 2  90 k  81  81  25 k 2  90 k  5k 5k 2  18
2
 
   
5k 5k 2  18  100 y  k 5k 2  18  20 y  5  4 y
 
Así pues, k 5k 2  18 es múltiple de 5, y está claro que 5k 2  18 no puede ser múltiple de 5,
luego k debe ser un múltiple de 5: k  5q , y por tanto
x  5  5q  9  25 q  9 .Vamos obteniendo los posibles valores de x, y comprobando si
cumplen la condición del enunciado:
q  0  x  25  0  9  9  x 2  81 sí es aceptable.
q  1  x  25  1  9  34  x 2  1156 no es aceptable.
q  2  x  25  2  9  59  x 2  3481 sí es aceptable.
q  3  x  25  3  9  84  x 2  7056 no es aceptable.
q  4  x  25  4  9  109  100
Segunda opción: x  9 es múltiple de 5. Entonces, con un razonamiento similar, llegamos a
x  25 q  9
q  1  x  25  1  9  16  x 2  256 no es aceptable.
q  2  x  25  2  9  41  x 2  1681 sí es aceptable.
q  3  x  25  3  9  66  x 2  4356 no es aceptable.
q  4  x  25  4  9  91  x 2  8281 sí es aceptable.
Las soluciones son: 9, 59, 41, 91 y la suma es 200 (A)

6.93
En primer lugar vemos que podemos reducir las factorizaciones de a, b y c a los factores 2 y 3:
a  2a1 3b1 , b  2a2 3b2 , c  2a3 3b3
Y por tanto:

a 2  2b 3  3c 5  2 a1 3b1 
2

 2  2 a2 3b2 
3

 3 2 a3 3b3 
5
 2 2 a1 32b1  23a2 133b2  25 a3 35b3 1 
2a1  3a2  1  5a3

2b1  3b2  5b3  1
Buscamos encontrar los valores mínimos, que encontraremos tomando
a3  2  a2  3  a1  5
b3  1  b2  2  b1  3
abc  2533  2332  2 231  21036
Los divisores de abc son todas las combinaciones posibles de a n b m , con 0  n  10 y
0  m  6 , en total 11  7  77 combinaciones posibles (D).

6.94
Puesto que 0  3000  5a  5b  5a  5b  a  b , y por tanto podemos sacar factor común 5 b :
 
5b 5ab  1  3000  23  3  53
5 a b  1 no puede ser múltiple de 5, luego la única opción posible es
5b  53  b  3
5ab  1  23  3  24  5a3  25  52  a  3  2  a  5
6.95
Sean  ,  estas dos raíces. Aplicando las identidades de Vieta tenemos:
    36  2 2  32

     k
Luego solo tenemos que ir probando todas las combinaciones posibles de los factores de
2 2  3 2 , teniendo en cuenta que no importa el orden:
  1 ,   36  k  37
  2 ,   18  k  20
  4 ,   9  k  13
  6 ,   9  k  15
  1 ,   36  k  37
  2 ,   18  k  20
  4 ,   9  k  13
  6 ,   9  k  15
Ocho combinaciones posibles, (B)

6.96
Tenemos n,18  180 y n,45  15 .
 
n,45   15  n , 32  5  3  5 luego n  3  5 k , y vemos que n  32  5 k

Por otro lado, n,18  180  3  5k , 32  2  32  22  5
El factor 3 2 puede salir del 18, el factor 5 aparece en 3 5k , pero el factor 2 2 tiene que venir de
k, así pues, n  3  5 k  3  5  22 q  60 q , para cierto entero q.
Tomando q  1 vemos que n  60 se adapta perfectamente a las condiciones de nuestro
enunciado, y por tanto la respuesta correcta es 6  0  6 (B).

6.97
a) Supongamos que podemos escribir n  a / b para ciertos enteros a y b. Entonces
2
a
2
a
n     2  b2 | a 2  b | a  n  a / b  Z
b b
b) Supongamos que n  m  k entero. Entonces
k2   
2
n  m nm2 n m

 
2
n  k  m  n  k  m  k  m  2k m  m 
2 k2  m  n
2k
k  m  n n  m  2 n m  m  n 2m  2 n m m  n m
2
m   
2k 2k 2k k
Vemos que m es un número racional, y por tanto, aplicando el apartado a, será un cuadrado
perfecto. Luego n  k  m también será un cuadrado perfecto.

6.98
a c
Supongamos que r  y 55 r  donde a y b son coprimos y c y d también son coprimos.
b d
Supongamos que 5 | b y 11 | b . Entonces 55r no se puede simplificar, y por tanto la única
posibilidad es c  55a y d  b . Luego
a  b  c  d  a  b  55a  b  a  55a  a  0 lo cual es imposible, pues suponemos r
positivo.
Luego se debe cumplir forzosamente 5 | b o 11 | b .
a) Supongamos que 5 | b pero 11 | b , es decir, b  5b' . Por lo tanto
a 55a 11a
r , 55r   es la fracción simplificada de 55r, y por tanto
5b' 5b' b'
a  5b'  11a  b'  4b'  10a  2b'  5a , y por tanto 5 | b ' , es decir, b  25b' ' . De nuevo
a 55a 11a
r , 55r   es la fracción simplificada de 55r, y por tanto
25b' ' 25b' ' 5b' '

a  25b' '  11a  5b' '  20b' '  10a  2b' '  a , es decir, a es un múltiplo de 2: a  2a' .

2a ' 55  2a' 11  2a'


r , 55r   y por tanto
25b' ' 25b' ' 5b' '
2
2a'25b' '  11  2a'5b' '  20b' '  20a'  b' '  a' y la fracción queda r 
25
Efectivamente esta fracción cumple las condiciones del enunciado:

2 55  2 11  2 22
r  55r    y efectivamente 2+25=22+5.
25 25 5 5

b) Supongamos que 11 | b pero 5 | b , es decir, b  11b' . Por lo tanto


a 55a 5a
r , 55r   es la fracción simplificada de 55r, y por tanto
11b' 11b' b'
a  11b'  5a  b'  10b'  4a  5b'  2a . De aquí deducimos que 2 | b ' , es decir, b  22b' ' ,
y también que 5 | a , es decir, a  5a'
luego
5a' 55  5a' 5  5a'
r , 55r   y por tanto:
22b' ' 22b' ' 2b' '
5a'22b' '  25a'2b' '  20b' '  20a'  b' '  a' y la fracción buscada es
5a' 5
r  .
22b' ' 22
Efectivamente, esta fracción cumple las condiciones del enunciado:
5 55  5 25
r  55r   y 5+22=25+2.
22 22 2
2 5 2 5 169
Así pues, S   ,  y    169  550  719 .
 25 22  25 22 550

6.99
13 ! 13  11  7  52  35  210

m m
Este número será un entero cuadrado perfecto si y sólo si m es un divisor del numerador de
forma que genere un cociente con potencias pares, es decir,
m  13  11  7  5a  3b  2 c con a  0,2 , b  1, 3, 5 y c  0, 2, 4, 6, 8, 10

Para sumar todos estos 2  3  6  36 términos, podemos sacar factor común:


S1  1  2 2  2 4  26  28  210  1365  3  5  7  13
S 2  S1  32  S1  34  S1  S1 1  9  81  S1  91  3  5  7  13  7  13  3  5  7 2  13 2
S3  S 2  52 S 2  S 2 1  25   S 2  26  3  5  7 2  13 2  2  13  2  3  5  7 2  13 3
S3  13  11  7  50  31  20 S3  13  11  7  50  31  20  2  3  5  7 2  133 
 2  32  5  73  11  13 4
Así pues, el resultado es 1  2  1  3  1  4  12 .

6.100
Los divisores de 2 20  323 son todos los números de la forma 2 a  3b con 0  a  20 y 0  b  23 .
Los divisores de 210  320 son todos los números de la forma 2 a  3b con 0  a  10 y 0  b  20 .
Luego buscamos todos los números de la forma 2 a  3b con 11  a  20 y 21  b  23 .
En total 10  3  30 números (B).

6.101
Llamaremos n al producto de seis números consecutivos para cierto x entero, es decir:
n  a b b c d d c d d a b b  x( x  1)( x  2)( x  3)( x  4)( x  5)

En primer lugar vemos que 23 | n . En efecto, si x es par,


2 | x , 2 | x  2 , 2 | x  4 y por tanto 23 | n
Si x es impar:
2 | x  1 , 2 | x  3 , 2 | x  5 y por tanto 23 | n .
En todo caso, 23 | n .
También vemos que 3 | n , pues en una secuencia de seis números consecutivos seguro que
encontramos un múltiplo de tres (de hecho encontramos dos).
Y vemos que 5 | n , pues en una secuendia de seis números consecutivos seguro que
encontramos al menos un múltiplo de cinco.
23 | n  2 | n 
  25| n
5| n 
Y puesto que n es un múltiplo de 10, seguro que acabará en 0, por lo tanto ya hemos deducido
que b  0 .
Ahora bien, vemos que nuestro número acaba en cifras b b , es decir en “00”, luego es un
múltiplo de 100  2 2  52 , es decir, es un número de la forma
n  a 0 0 c d d c d d a 0 0  a 0 0 c d d c d d a 100  a 0 0 c d d c d d a  22  52

Pero sabíamos que 23 | n , luego a 0 0 c d d c d d a sigue siendo par, es decir, acaba en par, y por
tanto a es par.

En el enunciado nos dicen que los números a, b, c, d son consecutivos. Antes hemos deducido
que b  0 , luego a,b y c tienen que ser forzosamente 1, 2, 3 (no necesariamente en este orden).
Puesto que a es par, solo es aceptable a  2 .

Así pues, solo nos quedan dos posibilidades:


a  2, b  0, c  3, d  1  n  2003113112 00
a  2, b  0, c  1, d  3  n  2001331332 00
Para decidir cual de los dos es el correcto, recordamos que 3 | n , luego la suma de sus cifras
tiene que ser múltiplo de 3. Esto no ocurre en el primer caso:
2  0  0  3  1  1  3  1  1  2  0  0  14
y sí ocurre en el segundo:
2  0  0  1  3  3  1  3  3  2  0  0  18
Luego el número buscado es 2001331332 00 y la respuesta correcta es d  3 (C).

6.102
a b c  (a  b  c)  d d d  100 a  10b  c  a  b  c  d d d
 99 a  9b  d d d  911a  b   d d d
Vemos que el número d d d es un múltiplo de 9, luego la suma de sus cifras 3d será también un
múltiplo de 9, es decir, que d es un múltiplo de 3. Veamos los tres casos posibles:
a) d  3  911a  b  333  11a  b  37  a  3, b  4
b) d  6  911a  b  666  11a  b  74  a  6, b  8
c) d  9  911a  b  999  11a  b  111 a  10, b  1 imposible.
Solo se pueden dar dos casos, y puesto que el valor de c no importa y por tanto son aceptables
las 10 cifras posibles, hay 2×10=20 casos posibles. (D).

6.103
n  ab c  a  b  c  5
Está claro que estamos hablando de un número múltiplo de 5, por lo tanto acabará en 0 o en 5,
es decir: c  0 o c  5 .
Si c  0 entonces n  a b 0  a  b  c  0  0 y este caso queda descartado porque no tiene tres
cifras.
Si c  0 entonces
a  100  b  10  5  a  b  5  5  100 a  10b  5  25 a b  20 a  4b  1  5a b
.
 (5a  2)(b  4)  9 32
De las seis opciones posibles:
5a  2  1 5a  2  3 5a  2  9 5a  2  1 5a  2  3 5a  2  9
, , , , , 
b  4  9  b  4  3  b  4  1  b  4  9  b  4  3  b  4  1 
La única que ofrece soluciones enteras es la segunda:
5a  2  3 a  1

b  4  3  b  7
Obteniendo el número 175. Por lo tanto hay una única solución (D).

Fuente de esta solución: Ricard González Blanch


6.104
Está claro que este número tiene que ser de la forma n  2 k , con k  0 .
Con k  1 , n  2 se cumple la condición del enunciado.
Con k  2 , n  4 se cumple la condición del enunciado.
Con k  3 , n  8 ya no se cumple la condición del enunciado.
Y observamos que son los dos únicos números válidos.
La solución es 2.

6.105
6100 A  10 B  C   100 C  10C  C  600 A  60 B  6C  C 100  10  1  111C
 600 A  60 B  105 C  40 A  4 B  7C  410 A  B   7C
De aquí se deduce que C debe ser un múltiplo de 4, es decir, C  4C' con C ' 1, 2 , luego
10 A  B  7C'
Esto solo puede ocurrir para C '  2  C  8 , A  1 y B  4 . Luego la solución es 8  1  4  13 .

6.106
a)
Vamos a utilizar el hecho de que los divisores di de n aparecen “por parejas”, esto es,
n
di  d k  i 1  n  di 
d k  i 1
Así pues,
n n n n n2
di di 1   
d k i 1 d k  (i 1) 1 d k i 1 d k i d k i 1  d k i
Y por tanto
k 1 k 1 k 1
n2 1
D  d1d 2  d 2d3  ...  d k 1d k   di  di 1    n2 
i 1 i 1 d k  i 1  d k  i i 1 d k  i 1  d k  i

Ya solo queda demostrar que


k 1
1

i 1 d k  i 1  d k  i
1

Para ello vamos a convertir esta suma en una suma telescópica. Puesto que
1 d  d k i 1 1
d k  i 1  d k  i  d k  i 1  d k  i  1   k i 1  
d k  i 1  d k  i d k i 1  d k  i d k i d k i 1
Con lo que
k 1
1 k 1
 1 1  1 1 1 1
d 
  
i 1  k  i
      1
i 1 k  i 1 d k i d d k  i 1  d1 d k 1 n

b)
Está claro que esta condición sucede siempre que n es primo. En efecto, Si n  p , con p primo,
tenemos la cadena de divisores 1  p de n , y por tanto
D  1  p es divisor de n 2 .

Por el contrario, supongamos que n no es primo. Sea p el mínimo primo divisor de n.


n
Tenemos d1  1, d 2  p , d k 1  , d k  n , y por tanto
p
n n2
D  d1d 2  d 2d3  ...  d k 1d k  d k 1d k  n
p p
n2
Pero si p es el mínimo divisor primo de n, también lo será de n 2 , y por tanto será el mayor
p
n2
divisor propio de n 2 , por lo tanto, si  D  n 2 , D no puede ser un divisor de n 2 .
p

Fuente de esta solución: Soluciones oficiales (IMO, página 770)

6.107
Reescribimos la cadena de divisores empezando por el índice 0 para mayor comodidad.
Sabemos que los divisores de la cadena
1  d0  d1  ...  d k  n
n
Se pueden agrupar por parejas: d i  d k i  n  d k i 
di

Por lo tanto, en particular, d k 2 | d k 1  dk .


Pero d k  n , y por tanto está claro que d k 2 | d k , con lo cual deducimos que d k 2 | d k 1 , es decir,
n n
d k 2  a  d k 1   a   d1  n  a  n  d 2  d1  a  d 2  d1 | d 2
d2 d1
Pero, por otro lado, sabemos que d1 | d 2  d3 , luego d1 | d3 .
Esta cadena la podemos continuar hasta el final:
d 2 | d3  d 4 
  d1 | d 3  d 4
d1 | d 2 
d1 | d 3  d 4 
  d1 | d 4
d1 | d 3 
De esta manera vemos que d1 | di para todo i  1 , y esto solo se cumple en los compuestos de la
forma n  p m , con m  1, para un único primo p. En efecto, si hubiera otro factor primo q,
db  q para algún índice b, pero d1 | d b , lo cual es imposible.

Por último, veamos que los números n  p m cumplen la condición del enunciado.
Si n  p m , entonces la cadena de divisores es
1  d1  p  d 2  p 2  ...  d k  n  p m
Y siempre se cumple di | di 1 y di | di  2 por tanto está claro que di | di 1  di 2 .

6.108
Está claro que basta con determinar los n para los cuales
n 2 n3 n 4 n5 n6
   
2! 3! 4! 5! 6!
sea un entero.
n2 n3 n 4 n5 n6 6  5  4  3  n 2  6  5  4  n3  6  5n4  6n5  n6
     
2! 3! 4! 5! 6! 6!
23  32  5 n2  23  3  5 n3  2  3  5 n 4  2  3 n5  n6

6!
Puesto que 3  5  2  6! , queremos determinar los valores de n para los cuales
2 4


32  5  2 4 | 23  32  5 n 2  23  3  5 n3  2  3  5 n 4  2  3 n5  n 6 
a)

2 4 | 2 3  32  5 n 2  2 3  3  5 n 3  2  3  5 n 4  2  3 n 5  n 6  

2 | 2 3  32  5 n 2  2 3  3  5 n 3  2  3  5 n 4  2  3 n 5  n 6  
2 | n6  2 | n
Luego n  2a para cierto a entero. Pero entonces
2 3  32  5 n 2  2 3  3  5 n 3  2  3  5 n 4  2  3 n 5  n 6 
 2 3  32  5  2 2 a 2  2 3  3  5  2 3 a 3  2  3  5  2 4 a 4  2  3  2 5 a 5  2 6 a 6 
 2 5  32  5a 2  2 6  3  5a 3  2 5  3  5a 4  2 6  3a 5  2 6 a 6 

 2 5 32  5a 2  2  3  5a 3  3  5a 4  2  3a 5  2a 6 

y por tanto se verifica 2 | 2  3  5 n  2  3  5 n3  2  3  5 n 4  2  3 n5  n 6
4 3 2 2 3

b)

32 | 2 3  32  5 n 2  2 3  3  5 n 3  2  3  5 n 4  2  3 n 5  n 6  

3 | 2  3  5 n  2  3 5 n  2  3 5 n  2  3n  n 
3 2 2 3 3 4 5 6

3| n  3| n
6

Luego n  3b para cierto b entero. Pero entonces


2 3  32  5 n 2  2 3  3  5 n 3  2  3  5 n 4  2  3 n 5  n 6 
 2 3  3 2  5  3 2 b 2  2 3  3  5  33 b 3  2  3  5  3 4 b 4  2  3  35 b 5  3 6 b 6 
 2 3  34  5  b 2  2 3  34  5b 3  2  35  5 b 4  2  36 b 5  36 b 6 

 34 2 3  5  b 2  2 3  5b 3  2  3  5 b 4  2  32 b 5  32 b 6 

y por tanto se verifica 3 | 2  3  5 n  2  3  5 n  2  3  5 n 4  2  3 n5  n 6
2 3 2 2 3 3

c)
  
5 | 23  32  5 n 2  23  3  5 n3  2  3  5 n 4  2  3 n5  n 6  5 | 2  3 n5  n 6  n5 6  n 
Puesto que 4 es primo, se pueden dar dos casos:
c1) 5 | n5  5 | n
Así pues, nuestro número n es de la forma 5  2  3  5k  30k
Los números 0  30 k  2017 corresponden a los valores de k entre k  1 y k  67 . Hay un
total de 67  1  1  67 valores aceptables para n.
c2) 5 | 6  n  6  n  5k  n  5k  6
Los números 0  30k  6  2017 corresponden a los valores de k entre k  1 y k  67 . Hay un
total de 67  1  1  67 valores aceptables para n.
Así pues, tenemos un total de 67  67  134 valores aceptables para n.

6.109
Vemos que para los números producto de dos primos esto no es posible.
En efecto, si n  p q , entonces D(n)  1, p, q, pq, y no hay forma posible de colocar p, q y pq,
de forma que p y q no queden adyacentes, y  p, q  1 .

Veamos otros casos.


Si n  abc , con a,b,c primos, sí es posible: a, ac, c, bc, b, abc, ab . Vemos que cada pareja
adyacente no es de números coprimos, y tampoco lo es la primera con la última.

 
Si n  p 2q , con p y q primos, sí es posible: D(n)  1, p, p 2 , q, pq, p 2 q , y los podemos ordenar
de la forma
 
p, p 2 , pq, q, p 2q .

En general vemos que cualquier número de la forma n  p k q satisface las condiciones del
enunciado. En efecto, basta tomar la cadena
 
p, p 2 , p3 ,..., p k , pq, q, p 2q,..., p k q

Vemos que, si sabemos que todos los divisores de m se pueden ordenar satisfaciendo las
condiciones del enunciado, también lo podremos hacer con n  m p , para cualquier primo p.
En efecto, sea a1 , a2 , ..., ak  una cadena circular con todos los divisores mayores que 1 de m
satisfaciendo las condiciones del enunciado. Entonces todos los divisores mayores que 1 de
n  m p serán a1 , a2 , ..., ak  p a1 p, a2 p, ..., ak p
Y los puedo colocar en la cadena
a1, a2 , ..., ak , ak p, p, ak 1 p, ak  2 p,..., a1 p 
que cumplirá todas las condiciones del enunciado.

6.110
En primer lugar vemos que ninguno de estos primos es 2.
El número 2 es el único primo par. Supongamos que tenemos la expresión
2  p2  ...  p1012  p1013  p1014  ...  p2024
La parte de la derecha será la suma de 1012 números impares, cada pareja de impares será un
número par, luego será un número par.
La parte de la izquierda es la suma de un par más un número impar de números impares, luego
será 2 más un número impar, es decir, un número impar, llegando a contradicción.

Sean A  p1 p2 ...p1012 y B  p1013 p1014... p2024 . Sin falta de generalidad podemos suponer que
A B.
Por el Teorema Fundamental de la Aritmética es imposible que A  B .
Los números A y B son el producto de números impares, luego es imposible que A  B  1 o
A  B  3 , pues en ese caso A sería par. Solo queda demostrar que A  B  2 , y para ello
vamos a demostrar un resultado más general, sin necesidad de que sean necesariamente primos.

Lema.
a) Si a1 ,a2 , a3 , a4 son números impares, y se cumple a1  a2  a3  a4 , entonces
a1a2  a3a4 es un múltiplo de 4.
b) En general, si a1 ,..., a2 n son números impares, y se cumple a1  ...  an  an1  ...  a2 n ,
entonces a1...an  an1...a2 n es un múltiplo de 4.

Demostración.
a) Puesto que son impares, ai  2ki  1 para ciertos enteros ki
a1  a2  a3  a4  2k1  1  2k2  1  2k3  1  2k3  1  k1  k2  k3  k4  0
Luego
a1a2  a3a4  2k1  12k 2  1  2k3  12k4  1 
 4k1k 2  2k1  2k2  1  4k3k4  2k3  2k 4  1 
 4k1k2  k3k4   2k1  k 2  k3  k 4   4k1k 2  k3k 4 
b) Basta aplicar el apartado anterior a a'1  1 , a'2 a1...an , a'3  1 , a'3 an1...a2n .

Volviendo a nuestro problema, todos los pi son impares, luego A  B es un múltiplo de 4, y


como sabemos que no puede ser cero, en particular, A  B  4 .

6.111
Podemos descomponer la figura en rectángulos y cuadrados:

para deducir que el área buscada es

n2 
n  m2  n  m  n  m  2024  2n 2  n  m2  2nm  4048 
2 2 2
2n  n  2nm  m   2nm  4048  n 2  m 2  4048 
2 2 2

(n  m)(n  m)  4048  2 4  11  23
Sabemos que n-m y n+m tienen la misma paridad, luego ambos factores deben contener al
menos un “2”. También está claro que n  m  n  m , luego descartaremos las marcadas con
asterisco (*):
n  m  23  11  23 n  m  2 3  23 n  m  23  11
  
n  m  2 n  m  2  11 n  m  2  23

n  m  2  11  23
2
n  m  2 2  23 n  m  2 2  11
   (*)

n  m  2
2
n  m  2 2  11 n  m  2 2  23
n  m  21  11  23 n  m  2  23 n  m  2  11
  (*)  (*)
n  m  23 n  m  2  11 n  m  2  23
3 3

En total hay 6 soluciones posibles (B).

6.112
(C) Basta comprobar las condiciones en cada número e ir descartando.
6.113
El número obtenido tiene que ser forzosamente de la forma
6 n  10 m  2  3  2  5  2 n  3n  2 m  5m  2 nm  3n  5m
n m

y vemos que el único número que no se adapta a este formato es (B).

6.114
Supongamos que es A.
Tenemos un número divisible por 3 pero no por 6, par, diferente de 2 y compuesto.
Es imposible, porque si es par y divisible entre 3, será divisible entre 6.
Supongamos que es B.
Tenemos un número divisible por 6 pero no por 3. Esto es contradictorio.
Supongamos que es C.
Tenemos un número que no es divisible ni entre 3 ni entre 6, impar, diferente de 2 y
compuesto.
El 35 cumple las condiciones. La respuesta correcta es C.
Aunque no es necesario, veamos que el resto de las opciones no son aceptables:
Supongamos que es D. Es contradictorio con ser compuesto.
Supongamos que es E. Tenemos un número primo, que no es dos pero es par, esto es
contradictorio.

6.115
Dentro de dos años, está claro que la edad de la nieta es 2. Luego, dentro de dos años,
tendremos la ecuación
m  h  2  2024  23  11  23  m  h  2 2  11  23
m y h deben ser pares, luego la única combinación aceptable con m  h es:
m  2  23 , h  2  11
Dentro de dos años María tendrá 46 años, luego ahora tiene 44 años (B).

6.116
Vemos que el factor 13 aparece elevado a 4, luego forzosamente será, como mínimo, la
factorización de 13  4  52 , esto quiere decir que aparecerá el 17 en
17 1  17 , 17  2  34 y 17  3  51
es decir, el 17 aparecerá en la factorización de tres números, luego tendremos un 17 3 (C).

6.117
Tenemos 44  7 x  5 y , donde x es el número de días que este pingüino ha comido 7 peces e y
es el número de días en que este pingüino ha comido 5 peces.
Mediante tanteo vemos que esta ecuación se resuelve con x  2 , y  6 , luego para el otro
pingüino tendremos 7 y  5 x  7  6  5  2  52 peces (D).
6.118
Llamaremos x, y, z, t a las casillas de la figura, tal y como aparece en la siguiente imagen:

Entonces
720  xy  znnt  zn 2t , por lo que n 2 es un cuadrado perfecto dentro de la factorización de
720  2 4  32  5 . Las únicas posibilidades son
n 2  32 , n 2  4 2 , n 2  2 2 , n 2  12 , n 2  2  3 , n 2  4  3 , seis en total (D).
2 2

6.119
Está claro que un entero es divisible entre 5 sí y solo si lo es su cuadrado, luego buscamos
todos los enteros positivos múltiplos de 5 de forma que su cuadrado sea menor que 2023.
Probando números observamos que 40 2  1600 y 45 2  2025 , luego serán todos los enteros de
la forma 52 ,10 2 ,15 2 ,..., ,40 2 , es decir, (5k ) 2 con 1  k  8 , ocho números en total (A).

6.120
Pasando a común denominador la expresión de la izquierda, vemos que c  15a  14b .

La afirmación I no es cierta. Basta tomar


a  1, b  3  c  15  14  3  57 , Mcd (57 ,210 )  3

Veamos que III es cierta.


Si Mcd (c,210 )  1 , aplicando el Teorema de Bezout, existirán enteros  ,  tales que
1   15a  14b  15 14  15a  14b  15 14  15 a  14b  15
Luego, de nuevo por el Teorema de Bezout, Mcd ( a,14 )  1 .
De la misma forma, tenemos
1   15a  14b  15 14  14b  15a  15 14  14 b  15a   14
y por tanto, por el Teorema de Bezout, Mcd (b,15)  1 .

Una manera de demostrarlo alternativa sería por contrapositivo:


Supongamos que no se cumple “ Mcd (a,14 )  Mcd (b,15)  1 ”, es decir, supongamos que
Mcd (a,14 )  1  a  pa' donde p  2 o p  7 . Por ejemplo, p  2 .
c  15a  14b  15  2a'14b  215a'7b  2 | c  Mcd(c,210)  1
y con razonamientos similares demostramos cualquiera de las otras opciones.

Supongamos que, por el contrario, Mcd (c,210 )  1 . Entonces existirá un primo p tal que
 p | 15a  14b

 p | 210  2  3  5  7
Entonces p tiene que ser 2, 3, 5 o 7. Supongamos que p  2 . Entonces
 p | 3  5a  2  7b
  p | 3  5a  p | a
 p | 2  7b
y puesto que p | 14 por tanto Mcd (a,14 )  p  1 .

La afirmación II es cierta, pues III es una versión más fuerte que II.

Luego son ciertas II y III solamente (E).

6.121
Sea x  f 2023
Sabemos que 2023  7  17 2 , luego el enunciado nos dice que
 7  17 2   7  17 2   7  17 2   7  17 2 
1  x  7  f    17  f    17  f 
2
2 
  7  17  f    2023 f 1  1
 7   17   17   7  17 
Aplicando la identidad del enunciado al caso n  1
1 f 1  1  f 1  1
Luego
 
x  7  f 17 2  17  f 7 17   17 2  f 7  7 17  f 17   2023  1
Aplicando la identidad del enunciado al caso n  17 2
 17 2   17 2   17 2 
1  f    17  f    17 2  f  2   1  f 17 2   17  f 17   17 2  1
 1   17   17 
 
 7  f 17 2  7  17  f 17   7  17 2  7
Substituyendo en la ecuación anterior:
x  17  f 7 17   17 2  f 7  7  7 17 2  2023  1
Aplicando la identidad del enunciado al caso n  7 17
 7 17   7 17   7  17   7 17 
1 f    17  f    7 f    7 17  f   1
 1   17   7   7 17 
1  f 7 17   17  f 7   7  f 17   7  17  1 
17  f 7 17   17 2  f 7   17  17  7  f 17   7 17 2
Substituyendo en la ecuación anterior:
x  17  17  7  f 17   7 17 2  7  7 17 2  2023  1
Aplicando la identidad del enunciado al caso n  17
 17   17 
1  f    17  f    1  1  f 17   17  1  f 17   1  17  16
1  17 
Substituyendo en la ecuación anterior:
x  17  17  7   16   7  17 2  7  7  17 2  2023  1 
x  1  17  17  7   16   7  17 2  7  7  17 2  2023  96

7.1.1
a) 7  7 b) 23  23 c) 29  5 d) 168  0 e) 773  5

7.1.2
a) 15  15 b) 21 21 c) 30  6 d) 200  8 e) 1441 1
7.1.3
a) 9 b) 23 c) 0 d) 14

7.1.4
a) 5 b) 9 c) 13 d) 15

7.1.5
a) 1 b) 11 c) 14 d) 1 e) 3 f) 0 g) 8 h) 16

7.1.6
a) 2 b) 6 c) 3 d) 4 e) 8

7.1.7
2023  289  7  0 , luego volverá a a ser jueves (C)

7.2.1

7.2.2

7.6.1
Si n  6 , está claro que los factores 3 y 6 están dentro de n ! , luego 6  3  9  2 | n! .
Por lo tanto, solo nos tenemos que ocupar de 1! 2! 3! 4! 5! 153  17  9
Así pues:
1! 2! 3! ...  5! 0 (mod 9)
  1! 2! 3! ...  n! 0  0  0 (mod 9)
6! 7! ...  n! 0 (mod 9) 
Y por tanto el residuo al dividirlo entre 9 es 0.

7.6.2
Por ejemplo: 62  42 (mod 5) , pero sin embargo, 6  4 (mod 5)

7.6.3
25  32  4  22 (mod 7)

  2 50
 2 
5 10
 2 
2 10
(mod 7 )  2 
5 4
(mod 7 )  2  (mod 7) 
2 4

24  16  2 (mod 7) 

  2
 24 (mod 7)  22 (mod 7)  4 (mod 7) , luego el residuo es 4.
41  6 (mod 7)  1 (mod 7)  4165 (mod 7)  (1)65 (mod 7)  1 (mod 7)  6 (mod 7) , luego el
residuo es 6.

7.6.4
  4
211  2048  1 (mod 89 )  2 44  2 411  211  14 mod 89  1 mod 89 
2 44  1 (mod 89 )  1  1 (mod 89 )  0 (mod 89 )  89 | 2 44  1

212  4096  22 (mod 97 )  2 24  212212  22  22 (mod 97 ) 


 484 (mod 97 )  96 (mod 97 )  1 (mod 97 ) 
 
2 48  2 24 (mod 97 )   1 (mod 97 )  1 (mod 97 ) 
2 2

2 48  1 (mod 97 )  1  1 (mod 97 )  0 (mod 97 )  97 | 2 48  1

7.6.5
9  1mod 10  91003  (1)1003 mod 10  1mod 10  9 mod 10
72  49  1 mod 10  7902  72  451
  1
451
mod 10  1 mod 10  9 mod 10
32  9  1 mod 10  3801  3  3800  3  32   400
 3 mod 10  32 400
mod 10 
 3 mod 10   1 mod 10  3 mod 10  1 mod 10  3 mod 10
400

Luego 91003  7902  3801 mod 10  9  9  3 mod 10  3 mod 10 , luego acaba en 3

7.6.6
11n  2  11n  112 
 n2
  11  12  11 mod 133
n

11  121  12 mod 133 


2

 
12 2 n 1  12 2 n  12  12 2  12 
n

  12  11 mod 133
n

12  144  11 mod 133


2


n2 2 n 1
an  11  12  12 11  12 11n mod 133  0 mod 133  133 | 11n  2  12 2 n 1
n

7.6.7
Está claro que 2137 2 acaba en 9, luego:
2137 2  9 mod 10  1 mod 10  2137 2  
376
  1
376
 1 mod 10

Por otro lado, está claro también que 2137  7 mod 10

Por lo tanto
2137 753  2137 752  2137  2137 2376  2137  2137 2  
376
 2137  1 7 mod 10  7 mod 10 ,
Luego acaba en 7.

Nota: Este problema también se puede resolver sin congruencias, observando en qué dígito van
acabando las potencias de 2137 n :
n  0 1
n 1 7
n29
n 33

Y para exponentes mayores entramos en un bucle.

7.6.8
Primera versión.
Estudiamos el dígito de las unidades de 31001:
30  1 , 31  3 , 32  9 , 33  27 , 34  81 , 35  253 , 36  759 ...

Vemos que se va repitiendo en grupos de 4: {1,3,9,7} , y puesto que 1001  1mod 4 ,


31001 acabará como 31 , es decir en 3.

El mismo análisis hacemos para estudiar el dígito de las unidades de 71002 :


70  1 , 71  7 , 72  49 , 73  343 , 74  2401 ,...
Vemos que se va repitiendo en grupos de 4: {1,7,9,3} , y puesto que 1002  2 mod 4 ,
71002 acabará como 7 2 , es decir en 9.

Finalmente, estudiemos el dígito de las unidades de 131003 :


130  1 , 131  13 , 132  169 , 133  2197 , 134  28561 ,...
Vemos que se va repitiendo en grupos de 4: {1,3,9,7} , y puesto que 1003  3 mod 4 ,
131003 acabará como 13 3 , es decir en 7.

El producto de un número acabado en 3, un número acabado en 9 y un número acabado en 7 es


un número acabado en 9.

Segunda versión.
Vamos a aprovechar el hecho de que toda potencia de un número acabado en 1 acaba siempre
en 1. En nuestro caso, cualquier potencia de 7  13  91 acaba en 1, y cualquier potencia de
34  81 acaba en 1. Luego:

3100171002131003  13  3100171002131002  13  310017  13   13  31001911002 


1002

 
 13  3  31000911002  13  3  34250911002  13  3  34
250
911002 
 13  3  81250911002  39  3  81250911002

Que claramente acabará en 9.

Fuente de esta versión: The Contest Problem Book V 198S-1988, pág. 62

7.6.9
  2
Para k  1 : 183  5832  833  7  1  186  183  12  1 (mod 7)
Para k  2 : 183  5832  12  49  1  186  18 
3 2
 12  1 (mod 49)
  2
Para k  3 : 183  5832  2  2401  1  186  183  12  1 (mod 2401)

Observación: Sin embargo, para k  4 cambia la pauta: 183  1030 (mod 74 )

7.6.10
Si n es impar, entonces n  2k  1 , y por tanto n2  4k 2  4k  1  4k (k  1)  1
k o k  1 es par, luego k ( k  1) es par, y por tanto 4k ( k  1) es un múltiplo de 8. Luego
n2  4k (k  1)  1  0  1  1 (mod 8) .

7.6.11
0  n2  3n  2  (n  1)(n  2) (mod 6)
La tabla de congruencias módulo 6 es suficientemente pequeña para enunciar todas las
combinaciones posibles:
0  2  3  5  6  6  1 (mod 6) , y por tanto: (n  1)  2, 5, 6 (mod 6)  n  1,4, 5 (mod 6)
Y cumpliendo la condición 1  n  25 , tenemos:
n  1(mod 6)  n  1,7,13,19,25
n  4 (mod 6)  n  4,10,16,20
n  5 (mod 6)  n  5,11,17 ,23
En total, 5  4  4 posibilidades.

7.6.12
Primera versión.
Vemos el comportamiento de 2 n  6  9 n para los primeros valores de n:
Antes de nada, observamos que 6  1 (mod 7)
n  1  21  2 6  91  9  2 (mod 7)  2n  6  9n  2  2  0 (mod 7)
n  2  22  4 6  92  81  4 (mod 7)  2n  6  9n  4  4  0 (mod 7)
n  3  23  8  1 (mod 7) 6  93  729  1 (mod 7)  2n  6  9n  1  1  0 (mod 7)

Y, a partir de este valor, se van repitiendo la pauta:


n  4  24  232  1  2 (mod 7)  2 (mod 7)
6  94  93  9  1  9  9 (mod 7)  2n  6  9n  2  2  0 (mod 7)
n  5  25  2322  1  22 (mod 7)  22 (mod 7)
6  95  93  92  1  92  92 (mod 7)  2n  6  9n  4  4  0 (mod 7)
n  6  26  2323  1  23 (mod 7)  23 (mod 7)
6  96  93  93  1  93  93 (mod 7)  2n  6  9n  1  1  0 (mod 7)
Y siempre llegamos al mismo resultado: 2n  6  9n  0 (mod 7) .

Segunda versión.
Partimos de la igualdad 9  2 (mod 7) , luego 9n  2n (mod 7) , y por tanto:
2n  6  9n  9n  6  9n  7  9n  0  9n  0 (mod 7)

7.6.13
Primera versión.
Vamos a calcular 6  8 (mod 49) directamente, mediante el método de las potencias de
83 83

dos:
61  6 (mod 49 )
62  36 (mod 49 )
64  36  36  1296  22 (mod 49 )
68  22  22  484  43 (mod 49 )
616  43  43  1849  36 (mod 49 )
632  36  36  1296  22 (mod 49 )
664  22  22  484  43 (mod 49 )
83  64  16  2  1  683  664616626  43  36  36  6
 43  22  6  15  6  41 (mod 49 )

81  8 (mod 49 )
82  64  15 (mod 49 )
84  15  15  225  29 (mod 49 )
88  29  29  841  8 (mod 49 )
816  8  8  64  15 (mod 49 )
832  15  15  225  29 (mod 49 )
864  29  29  841  8 (mod 49 )
83  64  16  2  1  883  864816828  8  15  15  8 
 15  29  435  43 (mod 49 )
Finalmente, 6  8  41  43  84  35 (mod 49)
83 83

Segunda versión.
Aplicando el Teorema del Binomio:
 n  n  n  1 n 1
(a  b) n  a n   a n 1b1   a n  2b 2  ...   a b  b n
1  2  n  1

Aplicado a nuestro caso, y teniendo en cuenta que n  83 es impar, y por tanto (1) n  1 ,

n n  n 
6n  (7  1) n  7 n    7 n 1    7 n  2  ...    7  1
1  2  n  1
n n  n 
8n  (7  1) n  7 n    7 n 1    7 n  2  ...    7  1
1  2  n  1

Y por tanto:
n n  n  3  n 
6n  8n  7 n    7 n  2    7 n  4  ...    7    7 
 2  4  n  3  n  1
 n n  n  3 
 2 7 n    7 n  2    7 n  4  ...    7  7n  
  2  4  n  3 
 n n  n  
 2  7 2  7 n  2    7 n  4    7 n  6  ...    7   14 n  49 k  14 n
  2   4   n  3  

En particular, para n  83, a83  683  883  49 k  14  83  14  83 (mod 49 )


Luego solo nos queda calcular este último residuo:

14  83  1162  49  23  35  14  83  35 (mod 49 )

Fuente de la segunda versión: The Contest Problem Book V 1983-1988 (George Berzsenyi, 1997).

Tercera versión.
Aplicando el Teorema de Euler (que se introducirá en el apartado 13.2)
 1
49  7 2   (49 )  491    42 , y a 42  1 (mod 49) si (a, n)  1 .
 7
En nuestro caso (6,49 )  (8,49 )  1 , y por tanto:
  2
642  1 (mod 49)  683  62421  642 61  61 (mod 49)
842  1 (mod 49)  883  82421  8  8
42 2 1
 81 (mod 49)
1 1 8  6 14
683  883  61  81      14  35 (mod 49)
6 8 48 1
Fuente de esta versión: [Link]
7.6.14
 
9  99  999  ...  99 ...9  9  99  (1000  1 )  (10000  1)  ...  100 ...00  1 
   
999 nueves  999 ceros 

 9  99  (10 3  1)  (10 4  1)  ...  10 999  1 
Y observamos que si k  3  10 k es múltiplo de 1000, y por tanto 10 k  0 (mod 1000 ) , luego
10 k  1  0  1  1 (mod 1000 )
Y por tanto:
 
(10 3  1)  (10 4  1)  ...  10999  1  (1)99931  (1)997  1 (mod 1000 )
Y, finalmente,
 
9  99  (10 3  1)  (10 4  1)  ...  10 999  1 mod( 1000 )  9  99 (1) mod( 1000 ) 
 891 mod( 1000 )  109 mod( 1000 )

7.6.15
Calculamos directamente los primeros valores de f (n) :
f (0)  0 , f (1)  2 , f (3)  6 , f ( 4)  14 , f (5)  30 , f (6)  62
Analizando cómo se obtiene f (n) llegamos a la conclusión de que
f (n)  2 f (n  1)  2(n  1)  2n  2 f (n  1)  2

Que es el típico comportamiento de una función exponencial. Mirando los primeros valores
vemos que un buen candidato puede ser f (n)  2n  2 .
Lo vamos a demostrar por inducción:
Para n  1,2,3 es cierto.
Suponiendo que f (n)  2n  2 , entonces
f (n  1)  2 f (n)  2  2(2n  2)  2  2n 1  4  2  2n 1  2 .
Luego es cierto para todo n.

f (100 )  2100  2 , y queremos determinar 2100  2 (mod 100 ) .


Calculamos 2100 (mod 100 ) con el “Método de las potencias de 2”:
22  4 (mod 100 )
24  16 (mod 100 )
28  16  16  256  56 (mod 100 )
216  56  56  3136  36 (mod 100 )
232  36  36  1296  96  4 (mod 100 )
264  (4)  (4)  16 (mod 100 )
Luego
2100  264 32 4  26423224  16  (4)  16 (mod 100 )  56  (4) (mod 100 )
 224 (mod 100 )  24 (mod 100 )  24 (mod 100 )  76 (mod 100 )

Finalmente: 2100  2  76  2 (mod 100 )  74 (mod 100 ) , y el residuo pedido es 74.

7.6.16
Queremos determinar k 2  2k (mod 10) . Estudiemos los residuos 2n (mod 10) :
21  2 (mod 10 )
22  4 (mod 10 )
23  8 (mod 10 )
24  4  4  16  6 (mod 10 )
25  6  2  12  2 (mod 10 )
26  2  2  4 (mod 10 )
27  4  2  8 (mod 10 )
28  8  2  6 (mod 10 )
...
En general: 2n (mod 10) genera un ciclo 2,4,8,6 , y en particular, para todo n  4k múltiplo
de 4, 24k  6 (mod 10) .

En particular, en nuestro caso:


4 | 2008  4 | 2008 2 

  4 | 2008  2  k
2 2008

2 2
2008 2 4 251
 
2 4 251
 2 4 4 251
 4| 2 
2008

k es un múltiplo de 4, y por tanto 2  6 (mod 10) .
k

También vemos que 2008 es múltiplo de 4, luego 22008  6 (mod 10) .


Por otro lado, 2008  8 (mod 10)  2008 2  82  64  4 (mod 10)
Y por tanto k  2008 2  22008  4  6  10  0 (mod 10)  k 2  02  0 (mod 10)
Finalmente, k 2  2k (mod 10)  0  6  6 (mod 10) , y el dígito de las unidades es 6.

Fuente de la solución: [Link]

7.6.17
n y 107 n tienen las dos últimas cifras iguales si y solo si
107 n  n (mod 100 )  7n  n (mod 100 )  6n  0 (mod 100 )  6n  100 k
 3n  50 k
De aquí deducimos que n es un múltiplo de 50. El múltiplo de 50 más pequeño es n  50 y ya
satisface la condición del enunciado, pues 107  50 acaba en 50.

7.6.18
Pasando a módulo 3 la ecuación se convierte en x 2  y 2  0
Hacemos la tabla x 2  y 2 (mod 3)

x0 x 1 x2
y0 0 1 1
y 1 1 2 2
y2 1 2 2

Vemos que la única posibilidad es x  0 , y  0 , es decir, x, y múltiplos de 3. Pero entonces:


2 2
   
3z 2  x 2  y 2  3x'  3 y'  9 x'2 9 y'2  9 x'2  y '2  z 2  3 x'2  y'2 , es decir, z 2 es
múltiplo de 3, y por tanto z es múltiplo de 3.

Pero entonces x 2  y 2  3z 2  3x'  3 y'  33z '  9x'2  y'2   9  3z '2  x'2  y'2  3z '2
2 2 2

Este proceso lo puedo repetir una y otra vez, y esto solo puede ocurrir si x  y  z  0 .

7.6.19
n  18 2  n2  36 n  324 .
Realizamos la división sintética n 2  36 n  324 entre n  2 obtenemos un cociente igual a
n  34 y un residuo igual a 256, luego n  18   256 (mod n  2)
2

Así pues
n  2 | n  18   n  18   0 (mod n  2)  256  0 (mod n  2)
2 2

 n  2 | 256  28

Luego n  2 es una potencia 2 k , 0  k  8


1  n  1 , 21  2  n  0 , 2 2  4  n  2 , 23  8  n  6 , 2 4  16  n  14 ,
25  32  n  30 , 26  64  n  62 , 2 7  128  n  126 , 28  256  n  254
Se comprueba que todos estos valores satisfacen la condición del enunciado.

7.6.20
Observamos la secuencia de potencias 2n (mod 7) :
21  2 (mod 7)
22  4 (mod 7)
23  1 (mod 7)
...
Siempre es 1, 2 o 4 y por tanto 2n  1 será congruente con 2 , 3 o 5, nunca con 0.

7.6.21
72  49  1 mod 10  , luego 7 4   1  1 mod 10  , y por tanto
2

77  7  73  74  7  (1)  1  7mod 10 
 
Y por tanto 77   7   1 7   7  7 mod 10 
7 7 7 7

Así pues, por cada pareja de potencias de 7 el resultado es 7, y como elevamos 1000 veces, que
es par, el resultado es 7.

7.6.22
Está claro que pasando estos números a módulo 3 solo pueden haber tres diferentes, luego
forzosamente encontraremos una pareja equivalente, digamos, por ejemplo, que
a  b (mod 3)  3 | a  b

Y ya tenemos un factor divisible entre 3 en el producto del enunciado.

De la misma forma, pasando a módulo 2 solo pueden haber 2 números diferentes, luego puede
suceder uno de los siguientes dos casos:
a  b (mod 2)  2 | a  b
a) 
b  c (mod 2)  2 | b  c
a  b (mod 2)  2 | a  b
b) 
c  d (mod 2)  2 | c  d

En todo caso, encontramos dos factores diferentes múltiples de 2, con lo que el resultado será
múltiple de 4. Puesto que 3 y 4 son coprimos, el producto del enunciado será múltiple de
3  4  12 .

7.6.23
Primera versión.
Aplicando el criterio de divisibilidad del 3 y del 9.
0  1  2  3  4  5  6  7  8  9  45
Agrupando dígitos en grupos, vemos que la suma de dígitos de N es:
7  45  10  2  3  4  5  6  7  8  1  9  9  0  9  1  9  2  705
705 es divisible entre 3 pero no entre 9, luego k  1 .
Segunda versión.
Aplicamos el criterio general N  S ( N )  S (S ( N ))  S (705)  12  3 mod 9 , y por tanto es
divisible entre 3 pero no entre 9, luego k  1 .

7.6.24
Supongamos que el año N es de 365 días, es decir, no es bisiesto.
300  6 mod 7
y el día 200 del año N  1 es el día 300  65  200  765  5 mod 7 , lo cual no puede ser.
Por lo tanto deducimos que el año N es bisiesto, de 366 días. Ahora sí es coherente:
300  66  200  765  6 mod 7

El día 100 del año N  1 es el día -265 respecto al primero, y  265  1 mod 7 , que
corresponde a un jueves.

7.6.25
n 2  3n  2  (n  1)(n  2)
6 | (n  1)(n  2)  (n  1)(n  2)  0 mod 6
Las únicas posibilidades son:
n  1  0 mod 6  n  1  5 mod 6  n  5 , 11 , 17 , 23
n  2  0 mod 6  n  2  4 mod 6  n  4 , 10 , 16 , 22
n  1  2 mod 6  n  1 mod 6
   n  1 mod 6  n  1 , 7 , 13 , 19 , 25
n  2  3 mod 6 n  1 mod 6
n  1  3 mod 6  n  2 mod 6
  no existe.
n  2  2 mod 6 n  0 mod 6

7.6.26
Aplicando el criterio de divisibilidad del 3, 7  4  A  5  2  B  1  A  B  19 es múltiple de
3, y 3  2  6  A  B  4  C  A  B  C  15 es múltiple de 3, luego su resta también lo será:
A  B  19   A  B  C  15  4  C  0 mod 3  C  4  1 mod 3
Los únicos dígitos aceptables son C  1,4,7 y su suma es 12.

7.6.27
Sabemos que n  ck ck 1 ck 2 ... c1 c0 es equivalente a
n  ck 10 k  ck 110 k 1  ck 210 k 2  ...  c110  c0
Es decir,
n  p (10 ) con p( x)  ck x k  ck 1 x k 1  ck 2 x k 2  ...  c1 x  c0 .

Y que las suma de sus cifras será S  p (1) .


Por otro lado, 9  10  1  9 | 10  1  9 | p(10 )  p(1)  n  S  n  S (mod 9)

Luego 9 | n  n  0 (mod 9)  S  0 (mod 9)  9 | S

Sabemos que n  ck ck 1 ck 2 ... c1 c0 es equivalente a


n  ck 10 k  ck 110 k 1  ck 210 k 2  ...  c110  c0
Es decir,
n  p (10 ) con p( x)  ck x k  ck 1 x k 1  ck  2 x k  2  ...  c1 x  c0 .

Y que la suma alternada de sus cifras es T  p (1) .

Por otro lado, 10  1 (mod 11)  p(10 )  p(1) (mod 11) , es decir n  T (mod 11) .
Luego 11 | n  n  0 (mod 11)  T  0 (mod 11)  11 | T

7.6.28
Pasamos los números del 1 al 90 a módulo 3, y vemos que se separan entre el 0 y el 1:

Puesto que 39  0 mod 3 y 40  1 mod 3 , al colocarse en la segunda columna quedarán


separados de la siguiente forma:

Y por tanto habrá 4 números entre ellos (E).

7.6.29
Estudiando como se construye esta pirámide de números es fácil ver que la suma S n de los
números de la fila n, es igual a
Sn  2Sn 1  n
Luego S n será una función cercana a una potencia de 2. En efecto,
S n  2n  n
Para determinar la cifra de las unidades de S 2023 vemos que las potencias de 2 van haciendo un
ciclo de 4 valores:
20  1 , 21  2 , 22  4 , 23  8 , 24  16 , 25  32 , 26  64 , ...
2023  4  500  3
Luego 2 2023 acaba igual que 2 3 , en ocho, y por tanto S 2023 acaba igual que 8-3 , es decir, en 5
(C).

8.1.1
11  1 mod 7 , 2 1  4 mod 7 , 31  5 mod 7 , 4 1  2 mod 7 , 5 1  3 mod 7 ,
6 1  6 mod 7 
8.1.2
Escribimos la tabla de multiplicar de Z 6 y marcamos los unos:

Podemos ver claramente que los valores invertibles son 1, 5, 7 y 11, cuyos repectivos inversos
son ellos mismos: 1, 5, 7 y 11.

8.1.1
d  (9,30 )  3 , y 3 | 21 , luego la ecuación anterior tendrá 3 soluciones diferentes.

Encontramos la primera solución por tanteo:


x0  9 , pues 9  9  81  2  30  21 .
Luego el resto de soluciones serán:
30
x1  9   1  9  10  19 , efectivamente: 9 19  171  5  30  21
3
30
x2  9   2  9  20  29 , efectivamente: 9  29  261  8  30  21
3

8.1.2
(3,10 )  1 , luego la congruencia lineal anterior tendrá una única solución módulo 10.
Aunque no sea la más elegante, una manera de resolverla es ir probando números del 1 al 9:
3  1  3  7 mod 10 , 3  2  6  7 mod 10 , 3  3  9  7 mod 10 ,
3  4  12  2  7 mod 10 , 3  5  15  5  7 mod 10 , 3  6  18  8  7 mod 10 ,
3  7  21  1  7 mod 10 , 3  8  24  4  7 mod 10 , 3  9  27  7 mod 10 .

Luego la solución es x  9 .

8.3.1
244  2  117  10
117  11  10  7
10  1  7  3
7  2 3 1

1  7  2  3  7  2  10  7   7  2  10  2  7  3  7  2  10  3  117  11  10   2  10 
 3  117  33  10  2  10  3  117  35  10  3  117  35  244  2  117  
 3  117  35  244  70  117  73  117  35  244

Luego
1  73  117  35  244  73  117 mod 244   117 1  73 mod 244 

9.1.1
a) Resolvemos la congruencia sumando 4 a ambos lados de la igualdad:
x  4  0 mod 5  x  4  4  0  4 mod 5  x  4 mod 5
b) Resolvemos la congruencia sumando 1 a ambos lados de la igualdad:
x  1  1 mod 5  x  1  1  1  1 mod 5  x  2 mod 5
c) Resolvemos la congruencia restando 3 a ambos lados de la igualdad:
x  3  1 mod 5  x  3  3  1  3 mod 5  x  2  2  5  3 mod 5
d) Resolvemos la congruencia restando 12 a ambos lados de la igualdad:
x  12  3 mod 5  x  12  12  3  12 mod 5  x  9  9  10  1 mod 5

9.1.2
Observamos la tabla de multiplicar de Z 5 :

a) El inverso multiplicativo de 3 es 2, luego multiplicamos a ambos lados por 2:


3x  1 mod 5  2  3x  2 1 mod 5  1x  2 mod 5  x  2 mod 5

b) De la misma manera: 3x  2 mod 5  2  3x  2  2  4 mod 5  x  4 mod 5


c) El inverso multiplicativo de 2 es 3, luego:
2x  3 mod 5  3  2x  3  3 mod 5  x  9  4 mod 5

d) 12  2 mod 5  12x  2x mod 5 y la congruencia queda:


2x  4 mod 5  3  2x  3  4 mod 5  x  12  2 mod 5
e)
2 x  4  2 mod 5  2 x  4  4  2  4 mod 5  2 x  6  1 mod 5 
3  2 x  3 1 mod 5  x  3 mod 5

9.2.1
Primera versión.
Un número n da residuo 5 cuando lo dividimos entre 6, 7, 8 y 9 si n  5 es divisible entre 6, 7,
8 y 9. Luego n  5 será divisible entre el mínimo común múltiplo de 6, 7, 8 y 9, que es 504.
Así pues, n  5  504 a  n  504 a  5 para cierto a.
Vamos dando valores de a  0, 1, 2, 3... y vemos que para a  4 , n  504  4  5  2021 ya es
demasiado grande. Luego hay 4 números (D).

Segunda versión. (un método más general)


Nos vamos a entrar en las divisiones entre 7, 8 y 9, por ser los tres coprimos.
Sea n un número que, dividido entre 7, da residuo 5. Luego n se podrá escribir de la forma
n  7a  5 para cierto entero a.
Pero este mismo número da residuo 5 cuando lo dividimos entre 8, luego n  7a  5  8b  5
para cierto entero b.
Luego 7 a  5  8b  5  7 a  8b  7 | 8b , y puesto que (7,8)  1 , se deduce que 7 | b , es decir,
b  7k para cierto entero k. Así pues, n  8b  5  8  7k  5  56k  5 .
Este número da residuo 5 cuando lo dividimos entre 9, es decir:
n  56k  5  9c  5 para cierto entero c, luego
56 k  5  9c  5  56 k  9c  9 | 56 k
De nuevo, puesto que (9,56 )  1 , se deduce que 9 | k , es decir, k  9q para cierto entero q.
Así pues,
n  56 k  5  56  9q  5  504 q  5

Con esto ya hemos conseguido una criba suficientemente fina para determinar todos los
candidatos posibles. Puesto que 6  2  3 , y el 2 lo hemos incluido en el 8, y el 3 en el 9, no nos
tenemos que preocupar por él. O dicho de otro modo, n  504 q  5  84  6q  5 , y todos estos
números darán 5 como residuo al dividirlos entre 6.
Puesto que 2021  504  4  5 , los valores de q aceptables estarán entre 0 y 3, es decir, habrán 4
números cumpliendo las condiciones del enunciado (D).

9.2.2
N  9a  1  10b  3  9a  10b  2  9b  b  2  9 | b  2  b  2  9k  b  9k  2
Luego n  10b  3  10 (9k  2)  3  90 k  20  3  90 k  17

Vemos que el único valor aceptable de dos dígitos es para k  1  N  90  1  17  73 , y el


residuo de 73 al dividirlo entre 11 es 7 (E).

9.2.3
n  6a  2  9b  5  11c  7
para ciertos enteros a, b, y c.
94 992
101  11c  7  999  94  11c  992  8  c  91  9  c  90
11 11

Puesto que 6a  2  11c  7  6a  11c  5  6c  5c  5  6c  5(c  1)

Luego c  1 tiene que ser un múltiple de 6.

De la misma manera, puesto que


9b  5  11c  7  9b  11c  2  9b  2c  2  9b  2(c  1)

Luego c  1 tiene que ser un múltiple de 9.


Así pues, c  1 será un multiple del mínimo común múltiple de 6 y 9, es decir, 18.
Así pues, c  1  18k para cierto k, y por tanto c  18k  1, y
n  11c  7  1118k  1  7  198k  11  7  198k  4
Para k  0  198 k  4  4 ,
k  1  198  1  4  194
k  2  198  2  4  392
...
k  5  198  5  4  986
k  6  198  6  4  1184 es excesivo.
Luego hay 5 casos aceptables (E).

9.3.4
y | x  x  0 mod y 
y  1 | x  1  x  1  0 mod y  1  x  1 mod y  1  x  y mod y  1
Puesto que  y, y  1  1, aplicando el TCR sabemos que el sistema
 x  0 mod x 

 x  y mod y  1
tendrá solución única y  a mod y( y  1) . Esta solución es x  y mod y( y  1) .
En efecto, está claro que x  y mod x( x  1)  x  y mod x  1, y
x  y mod x( x  1)  x  ay( y  1)  y  ay  a  1y  0 mod y 

Ahora vamos a ir observando los casos uno por uno:


a) y  1  x  1 mod 2  1  2a  1  100  1  a  49  49 casos.
b) y  2  x  2 mod 6  1  6a  2  100  1  a  16  16 casos.
c) y  3  x  3 mod 12  1  12a  3  100  1  a  8  8 casos.
d) y  4  x  4 mod 20  1  20a  4  100  1  a  4  4 casos.
e) y  5  x  5 mod 30  1  30a  5  100  1  a  3  3 casos.
f) y  6  x  6 mod 42  1  42a  6  100  1  a  2  2 casos.
g) y  7  x  7 mod 56  1  56a  7  100  1  a  1  1 caso.
h) y  8  x  8 mod 72  1  72a  8  100  1  a  1  1 caso.
i) y  9  x  9 mod 72  1  90a  9  100  1  a  1  1 caso.
Para y  10  x  10 mod 110  1  110a  10  100  a  0  x  y  10 y ya no se
cumple y  x .
El total es 49+16+8+4+3+2+1+1+1=85 casos.

9.4.4
En primer lugar vemos que si n  0 mod 2 , es decir, n es par entonces n mod 4  2 , pues
n  1, 3 mod 4 implicaría n impar, y n  0 mod 4 porque no se pueden repetir los módulos.
Con el mismo argumento, n  0 mod 2  n  3,5 mod 6 .

De la misma forma, n  1 mod 2  n  0,2 mod 4 , n  1 mod 2  n  0,2 mod 4 .


También vemos que n  5 mod 6  n  6k  5  n  2 mod 3 .

Haciendo el árbol de todas las posibles combinaciones, y descartando las ramas incompatibles
anteriores, vemos que solo quedan tres combinaciones aceptables:
a) n  0 mod 2, n  1 mod 3, n  2 mod 4, n  3 mod 5, n  4 mod 6
b) n  1 mod 2, n  2 mod 3, n  3 mod 4, n  0 mod 5, n  5 mod 6
c) n  1 mod 2, n  2 mod 3, n  3 mod 4, n  4 mod 5, n  5 mod 6

a)
n  4 mod 6  n  0 mod 2, n  1 mod 3 , luego el sistema a resolver es:
n  2 mod 4, n  3 mod 5, n  4 mod 6 .
n  2 mod 4  n  4a  2
n  3 mod 5  n  5b  3
n  4 mod 6  n  6c  4
Resolvemos el sistema de la segunda con la tercera congruencia:
n  5b  3  b  5 (mod 6)  b  6b'5
  5b  3  6c  4  5n  6c  1   
n  6c  4 c  4 (mod 5)  c  5c'4
n  5b  3  5(6b'5)  3  30b'28 

n  6c  4  6(5c'4)  4)  30 c'28 
Y ahora resolvemos con la primera congruencia:
n  4a  2  a  15 a'14
  4a  2  30 c'28  4a  30b'  26   
n  30 c'28  b'  2b' '1
n  415a'14   2  60 a'58 
  n  60 k  58
n  30 (2b' '1)  28  60b' '58
Los números de la forma n  60k  58 entre 1 y 999 corresponden a los valores entre
k  0  n  58 
  16 números en total.
k  15  n  60 15  58  958 

b)
n  5 mod 6  n  1 mod 2, n  2 mod 3 , luego el sistema a resolver es:
n  3 mod 4, n  0 mod 5, n  5 mod 6 .
n  3 mod 4  n  4a  3
n  0 mod 5  n  5b
n  5 mod 6  n  6c  5
Tomando la segunda y la tercera congruencia:
n  5b  b  6b'1
  5b  6c  5  5b  6c  5   
n  6c  5 c  5c'
n  5(6b'1)  30b'5 

n  6(5c' )  5  30 c'5
Ahora tomamos este resultado con la primera congruencia:
n  4a  3  a  15 a '8
  4a  3  30 c'5  4a  30 c'  2   
n  30 c'5 c'  2c' '1
n  4(15 a'8)  3  60 a '32  3  60 a'35 
  n  60 k  35
n  30 (2c' '1)  5  60 c' '30  5  60 c' '35 
Los números de la forma n  60k  35 entre 1 y 999 corresponden a los valores entre
k  0  n  35 
  17 números en total.
k  16  n  60 16  35  995 

c)
n  1 mod 2, n  2 mod 3, n  3 mod 4, n  4 mod 5, n  5 mod 6
n  5 mod 6  n  1 mod 2, n  2 mod 3 , luego el sistema a resolver es:
n  3 mod 4, n  4 mod 5, n  5 mod 6 .
n  3 mod 4  n  4a  3
n  4 mod 5  n  5b  4
n  5 mod 6  n  6c  5
Tomando la segunda y la tercera congruencia:
n  5b  4 b  6b'5
  5b  4  6c  5  5b  6c  1   
n  6c  5  c  5c'4
n  5(6b'5)  4  30b'29 

n  6(5c'4)  5  30 c'29 
Ahora tomamos este resultado con la primera congruencia:
n  4a  3  a  15 a '14
  4a  3  30 c'29  4a  30 c'  26   
n  30 c'29  c'  2c' '1
n  4(15 a '14 )  3  60 a '59 
  n  60 k  59
n  30 (2c' '1)  29  60 c' '59 
Los números de la forma n  60k  59 entre 1 y 999 corresponden a los valores entre
k  0  n  59 
  16 números en total.
k  15  n  60 15  59  959 

En total hay 16+17+16=49 números que satisfacen la condición del enunciado.

9.6.2
Sea n  a b c d .
7 | 1b c d  7 | 1000  100 b  10 c  d  1000  100 b  10 c  d  0
 6  2b  3c  d  0
7 | a1c d  7 | 1000 a  100  10 c  d  1000 a  100  10 c  d  0
 6a  2  3c  d  0
7 | a b 1d  7 | 1000 a  100 b  10  d  1000 a  100 b  10  d  0
 6a  2b  3  d  0
7 | a b c1  7 | 1000 a  100 b  10 c  1  1000 a  100 b  10 c  1  0
 6a  2b  3c  1  0

Con lo que nuestro problema se reduce a resolver el siguiente sistema de cuatro congruencias
(que siempre serán módulo 7):
6  2b  3c  d  0
6a  2  3c  d  0


6a  2b  3  d  0
6a  2b  3c  1  0
Restando la primera a la segunda:
6a  2b  4  3a  b  2  b  3a  2
Substituyendo en la tercera:
6a  23a  2  3  d  0 
6a  6a  4  3  d  0 
12 a  1  d  0 
d  1  12 a  1  5a
Substituyendo en la cuarta:
6a  23a  2  3c  1  0 
6a  6a  4  3c  1  0 
12 a  3  3c  0  4a  c  1  c  1  4a
Finalmente, subtituyendo en la primera:
23a  2  31  4a   1  5a  1 
6a  4  3  12 a  1  12 a  1 
 18 a  1  4a  1  3a  1  a  5
Puesto que sabemos que 0  a  9 llegamos a a  5
Luego
b  3  5  2  13  6  b  6
c  1  4a  1  4  5  19  2  b  2 , 9
Puesto que buscamos el máximo, tomaremos b  9
d  1  12  5  1  60  59  4  d  4
El número buscado es 5694, y la respuesta correcta es 5  694  699 .

10.1.7
   1  3 es divisible por p es equivalente a    1  3  0 mod p , es decir, nuestro
problema se reduce a demostrar que la congruencia xx  1  3  0 mod p tiene solución si y
solo si la congruencia y y  1  25  0 mod p tiene solución.

Supongamos, en primer lugar, que p  3 . Completando cuadrados en la primera congruencia


cuadrática:
xx  1  3  0 mod p  
x 2  x  3  0 mod p  
4 x 2  4 x  12  0 mod p  
(2 x) 2  2  2 x  12  12  12  0 mod p  
2 x  12  12  12  0 mod p  
2 x  12  11 mod p 
y esta congruencia tendrá solución cuando -11 sea un residuo cuadrático módulo p.

Completando cuadrados en la segunda congruencia cuadrática:


y  y  1  25  0 mod p  
y 2  y  25  0 mod p  
4 y 2  4 y  100  0 mod p  
(2 y ) 2  2  2 y  12  12  100  0 mod p  
2 y  12  12  100  0 mod p  
2 x  12  99  32  11mod p 
y esta congruencia tendrá solución cuando -11 sea un residuo cuadrático módulo p, es decir, las
dos congruencias tendrán soluciones o no en los mismos casos.

Si p  2 ,
xx  1  3  0 mod 2 no tiene solución porque entonces o bien x  0 mod 2 o bien
x  1  0 mod 2 , y en todo caso xx  1  0 mod 2 y por tanto
xx  1  3  xx  1  1  1  0 mod 2
Lo mismo sucede con la segunda congruencia. Así pues, ninguna de las dos tiene solución.

Si p  3 ,
 x  0 mod 3
0  xx  1  3  xx  1mod 3  
 x  1 mod 3
0  y y  1  25  y y  1  1 mod 3  y  2 mod 3

Es decir, ambas congruencias tienen solución (el enunciado no exige que tengan las mismas
soluciones).

10.2.3
Primera versión.
2005  5 (mod 1000 )
2005 2  25 (mod 1000 )
2005 3  25  2005  25  5  125 (mod 1000 )
2005 4  125  2005  125  5  625 (mod 1000 )
2005 5  625  2005  625  5  125 (mod 1000 )
2005 6  125  2005  125  5  625 (mod 1000 )


2005  125 (mod 1000 ) si n es impar
n

Vemos la pauta: n  3  

2005  625 (mod 1000 ) si n es par
n

En N  2005 11  2005 12  ...  2005 2006 hay 1996 sumandos: 998 pares y 998 impares, luego,
tabajando módulo 1000, tenemos:
N  998  125  998  625  998 (125  625 )  998  750
Haciendo esta última multiplicación (no hace falta hacerla entera) vemos que acaba en 500.

Segunda versión.
Ante todo vemos que 2005  5 (mod 1000 )

Por un lado calculamos el sumatorio módulo 125:


Si k  3  5k  10(mod 125 ) , luego el sumatorio es cero.

Por otro lado, calculamos el sumatorio módulo 8:

Observamos que 52  1 (mod 8)


Luego 5k  1 (mod 8) si es par, y 5k  5 (mod 8) si k es impar, luego
1996
511  512  ...  52006  (1  5)  1996  3  5988  4 (mod 8)
2
Ahora aplicamos el Teorema Chino del Residuo:
 N  125  8
x  0 (mod 125 )  N1  8

x  4 (mod 8)   N 2  125  125 y  1 (mod 8)  y  5
 x  8  0  y1  125  5  4  2500  500 (mod 1000 )

10.2.4
Si x no es divisible entre tres entonces x  1 mod 3 o x  2 mod 3 .
x  1 mod 3  x  3k  1  x 2  3k  1  9k 2  6k  1  3k (3k  2)  1  x 2  1 mod 3
2

x  2 mod 3  x  3k  2  x 2  3k  2  9k 2  12 k  4  3(3k 2  4k  1  1  x 2  1 mod 3


2

En todo caso, x 2  1 mod 3 .

Si x es impar, x  2k  1  x 2  2k  1  4k 2  4k  1  4k (k  1)  1 , aquí k o k+1 es par, y


2

por tanto 4k ( k  1) es múltiplo de 8, y por lo tanto x 2  1 mod 8 .


Así pues,
 x 2  1 mod 3
 2
 x  1 mod 8
Y ahora aplicamos el Teorema chino del residuo:
N  8  3  24
N1  3  3 y1  1 mod 8  y1  3
N 2  8  8 y2  1 mod 38  y2  5
x  3  3  1  8  5  1  9  40  49
x  49  1mod 24 

10.2.7
Queremos resolver la congruencia cuadrática x2  7 x  89  0 mod 77  .
Puesto que 77  7 11 , resolveremos las dos congruencias por separado:
i)
x2  7 x  89  0 mod 7  x2  2  0 mod 7  x2  2 mod 7
La resolvemos por tanteo: x  3, 4 mod 7
ii)
x 2  7 x  89  0 mod 11  x 2  4 x  1  0 mod 11
  x  2   3  0 mod 11   x  2   3 mod 11
2 2

La congruencia y 2  3 mod 11 tiene por soluciones y  5, 6 mod 11 , luego


x  2  5 mod 11  x  7 mod 11
x  2  6 mod 11  x  8 mod 11

Ahora resolvemos los cuatro sistemas de congruencias que determinan las soluciones anteriores
mediante el Teorema Chino del Residuo.
 x  3 mod 7 
a) 
 x  7 mod 11
N  7 11  77
N1  11 11y1  1 mod 7  y1  2
N2  7  7 y2  1 mod 11  y2  8
x  2 11 3  8  7  7  458  73 mod 77
 x  3 mod 7 
b) 
 x  8 mod 11
x  2 11 3  8  7  8  514  52 mod 77
 x  4 mod 7 
c) 
 x  7 mod 11
x  2 11 4  8  7  7  480  18 mod 77
 x  4 mod 7 
d) 
 x  8 mod 11
x  2 11 4  8  7  8  536  74 mod 77

y la menor solución positiva será 18.

10.4.3
Aplicando el Teorema de Wilson, sabemos que
 1 mod 17   16 ! 16  15  14! 14! (1)  15 1  (1) mod 17 
Ya solo queda calcular 15 1 mod 17  mediante el ADE:
17  15  2  2  17  15
15  7  2  1  1  15  7  2  15  7(17  15)  8  15  7  17  15 1  8 mod 17 
Así pues, 14! 8 mod 17 , y el residuo de la división es 8.

10.4.4
Aplicando el Teorema de Wilson, 2016! 1 mod 2017 y por otro lado, 2015 ! 1 mod p 
Luego 2016!2015! 1  (1)  2  2015 mod 2017

10.4.5
Multiplicando por 23! a ambos lados de la igualdad obtenemos
2  3  ...  23  1 3  ...  23  1 2  4  ...  23  ...  1 2  4  ...  22  a

Todos los sumandos de la izquierda tienen el factor 13 y por tanto se anulan al hacer módulo
13, menos 1 2  3 ... 12 14  ...  23 , por lo tanto todo se reduce a calcular
1 2  3 ... 12 14  ...  23 mod 13

Y aquí aplicamos el Teorema de Wilson:


1 2  3 ... 12  12! 1 mod 13 ,
14  15  16  ...  23  1  2  3  ...  10  12 1  111  12!  (1) 1  111   1 
 (1)  111   1  111 mod 13 

Determinamos 111 mod 13 mediante el ADE:


13  11  2  2  13  11
11  2  5  1  1  11  2  5  11  (13  11)  5  6  11  5  13  111  6 mod 13 
Así pues, finalmente:
a  (1)  111  6  7 mod 13

10.4.6
Se cumple i  ( p  i ) (mod p ) , por lo tanto:
1  3  5  ...  ( p  2)  (1)( p 1) / 2  p  1 p  3  ...  2 mod p 
Y multiplicando ambos lados por 1  3  5  ...  ( p  2) llegamos a:
12  32  52  ...  ( p  2)2  (1)( p 1) / 2  p  1 ! mod p 
Y finalmente aplicamos el Teorema de Wilson:
(1)( p 1) / 2  p  1 ! (1)( p 1) / 2 (1)  (1)( p 1) / 2 1  (1)( p 1) / 2 mod p 
tal y como queríamos ver.

Fuente de esta solución: Mathematical Excalibur, Vol. 22, No. 3, Feb. 19 – Apr. 19.

10.4.7
Vamos a estudiar la tabla de todos los valores 1  n  102 para observar qué valores
x  n2  n  n(n  1) mod 103  generan:
n n(n  1)
1 1 2  2
2 23  6
3 3  4  12
… …
99 99 100  (4)(3)  3  4  12
100 100 101  (3)(2)  2  3  6
101 101 102  (2)(1)  1  2  2

Vemos que los valores de x que vamos obteniendo van por parejas, puesto que
 n  103  n mod 103 , luego solo trabajaremos hasta la mitad:

n n(n  1)
1 1 2  2
2 23  6
3 3  4  12
… …
49 49  50
50 50  51
51 51 52

Pero observamos que, en módulo 103, 1  102 , 2  101 , 3  100 ,…, 51  52

Luego la tabla anterior la podemos escribir como

n n(n  1)
1  102  2  2
2  101  3  6
3  100  4  12
… …
49  54  50
50  53 51
51  52  52

Es decir, que el producto que buscamos es


2  3  4  ...  50  51  (52 )  (53)  ...  (101)  (102 )  52 
2  3  4  ...  50  51  52  53  ...  101  102   1
102521
 52 
102 ! 1
102521
 52  102 ! (1)  52
Ahora aplicamos el Teorema de Wilson (10.4.2) : 102! 1 mod 103
Para concluir que el producto es (1)(1)  52 mod 103  52 mod 103

Observación: En las soluciones oficiales (Compendium SMT, pág. 1078) se presenta una
solución más formal en términos algebraicos.

11.1
Queremos ver que 7n  3 (mod 10)  7n  43 (mod 100 )
Vamos viendo el comportamiento de las potencias de 7 módulo 100:
70  1
73  63  3 (mod 10 )
7 7
1

7 4  21  1 (mod 10 )
7  49  9 (mod 10 )
2

Vemos que hay un bucle de longitud 4, y que 7n  3 (mod 10)  n  3 (mod 4) .


Veamos ahora como son las potencias de 7 módulo 100:
70  1
73  343  43 (mod 100 )
71  7
7 4  301  1 (mod 10 )
7  49
2

Vemos que también hay un bucle de longitud 4, y que 7n  43 (mod 100 )  n  3 (mod 4) ,
luego 7n  3 (mod 10)  n  3 (mod 4)  7n  43 (mod 100 ) .

11.2
2222  3 (mod 7)
32  9  2 (mod 7)
33  6  1 (mod 7)

5555  3  1851  2
2222 5555  35555  331851 2  33   3   1
1851 2
2  2  5 (mod 7)
1851

5555  4 (mod 7)
4 2  16  2 (mod 7)
43  8  1 (mod 7)

2222  3  740  2
 
5555 2222  42222  43740 2  43 4  1
740 2
2  2 (mod 7)
1851

2222 5555  5555 2222  5  2  0 (mod 7)

11.3
n2  2 n4
an  1 2
n n2
2
n n2

Vemos que la sucesión está bien definida pues el denominador es siempre superior o igual a 2.

n4
Los valores diferentes de an  serán los mismos que de bn 
n n2
2

b4  0 y es el único cero de la sucesión.


1 n2  n  2
Si n  4 , los valores diferentes de bn  serán los mismos que de cn   .
bn n4
n2  n  2 14
cn   n 3
n4 n4

Veamos los elementos iguales de la sucesión cn :

14 14 14 14
cn  cm  n  3   m3 n m
n4 m4 n4 m4
14 14  1 1  n4m4 nm
 nm   14    14  14
m4 n4 m4 n4 m  4n  4 m  4n  4
 m  4n  4  14  7  2
1
 1  14
m  4n  4
Llegamos a una ecuación diofántica, que tiene dos soluciones positivas:
m  4  1 m  5 m42m6
,
n  4  14  n  18 n  4  7  n  11

Luego a5  a18 y a6  a11 , y el número de valores distintos es 98.

Observación.
En la solución oficial se sigue el siguiente razonamiento:
n4 m4
 2  (n  m)(nm  4n  4m  2)  0 
n n2 m m2
2

nm  4n  4m  2  14  14  (n  4)(m  4)  14
Llegando a las mismas dos soluciones: (5,18 ) y (6,11) .

11.4
Primera versión.
Vamos a ver que n19  n7  0 (mod 5) , n19  n7  0 (mod 3) y n19  n7  0 (mod 2) , y por tanto
n19  n7  0 (mod 5  3  2) .

a) n19  n7  0 (mod 5) .
Por el PTF, n5  n (mod 5) , luego
  3
n19  n53 4  n5 n4  n3n4  n7 (mod 5)

b) n19  n7  0 (mod 3)
Por el PTF, n3  n (mod 3) , luego
  6
n19  n631  n3 n  n6n  n7 (mod 3)

c) n19  n7  0 (mod 2)
Si n es par, n19 y n 7 son pares y por tanto n19  n 7 es par.

Si n es impar, n12 es impar, luego n12  1 es par y por tanto n19  n7  n7 n12  1 es par. 
Segunda versión. Sin usar aritmética modular ni PTF
n19  n7  n7 (n12  1)  n7 (n6  1)(n6  1)  n7 (n6  1)(n3  1)(n3  1)
Y utilitzando las identidades
 
n3  1  (n  1) n2  n  1
 
n3  1  (n  1) n2  n  1
 
n6  1  (n2  1) n4  n2  1
llegamos a
    
n19  n7  n7 (n2  1) n4  n2  1 (n  1) n2  n  1 (n  1) n2  n  1 
Ahora vemos que existen tres números consecutivos divisores de n19  n 7 , que son
n  1 , n , n  2 . Al menos uno de ellos será divisible entre 2 y exactamente uno de ellos será
divisible entre 3.
Veamos que n19  n 7 también es divisible entre 5:
Si ninguno de los números n  1 , n , n  1 es múltiple de 5 , entonces n  5k  2 o n  5k  3 , y
por tanto
n  5k  2  n 2  1  25 k 2  20 k  5 es divisible entre 5
n  5k  3  n 2  1  25 k 2  30 k  10 es divisible entre 5.
En todo caso, 5 | n 2  1 | n19  n7

Fuente de la segunda versión: Solución oficial (SE, pág. 883).

11.5
Antes de nada, observamos que el conjunto de números que al dividirlos entre 3 da residuo 1 es
cerrado por la multiplicación, es decir:
a  3a'1
  ab  (3a'1)(3b'1)  9a' b'3a'3b'1  3(3a' b' a'b' )  1
b  3b'1
Su producto también dará resto 1 al dividirlo entre 3.

Vamos a resolver este problema por reducción al absurdo.


Sea K1 el conjunto de números primos que al dividirlos entre 3 dan resto 1, y K 2 el conjunto
de números primos que al dividirlos entre 3 dan resto 2.
Todo número primo pertenece a K1 o K 2 , puesto que no puede ser múltiplo de 3.
Supongamos que el conjunto K 2 es finito:
K 2   p1 , p2 , ... , pn 

Y consideremos el número k  3 p1 p2 ... pn  2 .


No puede ser primo pues entonces pertenecería a K 2 lo cual es absurdo pues
k  p1 ,..., pn
Luego será producto de primos. Al menos uno de estos factores primos de k no será de la
forma 3k  1 , pues, por la observación anterior, si todos los factores de k son de la forma
3k  1 , su producto también sería de la forma 3k  1 , pero k da resto 2 al dividirlo entre 3, no
da resto 1.
Luego existirá un factor primo pi de k que pertenece al conjunto K 2 , lo cual es absurdo pues
cuando dividimos k  3 p1 p2 ... pn  2 entre pi da resto 2, no 0.
Con esto llegamos a contradicción, y por tanto el conjunto K 2 no puede ser finito.

Observación: En las soluciones oficiales (SE, pág. 1080) se presenta una variación ligeramente diferente.

11.6
Una raíz cuadrada de un entero es irracional o es un entero, luego se trata de determinar si
3n 2  2n  2 es un cuadrado perfecto para algún n  1 .

Supongamos que existe un entero k  0 tal que


3n 2  2n  2  k 2  3n 2  2n  2  k 2  0 
 2  4  4  3  (2  k 2 )  2  12 k 2  20  2  4(3k 2  5)  2  2 3k 2  5
n    
23 23 23 23
 1  3k 2  5

3

Pero entonces n es un múltiplo de 3, y por tanto 3n 2  2n  2 es de la forma 3q  2 , y no


puede ser un cuadrado perfecto como ya vimos en el problema 3.16.

11.7
Estudiando los primeros primos vemos, en efecto, una sucesión de 7 números seguidos
cumpliendo la condición:
29  29 , 30  2  3  5 , 31  31 , 32  25 , 33  3  11 , 34  2  17 , 35  5  7

Vamos a demostrar que no puede haber más.


Cada 8 números naturales habrá uno que dividido entre 8 dé residuo 4:
n  8k  4  22 (2k  1)

La descomposición factorial de dicho número tiene seguro el factor 2 elevado al cuadrado, pues
el paréntesis es impar, y por lo tanto no puede ser divisible entre 2. Así pues, dicho número no
puede cumplir la condición del enunciado.

11.8
Ver DE , problema #10.18

11.9
Se demuestra por inducción sobre n.
Para n  1 , S1  5  51 .
Para n  2 , S2  75  52  3
Supongamos que es cierto hasta n  1 , es decir, existe un S n 1 con n  1 dígitos todos impares
divisible entre 5 n 1 .
Para cualquier número impar 1  k  9 , el valor
S n  k  10 n 1  S n 1

Consiste en añadir el dígito k a la izquierda del número anterior. Por lo tanto, si S n 1 tiene todo
sus dígitos impares, también los tendrá S n .

Vamos a demostrar que existe un número impar 1  k  9 para el cual, además, S n sea divisible
entre 5 n .

Puesto que 5n 1 | Sn -1 , podemos escribir Sn 1  a  5n 1 , y por tanto



S n  k  10 n 1  S n 1  k  10 n 1  a  5n 1  5n 1 k  2n 1  a 
n 1 n 1
5 | Sn  5 | k  2
n
 a  k 2  a  0 (mod 5)

Y en efecto, basta tomar k  3n1 a (mod 5)

En efecto,  3n 1 a  2n 1  a  6n 1 a  a  1n 1 a  a  a  a  0 (mod 5)

Como estamos trabajando en módulo 5, si el valor de k obtenido es par, podemos sumarle 5


para que sea impar.

Nota: Se puede demostrar que el valor de k es único, y por lo tanto existe un único S n para
cada n.

11.10
2k  1
Sea Fkn  1 
n
Vamos a demostrar la identidad del enunciado por inducción sobre k .

Para k  1 es fácil:
21  1 2 1 1 1
F1n  1  1  1   1  , la solución es m1  n .
n n n m1

Supongamos que es cierto hasta hasta k , y veamos que también se cumple para k  1 :
n/2    2  1  
k
1 1
Fk  1  k 1   1  1  k 1 
 2  n2  n / 2  2  n  2 
 2k  1  n / 2  2k 1  n  2  1   2k 1  2  n  2k 1  n  2  1 
   k 1     k 1  
 n/2  2  n  2   n  2  n  2 
2k 1  n  1 2k 1  1
   1  Fkn1
n n

 1 
Fkn1  Fkn / 2  1  k 1  , y por tanto, si n es par, ya está demostrado, pues es cierto para
 2  n2
 1 
Fkn / 2 y basta multicarlo por 1   con mk 1  2k 1  n  2 .
 m k 1 

Si n es impar,
 1  2k  1  1 
Fk( n 1) / 2  1    1  1   
 n   (n  1) / 2  n 
 2k 1  2  1   2k 1  2  n  1  n  1  2k 1  1  n 2k 1  1
 1  1         1  Fkn1
 n  1  n   n  1  n  n n

 1
Es decir, Fkn1  Fk( n 1) / 2  1  
 n
Y se demuestra por inducción, pues ( n  1) / 2 es entero, tomando mk 1  n ..

Sea par o impar, queda demostrado el enunciado.

11.11
Pasamos a módulo n:
n | ai ai 1  1  ai ai 1  1  0 (mod n)  ai ai 1  ai  0 (mod n) 
ai ai 1  ai (mod n)

Tomando el primer elemento a1 , vemos que:


a1a2  a1 (mod n) 
  a1a2 a3  a1a2  a1 (mod n)
a2 a3  a2 (mod n)

Y después
a3a4  a3 (mod n) 
  a1a2 a3a4  a1a2 a3  a1 (mod n)
a1a2 a3a4  a1a2 a3  a1 (mod n)

Hasta que, finalmente,


a1...ak 1  a1 (mod n)

Pero si, además, n | ak a1  1  ak a1  a1 (mod n) , en cierta manera lo que hacemos es


completar el círculo, es decir:
a1  a1ak  a1...ak 1ak (mod n)

Pero esto lo podríamos haber hecho con todos y cada uno de los elementos a j , es decir:
a1  a2  ...  ak  a1...ak 1ak (mod n)

Lo cual es absurdo, pues 1  a j  n y son todos diferentes.

Fuente de la solución: "Mathematical Excalibur May-Sep. 2009"

11.12
Buscamos un número entero n con 10 4  n  10 5 y que cumpla n 2  m  10 5  n
para un cierto entero m .

n2  m  10 5  m  n2  n  10 5 m  n2  n  n(n  1) es divisible entre 10 5

Puesto que n y n  1 son coprimos, 2555  10 5 | n(n  1) si y solo si n o n  1 es divisible entre


2 5 y el otro es divisible entre 2 5 .

O, equivalentemente, queremos un número k divisible entre 55  3125 y que cumpla


k  1 (mod 32 )

3125 | k  k  3125 q 
  k  1 (mod 32 )  3125 q  1 (mod 32 )  21 q  1 (mod 32 )
3125  21 (mod 32 ) 

Puesto que 21 y 32 son coprimos, la ecuación 21 q  1 (mod 32 )

Tiene por solución q  3 . En efecto 21  3  63  2  32  1

Por otro lado, la condición 10 4  k  10 5 obliga a que 4  q  31 , y por tanto tomaremos el


valor q  32  3  29
21 (32  3)  21 (3)  ()1  1 (mod 32 )

Así pues, k  3125  29  90625 .

En efecto, 90625 2  8212890625

Fuente de la solución: [Link]


the-number

Observación: La solución oficial se puede considerar como un buen ejemplo de lo que no debe
ser la solución a un problema.
11.13
Primera versión.
n 2 n  1
2
13  2 3  33  ...  n 3 
4

n 2 n  1
2
Queremos resolver la congruencia  17 mod (n  5)
4

n 2 n  1 n 2 n  1
2 2
 17 mod (n  5)   k (n  5)  17
4 4
 n 2 n  1  4k (n  5)  4  17  4k (n  5)  68 
2

n 2 n  1  68 mod (n  5)
2

Pero observamos que


n  5 mod (n  5)  n 2   5 mod (n  5)
2

n  1  4 mod (n  5)  n  1   4 mod (n  5)
2 2

Y por tanto
n 2 n  1  (5) 2 (4) 2  25 16  400 mod (n  5)
2

Luego
400  68 mod (n  5)  400  68  0 mod (n  5)  332  0 mod (n  5)
 n  5 | 332  22  83

Con las condiciones del enunciado estamos suponiendo implícitamente que n  5  17 , luego
las únicas posibilidades son n  5  83 ,166 , 332  n  78 ,161 , 327 .
Comprobamos estas soluciones, y vemos que se cumple para 78 y para 161, pero no para 327:
n 2 n  1
2
n  327   100 mod (n  5)
4
Luego las soluciones son n  78 ,161 .

Observación. Las comprobaciones exigen cálculo manual con números grandes. Por ejemplo:
n 2  106929

n  327  

(n  1)  107584
2

Segunda versión. Mediante aritmética modular.


n 2 n  1
2
 17 mod (n  5) 
4
n 2 n  1  68 mod (n  5) 
2

n 2 n  1  68  0 mod (n  5) 
2

n  5 | n 2 n  1  68  n 4  2n 3  n 2  68
2
Realizando la división sintética tenemos que
n 4  2n3  n 2  68  n 3  3n 2  16 n  80 n  5  332 
n 4  2n3  n 2  68 332
 n3  3n 2  16 n  80 
n5 n5

Y por tanto
n  5 | n 2 n  1  68 
332
Z
2

n5

Y se sigue igual que en la primera versión.

Tercera versión.
n  m  5
Con un cambio de variable: m  n  5  
n  1  m  4
Y por tanto
n 2 n  1 (m  5) 2 (m  4) 2
2
 17 mod (n  5)   17 mod m 
4 4
(5) 2 (4) 2 400
 17 mod m   17 mod m  400  4  17 mod m 
4 4
400  68 mod m  400  68  0 mod m  332  0 mod m  m | 332  n  5 | 332

Y se sigue igual que en la primera versión.

Fuente de estas soluciones: [Link]

11.14
Primera versión. Mediante aritmética modular.
Buscamos conjuntos ordenados ( a, b, c, d ) , con a  0 , tales que
7 | 0abcd  7 | 10000  0  1000  a  100  b  10  c  d (1)
7 | a0bcd  7 | 10000  a  1000  0  100  b  10  c  d (2)
7 | ab0cd  7 | 10000  a  1000  b  100  0  10  c  d (3)
7 | abc0d  7 | 10000  a  1000  b  100  c  10  0  d (4)
7 | abcd 0  7 | 10000  a  1000  b  100  c  10  d  0 (5)

Puesto que: 10000  4 (mod 7) , 1000  6 (mod 7) , 100  2 (mod 7) y 10  3 (mod 7) ,


las condiciones del enunciado se pueden escribir como:
10000  0  1000  a  100  b  10  c  d  0 (mod 7)  6a  2b  3c  d  0 (mod 7)
10000  a  1000  0  100  b  10  c  d  0 (mod 7)  4a  2b  3c  d  0 (mod 7)
10000  a  1000  b  100  0  10  c  d  0 (mod 7)  4a  6b  3c  d  0 (mod 7)
10000  a  1000  b  100  c  10  0  d  0 (mod 7)  4a  6b  2c  d  0 (mod 7)
10000  a  1000  b  100  c  10  d  0  0 (mod 7)  4a  6b  2c  3d  0 (mod 7)

Restando la primera de la segunda llegamos a


2a  0 (mod 7)  a  0, 7 , y solo puede ser a  7
Restando la tercera de la segunda llegamos a
2b  0 (mod 7)  b  0, 7
Restando la tercera de la cuarta llegamos a
c  0 (mod 7)  c  0, 7
Restando la quinta de la cuarta llegamos a
2d  0 (mod 7)  d  0, 7

Así pues, las soluciones posibles son los todos los números de cuatro cifras que se forman con
los dígitos 0 y 7, es decir, los siete números: 7000, 7007, 7070, 7077, 7700, 7707 y 7777.

Segunda versión. Sin aritmética modular.


El tratamiento sería similar. Por ejemplo:
x  0abcd  10000 0  1000  a  100  b  10  c  d
y  a0bcd  10000  a  1000  0  100  b  10  c  d
7 | x
  7 | y  x  9000 a
7 | y
Y puesto que 7 no es divisor de 9000, se llega a 7 | a , es decir, a  0, 7 . Y como no puede ser
cero puesto que estamos suponiendo un número de cuatro cifras, solo queda a  7 .
Y con un argumento similar se deducen los otros tres dígitos.

11.15
a)
2n  1 y 2n  1 son coprimos.
2n  1  k  a 
  2n  1  2n  1  2  kb  ka  k (b  a)
2n  1  k  b 
Y esto es imposible para cualquier k  1 , pues entonces tendríamos k  2 y los números 2n  1
y 2n  1 serían pares, lo cual es absurdo.

b)
Está claro que podemos reducir nuestra búsqueda a divisores comunes primos.
Sea p un divisor común de n 2  1 y 3n  1
Puesto que n2  1  (n  1)(n  1) , p | n2  1 si y solo si p | n  1 o p | n  1 . (ver 4.5)
Supongamos que p | n  1 .
p | n 1 
  p | 3n  1  (n  1)  2n  p | n
p | 3n  1
Sabemos que dos números consecutivos son coprimos, luego es imposible que p sea divisor de
n y de n  1 .
Supongamos que p | n  1 .
p | n 1 
  p | 3n  1  (n  1)  2n  2  2(n  1)  p | n  1
p | 3n  1
Luego p | n  1 y p | n  1 , lo cual es absurdo por el apartado a).

Fuente de esta solución: F.J.G.C. en Facebook.

11.16
2 2001  3 2001 es divisible entre 7 equivale a demostrar que 2 2001  32001  0 mod 7 .
Veamos la sucesión de potencias de 2 módulo 7:
21  2  2 mod 7
2 2  4  4 mod 7 
2 3  8  1 mod 7
 
Puesto que 2001  667  3 , se tiene 2 2001  2 3
667
 1667  1 mod 7 .

Veamos la sucesión de potencias de 3 módulo 7:


31  3  3 mod 7
32  9  2 mod 7
33  27  6 mod 7
34  6  3  18  4 mod 7
35  4  3  12  5 mod 7
36  5  3  15  1 mod 7 
Puesto que 2001  333  6  3 , se tiene 32001  36    33  1333  6  6 mod 7 .
333

Luego, finalmente, 2 2001  32001  1  6  7  0 mod 7 , tal y como queríamos ver.

11.17
Mediante una división sintética vemos que

n3  100  n  10  n2  10 n  100  900 
Luego
 
0  n3  100  n  10  n 2  10 n  100  900  900 mod n  10 
 0  900 mod n  10   n  10 | 900
El mayor n positivo para el que esto ocurre es n  10  900  n  890

11.18
 n n n 
Estamos estudiando la función n     ,   ,    , y queremos ver en qué casos es
 4 5 6 
inyectiva, es decir, la imagen determina unívocamente la antiimagen.
Para que un entero n quede unívocamente determinado por esta terna se pueden cumplir varias
situaciones:

A)
n  1   a  1 , b , c 

n   a , b , c 
n  1   a , b  1 , c 

La única posibilidad de que
n  n  1
 4   a ,  4   a  1
es que se cumpla n  4k .
La única posibilidad de que
n  n  1
 5   b ,  5   b  1
Es que se cumpla n  5q  1
Luego se dará esta situación cuando
n  4k n  0 mod 4
   n  4 mod 20 
n  5q  1 n  4 mod 5
B)
n  1   a  1 , b , c 

n   a , b , c 
n  1   a , b , c  1 

De nuevo, tenemos n  4k .
La única posibilidad de que
n  n  1
 6   c ,  6   c  1
es que se cumpla n  6q  1
Luego se dará esta situación cuando
n  4k n  0 mod 4
 
n  6q  1 n  1 mod 6
Este sistema no tiene solución.
C)
n  1   a , b  1 , c 

n   a , b , c 
n  1   a , b , c  1 

Con los mismos razonamientos, aquí nos encontramos con n  5k y n  6q  1


n  5k n  0 mod 5
   n  5 mod 30 
n  6q  1 n  1 mod 6
D)
n  1   a , b  1 , c 

n   a , b , c 
n  1   a  1 , b , c 

Se dará esta situación cuando
n  4k  1 n  1 mod 4
   n  15 mod 20 
n  5q n  0 mod 5
E)
n  1   a , b , c  1 

n   a , b , c 
n  1   a  1 , b , c 

Se dará esta situación cuando
n  4k  1 n  1 mod 4
 
n  6q n  0 mod 6
Este sistema de congruencias no es compatible.
F)
n  1   a , b , c  1 

n   a , b , c 
n  1   a , b  1 , c 

Se dará esta situación cuando
n  5k  1 n  1 mod 5
   n  24 mod 30 
n  6q n  0 mod 6

Así pues, se cumplirá el enunciado cuando n sea de una de las cuatro formas siguientes:
n  4 mod 20   30
n  5 mod 30   20


n  15 mod 20   30
n  24 mod 30   20

De todos estos casos, debemos tener en cuenta que el grupo


n  5 mod 30 

n  15 mod 20 
tiene 10 repeticiones

Y el grupo
n  24 mod 30 

n  4 mod 20 
Tiene 10 repeticiones,
Por lo tanto, el total será 30  20  30  20 10 10  80 casos.

Observaciones: Una vez entendemos la mecánica profunda del problema, vemos que todo se
desarrolla en módulo 30 y módulo 20, luego podemos pasar resolver el problema en módulo
60, y después multiplicar por 3 el recuento obtenido. Al reducir el problema a 60 casos, incluso
se puede hacer un recuento "por fuerza bruta", uno por uno.

Podemos leer en [Link] que este problema recibió una queja formal
porque, en realidad, se podía haber incluido el caso n  600 , y por tanto el resultado sería 81.
Esta queja fue aceptada.

11.19
Observamos que A0  20  32  52  2  9  25  35  5  7 , luego el máximo común divisor
buscado será un divisor de 35, es decir: 1, 5, 7 o 35.
Veamos que 5 | An para algún n.
5 | An  An  0 (mod 5)  23n  36 n2  56 n2  0 (mod 5)  23n  36 n2  0 (mod 5)
  n
 23  32  36   n
 0 (mod 5)
23  8  3  23  
n
 3n
36  (33 ) 2  22  4  1  36 n
 
 (1) n  32  36
n
 (1)(1) n  (1) n1
Así pues, An  23   32  36   3n  (1) n1 (mod 5) .
n n

Luego, en particular,
A1  31  (1) 2  3  1  4  0 mod 5
Veamos que 7 | An para todo n.
 
An  23n  36 n2  56 n2  23  32  36
n
  n
 
 5 2  56
n
 (*)
23  8  1 mod 7
32  9  2 mod 7 
 
36  33
2
 (1) 2  1 mod 7 
52  25  4 mod 7 
 
56  53  125 2  (1) 2  1 mod 7 
2

Luego
(*)  1n  2  1n  4  1n  1  2  4  7  0 mod 7
Así pues, finalmente,  A0 , A1 , ... , A1999   7

11.20
Para n  2 es cierto:
 
n 6  1  63  32  7 , n 3  1 n 2  1  7  3 
y está claro que todos los factores primos del primero son factores primos del segundo.

Veamos que para n  2 no se cumple nunca.


Aplicando las fórmulas de factorización, tenemos
    
n6  1  n3  1 n3  1  n3  1 n  1 n2  n  1  
    
n3  1 n2  1  n3  1 n  1n  1

Luego nuestro problema se reduce a determinar los enteros n  2 para los cuales
p | n 2  n  1  p | n3  1 n 2  1   
Si p | n2  n  1 entonces p | n3  1 , puesto que n3  1  (n  1) n2  n  1  
Por otro lado, n2  n  1  n(n  1)  1 es un número impar, luego p  3 .
  
Luego p | n3  1 , puesto que n3  1, n3  1  n3  1  (n3  1) , n3  1  2 , n3  1  2   
Así pues, la única posibilidad aceptable es p | n2  1 .
Por otro lado,

p | n3  1
  p | n  1  n  1  n n  1
3 2 2

p | n  1
2

Y puesto que p | n (ya que p | n  p | n6  p | n6  1) , deducimos que p | n  1 .
De la misma forma,
p | n2  1  
  p | n  1  (n  n  1)  n  2
2 2

p | n  n  1
2

De p | n  1 y p | n  2 deducimos que p  3 , es decir, n 2  n  1  3k para cierto entero
positivo k.
n 2  n  1  3k  n 2  n  1  3k  0 , luego su discriminante debe ser un cuadrado perfecto:
  (1)2  4 1 (1  3k )  1  4(1  3k )  1  4  4  3k  3  4  3k  3 4  3k  1  
Pero para k  1 el número 4  3k  1 no es divisible entre 3, por lo tanto el único caso aceptable
es k  1  n 2  n  1  3  n  2 .

Fuente de esta solución: “Baltic Way 2002-2006 Problems and Solutions” Rasmus Villemoes

11.21
Primera versión.
Estos conjuntos son de la forma
{ 10 k  1 , 10 k  2 , 10 k  3 , ... , 10 k  10 } 0  k  99

En cada uno de ellos hay 10 números consecutivos, luego está claro que cada uno contendrá al
menos un múltiplo de 7, es imposible que contenga tres, y contendrá dos múltiplos de 7
cuando lo sea el primero, el segundo o el tercero.

Primer caso:
10 k  1  0 (mod 7)  10 k  1  6 (mod 7)  5k  3 (mod 7)
Estudiando los valores 5k (mod 7) :
k 0 1 2 3 4 5 6
5k 0 5 10≡3 15≡1 20≡6 25≡4 30≡2
Vemos que esto sucede para k  2 (mod 7)

Segundo caso:
10 k  2  0 (mod 7)  10 k  2  5 (mod 7)  2k  1 (mod 7)
Estudiando los valores 2k (mod 7) :
k 0 1 2 3 4 5 6
2k 0 2 4 6 8≡1 10≡3 12≡5
Vemos que esto sucede para k  4 (mod 7)

Tercer caso:
10 k  3  0 (mod 7)  10 k  3  4 (mod 7)  5k  2 (mod 7)
Estudiando los valores 5k (mod 7) :
k 0 1 2 3 4 5 6
5k 0 5 10≡3 15≡1 20≡6 25≡4 30≡2
Vemos que esto sucede para k  2 (mod 7)

Así pues, buscamos todos los valores de k , 0  k  99 tales que k  2,4,6 (mod 7) .
Puesto que 99  7 14  1 , existirán 14  3  42 conjuntos aceptables posibles (B)

Segunda versión.
Hemos visto que hay 100 conjuntos, y en cada conjunto hay 1 o dos múltiplos de 7. Entre el 0 y
el 1000 hay un total de 1000 / 7  142 múltiplos de 7, luego los conjuntos con dos múltiplos
de 7 serán 142-100=42.

11.22
Primera versión. Mediante factorización.
Opción A: Aplicamos Binomio de Newton:
 
2606  1  23
202
  
 1202  23  1 23
201
 
 23
200
 
 23
199

 ...
y por tanto está claro que es divisible entre 7, y queda descartado.
Opción E: Aplicando la identidad “suma de potencias”:
 
2607  3607  2  3 2606  2605  3  2604  32  ...
y por tanto está claro que es divisible entre 5, y queda descartado.
Opción D: Con el mismo razonamiento:
 
2607  1  2607  1607  2  1 2606  2605  2604  ...
y por tanto está claro que es divisible entre 3, y queda descartado.
Opción B: Aplicando la identidad “suma de cubos”
  3
  
2
2606  1  2202  13  2202  1 2202  2202  1 
El primer factor se puede escribir de la siguiente manera:
  101
2202  1  22  1  4101  1
Vemos como se comportan los números de la forma 4n  1
n  1  41  1  5
n  2  42  1  17
n  3  43  1  65
n  4  44  1  257
n  4  45  1  1025
Y observamos que si n es impar, 4n  1 es múltiple de 5, que es nuestro caso, luego también se
puede descartar.
Este último resultado se puede demostrar mediante aritmética modular:
4n  1 mod 5   1  1  1  1  0 mod 5 si n es impar.
n

La única opción sin descartar es C.

Segunda versión. Mediante exponenciación modular.


Vamos a estudiar la sucesión de potencias 2 n , 2n  1 y 2n  1 :

n 2n 2n  1 2n  1
1 2 1 3
2 4 3 5
3 8 7 9
4 16 15 17
5 32 31 33
6 64 63 65
7 128 127 129
8 256 255 267
9 512 511 513
10 1024 1023 1025

Observamos que los todos los números de la forma 2n  1 , con n impar, son todos múltiplos de
3. Es el caso D. En efecto,
2  1 mod 3  2n   1 mod 3
n

y en el caso n impar,
2n   1  1 mod 3  2n  1  1  1  0 mod 3
n
Observamos que los todos los números de la forma 2n  1 , con n par, son todos múltiplos de 3.
Es el caso A. En efecto,
2  1 mod 3  2n   1 mod 3
n

y en el caso n par,
2n   1  1 mod 3  2n  1  1  1  0 mod 3
n

Observamos que algunos de los números de la forma 2n  1 , con n par, son múltiplos de 5.
 
2n  1  22k  1  22  1  4k  1  (1)k  1 mod 5
k

y en el caso k impar,
(1)k  1  (1)  1  0 mod 5
Por lo tanto si podemos escribir n de la forma n  2k  22q  1  4q  2 , la potencia será
múltiple de 5, y quedará descartado. Y el número de la opción B se adapta a este patrón. En
efecto, 606  2  604  4 151  606  4 151  2 .

Finalmente, para descartar la opción E, vamos a trabajar módulo 5:


3  2 mod 5  3n   2 mod 5
n

Y por tanto, si n es impar,  2  2n mod 5 y por tanto


n

2n  3n  2n  2n  0 mod 5.


En el caso de la opción E tenemos que 607 es impar, por lo tanto este número será divisible
entre 5 y quedará descartado.
La única opción aceptable es C.

Observación. El número de la opción C, 2607  1 , es el número 14 de Mersenne, que se trabajan


en el apartado 19.2 de este mismo libro.

11.23
Sean a y b enteros tales que
3x  4 y  a 2

4 x  3 y  b 2

Sumando las dos ecuaciones:


a 2  b 2  3x  4 y  4 x  3 y  7 x  7 y  7( x  y)
Así pues, a 2  b 2 es un múltiplo de 7. Pero si hacemos toda la tabla a 2  b 2 mod 7 :

Vemos que la única posibilidad es a  b  0 mod 7 , es decir, que a y b sean ambos múltiplos
de 7: a  7n , b  7m para ciertos enteros n y m.
También lo podríamos haber demostrado observando que los cuadrados módulo 7 son 0, 1, 2 y
4, y la única manera de obtener 0 sumando dos de estos números es 0 + 0.

Resolviendo el sistema inicial tenemos


4b 2  3a 2 4(7m) 2  3(7n) 2 7 2 4m 2  3n 2 
x    74m 2  3n 2  ,
7 7 7
4a  3b 47 n   37m  7 4n  3m 2 
 74n 2  3m 2 
2 2 2 2 2 2
y  
7 7 7

Es decir, que x e y son múltiplos de 7, tal y como queríamos ver.

11.24
Si a  1 la ecuación queda de la forma 2  7b  c 2  4  7b  c 2  2  0  c 2  2 mod 7 , en
donde tenemos en cuenta que b  1 .
Pero vemos que en módulo 7 la expresión de la derecha nunca es cero:

c c2  2 mod 7
0 4
1 5
2 1
3 6
4 6
5 1
6 5

Luego no hay solución posible.

Si a  2 la ecuación queda de la forma 4  7 b  c 2  4  7 b  c 2 .


Aquí está claro que b debe ser par y c una potencia de 7, concretamente b  2k , c  7 k . Está
claro que cualquier combinación de esta forma satisfacerá la condición del enunciado. Luego
hay infinitas soluciones.

Si a  2 entonces 2 a es un múltiplo de 9. Luego, pasando la expresión a módulo 8 nos queda



c  4  1 (mod 8)
2

7  c  4 (mod 8)  (1)  c  4 (mod 8)   2


b 2 b 2
.

c  4  1 (mod 8)
Pero haciendo la tabla c 2  4 (mod 8) vemos que no hay ningún valor compatible con esta
condición:

c c2  4 mod 8
0 4
1 5
2 0
3 5
4 4
5 5
6 0
7 5

Luego no hay solución posible.

Así pues, las únicas soluciones aceptables son todas las de la forma a  2, b  2k , c  7 k , k  1 .

11.25
Vamos a resolver este problema por inducción. Sea an este número.
a1  5  51 1
a2  75  52  3
a3  375  53  5
a4  9374  54  15

Y vemos la pauta de construcción. Supongamos resuelto el problema hasta an  k n k n 1...k1 .


Por hipótesis de inducción, an será múltiple de 5 n , es decir, an  5n  kn
Tomemos los siguientes candidatos para an 1 , que construimos añadiendo una cifra impar al
anterior:
1kn kn 1...k1  1 10 n  an  1  2  5  5n  kn  5n 1  2n  kn 
n

3kn kn 1...k1  3  10 n  an  3  2  5  5n  kn  5n 3  2 n  k n 
n


5kn kn 1...k1  5  10 n  an  5  2  5  5n  kn  5n 5  2n  kn
n

7 kn kn 1...k1  7  10  an  7  2  5  5  kn  5 7  2  kn 
n n n n n

9kn kn 1...k1  9  10 n  an  9  2  5  5n  k n  5n 9  2 n  k n 
n

Está claro que todos estos números son divisibles entre 5 n , pero además, los cinco paréntesis de
la derecha tienen todos diferentes residuos módulo 5. En efecto, si

a  2n  kn  b  2n  kn mod 5  a  2n  b  2n mod 5  a  b mod 5 y esto nunca ocurre,


pues los números 1, 3, 5, 7 y 9 tienen todos residuos diferentes módulo 5. Así pues, uno de
ellos tiene que tener residuo cero módulo 5, es decir, uno de ellos debe ser divisible entre 5, lo
que dará lugar a un candidato divisible entre 5n 1 , tal y como queríamos ver.

Fuente de esta solución: Soluciones oficiales (USAMO, pág 84)

11.26
Este problema es equivalente a resolver la congruencia 2n  5n  n mod 1000 .
Puesto que 1000  23  53 , podemos dividir este problema en dos, con lo que nos queda el
siguiente sistema de congruencias:
2  5  n mod 2 
 n n 3

 n
2  5  n mod 5 
 n 3

Ahora bien, es fácil comprobar que no se cumple para n  2 . Pero si n  3 , está claro que
   
2n  0 mod 23 y 5n  0 mod 53 , con lo cual nuestro sistema se reduce al siguiente:

5n  n mod 23 
 n

2  n mod 53 
Veamos ahora estas congruencias por separado. La primera es fácil:
1 si n es par

52  25  1 mod 23 , luego 5n    mod 23  
5 si n es impar

 
Así pues, si n es par, 5n  1 mod 23 , pero para n par nunca se cumple n  1 mod 23 , pues  
 
n  1 mod 23  n  8k  1 que es impar. Luego n debe ser impar, y en particular n  8a  5
para cierto a  0 .

Veamos ahora la segunda congruencia: 2n  n mod 53 . Esta congruencia es mucho más 


difícil.
2  n mod 5

Aquí utilizaremos la propiedad 2 n  n mod 53    n
n

2  n mod 5 
 2

Primera reducción.

28a 5  8a  5 mod 53 . 
  2
28a 5  28 a 5
 256 a  32  32  6a mod 53  
 
28a 5  8a  5 mod 53  32  6a  8a  5 mod 53  
Resolvemos 32  6a  8a  5 mod 5
32  6a  8a  5 mod 5  7 1a  3a mod 5
 7  3a mod 5  31  7  a mod 5  a  2  7  14  4  1 mod 5 
a  5b  1
Y por tanto n  85b 1  5  40b  8  5  40b  3

Segunda reducción.
Volvamos a la congruencia 2n  n mod 53 :  
2  n mod 5   2
n
 40 b  3 mod 5   2
3 40b 3 3 40b

 23 40b  3 mod 53 
 2  320 b  24 mod 5 
40b 3

Estudiamos ahora la congruencia 240b  320 b  24 mod 52  



240b  320 b  24 mod 52  210    4b
 20b  1 mod 52 
Por otro lado, observamos que 210  1024  1 mod 52 , así pues:  
2 
10 4 b
 
 20b  1 mod 5   1  20b  1 mod 52
2 4b
 
 
 1  20b  1 mod 5  0  20b mod 5  b  5c
2
 2

Así pues, n  85b 1  5  85  5c 1  5  200c  3 .
Tercera reducción.
Volvemos a la congruencia 2n  n mod 53 :  

2200c3  200 c  3 mod 53  2    100 2c

23  200 c  3 mod 53 
 125   100  2100  1 mod 53   2100   1 mod 53 , luego aplicando Teorema de Euler,
2c

2 
100 2 c

2 3  200 c  3 mod 53  23    1
 200 c  3 mod 53  
  
 47  200 c  3 mod 5  50  200 c mod 5  c  4 mod 5  c  5d  4
3 3

Finalmente,
n  200c  3  2005d  4  3  1000d  800  3  1000d  797

Y el valor más pequeño lo encontraremos para d  0  n  797 .

12.2.1
32  9 (mod 191)   2
34  32  81 (mod 191)
 2
38  34  6561  67 (mod 191) 316  3   4489  96 (mod 191)
8 2

332  3   9216  48 (mod 191)


16 2
364  3   2304  12 (mod 191)
32 2

 3   144 (mod 191)


2
3128 64

Y ahora, puesto que 172  128  32  8  4 ,


3172 (mod 191)  31283323834 (mod 191)  144  48  67  81 (mod 191)  170 (mod 191)

13.1.1
Primera versión: Mediante el método de las potencias de dos.
Vamos calculando 1032 2  (mod 100 ) :
n

1032 1  32 (mod 100 )


1032 2  32  32  24 (mod 100 )
1032 4  24  24  76 (mod 100 )
1032 8  76  76  76 (mod 100 )
...

Observamos que 1032 2   76 (mod 100 ) para todo n  2 .


n

Por otro lado,


1032  1024  4  2  1032 1032  1032 1024 8  1032 1024  1032 8 
1032 1032 (mod 100 )  1032 2  1032 2 (mod 100 )  76  76 (mod 100 )  76 (mod 100 )
10 3

Segunda versión: Mediante el Teorema Chino del Residuo.


1032  0 (mod 4)  1032 1032  0 (mod 4)
1032  7 (mod 25)  1032 2  49  1 (mod 25) 
 1032 1032  1032 2516  1032 2  
516
 (1)516  1 (mod 25)
Nos queda el siguiente sistema de congruencias que resolveremos mediante el Teorema Chino
del Residuo:

1032
1032
 0 (mod 4)


1032
1032
 1 (mod 25)

N  4  25  100 

N1  25  25 y1  1 (mod 4)   x  25  y1  0  4  19  1  76
N 2  4  4 y2  1 (mod 25)  y2  19 

Y por tanto: 1032 1032  76 (mod 4  25) , tal como queríamos ver.

13.1.2
Queremos determinar 2003 2002 mod 1000  .
2001

Puesto que 2003 3 mod 1000, está claro que 2003 2002  32002 mod 1000 
2001 2001

Vamos a estudiar las potencias 3n mod 1000  . Para ello vamos a utilizar el siguiente
argumento:
  mod 1000   9
32k  32
k k
 (10  1) k mod 1000 

Para estudiar las potencias (10  1) k vamos a aplicar el Binomio de Newton:


k  k  k  k 
(10  1) k    10 0 (1) k    101 (1) k 1    10 2 (1) k 2    10 3 (1) k 3  ...
0 1  2  3

Todos los sumandos a partir del cuarto son múltiples de 1000, y por tanto se pueden despreciar.
Así pues,
(10  1) k  (1) k  10 k (1) k 1  50 k (k  1)(1) k 2 mod 1000 

En particular, tomando k  2n ,
(10  1) 2 n  (1) 2 n  10  2n(1) 2 n1  50  2n(2n  1)(1) 2 n2 mod 1000 
 1  20 n  100 n(2n  1) mod 1000 

Es decir:
34n  1  20 n  100 n(2n  1) mod 1000  (*)

En particular, para n  25 :
3100  3425  1  20  25  100  25  (2  25  1)  1  500  100  25  49
 1  500  2500  49  1  500  48  1  1000  24  1 mod 1000 

Puesto que 3100  1 mod 1000  , tenemos que determinar 2002 2001 mod 100  para resolver
nuestro problema.

2002  2 mod 100   2002 2001  22001 mod 100 


Utilizando que 22001  4  21999 mod 4  25  , aplicaremos la “Simplificación de congruencias
lineales”, con lo que tenemos que calcular  21999 mod 25  , y para determinar esta congruencia
utilizaremos el siguiente resultado:
210  1024  1 mod 25   21999  2 20001  2 200101  210   200
 2 1   1
200
 2 1 
 1  2 1  2 1  13 mod 25 
Ahora:
22001  4  21999 mod 4  25   4 13  52 mod 4  25 

En donde en el último paso hemos utilizado que 2 13  26  1 mod 25.

Así pues, finalmente,


32002 mod 1000   352 mod 1000 
2001

Volviento a utilizar (*),


352  3413  1  20 13  100 13(2 13  1)  1  260  100 13  25 
 1  260  32500  1  260  500  241 mod 1000 

2001
Las tres últimas cifras de 2003 2002 son 241.

Fuente: Soluciones oficiales (Compendium CMO, pág. 115)

13.1.3
 
5
a) Aplicando PTF, 36  1 (mod 7)  331  356 1  36 3  15  3  3 (mod 7)
b) 4 , c) 9

13.1.4
Aplicando el PTF, 212  1 (mod 13) , luego
 
21000  21283 4  212
83
 24  183  24  24 (mod 13)  16 (mod 13)  3 (mod 13)
El residuo es 3.

13.1.5
Por el PTF, 1116  1 (mod 17 ) .
 
104  16  6  8 , luego 11104  11166 8  1116 118  1 118  118 (mod 17 )
6 6

  4
112  121  2 (mod 17 )  118  112  24 (mod 17 )  16 (mod 17 )  1 (mod 17 )

Finalmente: 11104  1 (mod 17 )  11104  1  0 (mod 17 )  17 | 11104  1

13.1.6
El número a no puede ser múltiplo de 5, pues en ese caso mcd (a,35)  1 . Luego podemos
aplicar el PTF para garantizar que a 4  1 (mod 5) , y por tanto:
  3
a 4  1 (mod 5)  a12  a 4  13  1 (mod 5)
De la misma manera, a no puede ser múltiplo de 7, y de nuevo aplicamos el PTF para
garantizar que a6  1 (mod 7) , y por tanto:
  2
a6  1 (mod 7)  a12  a6  12  1 (mod 7)
Luego:
a12  1 (mod 5) 

  a  1 mod 5,7  a  1 mod 35 
12 12

a  1 (mod 7)
12

pues 5,7  mcm(5,7)  5  7  35 ya que mcd (5,7)  1

13.1.7
Sabemos que 4k  4 (mod 6) para todo k (se demuestra fácilmente por inducción sobre k)
Observamos que la secuencia an parece dar siempre residuo 4 módulo 7:
a1  4  4 (mod 7)
a2  44  256  7  36  4  4 (mod 7)

Veamos que, en general, si k  4 (mod 6)  4k  4 (mod 7) . En efecto:


Por el PTF, 46  1 (mod 7) luego
  j
k  4 (mod 6)  k  6 j  4  4k  46 j  4  46 44  44  4 (mod 7)
Luego an  4 (mod 7) para todo n, en particular, para n  100 .

13.1.8
Queremos ver (a,42)  1  a6  1  0 mod( 3  7  8)  a6  1 mod( 3  7  8)

Aplicamos tres veces el PTF:


(a,42 )  1  3 | a  a 2  1mod( 3)  a 6  a 2   13  1 mod( 3)
3

(a,42 )  1  7 | a  a 6  1mod( 7)
(a,42 )  1  2 | a  a  1mod( 2)  a 6  16  1mod( 2)

Y ahora:
a 6  1 mod( 3) 

a 6  1 mod( 7)  a 6  1 mod( 3,7,2)  a 6  1 mod( 3  7  8)
a 6  1 mod( 2)

13.1.9
Vemos que 7 | 2,3,4,5,6 y por tanto podemos aplicar el PTF:
  2
26  1 (mod 7)  220  236  2  26 22 (mod 7)  1222 (mod 7)  4 (mod 7)
3  1 (mod 7)  3  3  3  (mod 7)  1  1 (mod 7)
6 30 6 5 6 5 5

4  1 (mod 7)  4  4
6 40
 4   4 (mod 7)  1  4 (mod 7)  4 (mod 7)  4 (mod 7)
6 6  4 6 6 4 6 4 4

En donde hemos aplicado que 4  16  2 (mod 7)  4  4   2  4 (mod 7)


2 4 2 2 2

5  1 (mod 7)  5  5
6 50
 5   5 (mod 7)  1  5  5  25  4 (mod 7)
86  2 6 8 2 8 2 2
  10
66  1 (mod 7)  660  5106  56 (mod 7)  1 (mod 7)
Finalmente, 220  330  440  550  660  4  1  4  4  1  14  0 (mod 7)

13.1.10
Primera versión.
Aplicando el PTF a nuestro caso:
133 5  133  3 (mod 5) 

110 5  110  0 (mod 5)
  133  110  84  27  3  0  4  2  4 (mod 5)
5 5 5 5

84  84  4 (mod 5) 
5

27 5  27  2 (mod 5) 
n5  n (mod 5)  n  4 (mod 5)

Por otro lado:


133  1 (mod 3)  133 5  15  1 (mod 3) 

110  1 (mod 3)  110  (1)  1 (mod 3)
5 5

  133  110  84  27  1  1  0 (mod 3)


5 5 5 5

84  0 (mod 3)  84  0 (mod 3)
5

27  0 (mod 3)  27  0 (mod 3)
5 

Por el PTF, n3  n (mod 3)  n5  n2n3  n2n  n3  n (mod 3) , luego


n  0 (mod 3)

Está claro que n5  133 5  110 5  84 5  27 5  133 5  n  133

El primer número n  133 que cumple n  4 (mod 5) y n  0 (mod 3) es 144, y el siguiente es


176.

Enseguida (????) vemos que 176 es ya demasiado grande, por lo tanto la solución debe ser
n  144 .

Segunda versión.
133  3 (mod 10 )  133 5  35  243  3 (mod 10 )

110  0 (mod 10 )  110 5  0 (mod 10 ) 
  n  2  0  4  7  4 (mod 10 )
5

84  4 (mod 10 )  84  4  4 (mod 10 )
5 5

27  7 (mod 10 )  27  7  7 (mod 10 )
5 5 

Pero por otro lado,


n2  n (mod 2)  n5  n2n2n  n3  n2n  n2  n(mod 2) , y por tanto:
n5  n (mod 2)

  n  n (mod 10 )
5

n  n (mod 5) 
5

Así pues, n5  n  4 (mod 10) , y por lo tanto el número n buscado acaba en 4.


Observamos que 133 ,110 , 84 son números muy próximos a múltiplos de 27:
133  5  27  133 5  55  27 5 

110  4  27  110 5  45  27 5   n5  133 5  110 5  84 5  27 5  27 5 55  45  35  1  
84  3  27  84 5  35  27 5 
5
5
 5 5 5
 n5  n 
n  27 5  4  3  1  27  4393  4393  5   
5 5

27  27 

Como antes, sabemos que n  133 , y sabemos que acaba en 4, luego los candidatos son 134,
144, 154, 164…

Está claro que n  134 nos vamos a quedar cortos.

Probando (y utilizado mucho, pero mucho cálculo !!!!) con n  144 :


5
144 16  16  1048576
n  144       4315 , que se aproxima bastante a 4393.
27 3 3 243

Puesto que estamos trabajando con potencias quintas, que crecen de forma muy rápida, está
claro que el siguiente candidato n  154 ya no será muy próximo al valor buscado, por lo que
podemos asegurar que n  144 es la solución del problema.

Nota: El enunciado lleva implícito que este número existe. Ninguna de las dos soluciones
demuestra que 133 5  110 5  84 5  27 5  144 5 , son buenos argumentos que justifican que
n  154 es un buen candidato para este resultado.
Fuente de esta solución: [Link]

13.1.11
p | 29 p  1  29 p  1  0 (mod p)  29 p  1 (mod p) 
29    1
p 2 2
 1 (mod p)  29 2   p
 1 (mod p)

Pero, aplicando el PTF, sabemos que a p  a (mod p) para todo a , luego


  p
1  29 2  29 2 (mod p)  1  29 2 (mod p)  29 2  1  0 (mod p) 
p | 29 2  1  840  23  3  5  7  p  2 , 3 , 5 , 7
De los cuatro candidatos posibles, los tres primeros son satisfactorios:
p  2  29 2  29  1  1 (mod 2)
p  3  29 3  29  2  1 (mod 3)
p  5  29 5  29  4  1 (mod 5)
(en donde seguimos aplicando PTF : a p  a (mod p) )
Pero el cuarto primo no es satisfactorio: p  7  29 7  29  1  1 (mod 7)
Luego las soluciones son tres: p  2 , 3 , 5 .

13.1.12
Vamos a demostrar que solo el 1 es coprimo con toda la sucesión
an  2n  3n  6n  1 , n  1
Es decir, para todo k  1 , existe un m tal que k , am   1 .
Para ello es suficiente demostrarlo para los números primos.
En efecto, supongamos que se cumple para todo primo p .
Supongamos que no es cierto para cierto k  1 compuesto. Sea p uno de los factores primos
de k .
k , an   1 para todo n  1 .
p|k 
  p, an   1 para todo n  1 , contradiciendo la hipótesis.
k , an   1

Para p  2,3 , está claro que p | a2  48 .


Si p  3 , por el Pequeño teorema de Fermat,
2 p 1  1 (mod p)  3  2 p 1  3 (mod p)
3 p 1  1 (mod p)  2  3 p 1  2 (mod p)
6 p 1  1 (mod p)

y por tanto
6a p  2  6  2 p  2  6  3 p  2  6  6 p  2  6 
 3  2 p 1  2  3 p 1  6 p 1  6  3  2  1  6  0 (mod p)  p | 6a p  2

Y por el Corolario al Lema de Euclides, llegamos a p | a p  2   p, a p  2   p  1 .

Fuente de la solución: Soluciones oficiales (Compendium IMO , pág. 821)

13.1.13
Aplicando el PTF sabemos que x10  1 mod 11 , luego
 
4  x103  x10 x3  110 x3  x3 mod 11 , luego el problema se reduce a resolver la congruencia
10

x3  4 mod 11 .
Probando valores del 2 al 10 encontramos la única solución posible: 53  125  4 mod 11 .

13.1.14
Observamos que 385  5  7 11 , y también que
385 | n60  1  n60  1  0 mod 385   n60  1 mod 385 

Queremos demostrar 5 , 7 , 11 | n  n60  1 mod 385 

El PTF nos garantiza que


n 4  1 mod 5

5 , 7 , 11 | n  n 6  1 mod 7 
n10  1 mod 11

Luego
   1 mod 5 
n 4  1 mod 5  n 60  n 4
15

 1 mod 7   n  n   1 mod 7    n

n6 60 6 10 60
 1 mod 5  7 11

n10  1 mod 11  n  n   1 mod 11
60 10 6

tal y como queríamos ver.

13.1.15
El problema equivale a determinar los primos p para los cuales
29 p  1  0 mod p   29 p  1 mod p 

Por el PTF sabemos que se cumple 29 p  29 mod p  , luego nuestro problema se reduce a
determinar los primos p para los cuales
29  1 mod p  30  0 mod p  p | 30  2  3  5  p  2, 3, 5

13.1.16
p 1
9  111 1  999
... 9  10  1
...
p 1 p 1

Si p  7 entonces p | 10  2  5 , y por tanto, aplicando el PTF, 10 p1  1  1  1  0 mod p  .


Así pues, 9 111 1  0 mod p  .
...
p 1

Puesto que p  7  (9, p )  1 , podemos cancelar el 9 y llegar a 111 1  0 mod p  , tal y como
...
p 1

queríamos ver.

13.2.1
i | 2 j  1  2 j  1  0 (mod i)  2 j  1 (mod i)
Si i  1 , 2  0 (mod 1) , 1  0 (mod 1) , y por lo tanto, trivialmente 2 j  1 (mod i) para j  1 .
Si i  2 , 2  0 (mod 1) , 1  0 (mod 1) , y por lo tanto, trivialmente 2 j  0  0 (mod i) para todo
j , luego no se cumple la condición del enunciado.
Supongamos que i  3 .
Aplicando el Teorema de Euler, sabemos que si (2, i )  1 , es decir, si i es impar, entonces
2 (i )  1 (mod i) , y por tanto basta tomar j   (i ) , que sabemos cumplirá 1   (i )  i .

Si (2, i )  1 , es decir, si i es par, no existirá ningún j tal que 2 j  1 (mod i) , pues entonces:
2 j  1 (mod i)  2  2 j 1  1 (mod i)
Y por tanto x  2 j 1 sería solución para la congruencia 2 x  1 (mod i ) , pero ya vimos en en el
Tema 9 que no tiene solución.
Tomando j  1 , es imposible, pues 2  1 (mod i ) .
Luego se cumple para todos los 1  i  1000 impares, es decir, para 500 números.

13.2.2
Estudiar los ocho últimos dígitos de la expansión binaria de 271986 es equivalente a determinar
(en binario) 271986 (mod 256 )
 1
 (256 )   (256 )  256 1    128 ,
 2
Puesto que 256 | 27 , podemos aplicar el Teorema de Euler para garantizar que
27128  1 (mod 256 ) .
 15
Por otro lado, 271986  2715128 66  27128 27 66  11527 66  27 66 (mod 256 )

Calcularemos 27 66 (mod 256 ) con el “método de las potencias de dos”:


27 2  729  729  217  39 (mod 256 )  27 64   39  (mod 256 )
32

(39)2  1521  241  15 (mod 256 )   39    39 


32
 2 16
   15 (mod 256 )
16

 
(15)2  225  (31) (mod 256 )   15   (15)2  (31)8 (mod 256 )
16 8

 
4
(31)2  961  193  (63) (mod 256 )  (31)8  (31)2  (63)4 (mod 256 )
 (63) 
2
(63)2  3969  129 (mod 256 )  (63)4 2
 129 2 (mod 256 )
129 2  1(mod 256 )
Por lo anterior: 27 66  27 6427 2  1  217  217 (mod 256 )
Y la expresión binaria de 217 es "11011001"

13.2.3
Queremos determinar a2008 (mod 1000 ) , donde n se ha definido recursivamente:
a1  1 , a2  2a1 , a3  3a 2 , ... , a2008  2008 a 2007

Por un lado, 2008  251  8  2008  0 (mod 8)  n  2008 a2007  0 (mod 8) .

Por otro lado, 2008  16  125  8  2008  8 (mod 125 )

 1 4
 (125 )   (53 )  53 1    53 100 , luego, aplicando el Teorema de Euler, para cualquier
 5 5
número a , con (a,125 )  1 , se cumple a100  1 (mod 125 ) .

Observamos que
2007  7 (mod 100 )  2007 4k  74 k  2401 k  1k  1 (mod 100 )

a2006  2006 a 2005  (2  1003 ) a 2005  2a 2005  1003 a 2005 es múltiplo de 4, pues seguro que a2005  2 ,
Y por tanto a2007  2007 a 2006  1 (mod 100 )  a2007  100 k  1 para cierto k, luego
a2008  2008 a 2007  2008 100k 1  2008 100k  2008  2008  8 (mod 125 )

Finalmente aplicamos el Teorema Chino del Residuo:


n  0 (mod 8)

n  8 (mod 125 )
N  8  125 

N1  125  125 y1  1 (mod 8)   n  125  y1  0  8  47  8  3008  8 (mod 1000 )
N 2  8  8 y2  1 (mod 125 )  y2  47 

Así pues, la solución al problema es "008".

Fuente de la solución: Olympiad Number Theory Through Challenging Problems (Justin Stevens, 3E) pág.49

13.2.4
Queremos calcular f (17 )  f (18)  f (19 )  f (20 ) (mod 100 ) , donde f (x) se ha definido
recursivamente: f1 ( x)  x , f 2 ( x)  x f1 ( x ) ,..., f ( x)  f 4 ( x)  x f3 ( x ) .
 1
 (25)   (52 )  251    20  a 20  1 (mod 25) si (a,25)  1
 5
Los calcularemos por separado.

Primera parte: f (20 ) (mod 100 )


20 2  400  0 (mod 100 )  20 k  0 (mod 100 ) para todo k par, y puesto que
f 3 (20 )  20 f2 ( 20) es par, está claro que f (20)  20 f3 ( 20)  0 (mod 100 ) .

Segunda parte: f (19 ) (mod 100 )


f (19 )  19 f3 (19)  (1) f3 (19) (mod 4)

  f (19 )  1 (mod 4)
f 3 (19 )  19 f 2 (19)
es impar 

f 2 (19)  19 f1 (19) es claramente impar, luego
f 3 (19 ) (mod  (25))  f 3 (19 ) (mod 20 )  19 f2 (19) (mod 20 )  1 (mod 20 ) , y por tanto:
f (19)  19 f3 (19)  19 f3 (19) (mod  ( 25))  19 1  4 (mod 25) , pues 19  4  76  3  25  1
f (19 )  1 (mod 4) 
Resolvemos el sistema  mediante el Teorema Chino del Residuo:
f (19 )  4 (mod 25)
N  4  25  100 

N1  25  25 y1  1 (mod 4)  y1  1   f (19 )  25  1  (1)  4  19  4  279  79 (mod 100 )
N 2  4  4 y2  1 (mod 25)  y2  19 

Tercera parte: f (18 ) (mod 100 )


f 3 (18 )  18 f2 (19)  2  9  2  2 f2 (19)  9 f2 (19)  2k
f (19)

Y por tanto f (18)  18 f 3 (19)  2 f 3 (18)  22k  4k  0 (mod 4)


18 4  1 (mod 25) (!!!!)
f 3 (18)  18 f2 (18)  (2  9) f2 (18)  2 f2 (18)  9 f2 (18) es múltiplo de 4
f 4 (18)  18 f3 (18)  1 f3 (18)  1 (mod 25)
Resolvemos el sistema
f (18)  0 (mod 4) 

f (18)  1 (mod 25)
mediante el Teorema Chino del Residuo:
N  4  25  100 

N1  25  25 y1  1 (mod 4)  y1  1   f (18)  25  1  0  4  19  1  279  76 (mod 100 )
N 2  4  4 y2  1 (mod 25)  y2  19 

Cuarta parte: f (17 ) (mod 100 )


17  1 (mod 4)  f (17 )  17 f3 (17)  1 (mod 4)
f (17 )  17 f3 (17)  17 f3 (17) mod  ( 25) (mod 25)  (**)
f 3 (17 ) mod  (25)   f 3 (17 ) mod 20   17 f2 (17) (mod 20)  17 f2 (17) mod  ( 20) (mod 20)  (*)
 (20 )  8 , luego: f 2 (17 ) mod  (20)  f 2 (17 )  1717  117  1 mod 8
(*)  171  17 (mod 20)
(**)  1717 mod 25 

Vamos a calcular esta última potencia por el "método de las potencias de dos":
17 2  289  14 (mod 25)
17 4  14 14  196  4 (mod 25)
17 8  (4)  (4)  16 (mod 25)
  2
17 17  17 281  17 8 17  16 2 17  256  17  6  17  102  2 (mod 25)
Así pues, f (17 )  2 (mod 25 ) .
Como antes, aplicamos el Teorema Chino del Residuo para determinar f (17 ) (mod 100 )
f (17 )  1 (mod 4) 

f (17 )  2 (mod 25)
N  4  25  100 

N1  25  25 y1  1 (mod 4)  y1  1   f (17 )  25  1  1  4  19  2  177  77 (mod 100 )
N 2  4  4 y2  1 (mod 25)  y2  19 

Y finalmente,
f (17 )  f (18)  f (19 )  f (20 ) (mod 100 )  77  76  79  0  232 (mod 100 )  32 (mod 100 )

Fuente de la solución: Olympiad Number Theory Through Challenging Problems (Justin Stevens, 3E) pág.50

13.2.5
Queremos determinar 72014 (mod 1000 ) .

Primera versión.
Aplicamos el Teorema de Euler:
 (1000 )  400 , y puesto que (7,1000 )  1 , aplicando el Teorema de Euler:
1  7 (1000)  7400 mod (1000 )

  5
Luego 72014  7400514  74005  714  7400  714  1  714  714 mod (1000 )

7 2  49  73  343  7 4  2401  401  75  2807  807 


 76  5649  649  77  4543  543
  2
714  77  543 2  * * *849 (no hace falta hacer toda la multiplicación)
72014  714 mod (1000 )  849 mod (1000 )

Fuente de esta versión: Solución oficial (Ver SE, página 333)

Nota: En la Solución oficial (SE, página 333) se presenta otra solución alternativa.

Segunda versión.
Como es habitual en este tipo de problemas (ver Problemas #11.5 y #13.3), calcularemos por
separado 72014 (mod 8) y 72014 (mod 125 ) para después determinar 72014 (mod 1000 ) mediante
el Teorema Chino del Residuo.

a) 72014 (mod 8)
72  49  1 (mod 8)  72014  72   1007
 11007  1 (mod 8)

b) 72014 (mod 125 )


 1 4
 (125 )   (53 )  53 1    53 100 , luego, aplicando el Teorema de Euler, para cualquier
 5 5
número a , con (a,125 )  1 , se cumple a100  1 (mod 125 ) .

En nuestro caso se cumple (2014 ,125 )  1, luego 7100  1 (mod 125 )


 
Y por tanto 71000  7100
10
 110  1 (mod 125 )
 
2
72000  71000  12  1 (mod 125 )
7 2  49 (mod 125 )
73  343  93 (mod 125 )
7 4  651  151  26 (mod 125 )
75  182  57 (mod 125 )
7 6  219  24 (mod 125 )
7 7  168  43 (mod 125 )

Y por tanto 714  77  2


 432  1849  99 (mod 125 )

Y por último: 72014  72000  714  1  99  99 (mod 125 )

Finalmente, aplicamos el Teorema chino del Residuo:

c)
7 2014  1 (mod 8)
7 2014  99 (mod 125 )
N  8  125  1000
N1  125  125y1  1(mod 8)  y1  5
N2  8  8 y2  1(mod125)  y2  47 (*)
x  125  5  1  8  47  99  37849  849 (mod 1000 )

Nota: Esta segunda versión no es operativa en un contexto de una competición en la que no se pueden usar calculadoras, pues, por ejemplo, el
paso (*) anterior exige demasiado cálculo.

13.2.6
Está claro que 2 j  1 es siempre impar, luego i no puede ser par.
Si i es impar, tenemos que, aplicando el Teorema de Euler,
2 (i )  1 mod i   2 (i )  1  0 mod i   i | 2 (i )  1
Puesto que, por definición,  (i )  i  1000 , las condiciones del problema se cumplen para todo
i impar, y la solución es 500.

13.2.7
240  2 4  3  5 .
Aplicando el PTF:
p  5  p | 5  p8  p 4   2
 12  1 mod 5
p  5  p | 3  p8  p  2 4
 14  1 mod 3

Por otro lado, aplicando el Teorema de Euler:


 2 4   8 
  p  1 mod 2 
8 4

 2 , p   1
4

Finalmente, puesto que ( 8 , 5 , 3 )  1 ,


p 8  1 mod 5 

 
p 8  1 mod 3   p 8  1 mod 2 4  3  5 , tal y como queríamos ver.

p 8  1 mod 2 4  
13.3.1

1001 | 10 j  10 i  10 j 10 j  i  1 
 
  1001 | 10  1  10  1 mod 1001 
j i j i

( 1001 , 10 )  1
j

Observamos que ord1001(10)  6 , pues


10 3  1000  1 mod 1001   10 6   1  1 mod 1001 
2

y 101, 10 2 , 10 4 , 105  1 mod 1001  .

Con lo que llegamos a 6 | j  i .

Con lo anterior, hemos reducido nuestro problema a contar todas las parejas 0  i  j  99
cumpliendo j  i mod 6 .
Vemos que 99  16  6  3
Dibujamos un esquema de 99 filas (valores de j ) por 99 columnas (valores de i), podemos
eliminar todos los pares de la diagonal hacia arriba porque i  j , vemos que quedarán
completos 15  14  13  ...  2  1  15  16 / 2  120 bloques, y en cada uno hay 6 pares
aceptables, y en la parte inferior quedarán 16 medios bloques, con 4 pares aceptables en cada
uno. Luego hay un total de 6 120  4  16  784 pares i, j  aceptables.

Fuente: Olympiad Number Theory Through Challenging Problems (Justin Stevens, pág. 62)

13.3.2
Sean p, q primos.
1
q | 2 p  1  2 p  1  0 mod q   ord (2, q) | p  ord (2, q)  
p
ord (2, q)  1  2  1 mod q , lo cual es absurdo, luego solo puede ser ord (2, q)  p .
Pero, por otro lado, por el PTF, y puesto que q | 2 , tenemos que
2q 1  1 mod q  , y por tanto p  ord (2, q ) | q  1  p  q  1  p  q
También se puede aplicar el Teorema de Euler:
p  ord (2, q ) |  (q )  q  1  p  q  1  p  q

15.1.1
Queremos resolver la congruencia 30 n3  143 n 2  117 n  56  0 mod 13 .
Lo primero que hacemos es pasar los coeficientes a módulo 13:
30 n 3  143 n 2  117 n  56  0 mod 13   4n 3  0n 2  0n  4  0 mod 13  
 
4n 3  4  0 mod 13   4 n 3  1  0 mod 13   n 3  1  0 mod 13  
n 3  1 mod 13 

Esta última congruencia la podemos resolver directamente:


13  1 , 2 3  8 , 33  1 , 43  1 , 53  8 , 63  8 , 7 3  5 , 83  5 , 93  1 , 10 3  1 ,
113  5 , 12 3  1
y por tanto n3  1 mod 13  n  1, 3, 9 mod 13 .

Aunque es mucho más elegante seguir el siguiente razonamiento:


Sabemos que 212  1 mod 13  por el PTF, y sabemos que 2 es una raíz primitiva
módulo 13. Por lo tanto:
2 
a 3
 1 mod 13  23a  212 mod 13  12 | 3a  4 | a  a  4,8,12
a  4  2a  24  16  3 mod 13
a  8  2a  28  256  9 mod 13
a  12  2a  212  1 mod 13

Puesto que 2021 6 mod 13 , el primer valor 2021 k  1, 3, 9 mod 13 será para k  3 , y la
solución del problema será 2024 .

16.1
Primera versión.
Sea n  a  b .
a  b | a b  b a  a b  b a  0 (mod n)  a n  a  (n  a ) a  0 (mod n) 
a n  a  ( a ) a (mod n)

Supongamos que a es impar. Entonces la condición anterior es equivalente a:


aa
a n. a  a a (mod n)  1  n  a (mod n)  1  a 2 a  n (mod n)
a

Aplicando el Teorema de Euler, puesto que a, b  1  a, n  1 , sabemos que


a ( n )  1 (mod n)

 (n)  n
Luego basta con resolver  (n)  2a  n  a  .
2

Veamos que para n  2 p con p primo y p  1 (mod 4) se cumple esta condición.


 1  1
n  2 p   (n)  2 p1  1    p  1
 2  p
Luego:
 (n)  n p  1  2 p 3 p  1
a  
2 2 2

Observamos que la condición p  1 (mod 4) garantiza que a es par, puesto que


p  1 (mod 4)  3 p  1  2 (mod 4)

3 p 1 4 p  3 p 1 p 1
Y por tanto, deducimos que a  b  n  2 p  b  2 p   
2 2 2

Queda solamente garantizar que a y b así definidos son coprimos. En efecto:


 3 p  1 p  1  3 p  1, p  1  p  3 , 4  2
 ,    1
 2 2  2 2 2
tal y como queríamos ver.

Segunda versión.
Veamos que todo par de la forma a  2n  1 , b  2n  1, con n  2 satisface la condición del
enunciado.
En primer lugar vemos que (a, b)  1 aplicando el Algoritmo de Euclides:
( a, b)  ( 2n  1 , 2n  1 )  ( 2n  1 , 2n  1  2 )  ( 2n  1 , 2 )  1
Observamos que
a 2  2n  1  4n 2  4n  1  1 (mod 4n)
2

b 2  2n  1  4n 2  4n  1  1 (mod 4n)


2

a b  b a  a 2 n 1  b 2 n 1  a  a 2 n  b 2 n  2 1  a  a 2 n  b  b 2( n 1) 
  n
 a  a 2  b  b2   n 1
 a  1n  b  1n 1  a  b  4n  0 (mod 4n)  0 (mod a  b)

tal y como queríamos ver.

Fuente de estas soluciones: [Link]

16.2
abcd
Sabemos por aritmética básica que [Link]  , luego este problema nos pide encontrar
9999
todos los denominadores distintos posibles para todas las fracciones simplificadas equivalentes
n
a las fracciones de la forma , con 1  n  9999 .
9999

Caso 1.
El bloque principal de numeradores consta de todos los números 1  n  9999 coprimos con
9999, es decir,  n , 9999   1 .

Esta cantidad la podemos calcular fácilmente como aplicación directa de la función Phi de
Euler:
 1  1  1 
9999  32  11  101   (9999 )  9999 1  1  1    6000
 3  11  101 

Pero vemos que también pueden aparecer numeradores n con  n , 9999   1 . Por ejemplo:
605 112  5 11  5 55
 2  2 
9999 3  11  101 3  101 909

Tenemos una fracción simplificada cuyo numerador es múltiplo de 11.

Caso 2: Numeradores n múltiplos de 3 pero no de 11 ni de 101.


Si en el numerador queda un múltiplo de 3 es que proviene de haber simplificado un múltiplo
de 9, y por tanto el numerador original 1  abcd  9999 era múltiple de 9. Así pues, podemos
asegurar que n  1111 .

Entre 1 y 1111 hay 370 múltiplos de 3:


de 3  1  3 a 3  370  1110 → 370 números
a los que debemos quitar los múltiplos de 3 y 11, es decir, los múltiplos de 33:
de 33  1  33 a 33  33  1089 → 33 números
y debemos quitar los múltiplos de 3 y 101, es decir, los múltiplos de 303:
de 303  1  303 a 303  3  909 → 3 números

Puesto que 11 101  1111 , y este número no aparece, no hay casos superpuestos.

Así pues, tenemos 370  33  3  334 casos.


Caso 3: Numeradores múltiplos de 11, pero no de 3 ni de 101.

Si una fracción simplificada tiene numerador múltiplo de 11 es que proviene de una fracción
abcd
sin simplificar , con 1  abcd  9999 , y abcd múltiplo de 112  121 . Luego podemos
9999
garantizar que n  909 .

Los múltiplos de 11:


de 111  11 a 11  82  902 → 82 números.
A los que debemos quitar los múltiples de 11 y 3, es decir, múltiples de 33:
de 33  1  33 a 33  27  891 → 27 números.
y debemos quitar también los múltiples de 11 y 101, que está claro que no existen.

Asi pues, tenemos 82  27  55 casos.

Caso 4: Numeradores múltiplos de 101, pero no de 3 ni de 11.


Seguimos con el mismo razonamiento: Si una fracción simplificada tiene numerador múltiplo
abcd
de 101 es que proviene de una fracción sin simplificar , con 1  abcd  9999 , y abcd
9999
múltiplo de 101 2  10201 , lo cual es imposible.

Caso 5: Numeradores múltiplos de 3 y de 11, pero no de 101.


n 33 66 99
Son fracciones de la forma . Son tres casos: , , .
101 101 101 101

Así pues, N consta de 6000  334  55  3  6392 números diferentes, y la respuesta correcta al
problema es 392.

Fuente de esta solución: [Link]

16.3
Sea p un divisor primo de 20 22  1 , luego:
20 22  1  0 mod p   20 22  1 mod p   20 44  1 mod p 

Sea d  ord p (20 ) .


Está claro que (20, p)  1 , puesto que en caso contrario
p | 20  20  0 mod p   20 22  0 mod p   20 22  1  1  1 mod p 

Luego,/ por el Teorema Fundamental del Orden (ver 13.3)


d | 44 y d | 22 puesto que 20 22  1  1 mod p  .
Los dos únicos candidatos posibles son d  4 o d  11 . Estudiemos estos dos casos por
separado:

Primer caso: d  4 .
  
20 4  1 mod p   0  20 4  1  20 2  1 20 2  1  20  120  1 20 2  1   
 19  3  7  401 mod p 
20  1 mod 3, 7,19   20 2  1 mod 3, 7,19   20 22  20 2   11
 1 mod 3, 7,19 

  p  3,7,19
20 22  1 mod p  

El único candidato que queda es p  401 . Veamos que, en efecto, 20 22  1 mod 401 :
 
20 22  20 2
11
 400 11  (1)11  1 mod 401

Segundo caso: d  44 .
Aplicando el Pequeño Teorema de Fermat, y puesto que hemos visto que (20, p)  1 , se cumple
20 p1  1 mod p 

Luego 44 | p  1 , es decir: p  45, 89, 133 , 177 , 221, 265 , 309 , 353 , 397 ... y entre ellos los
primos son:
p  89, 353 , 397 ...

Tomemos como candidato p  1  88  p  89 .


 
2 6  64  25 mod 89   20 22  (2 2  5) 22  2 4  5 22  2 6  2 2  5 22   25   2 2  5 22  514  2 2  5 22 
7 7

 536  2 2  (*)

Por otro lado, 5 4  625  7  89  2 , luego:


 
(*)   54  2 2  2  2 2  211  2048  23  89  1  1 mod 89 
9 9

Así pues, ya tenemos dos primos 89  401 aceptables para las condiciones del enunciado.

Ahora ya solo nos queda comprobar que p  353 y p  397 no son aceptables. En efecto:

Supongamos que p  397 :


 
20 22  20 2
11
 400 11  311  36  35  729  243  (65)  243  13  5  243  13 1215 
 13  24  312  85  1 mod 397 

Supongamos que p  353 :



20 22  2 2  5  22
 2 44  522
  4
 
2 44  210  24  1024 4  24  70 4  2 4  70 2
2
 4900 2  (42) 2  42 2  1764  1 (mod 353)

Por lo tanto,
 
20 22  1 mod 353   522  1 mod 353   0  522  1  511  1 511  1 mod 353  
Pero
511  5 4  5 4  53  625 2 125  (81) 2 125  38 125  37  375  3  729  22  66  729 
 66  23  1518  106  1,1

Finalmente, la solución del problema es 89  401  490 .


Fuente de esta solución: "Math Gold Medalist" en Youtube: [Link]

16.4
Primera versión.
Sea N  5a  b , para ciertos enteros a  0 y 0  b  5 .
Luego, aplicando el Binomio de Newton,
16
N 16  5a  b    (5a )16 k b k
16

k 0
Todos los elementos de este sumatorio serán múltiples de 5 excepto el último, por tanto:
N 16  b16 mod 5

Veamos caso por caso:


b  0  N 16  016  0 mod 5
b  1  N 16  116  1 mod 5
 
b  2  N 16  216  22  48   1  1
8 8
mod 5
b  3  N 16  316  3   9   1
2 8
1 mod 5
8 8

b  4  N  4   1  1 mod 5
16 16 16

Todo esto se puede razonar directamente en el contexto de la aritmética modular, sin necesidad
de utilizar el Binomio de Newton:
N  0 mod 5  N 16  016  0 mod 5
N  1 mod 5  N 16  116  1 mod 5
 
N  2 mod 5  N 16  216  22  48   1  1
8 8
mod 5
N  3 mod 5  N 16  316  3   9   1
2 8
1 mod 5
8 8

N  4 mod 5  N  4   1  1 mod 5
16 16 16

Así pues, aplicando la fórmula de LaPlace habitual,


Casos favorables 4
P 
Casos posibles 5

En donde hemos tenido en cuenta que 2020  5  404 , es decir, los cinco casos son
equiprobables.

Segunda versión.
Aplicando el Pequeño Teorema de Fermat, sabemos que para todo N  0 mod 5 se cumplirá
N 4  1 mod 5 , luego:
 
N  0 mod 5  N 16  N 4  14  1 mod 5 .
4

Por otro lado, está claro que N  0 mod 5  N 16  016  0 mod 5 , y llegamos al mismo
resultado:
Casos favorables 4
P 
Casos posibles 5
17.2
Calculando a mano los primeros términos de esta suma observamos su pauta:
20 1 21 2
0 0 0 1  0  0
3 3 3 3
2 3
2 4 3 1 1 2 8 6 2 2
2    1 1 3     2 2
3 3 3 3 3 3 3 3 3 3
4 5
2 16 15 1 1 2 32 30 2 2
4    5 5 5     10   10
3 3 3 3 3 3 3 3 3 3
6 7
2 64 63 1 1 2 128 126 2 2
6     21   21 7     42   42
3 3 3 3 3 3 3 3 3 3
8 9
2 256 255 1 1 2 512 510 2 2
8     85   85 9     170   170
3 3 3 3 3 3 3 3 3 3

Si n  2 es par, 2n  1 es múltiple de 3.
En efecto: 2  1 (mod 3)  2n  (1)n  1 (mod 3) si n es par.
Por lo tanto:
 2n   2n  1 1  2n  1  1  2n  1
 3    3  3  3  3  3
     
 2 n 1   2n 
Y también vemos que si n  2 es par,    2  3  , luego
 3   
2 n 1
2 n 1
 2 1 1 
n
2 1 2
n
2 
n 1
 2 n  1
2  2    2     2  3 
3 3  3 3 3 3  3   
es decir, van por parejas: un valor y su doble.

Añadimos el sumando correspondiente a 1001 para tener 501 parejas completas:


1001
 2n  1001 2n  1001 2n  1  2n  1   1001  2n  1  1001 n
      3     3  2 3     3 3    2  1 
n0  3  n2   n2     n2   n2
n par n par n par

 1001 
   500   500 
   2n   500    22 n   500    4n   500  (*)
 n2   n 1   n 1 
 n par 
k
1  r k 1
Aplicamos la fórmula de la serie geométrica:  r n 
n 0 1 r
500
 500 n  1  4501 1  4501 4501  1

n 1
4  4  1 
n

 n0  1 4
1 
3
1 
3
1

Finalmente:
4501  1 4501  1
(*)   1  500   501
3 3

A este resultado le debemos restar el último valor que hemos contado de más:
 21001   2  21000     
 2 21000  1 2   2 21000  1 2  2 21000  1  
 3    3   2    
     3 3  3 3 3

Por lo tanto, el resultado es


4501  1
 501 
  
2 21000  1 4501  1  2 21000  1

 501 
3 3 3
4501  1  21001  2 4501  21001  1 2 2501  21001  1
  501   501   501 
3 3 3
21002  21001  1 2  21001  21001  1 21001  1
  501   501   501
3 3 3
1 (mod 3) si n es par
Nota: 2n  
2 (mod 3) si n es impar

17.3
Primera versión.
Dibujando una cuadrícula 10x10 y tachando manualmente uno por uno todos los múltiplos de
3, vemos que hay 50 múltiplos de 2 de los cuales 16 son múltiplos de 3 luego
50  16 34 17
P  
100 100 50

Segunda versión.
100 
Hay 100 / 2  50 números pares, y hay   16 números divisibles entre 2 y 3, luego hay
 6 
50  16  34 números divisibles entre 2 y no entre 3.
50  16 34 17
P  
100 100 50

17.10
La potencia más alta de 7 que divide a 1000 ! viene dada por:
1000  1000  1000  1000 
 71   142 ,  7 2   20 ,  7 3   2 , y si k  4  7 k   0

1000 
Luego   k   142  20  2  164
k 1  7 

La potencia de 7 más alta que divide a 500! viene dada por:


 500   500   500   500 
 71   71 ,  7 2   10 ,  7 3   1 y si k  4  7 k   0

 500 
Luego   k   71  10  1  82
k 1  7 

1000  1000 ! 1000 !


Finalmente, teniendo en cuenta que    
 500  500! 500! 500!
2

1000 
la potencia más alta de 7 que dividirá   será 164  2  82  0 , luego 7 no es un divisor.
 500 
17.11
 200  200! 200! 200  199  198  ...  2  1
n       
 100  100!100! 100! 100  99  98  ...  2  1
2 2 2 2 2 2

200  199  198  ...  102  101



100  99  98  ...  2  1

En el denominador aparecen todos los primos menores de 100 al menos una vez, luego
necesitamos el número primo p menor que 100 que aparezca al menos dos veces en el
200
numerador, es decir, de forma que 3 p  200  p   66
3
El mayor número primo menor que 66 es 61, y 100  2  61  122  200 , y
100  3  61  183  200 , y solo uno de estos dos factores 61 se anulará con el 61 del
denominador.
La solución es 61.

17.12
Estudiamos la parte de la derecha:

      
Dn  2n  1 2n  2 2n  4 ... 2n  2n 1  2n  1 2 2n 1  1 4 2n  2  1 ...2n 1 2  1   
 2  4  ...  2 2  12  12
n 1 n n 1 n2
 1...2  1 
2 2  12  12
1 2  ...  ( n 1) n n 1 n2
 1...2  1  2
n ( n 1) / 2
2 
n
12  12  1...2  1
n 1

n2

todos impares

n(n  1)
Luego el factor primo 2 aparece con multiplicidad v2 Dn  
2

Para determinar la multiplicidad del factor 2 en la parte de la izquierda, utilizaremos la fórmula


de Polignac:

k 
v 2 ( k !)    i 
i 1  2 


k   k 
1
Pero x  x , luego v2 (k!)    i    i  k  i  k
i 1  2  i 1 2 i 1 2

Así que llegamos a


n(n  1) n(n  1)
k! Dn  v2 (k!)  v2 ( Dn )  k  v2 (k!)  v2 ( Dn )  k 
2 2

Por otro lado,


   
Dn  2n  1 2n  2 2n  4 ... 2n  2n 1  2n 2n 2n ... 2n  2n          n
 2n .
2

 n(n  1) 
Demostraremos más adelante que, para todo n  6 , se cumple 2n  
2
!
 2 
y por tanto, para todo n  6 , no existe solución de la igualdad del enunciado, pues tendríamos

n(n  1)  n(n  1) 
k  k !  ! 2  Dn
n2

2  2 

Comprovamos manualmente los valores n  5 :



D1  21  20  1 , D2  22  1 22  2  6 , D3  23  123  223  22   168  
 4

D4  2  1 2  2 2  2 2  2  20160
4
 4 2
 4 3

D5  25  125  225  22 25  23 25  24   999360

1! 1 , 2! 2 , 3! 6 , 4! 24 , 5! 120 , 6! 720 , 7! 5040 , 8! 40320
9! 362880 , 10! 3628800

Y vemos que las únicas soluciones son n  1, k  1 y n  2 , k  3 .

La desigualdad
 n(n  1) 
2n   ! para n  6
2

 2 
Se demuestra de la siguiente manera:
 65
Para n  6 es cierta porque 26  6.9  1010 y  ! 15! 1.3  10
2
12

 2 

Observación:
Esto es lo que aparece en la solución oficial, Pero ¿Cómo se puede saber esto sin
calculadora? En Youtube encontramos el siguiente razonamiento:
 65
 ! 15! 15  14  13  12  11  10  9  8  7  6  5  4  3  2 
 2 
 8 8 8
 8
 8
 8
 8
 8  7  6  5  4  3  2  238  24  32  5  7 
88  2 38

 238  24  23  25  224 4  3 5  236  26


2

Para n  7 tenemos
 n(n  1)  n(n  1)
 ! 15!16  17  ...   236  16 n ( n 1) / 2 15  236  24 ( n ( n 1) / 2 15)  236  24 ( n ( n 1) / 2 15) 
 2  2
 2 n  24  n 2  2 n  24  2 n  24
 22 n  2n  2n  2n  2n
2 2 2 2 2

17.13
Puesto que 23 es primo, 236  x | 2000!  2000  1999  1998  ...  3  2  1 si y solo si 23 divide a
algún número de este producto.
2000  23  86  22 , luego hay 86 números divisibles entre 23, de los cuales, puesto que
86  23  3  17 hay 3 divisibles entre 23 2 , así pues, en el número 2000! encontramos
86  3  89 veces el factor 23 .
Así pues, 6  x  89  x  86 .

17.14
n 1 !
2

en un entero si y solo si n ! | n 2  1 .
n

n!n

Siguiendo con las definiciones introducidas en 17.4 sobre orden p-ádico de un número,
tenemos que
v p (a)  v p (b)  a | b

Luego vamos a ver para qué enteros n entre 1 y 50 se cumple:



v p n !  v p n 2  1 !
n
  
Por otro lado, se cumple
v p a b  b  v p a 

Así pues, queremos comprobar la desigualdad


n  v p  n !   v p n 2  1 ! (*)

Podemos reducir nuestro estudio a las potencias de números primos.

Primer caso: Si n  p , un número primo. Aplicando la fórmula de Polignac:


 n 2  1
 

v p n2 1 !      ( p  1)  0  ...  0  p  1
k 1  p 
v p n !  1  0  0  ...  1 (ver 17.14b)

Y por lo tanto la desigualdad (*) se convierte en


p  p 1

Lo cual no es cierto para ningún p.

Segundo caso: Si n  p 2 , el cuadrado de un primo. Aplicando la fórmula de Polignac:


v p n 2  1!  v p  p 4  1 !  p 3  p 2  p  3
v p n !  p  1 (ver 17.14c)

Y por lo tanto la desigualdad (*) se convierte en


p 2  p  1 p 3  p 2  p  3  0  p  3  3  p

Lo cual es cierto para todo primo excepto el 2.

Caso general: Se puede demostrar que para todo n  p i con i  2 se satisface la desigualdad
(*)

Así pues, hay 16 números para los cuales no se verifica la desigualdad, y 50  16  34 números
para los que sí se satisface (D).
Fuente de esta solución: [Link]
18.1
Por el TDB, (m, n)  am  bn para ciertos enteros a, b . Luego
(m, n)  n  am  bn  n   m  n  m n  n
       a  b    a    b 
n  m n  m  n  m  n  m  m

m n
Con lo que nuestro problema se reduce a demostrar que   es un entero.
n  m 

n  n 1
Ahora aplicamos la igualdad m   n  (Ver Tema 2 de PC)
 m  m  1

m  n  1  n  1  n 1  n 1
   m   n   que es un número combinatorio, y por tanto entero.
n  m  n  m  n  m  1  m  1

Fuente de la solución: Number Theory Concepts and problems (Andreescu, Dospinescu… 2017) pág. 65

20.5
Basta aplicar n  p1a1 p2a2 ... prar   (n)  a1  1a2  1...ar  1
 (n ) es impar  ai  1 es impar 1  i  r  ai es par 1  i  r  ai  2bi 1  i  r

 n  p1a1 p2a2 ... prar  p12b1 p22b2 ... pr2br  p1b1 p2b2 ... prbr 
2

20.6
Está claro que si n  p q1 con p, q primos, entonces  (n)  q  1  1  q es primo.
Sea n  p1a1 p2a2 ... prar y supongamos que  (n)  a1  1a2  1...ar  1 es primo. Entonces está
claro que r  1 . Luego
n  p a y  ( n)  a  1 primo, y podemos escribir n  p ( n )1 con  (n ) primo, como queríamos.

20.7
10 99  2  5  299  599
99

El número total de divisores es (99  1)(99  1)  100 2  10000


De estos divisores, los múltiplos de 10 88 serán aquellos de la forma 2 a 5b , con 88  a, b  99 , es
decir, 12 12  144 números.
144 6
Luego la probabilidad es 
10000 625

20.8
Puesto que 6 solo se factoriza como 2  3 o 61, el número n solo puede constar de dos factores
primos:
a  1  2
n  p a qb    a  1, b  2  n  p  q con p  q .
2

b 1  3
O bien:
n  p a ,  a  1  6  a  5  n  p5

20.9
1010  2  5  210 510 , y sus divisores son todas las combinaciones posibles de la forma
10

n  2 a 5b con 0  a, b  10 . Hay 112  121 en total.

15 7  3  5  37 57 , y sus divisores son todas las combinaciones posibles de la forma


7

n  3a 5b con 0  a, b  7 . Hay 82  64 divisores posibles.


Pero los divisores de la forma n  305b  5b con 0  b  7 ya aparecen como divisores del
primer número, por lo tanto los restamos. En total hay 64  8  56 divisores.

 
11
1811  2  32  211 322 , y sus divisores son todas las combinaciones posibles de la forma
n  2 a 3b con 0  a  11 y 0  b  22 . Son 12  23  276

Pero de estos están repetidos aquellos de la forma


n  2 a 30 con 0  a  10 : 11 divisores, y aquellos de la forma
n  2 03b con 1  b  7 : 7 divisores. (¡Atención! No contemos el divisor 1 dos veces)
Luego hay 276  11  7  258 nuevos.

Así pues, hay un total de 121  56  258  435 divisores.

20.10
Para n  1 ,  (1)  1 ,  (2)  2 y está claro que no cumple la condición. Luego n  2 , y por tanto
 (n) ,  (n  1)  2 y en consecuencia  (n)   (n  1)  7   (n) ,  (n  1)  5 .

 (n)  1  n  1 y ya hemos visto que no cumple la igualdad del enunciado.

Luego las posibilidades que quedan son:


a)  (n)  2 , (n  1)  5
b)  (n)  3 , (n  1)  4
c)  (n)  4 , (n  1)  3
d)  (n)  5 , (n  1)  2

Teniendo en cuenta que:


 ( n)  2  n  p
 ( n)  3  n  p 2
 ( n)  5  n  p 4
n  p  q , p  q
 ( n)  4  2  2  
n  p
3

Tenemos
a) n  p , n  1  p 4
b) n  p 2 , n 1  p q
n  p 2 , n  1  p3
c) n  p 3 , n  1  q 2
n  p q , n  1  q2
d) n  p 4 primo y n 1  q primo.
Observamos que en todos los casos, uno de los dos números consecutivos es la potencia par de
un número primo, y con esto vamos probando casos:
n  3 , n  1  4   (3)  2 ,  (4)  3
22  4   ninguna cumple.
n  4 , n  1  5   (4)  3 ,  (5)  2
n  8 , n  1  9   (8)  4 ,  (9)  3
32  9   ambas cumplen la condición del enunciado.
n  9 , n  1  10   (9)  3 ,  (10 )  4
n  15 , n  1  16   (15)  4 ,  (16 )  5
24  16   cumple la segunda.
n  16 , n  1  17   (16 )  5 ,  (17 )  2
n  24 , n  1  25   (24 )  8 ,  (25)  3
52  25   cumple la segunda.
n  25 , n  1  26   (25)  3 ,  (26 )  4
n  48 , n  1  49   (48)  10 ,  (49 )  3
7 2  49   ninguna cumple.
n  49 , n  1  50   (49 )  3 ,  (50 )  6
n  80 , n  1  81   (80 )  10 ,  (81)  5
34  81   ninguna cumple.
n  81 , n  1  82   (81)  5 ,  (82 )  4
n  120 , n  1  121   (120 )  16 ,  (121)  3
112  121   cumple la segunda.
n  121 , n  1  122   (121)  3 ,  (122 )  4
n  168 , n  1  169   (168 )  16 ,  (169 )  3
13 2  169   ninguna cumple.
n  169 , n  1  170   (169 )  3 ,  (170 )  8
n  288 , n  1  289   (288 )  18 ,  (289 )  3
17 2  289   ninguna cumple.
n  289 , n  1  290   (289 )  3 ,  (290 )  8
n  360 , n  1  361   (360 )  24 ,  (361)  3
19 2  361   cumple la segunda.
n  361 , n  1  362   (361)  3 ,  (362 )  4

Y paramos porque hemos conseguido las seis soluciones pedidas. El resultado es


8+9+16+25+121+361=540

Fuente de esta solución: [Link]/wiki/[Link]/2019_AIME_I_Problems/Problem_9

20.11

2004  22  3 167  2004 2004  22  3 167 2004
 24008  32004 167 2004 .

Los divisores de 2004 2004 son todos los números de la forma

d  2 a  3b  167 c , con 0  a  4008 , 0  b  2004 , 0  c  2004 .

Luego  d   a  1b  1c  1 , y por tanto

 d   2004  a  1b  1c  1  22  3 167

O equivalentemente, todas las posibilidades x, y, z con x y z  22  3 167 y


1  x  4009 , 1  y  2005 , 1  z  2005 .
Colocamos primero los "2": Hay 6 formas diferentes.
2–2-/
2-/-2
/-2–2
22 - / - /
/ - 22 - /
/ - / - 22

Ahora tenemos que colocar el 3 y el 167, en cualquiera de las tres posiciones, luego el total será
6  3  3  54 formas diferentes.

20.12
Sean J, C y Z las edades respectivas de Joey, Chloe y Zoe.
Tenemos que J  C  1 y Z  1
En cada aniversario tenemos la siguiente pauta:

Zoe 1 2 3 4 Z
Chloe C C+1 C+2 C+3 C+Z-1
Joey C+1 C+2 C+3 C+4 C+Z

Está claro que C es múltiple de 1, y existirán ocho números más Z1 , Z 2 ,..., Z8 tales que

Z1 | C  Z1  1 , Z2 | C  Z2  1, ..., Z8 | C  Z8  1

Puesto que Zi | Zi , entonces está claro que Zi | C  1 , es decir, el número C  1 tiene


exactamente ocho divisores diferentes de 1. Luego C  1 tiene nueve divisores contando el 1.
Aplicando 20.4, tenemos que
C  1  p1a1 p2a2 ... prar
entonces
9  a1  1a2  1...ar  1

Con un solo factor primo sería imposible, pues como mínimo tendríamos el resultado
C  1  28  256 , una edad absurda.

Con dos factores primos, la única posibilidad aceptable para una edad es
9  2  12  1  C  1  2232  36

Y por tanto C  36  1  37  J  C  1  38

Zoe 1 2 3 4 5 Z
Chloe 37 38 39 40 41 ...
Joey 38 39 40 41 42 C  37  Z

Nos piden determinar el siguiente Z tal que Z | J  Z | 37  Z  Z | 37 , pero puesto que 37 es


primo, la única posibilidad es Z  37  J  37  37  74 , y la respuesta correcta es 7  4  11
(E).
20.13
1 1 1 yx 1 xy
      N  xy  N ( x  y ) 
x y N xy N x y
xy  Nx  Ny  0  ( x  N )( y  N )  N 2  0  ( x  N )( y  N )  N 2

Que esta ecuación tenga exactamente 2005 soluciones es una forma de decir que existen
exactamente 2005 formas diferentes de escribir N 2 como producto de dos enteros positivos, es
decir, que N 2 tiene exactamente 2005 divisores, pues existe una biyección clara entre los
divisores de un número y sus expresiones como producto:
d  d  (N / d )
Ahora aplicaremos el problema 20.5: El número N es un cuadrado perfecto si y solo si  (N ) es
impar.
Si la factorización canónica de N es N  p1a1  p2a 2  ...  pna n , la factorización canónica de N 2 es
 
N 2  p12 a1  p22 a 2  ...  pn2 a n , y por lo tanto  N 2  (2a1  1)(2a2  1)...(2an  1)  2005

Por otro lado, 2005  5  401 , luego la única formas posibles son:
a) a1  2 y a2  200 , y por tanto  ( N )  (a1  1)(a2  1)  3  201  603 , un número impar, y por
tanto N es un cuadrado perfecto.
b) a1  1002  N  p110021   ( N )  1003 impar, luego es un cuadrado perfecto.

20.17
Por la teoría estudiada sabemos que si n  p1a1 p2a2 ... prar es la descomposición en factores primos
de n, entonces
d (n)  a1  1a2  1...ar  1

Por otro lado, 3


n  p1a1 / 3 p2a2 / 3 ... prar / 3 ,
Y por tanto:
d (n) a1  1a2  1...ar  1 a 1
f ( n)  3
 a1 / 3 a 2 / 3 ar / 3
  ka k / 3
n p1 p2 ... pr 1 k  r pk

Puesto que en todo momento estamos trabajando con números positivos, y la función 3 x es
estrictamente positiva, el n que determina el máximo de la función anterior será el n que
determina el máximo de la función

f ( n)  
3 ak  1
3

1 k  r pka k

Fijado un número primo pk , la sucesión


ak  13
es una sucesión con límite 0, pues es un
pka k / 3
cociente entre una potencial y una exponencial. Tenemos que estudiar el comportamiento para
los diferentes valores de pk   2 , 3 , 5 , 7 , 11 , ...

pk  2 , ak  1  ak  1 / pka k  4
3

pk  2 , ak  2  ak  1 / pka k  27 / 4
3

pk  2 , ak  3  ak  1 / pka k  8
3
 Máximo
pk  2 , ak  4  ak  1 / pka k  125 / 16  7.81
3

pk  3 , ak  1  ak  1 / pka k  8 / 3  2.67


3

pk  3 , ak  2  ak  1 / pka k  3  Máximo


3

pk  3 , ak  3  ak  1 / pka k  64 / 27  2.37


3

pk  5 , ak  1  ak  1 / pka k  8 / 5  1.6  Máximo


3

pk  5 , ak  2  ak  1 / pka k  27 / 25  1.08


3

pk  7 , ak  1  ak  1 / pka k  8 / 7  1.14  Máximo


3

pk  7 , ak  2  ak  1 / pka k  27 / 49  0.55


3

pk  11 , ak  1  ak  1 / pkak  8 / 11  1
3

Vemos que, para todo pk  11 y para todo ai  1 el factor resultante es menor que 1 y por tanto
disminuye el valor de la función. Así pues, el máximo de esta función lo encontraremos para
n  23  32  51  71  2520
Y por tanto la respuesta correcta es 2  5  2  0  9 (E)

Observación: Para no tener que calcular el producto final y ganar tiempo, se puede argumentar
que el resultado será múltiplo de 32  9 , y por tanto la suma de sus cifras será también múltiplo
de 9, y la respuesta (E) es la única con esta condición.

Fuente de esta solución: [Link]

21.5
1815  3  5  112
Sabemos que n  2 a 3b para ciertos enteros positivos a, b . Sabemos que entonces
2a 1  1 3b 1  1 1 a 1
 ( n) 
2 1

3 1
 
 2  1 3b 1  1
2

A falta de algo mejor, vamos viendo los diferentes valores que se obtienen:
k 2k 1  1 3k 1  1
1 3 8
2 7 26=2·13
3 15=3·5 80=24·5
4 31 242=2·112
5 63=3·31 728=23·7·13

Vemos que la única combinación que se adapta a nuestro problema es a  3 y b  4 :

2

1 31
 
1
2  1 34 1  1  3  5  2  112  1815 . Así pues, n  2334  8  81  648 .
2

22.1.1
Sabemos que existirá solución pues (7,9)  1 .
7 x  9 y  3  7 x  3  9 y  31  3 y   3 | x  x  3k 
7  3k - 9y  3  7 k - 3y  1  7 k  1  3 y
Claramente una solución de esta última ecuación es y  2, k  1 , luego y  2, x  3, k  3 será
una solución particular de la ecuación del enunciado.
En general, las soluciones serán de la forma y  2  9k , x  3  7k , con k  Z

22.1.2
Sean N a y N b la cantidad de manzanas y naranjas compradas, respectivamente.
Sean Pa y Pb el precio de cada manzana y de cada naranja, respectivamente, en céntimos.

Tenemos las siguientes condiciones:


 N a  N b  12
P  P  3
 a b

 aN  N b
 Pa N a  Pb N b  132

Luego
Pb  3N a  Pb 12  N a   132  Pb N a  3N a  12 Pb  Pb N a  132 
3N a  12 Pb  132  N a  4 Pb  44

Esta última ecuación diofántica tiene solución pues d  mcd (1,4)  1 , y 1 | 44 .

Una solución particular de esta ecuación es N a  4, Pb  10 , pero no satisface la condición


N a  N b . El resto de soluciones son de la forma
4
N a  4  k , Pb  10  k
1
k  0  N a  4, Pb  10  N b  8
k  1  N a  8, Pb  9  N b  4, Pa  12
k  2  N a  12, Pb  8  N b  0, Pa  11

Las soluciones posibles son 4 naranjas a 9 céntimos, (con 8 manzanas a 12 céntimos) o 12


manzanas a 11 céntimos (y sin ninguna naranja)

22.1.3
Aplicamos el algoritmo de Euclides:
858  3  253  99 
253  2  99  55 

99  1  55  44   (858 ,253 )  11 , y 11 | 33 , luego la ecuación tiene solución.
55  1  44  11 

44  4  11 

Deshacemos los pasos del algoritmo de Euclides:


11  55  44 
44  99  55 

  11  55  44  253  2  99  99  55   253  2  99  99  55 
55  253  2  99 
99  858  3  253 
 253  3  99  253  2  99  2  253  5  99  2  253  5858  3  253  
 2  253  5  858  15  253  5  858  17  253

Y multiplicando ambos lados por 3 obtenemos la solución deseada:


11  5  858  17  253  33  15  858  51  253  x  15, y  51

El conjunto general de soluciones serán todas las parejas de la forma:


253 858
x  15  k  15  23k  8  23k , y  51  k  51  78k
11 11

22.1.4
Calculamos el (858 ,253 ) mediante el algoritmo de Euclides:
258  147  111
147  111  36 
  (858 ,253 )  3 , y 3 | 369 , luego la ecuación tiene solución.
111  36  3  3 
36  3 12 

Deshaciendo los pasos del algoritmo de Euclides:


3  111  3  36 

36  147  111   3  111  3  36  258  147  3147  111   258  147  3  147  3  111 
111  258  147 
 258  4  147  3  111  258  4  147  3258  147   258  4  147  3  258  3  147 
 4  258  7  147
Multiplicando por 123 tenemos 369  123  3  492  258  861 147
luego una solución concreta es x  492 , y  861 , y el conjunto de soluciones es
x  492  49 k , y  861  86 k

22.1.5
x  15  11k , y  27  20 k

22.1.6
La sucesión será de la forma a , a  k , a  2k , a  3k , a  4k , a  5k para un cierto ángulo inicial
a, y sabemos que los ángulos internos de un hexágono suman (6  2)  180 º  720 º , así pues:
a  a  k  a  2k  a  3k  a  4k  a  5k  720  6a  15 k  720  2a  5k  240

Esta última ecuación diofántica tendrá solución pues mcm (2,5)  1 .


Una solución es a  100 , k  8 , con lo que el conjunto de soluciones será de la forma:
a  100  5n
 , con n  Z
b  8  2 n
Tenemos, además, la restricción 0  a  180 , luego
0  100  5n  180  100  5n  80  20  n  16

El valor máximo se toma con n  15  a  100  5 15  175 º .

22.1.7
Primera versión.
Sea n dicho número.
n  10 a  9
  10 a  9  9b  8  10 a  9b  1
n  9b  8 
Esta ecuación diofántica tiene por solución
a  1  9k 
  n  10 (1  9k )  9  90 k  1 , o equivalentemente, n  90c  1
b  1  10 k 

Añadimos una condición más:


n  90 c  1
  90 c  1  8d  7  90 c  8d  8  45c  4d  4
n  8d  7 
Esta ecuación diofántica tiene por solución:
c  4e 
  n  90 (4e)  1  360 e  1
d  45e  1

Añadimos una condición más:


n  360 e  1
  360 e  1  7 f  6  360 e  7 f  7
n7f 6 
Esta ecuación diofántica tiene por solución:
e  7g 
  n  360 (7 g )  1  2520 g  1
d  1  360 g 

Con este resultado ya podemos buscar candidatos, y vemos que con g  1  n  2519 ya
cumple todas las condiciones del enunciado.

Segunda versión.
Sea n el número buscado. Este número se puede escribir como
n  10 a9  9  9a8  8  8a7  7  ...  2a1  1

Pero entonces:
n  1  10(a9  1)  9(a8  1)  8(a7  1)  ...  2(a1  1) , es decir:
2,3,4,5,6,7,8,9 | n  1  2520  (2,3,4,5,6,7,8,9) | n  1  n  1  2520 k  n  2520 k  1

De nuevo, comprobamos que, con k  1 , n  2519 cumple todas las condiciones exigidas.

22.1.8
19  17  1  2  2  19  17 
  1  17  8(19  17 )  17  8  19  8  17  8  19  9  17
17  8  2  1  1  17  8  2 
22.2.1
Para resolver este problema vamos a utilizar que todo cuadrado perfecto es 0 o 1 módulo 3 (ver
Problema 3.22).

Entre el conjunto de todas las soluciones posibles, tomaremos aquella tal que a  b  c sea
mínimo.

Pasando a módulo 3, a 2  2b2  3c 2  2b2 (mod 3) .


Puesto que b 2 es 0 o 1 módulo 3, 2b 2 será 0 o 2 módulo 3. Puesto que es un cuadrado perfecto,
no podrá ser 2 módulo 3, por lo que la única posibilidad es que sea 0 módulo 3:

b2  0 (mod 3)  3 | b2  3 | b

3 | b  3 | 2b 2 

  3 | a  2b  3c  3 | a
2 2 2

3 | 3c 2

3 | b  b  3b'  b 2  9b'2 

2
 
 3c 2  2b 2  a 2  2  9b'2 9a'2  9 2b'2 a'2  9 | 3c 2  3 | c 2
3 | a  a  3a'  a  9a' 
2

3 | c 2  3 | c  c  3c'

2 2 2

Luego 2b'2 3c'2  2   3   2  3  2b 2  3c 2   a 2     a'2


b c b2 c2 1 1 a
3 3 9 9 9 9 3

Es decir, la terna (a ' , b' , c' ) también es solución de la ecuación del enunciado, pero
a b c
a '  b'  c '     a  b  c
3 3 3
Contradiciendo la hipótesis de que esta suma era mínima.
Así pues, no existe ninguna solución a esta ecuación que no sea la trivial a  b  c  0

Fuente de la solución: Solución oficial (OME, página 247)

22.3.8
Siguiendo los razonamientos de los teoremes de este apartado, distinguiremos los siguientes
casos:
a) Si n no es un cuadrado perfecto.
a1) Si n es impar.
El número de soluciones positivas será
1 n
 (n)    i  1
2 i 1
Luego
1 n n
43  47    i  1  2  43  47    i  1
2 i 1 i 1
Y el valor de n más pequeño lo encontraremos tomando los factores primos más pequeños con
los exponentes más pequeños posibles (y en orden inverso):
n  3471 5431 72 1  346 542 7

a2) Si n es par.
El número total de soluciones será
n n
 (n)  (1  1)  i  1  2021  43  47  (1  1)  i  1  2  43  47
1
2 i2 i2
Y el número n más pequeño que podemos conseguir será
n  2471 3431 52 1  248 342 5

b) Si n es un cuadrado perfecto.
b1) Si n es impar.
n
La fórmula  
i 1
i  1 determina el total de soluciones con x positiva, que pueden ser de la

forma ( x, y ) o ( x, y ) . Entre ellas tenemos la solución la solución  


n , 0 , que no se
contempla en este problema, luego:
n

 
i 1
i  1  2  2021  1  13  311

Y el número n más pequeño que podemos conseguir será n  3310  512

b2) Si n es par.
n
La fórmula (1  1)  i  1 determina el total de soluciones con x positiva, que pueden ser
i2

de la forma ( x, y ) o ( x, y ) . Entre ellas tenemos la solución la solución  


n , 0 , que no se
contempla en este problema, luego:
n
(1  1)  i  1  2  2021  1  13  311
i 2

Los valores de n que vamos obteniendo: 214 p310 , 2312 p12 , 24044 son todos mayores que el
encontrado en el apartado a2 (falta por comprobar).

Así pues, el menor número olímpico es n  248 342 5 y si exigimos que además sea impar es
n  346 542 7 .

Fuente: Soluciones oficiales (OME, pág. 886)

22.3.9
a  a 2  8a
Las raíces de f ( x)  x  a x  2a son x 
2

Luego a 2  8a debe ser un cuadrado perfecto: a 2  8a  b 2


a 2  8a  b 2  a 2  8a  16  b 2  16  a  4  b 2  16 
2

a  42  b2  16  a  4  ba  4  b  16
Resolvemos todas las combinaciones posibles:
a  4  b  1 a  25 / 2 a  4  b  2 a  9
   
a  4  b  16 b  15 / 2 a  4  b  8 b  3
a  4  b  4 a  8 a  4  b  8 a  9
   
a  4  b  4 b  0 a  4  b  2 b  3
a  4  b  16 a  25 / 2 a  4  b  1 a  9 / 2
   
a  4  b  1 b  15 / 2 a  4  b  16 b  15 / 2
a  4  b  2 a  1 a  4  b  4 a  0
   
a  4  b  8 b  3 a  4  b  4 b  0
a  4  b  8 a  1 a  4  b  16 a  9 / 2
   
a  4  b  2 b  3 a  4  b  1 b  15 / 2

Las soluciones aceptables son 9, 8, 0 y -1 , luego la respuesta correcta es 9  8  0  (1)  16


(C)

Observación.
Una manera mucho más elegante de resolver la ecuación
a  4  ba  4  b  16
Es la siguiente:
a4b u
a  4  ba  4  b  u  v    2(a  4)  u  v  a 
uv
4
a  4  b  v 2

Luego
17 10
u  1 , v  16  a  4 u  2,v 8a  49
2 2
8  17
u 4,v 4a  48 u  1 , v  16  a  4
2 2
 10 8
u  2 , v  8  a   4  1 u  4 , v  4  a  40
2 2
Fuente de esta solución: [Link]

22.4.1
x 2  y 2  3z 2  x 2  y 2  0 (mod 3) .
Ahora aplicamos el resultado de que todo cuadrado es 0 o 1 módulo 3, luego
x 2  0 (mod 3) , y 2  0 (mod 3)  x 2  y 2  0  0  0 (mod 3)
x 2  1 (mod 3) , y 2  0 (mod 3)  x 2  y 2  1  0  1 (mod 3)
x 2  0 (mod 3) , y 2  1 (mod 3)  x 2  y 2  0  1  1 (mod 3)
x 2  1 (mod 3) , y 2  1 (mod 3)  x 2  y 2  1  1  2 (mod 3)

La única posibilidad válida es x 2  y 2  0 (mod 3)  x  y  0 (mod 3)


Luego
 
x  3 x' , y  3 y '  x 2  y 2  (3 x' ) 2  (3 y ' ) 2  9 x'2 9 y '2  9 x'2  y '2  3 z 2 
 
3 x'2  y '2  z 2  3 | z 2  3 | z  z  3 z '  z 2  3 z '  9 z '2
2

Luego:
 
3 x'2  y'2  9 z '2  x'2  y'2  3z '2

x y z
Llegando a la misma ecuación para valores  x' , y ' , z '   , ,  estrictamente más pequeños
 3 3 3
que los primeros. Aquí aplicamos el Principio del Descenso Infinito para deducir que esta
ecuación no tiene ninguna solución exceptuando la trivial x  y  z  0

22.4.2
a 2  b 2  c 2  a 2b 2  c 2  a 2b 2  a 2  b 2

Puesto que todos los elementos involucrados son cuadrados, podemos reducir nuestro estudio a
las soluciones positivas, y después duplicar las soluciones encontradas.

Vamos a utilizar que todo cuadrado es 0 o 1 módulo 4.


a 2  0 (mod 4), b 2  0 (mod 4)  a 2b 2  a 2  b 2  0  0  0  0  0 (mod 4)
a 2  1 (mod 4), b 2  0 (mod 4)  a 2b 2  a 2  b 2  1  0  1  0  1  3 (mod 4)
a 2  0 (mod 4), b 2  1 (mod 4)  a 2b 2  a 2  b 2  0  1  0  1  1  3 (mod 4)
a 2  1 (mod 4), b 2  1 (mod 4)  a 2b 2  a 2  b 2  1  1  1  1  1  3 (mod 4)

Así pues, la única posibilidad aceptable es: a 2  0 (mod 4) y b2  0 (mod 4) .


Escribamos a 2  4a ' y b 2  4b' para ciertos a ' , b' .
Luego c 2  a 2b2  a 2  b2  4a'4b'4a'4b'  44a' b'a'b'  22 4a' b'a'b'

y por tanto 4a' b'a'b' debe ser tambíén un cuadrado, y por tanto 0 o 1 módulo 4.

Pero observamos que este proceso lo estamos repitiendo una y otra vez, obteniendo una cadena
decreciente de divisores de 4, con lo que, aplicando la técnica del "descenso infinito de
Fermat", llegamos a la conclusión de que la única solución aceptable es a  b  0 , y por tanto
también c  0 .

22.4.3
Está claro que (0,0,0) es solución de la ecuación. Veamos que es la única solución posible
aplicando la técnica del descenso infinito de Fermat.
Supongamos que ( x, y , z ) es solución de la ecuación.

x3  2 y 3  4 z 3  2 y 3  2 z 3 
Luego x 3 es par, y por tanto x es par: x  2 x1  x3  2 x1   8x13 , y por tanto

8x13  2 y 3  4 z 3  4 x13  y 3  2 z 3  y 3  4 x13  2 z 3  2 2 x13  z 3 
Luego y 3 es par, y por tanto y es par: y  2 y1  y 3  2 y1   8 y13 , y por tanto

8 y13  4 x13  2 z 3  4 y13  2 x13  z 3  z 3  2 x13  4 y13  2 x13  2 y13 
Luego z 3 es par, y por tanto z es par z  2z1
Así pues,
x3  2 y 3  4 z 3  2 x1   22 y1   42 z1   8 x13  2  8 y13  4  8 z13
3 3 3

 x13  2 y13  4 z13

Así pues, tenemos otra solución ( x1, y1, z1 ) de la misma ecuación, con valores más pequeños en
valor absoluto (dividiendo entre dos los respectivos números), cosa que nos lleva al absurdo
pues estamos trabajando con números enteros.

22.5.1
Supongamos que y  7 . Pasando a módulo 7 , y! 0 , 2001  1 y la ecuación se convierte en
x 2  1  6 mod 7

Pero en Z 7 esta ecuación no tiene solución. En efecto:


0 2  0 , 12  1 , 22  4 , 32  9  2 , 4 2  16  2 , 5 2  25  4 , 6 2  36  1

Así pues, y  6 . Ahora tenemos que ir estudiando los casos uno a uno:
y  6  x 2  6! 2001  x 2  2721 no es ningún cuadrado. Esto se puede deducir fácilmente
viendo 2721 que es divisible entre 3 pero no entre 9.
y  5  x 2  5! 2001  x 2  2121 no es ningún cuadrado. Como en el caso anterior, esto se
puede deducir fácilmente viendo que 2121 es divisible entre 3 pero no entre 9.
y  4  x 2  4! 2001  x 2  2025  45 2 y tenemos el par de soluciones   45, 4  .
Los casos y  3 no generan soluciones, y por tanto las únicas soluciones al problema son
  45, 4  .
Fuente: Olympiad Number Theory Through Challenging Problems (Justin Stevens), pág. 86

22.5.2
Supongamos que 3m  3n  1  x 2 para cierto entero x .
En primer lugar vemos que 3m  3n  1 es impar, luego x es impar.
Si x es impar, x 2  1 mod 8
En efecto: x  2k  1  x 2  2k  1  4k 2  4k  1  4k k  1  1 , y
2

k  1  k  1  2  4k (k  1)  0 mod 8
k  0  x  1  x 2  1  1 mod 8
Pero, por otro lado, las potencias de 3 siempre son 1 o 3 módulo 8:
30  0 , 31  3 , 32  9  1 , 33  3 ...
Luego todo número de la forma 3m  3n  1 será congruente con 1  1  1  3 , 1  3  1  5 o
3  3  1  7 , en todo caso nunca será 1 módulo 8.

Observación: No era necesario especificar los impares: En general, todo cuadrado módulo 8 es
0, 1 o 4, y ninguno de los valores 3m  3n  1 es 0, 1 o 4.

22.5.3
La clave está en pasar a módulo 13:
Por un lado, 19  6 , y por el PTF, 613  6 . Además,
 2
62  36  10  63  60  8  66  63  82  64  12
1919  619  613  6 6  6  12  72  7 .
(Otra manera de verlo sería 1919  619  612  6 7  1  6 7  6 6  6  12  6  7 )

Por otro lado, veamos las potencias x 3 mod 13 , y 4 mod 13 .
Por el Teorema de Euler, suponiendo  x , 13   1 , tenemos
 13  12  x3   x12  1 mod 13 .
4

 
Luego x3  z mod 13  1  x3  z 4 mod 13 , y la ecuación z 4  1 mod 13 tiene las
4

siguientes soluciones: z  1, 5, 8, 12 mod 13 , luego x 3  0, 1, 5, 8, 12 mod 13 .


De la misma forma, puesto que la ecuación z 3  1 mod 13 tiene las soluciones
z 3  1, 3, 9 mod 13 , los posibles valores y 4 mod 13 son x 3  0, 1, 3, 9 mod 13 .

Finalmente, solo hay que comprobar que ninguna de las combinaciones posibles para x 3  y 4
nunca es 7.

Observación. Como es habitual, no queda claro cómo “adivinar” que debemos pasar a módulo
13. De hecho, mediante el uso del ordenador observamos que el módulo 13 (y 26 y
siguientes...) es el único caso en el que se observa la “anomalía” de tener ningún valor
x 3  y 4  7 mod n  . Basta buscar una combinación “bonita” de la forma 1919  7 para contruir
un problema digamos “difícil”.

22.5.4
Pasamos a módulo 11.
Sabemos que x10  1 mod 11 para todo x con ( x ,11 )  1 .
 
Luego x5  z mod 11  x5  z 2 mod 11
2

Sabemos, aplicando el PTF o el Teorema de Euler, que x10  1 mod 11 .


Por otro lado, vemos los cuadrados módulo 11:
0 2  0 , 12  1 , 22  4 , 32  9 , 4 2  5 , 52  3 ,
6 2  3 , 7 2  5 , 82  9 , 9 2  4 , 10 2  1

Así pues, y 2  z mod 11  y 2   0 , 1 , 4 , 5 , 9 


y también se cumple z 2  1 mod 11  z  1 mod 11
 
Por lo tanto x5  z mod 11  x5  z 2 mod 11  x5  1 mod 11
2

Observamos se cumple x5  y 2  4 mod 11 para todo x, y , luego la ecuación no tiene


solución.

22.5.5
Pasamos a módulo 9.
Los residuos cúbicos módulo 9 son -1, 0 y 1.
Los residuos de grado 6 módulo 9 son 0 y 1.
Luego tenemos
a b  3  1,  0,1 mod 9 
 a b  5, 6, 7 mod 9
5 5

 5  5
ab  3  1,  0,1 mod 9 
 ab  5, 6, 7 mod 9

Pero a 5b  ab5  a 6b 6  (ab) 6  0, 1 mod 9

Comprobando todas las combinaciones posibles de 5, 6 y 7 :


5  5  7 mod 9 , 5  7  8 mod 9 , 7  7  4 mod 9 ...

Vemos que la única posibilidad aceptable es


a b  6 mod 9
5

 5
ab  6 mod 9

Pero entonces a 5b  6 mod 9  a 5b  9k  6  33k  2  3 | a 5b


Y por otro lado, ab5  6 mod 9  ab5  9q  6  33q  2  3 | ab5
En todo caso, 3 | a  9 | a 5b  a 5b  0 mod 9 o bien 3 | b  9 | ab5  ab5  0 mod 9
llegando a contradicción.
Otra forma de justificarlo sería a 5b  6 mod 9 implica que tiene exactamente un factor de 3, y
ab5  6 mod 9 implica que tiene exactamente un factor de 3, pero entonces a 5 b  ab 5 tendría
exactamente dos factores de 3, lo que es imposible porque es una potencia sexta.

Fuente de esta solución: [Link]

22.5.6
x  12  x  22  ...  x  2001 2 
k  2001 k  2001 k  2001 k  2001 k  2001
  x  k   x  2kx  k 2  x   2kx   k 
2 2 2 2

k 1 k 1 k 1 k 1 k 1

2001  2002 2001  2002  4003


 2001 x 2  2 x  
2 2
 2001 x 2  2001  2002 x  667 1001  4003  2 mod 3
Luego no puede ser ningún cuadrado perfecto (ver 7.3.9a)

22.5.7
Vemos que uno de los dos debe ser múltiple de 3.
Supongamos, por el contrario, que p  1, 2 mod 3 y q  1, 2 mod 3. Ninguna de las
combinaciones posibles es aceptable:

p  q  0
3 5

p  1, q  1   ç

 ( p  q ) 2
 2 2
 4  1

 p  q  1  (1)  2
3 5

p  1, q  2  1  

( p  q)  0
2


 p  q  (1)  1  2  1
3 5

p  2  1, q  1  

( p  q)  0
2

 p  q  (1)  (1)  0
3 5

p  2  1, q  2  1  

( p  q)  (2)  1
2 2

Así pues, al menos uno de los dos debe ser múltiple de 3, y puesto que son primos, al menos
uno de los dos debe ser igual a 3.
Supongamos que p  3 . La ecuación queda de la forma:
p 3  q 5   p  q   27  q 5  (3  q) 2  0  q 5  27 absurdo.
2

Supongamos que q  3 . La ecuación queda de la forma:


p 3  q 5   p  q   p 3  35   p  3  p 3  243   p  3 , cuya única solución
2 2 2

entera es p  7 .
Así pues, la única solución con números primos es ( 7 , 3 ).

22.6.1
xy  7 2  x 2  y 2  x 2 y 2  14 xy  49  x 2  y 2 
x 2 y 2  14 xy  49  x  y   2 xy  x 2 y 2  12 xy  49  x  y  
2 2

x 2 y 2  2  6 xy  6 2  6 2  49  x  y   xy  6  13  x  y  
2 2 2

13  x  y   xy  6    x  y  xy  6 x  y  xy  6 
2 2

Sumando y restando las ecuaciones observamos que el sistema


 x  y  xy  6  a

 x  y  xy  6  b
es equivalente al sistema
 x  y  ( a  b) / 2

 xy  6  (a  b) / 2

Veamos casos posibles:


x  y  7  x  0, y  7
a  1, b  13   
 xy  6  6  x  7, y  0
x  y  7  x  3, y  4
a  13, b  1   
 xy  6  6  x  4, y  3
 x  y  7  x  4, y  3
a  1, b  13   
 xy  6  6  x  3, y  4
 x  y  7  x  7, y  0
a  13, b  1   
 xy  6  6  x  0, y  7

Las soluciones con valores no negativos son los pares (0,7) , (7,0) , (3,4) , (4,3).

22.6.2
Primera versión.
x 2  y  1  y 2  x  1  1  x 2 y  x 2  xy 2  y 2  1 
  
xy x  y   x 2  y 2  1  xy x  y    x  y   2 xy  1
2

Realizando el cambio de variable a  xy, b  x  y , la ecuación se transforma en:
 
ab  b 2  2a  1  ab  b 2  2a  1  ab  b 2  2a  4  5 
a  b  2b  2  5
Veamos las combinaciones posibles:
a  b  2  1  xy  2  x  1, y  2
  b  3, a  2   
b  2  5  x  y  3  x  2, y  1
a  b  2  5  xy  2
  b  1, a  2   no tiene soluciones reales.
b  2  1  x  y  1
a  b  2  1  xy  10
  b  7, a  10   no tiene soluciones enteras.
b  2  5  x  y  7
a  b  2  5  x  1, y  2  x  5, y  2
  b  3   
b  2  1  x  2, y  1  x  2, y  5

Así pues, las soluciones son x  1, y  2 ; x  2, y  1 ; x  5, y  2 ; x  2, y  5 .

Segunda versión.
Realizamos el cambio de variable a  x  1 , b  y  1 .
x  a  1  x 2  a  1  a 2  2a  1
2

y  b  1  y 2  b  1  b 2  2b  1
2

x 2  y  1  y 2 x  1  1  (a 2  2a  1)b  (b 2  2b  1)a  1 
a 2b  2ab  b  ab 2  2ab  a  1  ab(a  b)  4ab  a  b  1 
ab(a  b  4)  a  b  1  ab(a  b  4)  a  b  4  5 
ab  1(a  b  4)  5
y se continúa estudiando los casos posibles como en la primera versión.

Fuente de la segunda versión: An Introduction to Diophantine Equations A Problem-Based Approach (Andreescu, Cucurezeanu, Andrica) ,
página 7.

22.6.3
xy 2  2 y 2  x  107  0  xy 2  2 y 2  x  107  ( x  2)( y 2  1)  2  107 
( x  2)( y 2  1)  105  3  5  7
Ahora estudiamos caso por caso:
x  2  1 x  2  3 x  1
 2  x  1 imposible  2  2
 y  1  3  5  7  y  1  5  7  y  36  y  6
x  2  5 x  2  7 x  5
 2  y 2  22 imposible  2  2
 y  1  3  7  y  1  3  5  y  16  y  4
x  2  3  5 x  2  3  7
 2  y 2  8 imposible  2  y 2  6 imposible
 y 1  7  y 1  5
 x  2  5  7  x  33  x  2  3  5  7  x  105
 2  2  2  2 imposible
 y 1  3 y  4  y  2  y 1  1 y  0

Las soluciones son: (1,6), (5,4), (33,2)

Nota: Tal vez más elegante hubiera sido seguir con la descomposición:
( x  2)( y 2  1)  105  ( x  2)( y  1)( y  1)  105  3  5  7

22.6.4
Sea x el número de columnas, y sea y el número de filas de este teatro.
La ecuación diofántica que se nos plantea es
xy  12 y  15 x  7 
xy  12 y  15 x  7 
( x  12 )( y  15)  12  15  7 
( x  12 )( y  15)  7  12  15  187  11  17

Estudiemos los casos:


 x  12  1  x  13
1)    xy  2626
 y  15  187  y  202
 x  12  11  x  23
2)    xy  736
 y  15  17  y  32
 x  12  17  x  29
3)    xy  783
 y  15  12  y  27
 x  12  187  x  199
4)    xy  3184
 y  15  1  y  16

Así pues, vemos que el mínimo número de butacas se obtiene para 736 (C).

22.6.5
Sea x el número de columnas, y sea y el número de filas de este teatro.
La ecuación diofántica que se nos plantea es
xy  11 y  14 x  17 
xy  11 y  14 x  17 
( x  11)( y  14 )  11  14  17 
( x  11)( y  14 )  17  11  14  171  32  19

Estudiemos los casos:


 x  11  1  x  12
1)    xy  2220
 y  14  171  y  185
 x  11  3  x  14
2)    xy  994
 y  14  3  19  y  71
 x  11  32  x  20
3)    xy  660
 y  14  19  y  33
 x  11  3  19  x  68
4)    xy  1156
 y  14  3  y  17
 x  11  171  x  182
5)    xy  2730
 y  14  1  y  15
Y vemos que el número mínimo de asientos es 660 (B).

22.6.6
Para resolver este problema utilizaremos la siguiente factorización:
2x  y  z  2 xyz   2 xy  2 yz  2 zx  1  2 x 2  12 y 2  12 z 2  1
2 2

En Internet se pueden encontrar diversas formas más o menos largas de demostrar esta
igualdad, pero todas ellas son significativamente farragosas y difíciles.

Con la identidad anterior el problema se resuelve fácilmente:


2x  y  z  2 xyz   2 xy  2 yz  2 zx  1  2023 
2 2

2x  y  z  2 xyz   2 xy  2 yz  2 zx  1  2023


2 2

2 x 2
  
 1 2 y 2  1 2 z 2  1  2023  7 17 2

Los únicos divisores de 7  17 2 que se pueden escribir de la forma 2 x 2  1 son


x  2  2x2 1  7
x  3  2 x 2  1  17

Así pues, las soluciones serán ( 2,2,3) y todas sus posibles permutaciones.

22.7.1
En primer lugar vemos que esta ecuación se verifica para todas las parejas x, y   k ,k  .
Supongamos que y   x .
 
x3  y 3  x  y  x 2  xy  y 2 , y por tanto
3 3 2
 
x  y  x  y   x  y  x 2  xy  y 2  x  y 
2

y puesto que estamos suponiendo que y   x  x  y  0 , podemos cancelar este factor para
llegar a:
x 2  xy  y 2  x  y

Y esta ecuación es equivalente a:


x  y 2  x  12   y  12  2
De aquí se deduce que
 x  y 2  2

 x  1  2
2


 y  1  2
2

x  1  1  x  2 y 1  1  y  2
x  1  2   x  1  0  x  1  y  1  2   y  1  0  y  1
2  2

 x  1  1  x  0  y  1  1  y  0
 
x  y  1
x  y   2   x  y  0
2

 x  y  1

De los 3  3  9 posibles candidatos x, y  , al final son válidas las parejas (0, 1), (1,0), (1,2),
(2,1 ), (2,2 ), a los que debemos añadir todas las parejas posibles de la forma ( k , k ) .

Fuente: An Introduction to Diophantine Equations A Problem-Based Approach (Andreescu, Cucurezeanu, Andrica) , página 13

22.7.2
Si a, b, c  2 ,
1 1 
 
ab 4

1

1 
bc 4  1 1 1 1
    1
1 1  a  b b  c c  a a  b  c  2

ca 4 
1 1
abc2  62  4   
a  b  c  2 4

Y la igualdad solo puede darse si a  b  c  2 , y se comprueba que este caso es solución de la


ecuación.
Supongamos ahora que al menos una de las tres incógnitas es menor que 2. Podemos suponer,
sin pérdida de generalidad, que a  1 .
La ecuación nos queda
1 1 1 1
   1
b 1 b  c c 1 b  c 1
De nuevo, si b, c  3 ,
1 1 
 
b 1 4

1 1 1 
 
bc 6 4  1 1 1 1
     1 . Luego 1  b, c  2
1 1  b  1 b  c c  1 b  c  1

c 1 4 
1 1 1
  
b  c 1 5 4 
1 1 1 1
El caso b  c  1 no satisface la ecuación:    1
2 2 2 1
1 1 1 1
El caso b  1, c  2 no satisface la ecuación:     1
2 3 3 2
El caso c  1 , b  2 tampoco por simetría.
1 1 1 1
Finalmente, el caso b  c  2 tampoco satisface la ecuación:     1
3 4 3 3
Así pues, el único caso aceptable es a  b  c  2

22.7.3
Primera parte: Demostración de la desigualdad.
Supongamos que no es cierto, es decir, que existen a , b, n enteros positivos tales que a  b y
ab  1  n 2 , pero a  b  4n  3

 
(a  b) 2  (a  b) 2  4ab  4n  3  4ab  4n  3  4 n 2  1  4n 2  4n  1  2n  1
2

Luego
a  b  2n  1  a  b  2n

ab
2

Por otro lado, por la desigualdad AM-GM: ab   


 2 

Y por tanto:
 a  b   2n 
2 2

n  1  ab  
2
   n
2

 2   2
llegando a contradicción.

Segunda parte. Resolución de la igualdad.


De nuevo utilizamos identidad (a  b) 2  (a  b) 2  4ab .
 
(a  b) 2  (a  b) 2  4ab  4n  3  4 n 2  1  4n 2  4n  1  2n  1  a  b  2n  1.
2

La ecuación a  b  4n  3 implica que 4n  3 sea un cuadrado perfecto impar, es decir,

4n  3  2u  1 para cierto u entero no negativo, y a  b  4n  3 


2
2u  12  2u  1 .

4n  3  2u  1  4u 2  4u  1  4n  4u 2  4u  4  n  u 2  u  1
2

a  b  2n  1
  2b  2n  1  2u  1  2n  u   b  n  u  u  u  1  u  u  1
2 2

a  b  2u  1

a  b  2u  1  a  2u  1  b  2u  1  u 2  1  u 2  2u  2

Luego el conjunto solución son todos los números a , b, n tales que


a  u 2  2u  2

b  u2 1 
 para todo entero u positivo.
n  u  u 1 
2

a  b  2u  1 

En efecto, se cumple:
  
ab  u 2  2u  2 u 2  1  u 4  2u 3  2u 2  u 2  2u  2  u 4  2u 3  3u 2  2u  2 
 
2
 u 2  u  1  1  n2  1

Fuente de esta solución: Soluciones oficiales OME.

22.7.4
x4  y3
x y
2
 
 x  y  x 4  y 3  x 2  y x  y   x 3  x 2 y  xy  y 2

Vamos a ver que si x  2 y y  4 , entonces


x 4  y 3  x 3  x 2 y  xy  y 2

y por tanto no se puede dar la igualdad.

En efecto, aplicando la desigualdad AM-GM,


x4  y3
 x4 y3  x2 y y
2

Y puesto que y  4  y  2 , y por tanto


x 2 y y  2 x 2 y  x 2 y  x 2 y  x 2 y  xy (1)
en donde hemos aplicado además que x  2  x 2  x .

Por otro lado, x  2  x 4  x  x 3  2 x 2 y y  4  y 3  y  y 2  4 y 2  2 y 2


x 4  y 3 2x3  2 y 2
  x 3  y 2 (2)
2 2

Con (1) y (2) llegamos a


x4  y3 x4  y3
x4  y3    x 3  x 2 y  xy  y 2
2 2

tal y como queríamos ver.

Luego la ecuación solo puede tener solución cuando x  1 o bien y  3 .

Primer caso: x  1. Entonces:


y  0
  
1  y 3  1  y  y  y 2  0  y 3  y 2  2 y  y y 2  y  2  y  y  2  y  1   y  2
 y  1

La única solución aceptable es x  1, y  2 . En efecto:


14  23 9
  3 1 2
12  2 3

Segundo caso: y  1 . Entonces:



x 4  13  x 3  x 2  x  12  0  x 4  x 3  x 2  x  x x 3  x 2  x  1 
La solución x  0 no es aceptable, luego x 3  x 2  x  1  0 , cuyas soluciones enteras tienen
que ser divisores de 1:
x  1  13  12  1  1  2  0
Luego no hay solución positiva.

Tercer caso: y  2 . Entonces:


x 4  23  x 3  2 x 2  2 x  2 2  x 4  x 3  2 x 2  2 x  4  0

Debemos buscar las soluciones enteras de esta ecuación entre los divisores de 4:
x  1  14  13  2  12  2  1  4  0
x  2  2 4  23  2  2 2  2  2  4  0
x  4  4 4  43  2  4 2  2  4  4  156  0

Aquí aparece de nuevo la solución x  1, y  2 y encontramos otra solución: x  2, y  2 . En


efecto:
24  23 23 2  1
 4 22
22  2 22  1

Cuarto caso: y  3 . Entonces:


x 4  33  x 3  3 x 2  3x  32  x 4  x 3  3x 2  3 x  18  0

Se verifica que ningún divisor positivo de 18 es solución de esta ecuación, por lo que las
soluciones ( x, y ) enteras positivas de la ecuación del enunciado son (1,2) y (2,2) .

Fuente de la solución: Soluciones oficiales (OMEC , pág. 424)

22.7.5
En primer lugar vemos que esta ecuación se cumple para x  y  z  5 .
Por otro lado, está claro que
1 1 1 1 1 1 1 1 1 1 3
x, y , x  5  , ,        
x y z 5 x y z 5 5 5 5
por lo tanto, al menos una de las incógnitas debe estar entre 1 y 5. Podemos suponer, sin
pérdida de generalidad que esa incógnita es la x. Veamos los casos:

a) x  1
1 1 1 3 1 1 3 2
      1   0 absurdo. No hay solución aceptable.
1 y z 5 y z 5 5

b) x  2
1 1 1 3 1 1 3 1 1 z y 1
           10 y  10 z  yz 
2 y z 5 y z 5 2 10 zy 10
 10 y  10 z  yz  100  100   y  10 10  z   100
y  10  1  y  10  2 
  y  11 , z  110   y  12 , z  60
10  z  100  10  z  50 
y  10  4  y  10  5 
  y  14 , z  35   y  15 , z  30
10  z  25  10  z  20 
y  10  10  y  10  20 
  y  20 , z  20   y  30 , z  15
10  z  10  10  z  5 
y  10  25  y  10  50 
  y  35 , z  14   y  60 , z  12
10  z  4  10  z  2
y  10  100 
  y  110 , z  11
10  z  1 

Otra manera de verlo es la siguiente:


1 1 1 1 1 1 y  10
      
y z 10 z 10 y 10 y
10 y 10 y  100  100 10  y  10   100 100
z    10 
y  10 y  10 y  10 y  10
De lo que se deduce que z  10 y que y  10 | 100

c) x  3
1 1 1 3 1 1 3 1 4 yz 4
           15 y  15 z  4 yz 
3 y z 5 y z 5 3 15 yz 15
60 y  60 z  16 yz  0  60 y  60 z  16 yz  225  255  4 y  15 15  4 z   225
 4 y  15 4 z  15   225  32  52
4 y  15  25 
  y  10, z  6
4 z  15  9 
Obtenemos las soluciones (3, 4, 60) , (3, 5, 15) , (3, 6, 10) , (3, 10, 6) , (3, 15, 5) ,
(3, 60, 4).

d) x  4
1 1 1 3 1 1 3 1 4 yz 4
         
4 y z 5 y z 5 4 15 yz 15
y la única solución aceptable es (4, 4, 10) y sus permutaciones.

e) x  5
1 1 1 3 1 1 3 1 2
       
5 y z 5 y z 5 5 5
y la única solución aceptable es (5, 5, 5)

22.8.2
En primer lugar vemos que esta ecuación es perfectamente simétrica, por lo que podemos
suponer que las incógnitas están ordenadas:

3
1 1 1 1 1 1  1  1  1   1 
1  a  b  c  1     2  1   1   1   1  1  1    1  
a b c a b c  a  b  c   a 

Así pues, para cualquier terna a  b  c de soluciones, se cumplirá la desigualdad


3
 1
2  1  
 a
3
 1 1
La función f ( x)  1   es estrictamente decreciente, pues 1 es estrictamente
 x x
3
decreciente y x es estrictamente creciente.

Por otro lado, observamos que


3 3
4 64  5  125
f (3)      2 y f (4)     2
3 27 4 64

Luego el valor de a (que, recordemos, es el mínimo de la lista) debe cumplir a  3 . Veamos


los casos posibles.

Para a  1 :
 1  1  1   1  1   1  1 
2  1  1  1    21  1    1  1    1
 1  b  c   b  c   b  c 
1 1
es imposible puesto que ,  0
b c

Para a  2 :
 1  1  1  3  1  1   1  1   b  1  c  1 
2  1  1  1    1  1    4  31  1    3  
 2  b  c  2  b  c   b  c   b  c 
4bc  3(b  1)(c  1)  3(bc  b  c  1)  3bc  3b  3c  3 
0  bc  3b  3c  3

Completamos esta última ecuación:


0  bc  3b  3c  3  bc  3b  3c  9  12  (b  3)(c  3)  12 
12  (b  3)(c  3)

Las posibilidades son (recordemos que estamos suponiendo b  c ) :


1 b 3  2  b  3
  b  4, c  15 ;   b  5, c  9
12  c  3 6  c  3
3  b  3
  b  6, c  7
4  c  3

Para a  3 :
 1  1  1  4  1  1   1  1 
2  1  1  1    1  1    6  41  1  
 3  b  c  3  b  c   b  c 
 1  1   b  1  c  1 
 3  21  1    2  
 b  c   b  c 
3bc  2(b  1)(c  1)  2bc  2b  2c  2  0  bc  2b  2c  2

De nuevo completamos cuadrados para resolver esta ecuación:


0  bc  2b  2c  2  (b  2)(c  2)  6  6  (b  2)(c  2)
1 b  2
  b  3, c  8
6  c  2
2  b  2
  b  4, c  5
3  c  2

Llegamos así a la lista final de posibles soluciones ordenadas:


a  2, b  4, c  15 ; a  2, b  5, c  9 ; a  2, b  6, c  7 ; a  3, b  3, c  8 ;
a  3, b  4, c  5

El conjunto total de soluciones son todas las permutaciones posibles que se pueden realizar con
las ternas anteriores.

Fuente de esta solución: [Link]

22.8.1
2009  7 2  41 luego
a  b  2009  7 41  a  7 41  b 
 
2
 a  7 41  b  72  41  b  2  7 41 b  72  41  b  2  7 41b
Sabemos que a es un entero, y en la parte de la derecha de la igualdad, el único elemento
susceptible de no ser entero es 41b . Luego 41b también debe ser entero, y puesto que 41 es
primo, deducimos que b es un múltiplo de 41 multiplicado por un cuadrado perfecto. De la
misma forma deducimos que a debe ser un múltiplo de 41 multiplicado por un cuadrado
perfecto:
a  41x 2 

2
 a  b  41x 2  41 y 2  41x  41 y  7 41  x  y  7
b  41 y 
Buscamos parejas 0  x, y tales que x  y  7 :
x  0, y  7  a  41  02 , b  41  72
x  1, y  6  a  41  12 , b  41  62
x  2, y  5  a  41  22 , b  41  52
x  3, y  4  a  41  32 , b  41  42
Y las tres que quedan, invertiendo los valores de x e y . Son ocho soluciones en total:
( 0 , 2009 ) , ( 41 , 1476 ) , ( 164 , 1025 ) , ( 369 , 656 ) , ( 656 , 369 ) , ( 1025 , 164 ) ,
( 1476 , 41 ) , ( 2009 , 0 )

Fuente: [Link]

22.8.3
1 2 3 1 2 3
  1   1
x y z x y z

1 2 1 2 3
Si x, y  3      1  1  para todo z  1, y la ecuación no tendrá solución.
x y 3 3 z

Caso 1. x  1 .
2 3 2 3
1   1     2 z  3 y , cuyas soluciones son todas las ternas de la forma
y z y z
x  1

 y  2k , k  1
 z  3k

1 2 3 1 1
En efecto,   1  1
1 2k 3k k k

Caso 2: y  1
1 2 3 1 3 1 3 3 1 3x  z
 1   2 1  1   1    1 
x 1 z x z x z z x xz
 xz  3x  z  0  ( x  1)( z  3)  3  0  ( x  1)( z  3)  3

Puesto que x  1  x  1  2 , la única solución posible es


x  1  3  x  2
 
z  3  1 z  2 

1 1 3
En efecto:   1
2 1 2

A partir de aquí podemos suponer x, y  1

Caso 3. x  2 .
1 2 3 2 1 3 z6
 1      4 z  y ( z  6)  yz  6 y  4 z  yz  6 y  0 
2 y z y 2 z 2z
 (4  y )( z  6)  24  0  (4  y )( z  6)  24

z  6 es positivo, luego 4  y  0  1  y  4

y  2  2( z  6)  24  z  6  12  z  6 , obteniendo la solución x  2, y  2, z  6
y  3  z  6  24  z  18 , obteniendo la solución x  2, y  3, z  18

Caso 4. y  2
1 2 3 1 3 1 3
  1       z  3x dando lugar al conjunto de soluciones
x 2 z x z x z
x  k

y  1 , k  1
 z  3k

1 2 3 1 3 k  3k  2k
En efecto:     2 2  2  2  2  0
k 1 k k k k k

Así pues, las soluciones de esta ecuación son todas las ternas siguientes:
( 1 , 2k , 3k ) , k  1 ; ( 2 , 1 , 2 ) ; ( 2 , 3 , 18 ) ; ( k , 2 , 3k ) , k  1

22.8.4
1 4 1 n  4m 1
     12 (n  4m)  m n  0  m n  12 n  48 m
m n 12 mn 12
Para resolver esta ecuación aplicamos el método de completar cuadrados:
0  m n  12 n  48 m  (n  48)(m  12 )  12  48 
12  48  (n  48)(m  12 ) 
26  32  (n  48)(m  12 )
Puesto que, además, n debe ser impar, el factor n  48 no puede ser divisible entre 2, y por
tanto las únicas posibilidades aceptables son:
1  n  48 
  n  49, m  588
2  3  m  12 
6 2

3  n  48 
  n  51, m  204
2 6  3  m  12 
32  n  48 
  n  57 , m  76
2 6  m  12 

22.8.5
1 1 3 ab 3 2018 2018
     2018 (a  b)  3ab  0  ab  a b
a b 2018 ab 2018 3 3
 2018  2018  2018 2
 0  a   b   
 3  3  32
2018 2  2018  2018 
 a   b  
 3  3 
2
3
2 2  1009 2  2018 2  3a  2018 3b  2018 

Estudiemos los posibles casos:


1  3a  2018  a  673

 
2 2  1009 2  3b  2018  b  2 2  1009 2  2  1009 / 3  2  1009  673  1358114
2  3a  2018 
  a  2020 / 3 , no es entero.
2 1009  3b  2018 
2

2 1009  3a  2018 
  a  4036 / 3 , no es entero
2 1009 2  3b  2018 

Nota: Estos dos casos anteriores se podrían haber rechazado teniendo en cuenta que
3a  2018  1 (mod 3) .

22  3a  2018 
 a  674

 b  340033
1009  3b  2018 
2

2 2  1009  3a  2018  a  2018


 
1009  3b  2018  b  1009

El resto de soluciones se obtienen intercambiando los valores. Así pues, las soluciones son:
( 673 , 1358114 ) , ( 674 , 340033 ) , ( 2018 , 1009 ) , ( 1009 , 2018 ) ,
( 1358114 , 673 ) , ( 340033 , 674 ) .

22.8.6
  
2009  x 2  y 4  x  y 2 x  y 2  a  b

Supongamos en primer lugar, que a y b no son primos entre sí. Entonces mcd (a, b)2 es un
divisor de 2009  7 2  41 , y por tanto la única posibilidad es que mcd (a, b)  7 . Luego:
a  x  y 2  7a' 
  2 x  7(a'b' )  7 es divisor de x, y puesto que es también divisor de a,
b  x  y  7b' 
2

será también divisor de y. Luego:
x  7 x' 
y  7 y '
 2

2 2
 2 2
 
2

( x  y )( x  y )  7 x'7 y ' 7 x'7 y '  7 x'7 y ' x'7 y '  7  41 
2 2 2 2

x'7 y' x'7 y'   41


2 2

La única posibilidad es
x'7 y'2  41 , x'7 y'2  1  2 x'  41  1  42  x'  21  7 y'2  41  x'  41  21  20 absurdo.

Luego a y b son primos entre sí.


x  y  a
2

  x  a  y 2  a  y 2  y 2  b  2 y 2  b  a . Luego b  a , b  a es positivo,

x  y  b
2

ba
par y es un cuadrado perfecto.
2
Veamos todas las posibilidades:
a)
a7  ba
  b  a  287  7  280   140 no es un cuadrado.
b  7  41 2

b)
a  41  ba
  b  a  49  41  8   4  2 2 , sí es un cuadrado.
b  7  7 2

x  y 2  41 

  2 x  41  49  90  x  45, y  49  45  4  y  2
2
En este caso:
x  y  49 
2

  
En efecto, 45  2 45  22  41  49  2009
2

c)
a 1  ba
  b  a  2009  1  2008   1004 no es un cuadrado.
b  7  7  41  2009  2

Luego las soluciones son x  45 , y  2 .

Fuente de esta solución: Solución oficial de la OME.

22.8.7
Primera versión.
Vamos a resolver la ecuación diofántica y 2  3x 2 y 2  30 x 2  517
Sean a  x 2 y b  y 2 . La ecuación anterior es equivalente a
b  3ab  30 a  517  b  3ab  30 a  517  b(1  3a )  30 a  10  517  10 
b(1  3a)  10 (3a  1)  507 (*)

Sea c  1 3a . Puesto que a y b son cuadrados, serán positivos, y también lo será c.

(*)  bc  10c  507  c(b  10)  507  3  13 2

22.8.8
Realizamos la sustitución a  p  q  p  a  q  p  q  a  q  q  a  2q
La ecuación se ha transformado en a  2q  a3
 
a  2q  a3  2q  a3  a  a a 2  1  aa  1a  1

Forzosamente uno de los tres factores deben ser 2.


a  2  2q  2  1  3  6  2  3  q  3  p  a  q  2  3  5
Y en efecto, 8  5  3  (5  3)3

Las otras opciones no son aceptables:


a  1  2  a  3  2q  3  2  4  24  2  12  q  12 no es primo.
a  1  2  a  1  2q  1  0  2  0  q  0 no es primo.

La única solución es p  5 , q  3 .

De todas las soluciones posibles (tomando combinaciones de 3,13,13 2 ), la única que genera
como soluciones dos cuadrados es:
b  10  3  13  b  49  7 2  c  13  1  3a  a  4  22
Y por tanto 3x 2 y 2  3ab  3  4  49  588 .

Segunda versión.
 
La ecuación original se puede escribir como y 2  10 3x 2  1  3 132 .
Observamos que, puesto que y es un entero y 3 x 2  1 es positivo, y 2  10 debe ser también
positivo. Luego y 2  10  1,3,13,39,169 ,507  , y por tanto
y 2  11,13,23,49,179 ,517  . El único cuadrado perfecto de esta última lista es 49, luego
y 2  10  39 , y por tanto 3x 2  1  12  x 2  4  3x 2 y 2  12  49  588 .

Fuente de la segunda versión: The Contest Problem Book V (George Berzsenyi)

22.8.9
Primera versión.
En primer lugar, vemos que al menos uno de estos tres números tiene que ser 1.
Supongamos, por el contrario, que a , b , c  1 . Entonces podemos escribir
a  x  1 , b  y  1 , c  z  1 con x, y , z  0 . Pero entonces:
abc  a  b  c  1  ( x  1)( y  1)( z  1)  x  1  y  1  z  1  1 
1  x  y  z  xy  yz  xz  xyz  x  y  z  4  xy  yz  xz  xyz  3
Lo cual es imposible pues x, y, z  0  xy  yz  xz  xyz  4

Así pues, podemos suponer que, por ejemplo, b  1 . Entonces la ecuación queda de la forma:
ac  a  c  2  ac  a  c  2  ac  a  c  1  3  (a  1)(c  1)  3
Y los casos posibles son:
a  1  1 a  1  3
  a  2, c  4   a  4, c  2
c  1  3 c 1  1 
Así pues, la única solución es la terna ( 1 , 2 , 4 ) y todas sus permutaciones, pues las incógnitas
a , b , c de la ecuación del enunciado son perfectamente intercambiables.

Segunda versión.
abc  a  b  c  1  abc  a  b  c  1  a(bc  1)  b  c  1 
bc  1 | b  c  1  bc  1  b  c  1  bc  b  c  2  bc  b  c  1  3
 (b  1)(c  1)  3
Veamos los casos posibles:
b  1  1
  b  2, c  4  a  1
c  1  3
b  1  2
  b  3, c  2  a  6 / 5 , y esta solución no es aceptable, pues no es entera.
c 1  1 
b  1  1
  b  2, c  2  a  5 / 3 , y esta solución no es aceptable, pues no es entera.
c  1  1

Y un último caso: b  1  0  b  1, que ya se consideró en la primera versión.

Tercera versión.
Supongamos que los tres son iguales. Entonces la ecuación queda de la forma
a 3  3a  1 , y entonces:
 
a3  3a  1  a3  3a  1  a a 2  3  1  a | 1  a  1
pero 13  3  1  1 , por lo tanto los tres números son iguales.
Los ordenamos: a  b  c , y al no ser iguales, a  c  a  1  c  a  b  1  b  c  c  c  2c .
Ahora volvemos a la ecuación del enunciado:
abc  a  b  c  1  abc  c  a  b  1  cab  1  a  b  1  c | a  b  1  2c

Solo hay dos posibilidades:


c  a  b  1  ab(a  b  1)  a  b  a  b  1  1 
ab(a  b  1)  2a  2b  2  2(a  b  1)  ab  2  a  1, b  2  c  4
2c  a  b  1  abc  2c  c  3c  ab  3  a  1, b  3  c  5 .
Pero esta solución no es aceptable pues no se satisface la ecuación original:
1 3  5  1  3  5  1
La única solución es a  1, b  2 , c  4 y todas sus permutaciones.

Fuente de las versiones 2 y 3: “Teoría de Números. Entrenamiento de Hidalgo para la Olimpiada Mexicana de matemáticas” pág. 23.

22.8.10
Pasando a módulo 8: 8m  7  n 2  n 2  7  1 (mod 8)
Las soluciones de la congruencia n 2  1 (mod 8) son: 1 , 3 , 5 , 7 (mod 8) , es decir:

n  8k  1 , n  8k  3 , n  8k  5 , n  8k  7

n2  7
Para cualquier entero k, y en todo caso n 2  7 será divisible entre 8 y por tanto: m 
8
Segunda parte:
n2  7
m  1959  n 2  7  1959  8  n 2  1959  8  7  15665
8
El primer cuadrado superior a 15665 es 125 2  15625  15625  7 , pero no sirve pues es impar
y por tanto no es divisible entre 8.
El segundo cuadrado superior a 15665 es 126 2  15876  15876  7 , pero no sirve pues es
impar y por tanto no es divisible entre 8.
El tercer cuadrado superior a 15665 es 127 2  16129  16129  7  16136  8  2017 , luego
m  2017 .

22.8.11
En primer lugar, vemos que x  0, y  0 es solución trivial de la ecuación. También vemos que
la parte izquierda es positiva, y por tanto x, y deben tener el mismo signo. Si x, y  es
solución, también lo será  x, y  , así pues, podemos suponer que ambos son positivos.

Vamos a estudiar la presencia del factor 3 en la descomposición factorial de x, y .

Supongamos que x  3m a , y  3n b , con m, n  0 y a, b no divisibles entre 3.


Podemos suponer, además, que m  n .

34 23 x 2  y 2   x 3 y 3  34 23 32 m a 2  32 n b 2   33m a 3 33n b3  23 32 m a 2  32 n b 2   33m  3n  4 a 3b3


 2332 n 32 m  2 n a 2  b 2   33m  3n  4 a 3b3  23 32 m  2 n a 2  b 2   33m  n  4 a 3b3
 
 8 3m  n a   b 2  33m  n  4 a 3b3
2

Todo cuadrado es 0 o 1 módulo 3 (ver problema #3.16a), luego la suma de dos cuadrados será
0, 1 o 2 módulo 3, es decir, no será divisible entre 3, y por lo tanto, multiplicada por 8 tampoco
será múltiplo de 3. Así pues, la parte de la izquierda no es múltiplo de 3, y por tanto,
observando la parte de la derecha, necesariamente 3m  n  4  0 , y esto solo pasa cuando
m  n  1.


Por lo tanto, la ecuación queda 8 a 2  b2  a3b3 . 
Por simetría podemos suponer que a  b

   
a3b3  8 a 2  b2  8 a 2  a 2  16 a 2  ab3  16

Las únicas opciones para b son b  1, 2 .

   
Si b  1  a  16 y 8 a 2  b2  a3b3  8 a 2  1  a3 ningún posible valor de a resuelve esta
ecuación.

Si b  2  a 23  16  a 8  16  a  2 . La pareja b  2, a  1 no soluciona 8 a 2  b2  a3b3 .  


Finalmente, la pareja b  2, a  2 sí es solución de 8 a 2  b2  a3b3 :  
 
8 22  22  64  43  2323

Luego x  3m a  3  2  6 , y  3n b  3  2  6 , es la única solución posible (junto a x  0, y  0


y x  6, y  6 ).
Fuente de esta solución: Solución oficial.

22.8.12
Está claro que xy  61  0  x 3  y 3  x  y .
Aplicando la identidad x  y   x 3  3xy( x  y )  y 3  x 3  y 3  x  y   3xy( x  y )
3 3

Transformamos la ecuación anterior en


x  y 3  3xy( x  y)  xy  61
Realizamos el cambio de variable
a  x  y  0

b  xy  0

La ecuación se transforma en a 3  3ab  b  61  a 3  3ab  b  61  a 3  b(3a  1)  61


Vamos dando valores de a :
a  1  13  b(3  1  1)  61  b  30
a  2  23  b(3  2  1)  61 no tiene solución entera
a  3  33  b(3  3  1)  61 no tiene solución entera
a  4  43  62  61 la ecuación a 3  b(3a  1)  61 no tiene solución.

a  x  y  1
Así pues, la única solución es   x  6, y  5
b  xy  30

Observación. Otro razonamiento alternativo es: Puesto que x  y


x 3  y 3   x  y x 2  xy  y 2   xy  61  x 2  xy  y 2  xy  61  x 2  y 2  61 
 61  x 2  y 2  2 y 2  y  1,2,3,4,5
Y a partir de aquí vamos probando estos candidatos hasta dar con el único y aceptable.

22.8.13
x 2  y x  y 2   x  y 3 
x 3  y 3  xy  x 2 y 2  x 3  3 x 2 y  3 xy 2  y 3 
2 y 3  xy  x 2 y 2  3 x 2 y  3 xy 2  0 
 
y 2 y 2  x  x 2 y  3 x 2  3 xy  0

Puesto que estamos suponiendo y  0 llegamos a


2 y 2  x  x 2 y  3x 2  3xy  0

Esta ecuación la podemos interpretar como una ecuación de segundo grado en y:


a  2
2 y 2  x 2  3x y  3x 2  x  0  ay 2  by  c  0 , b  x 2  3x

c  3x 2  x

b 
y por tanto y  ,   b 2  4ac
2a

Estudiemos el discriminante  , que deberá ser un cuadrado perfecto:


  b 2  4ac  ( x 2  3 x) 2  4  2  (3 x 2  x)  x 2 ( x  3) 2  8 x(3 x  1) 

 x x( x  3) 2  8 (3 x  1) 
x( x  3) 2  8 (3x  1)  x3  6 x 2  15 x  8  x  1 x  8
2
Luego   x x  1 x  8
2

  x x  1 x  8  x  1 x x  8
2
Y por tanto

Así pues, x( x  8) deberá ser un cuadrado perfecto. A partir de aquí podemos utilizar la técnica
el problema 4.1.6:

x( x  8)  n 2  x 2  8 x  n 2  0  x 

8  64  4  n 2
8  64  4n 2

2 2
8  2 16  n 2
  4  16  n 2  4  42  n 2
2

Luego 4 2  n 2 debe ser un cuadrado perfecto:


4 2  n 2  m 2  42  m 2  n 2   m  n  m  n 

Puesto que m  n  0 , se tiene que cumplir m  n  0  m  n


Veamos las diferentes posibilidades:
m  n 1  
  2 m  17  m  17 / 2 no es entero
m  n  16 

m  n  2

  2 m  10  m  5 , n  3 absurdo
m  n  8

m  n  4

  2 m  8  m  4 , n  0  m  4 , n  0
m  n  4

m  n  8

  2 m  10  m  5 , n  8  5  3
m  n  2

m  n  16 

  2 m  17  m  17 / 2 no es entero
m  n 1  
Todas las combinaciones que generan el producto de dos negativos llevan al absurdo:
m  n  k1 
  2 m  k1  k2  0 absurdo
m  n  k2 

Las únicas posibilidades aceptables son


a) m  4 , n  0 , y por tanto:
8
x  4  42  n 2  4  42  02  4  42  4  4  
0
b) m  5 , n  3  n  3
9
x  4  42  n 2  4  42  32  4  25  4  5  
 1

Así pues, llegamos a las posibles soluciones x   1 , 0 , 8 , 9 . Veamos caso a caso:


x  1    x x  1 x  8  0
2

b  x 2  3x  1  3  4
b  4
y   1
4 4
x  0    x x  1 x  8  0 esta solución se descarta en el enunciado.
2

x  8    x x  1 x  8  0
2

b  x 2  3 x  64  24  40
 b    40
y   10
4 4

x  9    x x  1 x  8  9  10 2  1  900
2

b  x 2  3 x  81  27  54
 b    54  900  54  30  21
y   
4 4 4  6

Así pues, hemos llegado a las soluciones   1 ,  1  ,  8 ,  10  ,  9 ,  21  ,  9 ,  6 .


22.8.14
Multiplicamos la ecuación por 4, completamos cuadrados y aplicamos la fórmula "diferencia de
cuadrados":
x 6  3x 3  1  y 4  4 x 6  12 x 3  4  4 y 4 
2 x   2  2 x  3  3  9  4  4 y  2 x  3  5  4 y
3 2 3 2 4 3 2 4

2 x  3  4 y  5  2 x  3  (2 y )  5 
3 2 4 3 2 2 2

2 x  3 (2 y ) 2 x  3 (2 y )  5


3 2 3 2

Ahora solo queda ir viendo las diferentes posibilidades:



2 x3  3  2 y 2  1 
  4 y  4  y  1  y  1  2 x  3  2 1  5  x  0
2 2 3

2 x  3  2 y  5
3 2

2 x  3  2 y  5
3 2

  4 y  4 absurdo
2

2x  3  2 y  1
3 2

2 x  3  2 y  1
3 2

  4 y  4 absurdo
2

2 x  3  2 y  5
3 2

2 x  3  2 y  5
3 2

  4 y  4  y  1  y  1
2 2

2 x  3  2 y  1
3 2
 absurdo.
 2 x 3  3  2 1  5  2 x 3  6  2 x 3  6  x 3  3

La única solución aceptable es x  0, y  1 .


Efectivamente, estos valores satisfacen la ecuación:
x  0, y  1  06  3  03  1  1  14
x  0, y  1  06  3  03  1  1  (1) 4

Fuente de esta solución: [Link] (GG Maths)

22.8.15
Está claro que x  0, y  0 es solución trivial de la ecuación.
Si x  0 la ecuación se reduce a py  0  y  0 , con lo que llegamos a la solución trivial.
Si y  0 por el mismo razonamiento llegamos a la misma solución (la ecuación es simétrica).

px  y   x y  p | x y  p | x o p | y
Supongamos que p | x  x  p x' , y por tanto

px  y   x y  p px' y   px' y  px' y  x' y  px'  x' y  y  ( x'1) y  p | ( x'1) y

Luego p | ( x'1) o p | y .

Si p | y , entonces y  py' y por tanto la ecuación queda


px'  ( x'1) y  px'  ( x'1) py'  x'  ( x'1) y '

Esta última ecuación solo tiene soluciones x'  0, y '  0 y x'  2, y '  2 , es decir, la solución
trivial y x  2 p , y  2 p .

En efecto: p2 p  2 p   4 p 2  2 p  2 p  x y .

Nos queda estudiar el caso p | ( x'1) , es decir x '1  pk , y la ecuación queda


pk  1 1
px'  ( x'1) y  p( pk  1)  pky  pk  1  ky  y   p
k k

La única posibilidad que y sea entero es k  1 , luego y  p  1 , x '  p  1 , x  p ( p  1)

En efecto:
   
p p( p  1)  p  1  p p 2  p  p  1  p p 2  2 p  1  p( p  1)2  p( p  1)( p  1)  xy

Luego las soluciones son tres:


x  y  0 ; x  y  2 p ; x  p ( p  1), y  p  1

Observación.
En las soluciones oficiales (SE, pág. 849) se presenta un razonamiento parecido:
px' y  x' y  0  x' y  px' y  x' ( y  p)  y  p  x' ( y  p)  y  p 
 x'1 | p
 x' ( y  p)  ( y  p)  ( x'1)( y  p)  
y  p | p
 x'1  p
 x'1   p

Y puesto que p es primo, tenemos que x'1 | p  
 x'1  1
 x'1  1
22.8.16
El valor y es solución si y solo si lo es  y , así que podemos suponer y  0 .
Si x  0 entonces la ecuación es y 2  1  20  220 1  1  1  2  4  y  2 , llegando a las
soluciones: 0,  2 .

Supongamos que x  0 .

 
1  2 x  22 x 1  y 2  2 x 1  2  2 x  2 x  2  22 x  2 x  22 x 1  y 2  1  ( y  1)( y  1) 
 
2 1  2  2  ( y  1)( y  1)
x x

De esta última igualdad deducimos que y tiene que ser impar, luego y  1 , y  1 son pares.
Pasando a módulo 4, tenemos
y  4q  0 descartado, pues es par.
y  4q  1  y  1 es múltiple de 4 , y  1 es múltiple de 2 .
y  4q  2 descartado, pues es par.
y  4q  3  y  1 es múltiple de 4, y  1 es múltiple de 2 .
En todo caso, ( y  1)( y  1) es múltiplo de 8, pero 1  2  2 x es impar, luego x  3 .
y  4q  1  y 2  1  16 q 2  8q  1  1  16 q 2  8q  8q2q  1 y la ecuación queda

 
2x 1  2  2x  y 2  1 
2 1  2  2   8q2q  1  2 q2q  1 
x x 3

2 1  2  2   q2q  1  q | 2  q  2
x 3 x x 3 x 3
k  y  4q  1  22 2 x  3 q  1  2 x 1 k  1

Además, k debe ser impar, pues en caso contrario


y  2 x t  1  y 2  22 x t 2  2 x 1t  1  y 2  1  22 x t 2  2 x 1t  2 x (2 x t 2  2t )
y la ecuación queda
 
2 x 1  2  2 x  y 2  1  2 x (2 x t 2  2t )  1  2  2 x  2 x t 2  2t , impar igual a par, absurdo.

Así pues, y  2 x 1 k  1 o bien y  2x 1 k  1 para cierto k  1 impar.

Primer caso: y  2x 1 k  1

Entonces y 2  1  22 x  2 k 2  2 x k  1  1  2 x 2 x  2 k 2  k y la ecuación queda 
2 1 2
x
 x 1
  2 2x x2

k  k 1 2
2 x 1
2 x2
k k 
2

1  k  2 x  2 k 2  2 x 1  2 x  2 k 2
 23 
1  k  2x2 k 2  8   (*)

 
Luego k  1  2 x  2 k 2  8  0  k 2  8  0  k 2  8  k  1 , 2

Hemos dicho anteriormente que k era impar, luego solo nos queda la posibilidad k  1 .


Pero sustituyendo k  1 en (*) tenemos 1  1  2 x  2 12  8  0  2 x  2  7 absurdo. No hay 
ninguna solución posible.
Segundo caso: y  2x 1 k  1
 
De la misma manera, Entonces y 2  1  22 x  2 k 2  2 x k  1  1  2 x 2 x  2 k 2  k y la ecuación
queda
2 x 1  2 x 1   2 x 2 x  2 k 2  k   1  2 x 1  2 x  2 k 2  k 
 
1  k  2 x  2 k 2  2 x 1  2 x  2 k 2  23 

1  k  2x2 k 2  8  (**)

Luego
1  k  2 x  2 k 2  8  2k 2  8 
2k 2  k  17  0  k  3  k  1 , 3

Sustituyendo en (**) vemos que k  1 no es aceptable:


 
2  1  1  2 x  2 12  8  2 x  2  7

Así que la única opción aceptable es k  3 , que sustituyendo en (**) nos determina el valor x :
 
1  3  2 x  2 32  8  4  2 x  2 1  x  4

Y finalmente y  2 x 1 k  1  24 1  3  1  24  1  23

Las soluciones son, por tanto, x  4 , y  23 .

Comprobemos que, efectivamente, los valores obtenidos satisfacen la ecuación:


1  24  224 1  1  16  512  529  23 2 .

El problema tiene cuatro soluciones: x  0, y  2 y x  4, y  23 .

Fuente de la solución: Soluciones oficiales (SE, página 880)

22.8.17
En primer lugar pasamos la igualdad a módulo 3:
3x  4 y  5z  3x  4 y  5z (mod 3)  0  1y  (1) z (mod 3)  1  (1) z (mod 3)

De aquí deducimos que z debe ser par: z  2k para cierto k  IN .


3x  4 y  52k  3x  52k  4 y  5k  2 y      5
2 2 k

 2 y 5k  2 y 
Por lo tanto los dos factores de la parte derecha deben ser ambos potencias de 3.
Pero estos dos factores no pueden ser ambos múltiples de 3, pues aplicando el Algoritmo de
Euclides:
 a,b    a, a  b 
En nuestro caso, sumando ambos factores:
  
5k  2 y , 5k  2 y  5k  2 y , 2  5k 
Y claramente 2  5k no es múltiple de 3.
La única opción válida es que sean factores coprimos, y por tanto

5  2  3
k y x

 k

5  2  1
y

Restamos las dos ecuaciones para llegar a


2  2 y  3x  1  2 y 1  3x  1 (*)

Ahora observamos que si y  3 entonces 2 y 1 es un múltiplo de 16 y por tanto, pasando a


módulo 16 llegamos a
0  3x  1 (mod 16)  1  3x (mod 16)

Estudiemos las potencias de 3 módulo 16:


x  0  30  1 (mod 16)
x  1  31  3 (mod 16)
x  2  32  9 (mod 16)
x  3  33  27 (mod 16)
x  4  34  81  5  16  1  1 (mod 16)
x  5  35  243  3 (mod 16)

Vemos que generan un ciclo, es decir, que x ha de ser un múltiple de 4: x  4s para cierto
s  IN . Luego la ecuación se convierte en
2 y 1  3x  1  34 s  1  81s  1  81s  2 y 1  1

Pero ahora, pasando a módulo 5, tenemos


2 y 1  1  1s  1 (mod 5)  2 y 1  0 (mod 5)
Lo cual es imposible, pues ninguna potencia de 2 es múltiple de 5.

Con todo esto hemos demostrado que la hipótesis inicial y  3 es inaceptable, y por tanto
y  3 , y ahora podemos testear caso por caso en la ecuación (*):
y  1  211  4  3x  1  4  1  3x  5  3x no tiene solución.
y  2  22 1  8  3x  1  8  1  3x  9  3x  x  2
Y resolvemos el sistema:
5k  2 y  3x
 5k  4  9
 5k  9  4  5

 k  k  k  k  1  z  2k  2

5  2  1
y

5  4  1 
5  1  4  5

Así pues, la única solución aceptable de este problema es la conocida terna pitagórica
32  42  52 .

22.8.18
3m  2n  1

  3  1 2  1  m  0  3  3
m n m

2 0
n


Por otro lado, 3m  2 n  1  2 n  3m  1  3  1  2  n  1

Supongamos, en primer lugar, que n  1 . Entonces 3m  1  21  3  m  1 , y la solución es


m  1 , n  1.

Supongamos ahora que n  1 , entonces 2 n es múltiplo de 4, y por tanto:


2n  3m  1  0 mod 4  3m  (1) m  1 mod 4  m  2k , es decir, es un número par.

 
2
 
3m  2n  1  2n  3m  1  32k  12  3k  12  3k  1 3k  1 
Así pues, 3k  1 y 3k  1 son dos potencias de 2, y difieren en dos unidades, y esto solo puede
ocurrir para

3k  1  4 
  k  1  m  2  n  3 , que es la otra solución de esta ecuación.
3  1  2
k

Nota: Se debe justificar que solo 4 y 2 son las potencias de 2 que difieren en dos unidades:
2b  1 , 2ab  1  2  imposible
2  2  2  2 2  1  2   b
a b b a b 
 a b
2  2 , 2  1  1  b  1 , a  2
22.8.19
Empezamos como es habitual manipulando algébricamente la ecuación:
1  x 2 y  x 2  2 xy  2 x  y 
x 2 y  x 2  2 xy  2 x  y  1 
x xy  x  2 y  2   y  1 
x  x  2  y  1  4  y  1

Si x  0  y  1, que es una solución de la ecuación. Supongamos que x  0 .

De aquí deducimos que x es un divisor de y  1 , es decir, y 1  a x para cierto entero a .


Luego:
x x  2a x  4  a x

Como estamos suponiendo que x  0 podemos cancelar dicho término y llegar a


x  2a x  4  a 
x  2a x  a  4 
a x  2 x  1  4 

a x2  2x 1  4
Ahora ya solo nos queda analizar todas las posibilidades:
a  1
 2 La segunda ecuación tiene soluciones irracionales.
x  2x 1  4
a  2

 2  x  1  y  1  2(1)  y  1
x  2x 1  2  x  3  y 1  2  3  y  7
 

a  4
 2 La segunda ecuación tiene soluciones irracionales.
x  2x 1  1

 a  1
 2 La segunda ecuación tiene soluciones complejas.
 x  2 x  1  4

a  2
 2
 x  2 x  1  2  x  1  y  1  (2) 1  y  1

a  4

 2  x  0 (descartada)
 x  2 x  1  1  
  x  2  y  1  (4)  2  y  7

Completando así todas las cinco soluciones posibles:


 0 ,1  ,  1,1  ,  3 ,7  ,  1,1  ,  2 ,7 
Fuente de esta solución: [Link] (GG Maths)

22.8.20
Dado un valor de n, sea T el número de casas que empiezan por “2”.
Vamos a dividir el problema en casos diferentes en función del valor de n.
a) 100  n  199
Los únicos números que empiezan por el dígito “2” son el 2, y entre 21 y 29, once valores,
luego
T 11 1
   11k  n  11 | n
n n k
Luego serán todos los múltiplos de 11: Del 11 10  110 al 11 18  198 , nueve casos.

b) 300  n
Los únicos números que empiezan por el dígito “2” son 2, del 21 al 29, diez valores, y los
números entre 200 y 299, 100 valores, luego hay 111 casas que empiezan por “2” y por tanto
T 111 1
   111k  n  111 | n
n n k
Luego serán todos los múltiplos de 111: 333, 444, 555,… 999, 7 casos en total.

c) 200  n  299
En este caso T depende del valor de n: Hay 11 casos fijos (el 2 y del “21” al “29”) y n  200  1
casos entre “200” y n. Luego T  11  n  200  1  n  188 y por tanto
T n  188 1
   k n  188   n
n n k
Para resolver esta ecuación diofántica completamos cuadrados:
T n  188 1
   k n  188   n  kn  188 k  n  0
n n k
0  kn  188 k  n  k  1n  188   188  k  1n  188   188  22  47
Vamos estudiando los casos uno a uno:
k  1  1 , n  188  22  47 absurdo.
k  1  2 , n  188  2  47  94  n  188  94  282 , k  3
k  1  22 , n  188  47  n  188  47  235 , k  5
k  1  47 , n  188  22  n  188  4  192  200 absurdo.
k  1  2  47 , n  188  2  n  188  2  190  200 absurdo.
k  1  22  47 , n  188  1  n  188  1  189  200 absurdo.

Luego las soluciones posibles son dos: n  282 , k  3 y n  235 , k  5

Finalmente, tenemos 9  7  2  18 casos aceptables.

Observación.
Otra forma alternativa de resolver el apartado (c) es la siguiente:
Sea 200  n  299
Sea x  n 199
T x  11 1 x  199 x  11  188 x  11 188 188
  k    1 
n x  199 k x  11 x  11 x  11 x  11 x  11
188
k 1 
x  11
Luego
k  1 | 188  22  47 , y los factores aceptables de 188 son 47 y 94.

22.9.1
Supongamos que

p  q  a
2



 p  7q  b
2

Para ciertos a, b que podemos suponer positivos. Luego


6q  p  7q  ( p  q)  b 2  a 2  b  a b  a 

Luego b 2  a 2 es par, y por tanto a y b tienen la misma paridad. Pero entonces b  a b  a  es
el producto de dos números pares, luego es múltiplo de 4. De aquí se deduce que q también
debe ser par, y el único par primo es 2. Así pues, q  2 , y por tanto:
b  ab  a  12
De todas las soluciones posibles del sistema (ver Apartado 10.3)

b  a  2 3
x y


 2  x 1 y
b  a  2 3

La única con soluciones positivas es a  2, b  4 , y finalmente p  a 2  q  2 2  2  2 .


22.9.2
Está claro que si p  q entonces p 2  7 pq  q 2  9 p 2  (3 p) 2 es siempre un cuadrado
perfecto. Supongamos que p  q .
Supongamos que p 2  7 pq  q 2  r 2 para cierto entero r . Luego
r 2  p 2  7 pq  q 2   p  q   5 pq 
2

5 pq  r 2   p  q   r  p  q r  p  q 
2

Puesto que a la izquierda tenemos 3 números primos, está claro que uno de los dos factores de
la derecha es igual a uno de ellos y el otro al producto de los otros dos.
El factor r  p  q es mayor que 5, luego tendrá que ser el producto de dos de los primos de la
izquierda, tal vez de los tres, mientras que el factor r  p  q deberá ser igual a p, q, 5 o 1.

Veamos los casos:

Caso 1.
r  p  q  p  r  2 p  q , y la ecuación original se convierte en
p 2  7 pq  q 2  2 p  q   4 p 2  4 pq  q 2  0  3 p 2  3 pq  3 p p  q  
2

p  0

p  q  0
Ninguno de estos dos casos es aceptable.

Caso 2.
r  p  q  5  r  p  q  5 , y la ecuación
5 pq  r  p  qr  p  q
Se convierte en:
pq  p  q  5  p  q  2 p  2q  5  5  2 p  2q  pq  ( p  2)(q  2)  4 
9  ( p  2)(q  2)
Las soluciones de esta última ecuación son p  q  5 ; p  3 , q  11 ; p  11 , q  3 .

Caso 3.
r  p  q  1  r  p  q  5 pq , y restando ambas ecuaciones obtenemos
2 p  2q  5 pq  1  2 p  2q  1  5 pq

Lo cual es imposible porque el lado de la derecha es mayor que el lado de la izquierda (solo se
podría cumplir para p  q  1 , caso que está descartado.

Así pues, las soluciones son p  q ; p  3 , q  11 ; p  11 , q  3

Fuente de esta solución: [Link]

22.9.3
p 1 p2 1
Supongamos que a 2
y  b 2 para ciertos enteros a , b .
2 2

Entonces, restando ambas expresiones tenemos


p p  1  p 2  p  p 2  1   p  1  2b 2  2a 2  2(b 2  a 2 )  2(b  a)(b  a) (*)

Por ser p un número primo, se debe cumplir p | b  a o p | b  a .

Pero
2b 2  p 2  1  p 2  p 2  2 p 2  b  p
2a 2  p  1  2 p 2  a  p

Luego b  a  p y b  a  2 p

De b  a  p se deduce que no se puede cumplir p | b  a , por lo que, forzosamente, p | b  a .


Pero teniendo en cuenta que b  a  2 p solo se puede cumplir b  a  p , y por tanto la
ecuación (*) se convierte en el sistema
a  b  p
  2(b  a)  a  b  1  2b  2a  a  b  1  b  3a  1  b  3a  1
2(b  a)  p  1

Y por tanto:
a  b  p  a  3a  1  p  4a  1  p

Luego:
p 1 4a  1  1
 a2   a 2  2a  a 2  a  2  p  4  2  1  7
2 2

7 1 72  1
Así pues, p  7 es la única solución aceptable. En efecto,  22 y  52
2 2
Fuente de esta solución: Soluciones oficiales (OMEC, página 433)

22.9.4
Primera versión.
Supongamos que 2n 2  d k con k  1 .
Supongamos, por otro lado, que n 2  d  m 2 para cierto entero m .
Por un lado, n 2  d  m 2  m 2  n 2
Pero por otro lado,
n 2  d  m 2  k n 2  d   k m 2  k n 2  k d  k m 2  k n 2  2n 2  k m 2 
n 2 k  2   k m 2  k n 2  (k  2) n 2  k m 2  k n 2  k m 2  n 2  m 2

Llegando a contradicción.

Segunda versión.
Supongamos que 2n 2  d k con k  1 .
Supongamos, por otro lado, que n 2  d  m 2 para cierto entero m .
2 2 2n 2 
  n 2 k 2  2k 
2n 2
Entonces m  n   k m   k  n 
2 2 2

k  k 
Luego k 2  2k también debe ser un cuadrado perfecto, pero esto es imposible, pues este
número está entre los cuadrados de dos enteros consecutivos. En efecto:
k 2  k 2  2k  k 2  2k  1  (k  1) 2
22.9.5
Para p  2 , 2 p 4  p 2  16  44 y no es un cuadrado perfecto.
Para p  3 , 2 p 4  p 2  16  169  13 2 y sí es un cuadrado perfecto.
Para todo primo p  3 , está claro que no puede ser múltiple de tres, luego p  1 , 2 mod 3
p  1 mod 3  2 p 4  p 2  16  2  1  1  1  2 mod 3
p  2 mod 3  2  24  22  16  2  1  1  1  2 mod 3
Por otro lado,
02  0 mod 3 , 12  1 mod 3, 22  4  1 mod 3
Luego es imposible que 2 p 4  p 2  16  k 2 .
Así pues, p  3 es la única solución.

Fuente de esta solución: Mathematical Excalibur Volume 21, Number 4, 2018

22.9.6
La ecuación es equivalente a
4x  14 y  1  4z 2  1
Sea p un factor primo de 4 x  1. Entonces 4 z 2  1 mod p  . En efecto:
p | 4 x  1  0  4 x  14 y  1  4 z 2  1 mod p   4 z 2  1 mod p 

Luego 2 z   1 mod p 
2

Por otro lado, aplicando el Pequeño Teorema de Fermat, 2 z   1 mod p  , y por tanto:
p 1

1  2 z 
p 1

 2 z  
2 ( p 1) / 2
  1
( p 1) / 2
mod p

Y por tanto p  1 mod 4 . Aquí hemos aplicado 1  (1) k mod p   k es par, y por tanto
p 1
 2a  p  1  4a  p  4a  1  p  1 mod 4
2

Así pues, todo factor p primo de 4 x  1 cumple p  1 mod 4 , por lo tanto, el producto de
todos estos factores, que es el propio 4 x  1 también lo será:
4x 1  1 mod 4

Lo cual nos lleva al absurdo  1  1 mod 4.

22.9.7
n(n  1)
Es de sobra conocido que tn  1  2  3  ...  n  , luego queremos determinar los
2
n(n  1)
números de la forma que son cuadrados perfectos:
2
n(n  1) n
tn   m2  n(n  1)  2m2  (n  1)  m2
2 2

para cierto entero positivo m .


Puesto que dos números consecutivos son coprimos, de la igualdad n(n  1)  2m 2 podemos
esperar que ambos números sean “casi” cuadrados perfectos, exceptuando un factor 2 “extra”:

a 2  n es un cuadrado perfecto y n  1  2b 2 es otro cuadrado perfecto multiplicado por 2.

Puesto que n es par, n+1 será impar, y por tanto su cuadrado será impar. Esto nos evita
chequear cuadrados de números pares.

a  3  n  a 2  32  9  n  1  8  2  4  2  2 2
a  5  n  a 2  52  25  n  1  24  2  12
a  7  n  a 2  7 2  49  n  1  48  2  24
a  9  n  a 2  9 2  81  n  1  80  2  40
a  11  n  a 2  112  121  n  1  120  2  60
a  13  n  a 2  13 2  169  n  1  168  2  84
a  15  n  a 2  15 2  225  n  1  224  2  112
a  17  n  a 2  17 2  289  n  1  288  2  144  2  12 2

Ya hemos encontrado un número que se adapta a nuestras condiciones:


2  12  17   2  12 2  17 2  288  289
2

Y por tanto:
288  289
t8   41616  4  1  6  1  6  18 (D)
2

22.10.1
Denotando p a las personas, h a los caballos, s a las ovejas, c a las vacas y d a los patos,
tenemos
p  3h 
 d  9h 
s  4c     T  p  h  s  c  d  3h  h  4c  c  9h  13h  5c
 s  4c 
d  3 p

Aplicando ahora el teorema “Chicken McNugget”, el valor máximo no representable es


T  13  5  13  5  47 , luego la solución es (B).

22.10.2
El problema que se plantea es estudiar todas las combinaciones posibles

x  5  y  n  z  n  1

con enteros x, y , z  0

Y determinar aquellos enteros positivos n para los cuales 91 no se puede formar pero sí se
pueden formar 92, 93, 94…
Está claro que, independientemente del valor n , se pueden formar todos los múltiplos de 5,
pues basta ir dando valores a x , con y  z  0 .

Para un valor n cualquiera, está claro que podremos formar seguro todos los valores de la
forma n , n  5 , n  10 , n  15 ,… También todos los valores de la forma n  1 ,
n  1  5  n  6 , n  1  10  n  11 ,…

Todo esto nos indica que la clave para resolver este problema es pasar a módulo 5:

1 2 3 4 5
6 7 8 9 10
11 12 13 14 15

91 92 93 94 95
96 97 98 99 100

Aplicamos el Teorema “Chicken McNugget”, a las combinaciones 5 y n:


91  5n  5  n  n  24
Luego podemos garantizar que n  24 .

Primer caso: n  1 mod 5


Supongamos que n se encuentra en la primera columna, es decir, que n  1 mod 5 .
Vemos que 91 también está en la primera columna, luego nos obliga a n  91 , es decir:
n  96 ,101 , ...
Pero entonces no podríamos obtener 92 (ni 93, 94…) con lo que dicho número no es aceptable.

Segundo caso: n  2 mod 5

Si n se encuentra en la segunda columna, n  1 se encontrará en la tercera, y por tanto 2n se


encontrará en la cuarta. En efecto:
n  5k  2  2n  25k  2  10k  4  4 mod 5

También vemos que 2n  1  2n  2  1 mod 5 se puede obtener, y está en la primera


columna.
Luego 2n  1  91, y el primer candidato interesante sería el siguiente: 2n  1  96

2n  1  96  n  47

Comprobamos que, efectivamente, n  47 satisface las condiciones del enunciado:


Están todos los múltiplos de 47, en particular 92  47  45  47  9  5 , todos los múltiplos de
48, 2  47  94 y todos los múltiplos de 94, y observamos que 91 no se puede representar como
combinación de 47, 48 y 5, pero sí podemos representar 96  2(47  1)  2  48 .

1 2 3 4 5
6 7 8 9 10
11 12 13 14 15

47 48

91 92 93 94 95
96 97 98 99 100

Tercer caso: n  3 mod 5

n  3 mod 5  2n  6  1 mod 5 , y por tanto 2n se encontrará en la primera columna. Como


en el caso anterior, deducimos que 2n  91 y el primer candidato interesante sería el siguiente:
2n  96  n  48

Pero con este valor nos encontramos con un problema en la segunda columna: No se puede
representar 92, y por tanto no es válido.

En efecto, todo consiste en ir viendo las posibles combinaciones:


x  0, y  1, z  1  x  5  y  48  z  49  97
x  1, y  1, z  1  x  5  y  48  z  49  102
x  9, y  1, z  0  x  5  y  48  z  49  93

Cuarto caso: n  4 mod 5

En este caso n  1  5 mod 5 y es redundante, 2n  8  3 mod 5 , 3n  12  2 mod 5 y


4n  1 mod 5 . Como en los casos anteriores, imponemos la condición

4n  96  n  24

Y vemos que con este valor todas las cinco columnas están cubiertas, es decir, es una segunda
solución al problema.

Quinto caso: n  5  0 mod 5


En este caso tendríamos n  1  1 mod 5 , es decir, en la primera columna, y por tanto
n  1  91  n  92 . El primer candidato sería n  95 , pero con este valor no podríamos formar
92 (ni 93…), luego no es aceptable.

Así pues, las soluciones de este problema son n  24 y n  47 .

Fuente de esta solución: Vídeo “Momentum Learning” [Link]

22.10.3
Si (a, b)  1 existirían infinitas puntuaciones no alcanzables, pues todas las puntuaciones
posibles serían múltiples de ( a , b ) . Así pues, (a, b)  1 .
En este caso, podemos aplicar el teorema anterior que dice que existirán exactamente
(a  1)(b  1)
puntuaciones no alcanzables:
2
(a  1)(b  1)
 30  (a  1)(b  1)  70  2  5  7 .
2
Puesto que además sabemos que a  b y (a, b)  1 , las posibilidades son las siguientes:
a  1  10, b  1  7  a  11, b  8
a  1  70, b  1  1  a  71, b  2
Puesto que 58  0  71  29  2 , descartamos la segunda opción, y por tanto la única solución
válida es a  11, b  8 .

Fuente de esta solución: Number Theory for Mathematical Contests (David A. Santos) pág. 57

22.10.4
La columna más baja será de 4  94  376 , y la columna más alta será de 19  94  1786 .
Como nos piden determinar el número de alturas diferentes, podemos simplificar el problema
con el siguiente argumento:
Cada ladrillo de 4×10×19 aportará un incremento de altura de 4, 10 o 19, o equivalentemente,
un incremento de altura de 0, 6 y 15. Pero estos tres números son múltiples de 3, luego
podemos reducir nuestro problema a determinar todos los incrementos de altura de 0, 2 y 5.

Así pues, tenemos una colección de incrementos entre 0  94  0 y 5  94  470 .

Por un lado, no todas las combinaciones posibles de incrementos n  2 x  5 y son posibles.


Sabemos por el Teorema de Frobenius que se podrán dar todos los incrementos
n  25  2  5  3

Para valores menores de 3 determinamos los casos posibles uno a uno:


0  2 0  5 0
2  2 1  5  0

Luego no serán aceptables los incrementos 1 y 3.

Pero también debemos considerar que tenemos un tope de 470. Y vemos que es lo mismo subir
2 que bajar 3. Por ejemplo, los valores más altos son:
464  2  2  92  5  470  2  3  0  5
465  0  2  93  5  470  0  3  1 5
466  ???
467  1 2  93  5  470  1 3  0  5
468  ???
469  ???
470  0  2  94  5  470  0  3  0  5

Es decir, tenemos que descartar las combinaciones imposibles de la forma m  3 x  5 y

De nuevo, aplicando el Teorema de Frobenius, garantizaremos todos los valores de


m  3 5  3  5  7 .
Entre 0 y 7 se pueden alcanzar los valores siguientes:
0  5 0  3 0
3  5  0  3 1
5  5 1  3  0
6  5 0  3 2
No es posible encontrar soluciones para m  1, 2, 4, 7 .

Así pues, tenemos 470  1  2  4  465 incrementos diferentes, que corresponderán a 465
alturas diferentes de la torre.

Fuente de esta solución: [Link]

22.10.5
Estamos estudiando todas las combinaciones posibles 6a  10b  15c con 0  a, b, c enteros.
Sabemos que si n, m son coprimos, el “número de Frobenius” asociado, es decir, el mayor
entero que no se puede representar de la forma  n   m , con 0   ,  es nm  n  m (Ver
Teoria de Números 20.10).

6a  10b  15c  32a  5c  10b y puesto que 3 y 10 son coprimos, podemos aplicar la fórmula
anterior para garantizar que el máximo entero no representable es 3 10  3  10  17 .

Pero ahora tenemos que aplicar la fórmula del número de Frobenius a 2a  5c , y su máximo
entero no representable es 2  5  2  5  3 .

Luego también podemos tener problemas cuando 2a  5c  3 . En este caso los valores serán:
32a  5c  10b  3  3  10b  9  10b
b  2  9  10  2  29 , y observamos que este número no se puede escribir de ninguna otra
forma 6a  10b  15c , luego 29 tampoco es representable.
Sin embargo, si b  3 , entonces
9  10b  9  10 (b  3  3)  9  10 (b  3)  30  39  10 (b  3)  3  13  10 (b  3)
y 13 sí es representable 2a  5c  2  4  5 1 .

Así pues, el valor máximo no representable de la forma 6a  10b  15c es 29, y la respuesta
correcta es 2+9=11 (D).

También podría haber problemas en los números de la forma 2a  5c  2 , pero entonces serían
de la forma
32a  5c  10b  3  2  10b  6 1  10b , y estos números son perfectamente representables.

Finalmente, también podría haber problemas en los números de la forma 2a  5c  1, pero


entonces serían de la forma
32a  5c   10b  3  10b, b  3  3  10b  3  10 (b  3  3)  3  10 (b  3)  30 
,
 33  10 (b  3)  3  11  10 (b  3)

También podría gustarte